*NURSING > TEST BANK > Test Bank - Physical Examination and Health Assessment 8 edition (by Jarvis) Chapter 01: Evidence-Ba (All)

Test Bank - Physical Examination and Health Assessment 8 edition (by Jarvis) Chapter 01: Evidence-Based Assessment,100% CORRECT

Document Content and Description Below

1. Aftercompletinganinitialassessmentofapatient,thenursehaschartedthathisrespirationsareeupneicand his pulse is 58 beats per minute. These types of data wouldbe: a. Objective. b. Reflective. ... c. Subjective. d. Introspective. ANS: A Objectivedataarewhatthehealthprofessionalobservesbyinspecting,percussing,palpating,andauscultating during the physical examination. Subjective data is what the person says about him or herself during history taking.Thetermsreflectiveandintrospectivearenotusedtodescribedata. DIF: Cognitive Level: Understanding (Comprehension) MSC: Client Needs: Safe and Effective Care Environment: Management of Care 2. ApatienttellsthenursethatheisverynervNouUsR,SisINnaGuTsBea.CteOdM,andfeelshot.Thesetypesofdatawouldbe: a. Objective. b. Reflective. c. Subjective. d. Introspective. ANS: C Subjectivedataarewhatthepersonsaysabouthimorherselfduringhistorytaking.Objectivedataarewhatthe health professional observes by inspecting, percussing, palpating, and auscultating during the physical examination.Thetermsreflectiveandintrospectivearenotusedtodescribedata. DIF: Cognitive Level: Understanding (Comprehension) MSC: Client Needs: Safe and Effective Care Environment: Management of Care 3. Thepatientsrecord,laboratorystudies,objectivedata,andsubjectivedatacombinetoformthe: a. Database. b. Admittingdata. c. Financialstatement. d. Dischargesummary. ANS: A Togetherwiththepatientsrecordandlaboratorystudies,theobjectiveandsubjectivedataformthedatabase. Theotheritemsarenotpartofthepatientsrecord,laboratorystudies,ordata. DIF: Cognitive Level: Remembering (Knowledge) MSC: Client Needs: Safe and Effective Care Environment: Management of Care 4. Whenlisteningtoapatientsbreathsounds,thenurseisunsureofasoundthatisheard.Thenursesnext action should beto: a. Immediately notify the patientsphysician. b. Document the sound exactly as it washeard. c. Validatethedatabyaskingacoworkertolistentothebreathsounds. d. Assessagainin20minutestonotewhetherthesoundisstillpresent. ANS: C NURSINGTB.COM Whenunsureofasoundheardwhilelisteningtoapatientsbreathsounds,thenursevalidatesthedatatoensure accuracy.Ifthenursehaslessexperienceinanarea,thenheorsheasksanexperttolisten. DIF: Cognitive Level: Analyzing (Analysis) MSC: Client Needs: Safe and Effective Care Environment: Management of Care 5. Thenurseisconductingaclassfornewgraduatenurses.Duringtheteachingsession,thenurseshouldkeep inmindthatnovicenurses,withoutabackgroundofskillsandexperiencefromwhichtodraw,aremorelikely to make their decisionsusing: a. Intuition. b. A set ofrules. c. Articles injournals. d. Advice fromsupervisors. ANS: B Novicenursesoperatefromasetofdefined,structuredrules.Theexpertpractitionerusesintuitivelinks. DIF: Cognitive Level: Understanding(Comprehension) MSC: Client Needs: General 6. Expertnurseslearntoattendtoapatternofassessmentdataandactwithoutconsciouslylabelingit.These responses are referred toas: a. Intuition. b. The nursingprocess. c. Clinicalknowledge. d. Diagnosticreasoning. ANS: A Intuitionischaracterizedbypatternrecognitionexpertnurseslearntoattendtoapatternofassessmentdataand act without consciously labeling it. The other options are notcorrect. DIF:CognitiveLevel:Understanding(Comprehension) MSC: Client Needs:General 7. Thenurseisreviewinginformationaboutevidence-basedpractice(EBP).Whichstatementbestreflects EBP? a. EBPreliesontraditionforsupportNoUfRbSesINtpGrTacBt.iCceOsM. b. EBPissimplytheuseofbestpracticetechniquesforthetreatmentofpatients. c. EBPemphasizestheuseofbestevidencewiththecliniciansexperience. d. ThepatientsownpreferencesarenotimportantwithEBP. ANS:C EBPisasystematicapproachtopracticethatemphasizestheuseofbestevidenceincombinationwiththe clinicians experience, as well as patient preferences and values, when making decisions about care and treatment. EBP is more than simply using the best practice techniques to treat patients, and questioning traditionisimportantwhennocompellingandsupportiveresearchevidenceexists. DIF: Cognitive Level: Applying (Application) MSC: Client Needs: Safe and Effective Care Environment: Management of Care 8. Thenurseisconductingaclassonprioritysettingforagroupofnewgraduatenurses.Whichisanexample of a first-level priorityproblem? a. Patient with postoperativepain b. Newly diagnosed patient with diabetes who needs diabeticteaching c. Individual with a small laceration on the sole of thefoot d. Individualwithshortnessofbreathandrespiratorydistress ANS:D First-levelpriorityproblemsarethosethatareemergent,lifethreatening,andimmediate(e.g.,establishingan airway,supportingbreathing,maintainingcirculation,monitoringabnormalvitalsigns). DIF: Cognitive Level: Understanding (Comprehension) MSC: Client Needs: Safe and Effective Care Environment: Management of Care 9. Whenconsideringprioritysettingofproblems,thenursekeepsinmindthatsecond-levelpriorityproblems include which of theseaspects? a. Lowself-esteem b. Lack ofknowledge c. Abnormal laboratoryvalues d. Severely abnormal vitalsigns ANS: C NURSINGTB.COM Second-levelpriorityproblemsarethosethatrequirepromptinterventiontoforestallfurtherdeterioration(e.g., mentalstatuschange,acutepain,abnormallaboratoryvalues,riskstosafetyorsecurity). DIF: Cognitive Level: Understanding (Comprehension) MSC: Client Needs: Safe and Effective Care Environment: Management of Care 10. Whichcriticalthinkingskillhelpsthenurseseerelationshipsamongthedata? a. Validation b. Clustering relatedcues c. Identifying gaps indata d. Distinguishingrelevantfromirrelevant ANS:B Clusteringrelatedcueshelpsthenurseseerelationshipsamongthedata. DIF: Cognitive Level: Understanding(Comprehension) MSC: Client Needs: Safe and Effective Care Environment: Management of Care 11. Thenurseknowsthatdevelopingappropriatenursinginterventionsforapatientreliesonthe appropriatenessofthe diagnosis. a. Nursing b. Medical c. Admission d. Collaborative ANS:A Anaccuratenursingdiagnosisprovidesthebasisfortheselectionofnursinginterventionstoachieveoutcomes forwhichthenurseisaccountable.Theotheritemsdonotcontributetothedevelopmentofappropriatenursing interventions. DIF: Cognitive Level: Understanding (Comprehension) MSC: Client Needs: Safe and Effective Care Environment: Management of Care 12. Thenursingprocessisasequentialmethodofproblemsolvingthatnursesuseandincludeswhichsteps? a. Assessment, treatment, planning, evaluation, discharge, andfollow-up NURSINGTB.COM b. Admission, assessment, diagnosis, treatment, and dischargeplanning c. Admission, diagnosis, treatment, evaluation, and dischargeplanning d. Assessment,diagnosis,outcomeidentification,planning,implementation,andevaluation ANS:D Thenursingprocessisamethodofproblemsolvingthatincludesassessment,diagnosis,outcome identification, planning, implementation, andevaluation. DIF: Cognitive Level: Understanding (Comprehension) MSC: Client Needs: Safe and Effective Care Environment: Management of Care 13. Anewlyadmittedpatientisinacutepain,hasnotbeensleepingwelllately,andishavingdifficulty breathing. How should the nurse prioritize theseproblems? a. Breathing,pain,andsleep b. Breathing,sleep,andpain c. Sleep,breathing,andpain d. Sleep, pain, andbreathing ANS: A First-levelpriorityproblemsareimmediatepriorities,rememberingtheABCs(airway,breathing,and circulation),followedbysecond-levelproblems,andthenthird-levelproblems. DIF: Cognitive Level: Analyzing (Analysis) MSC: Client Needs: Safe and Effective Care Environment: Management of Care 14. Whichofthesewouldbeformulatedbyanurseusingdiagnosticreasoning? a. Nursingdiagnosis b. Medicaldiagnosis c. Diagnostichypothesis d. Diagnostic assessment ANS:C Diagnosticreasoningcallsforthenursetoformulateadiagnostichypothesis;thenursingprocesscallsfora nursingdiagnosis. NURSINGTB.COM DIF:CognitiveLevel:Understanding(Comprehension) MSC: Client Needs:General 15. Barriers to incorporating EBPinclude: a. Nurseslackofresearchskillsinevaluatingthequalityofresearchstudies. b. Lackofsignificantresearchstudies. c. Insufficientclinicalskillsofnurses. d. Inadequate physical assessmentskills. ANS: A Asindividuals,nurseslackresearchskillsinevaluatingthequalityofresearchstudies,areisolatedfromother colleagueswhoareknowledgeableinresearch,andoftenlackthetimetovisitthelibrarytoreadresearch.The other responses are not consideredbarriers. DIF:CognitiveLevel:Understanding(Comprehension) MSC: Client Needs:General 16. Whatstepofthenursingprocessincludesdatacollectionbyhealthhistory,physicalexamination,and interview? a. Planning b. Diagnosis c. Evaluation d. Assessment ANS:D Datacollection,includingperformingthehealthhistory,physicalexamination,andinterview,istheassessmen step of the nursingprocess. DIF:CognitiveLevel:Remembering(Knowledge) MSC: Client Needs:General 17. During a staff meeting, nurses discuss the problems with accessing research studies to incorporate evidence-basedclinicaldecisionmakingintotheirpractice.Whichsuggestionbythenursemanagerwould best help theseproblems? a. Form a committee to conduct researchstudies. b. PostpublishedresearchstudiesonNthUeRuSnINitsGbTuBll.CetOinMboards. c. Encourage the nurses to visit the library to reviewstudies. d. Teachthenurseshowtoconductelectronicsearchesforresearchstudies. ANS: D Facilitating support for EBP would include teaching the nurses how to conduct electronic searches; time to visitthelibrarymaynotbeavailableformanynurses.Actuallyconductingresearchstudiesmaybehelpfulin the long-run but not an immediate solution to reviewing existingresearch. DIF: Cognitive Level: Applying (Application) MSC: Client Needs: Safe and Effective Care Environment: Management of Care 18. Whenreviewingtheconceptsofhealth,thenurserecallsthatthecomponentsofholistichealthinclude which ofthese? a. Disease originates from the externalenvironment. b. The individual human is a closedsystem. c. Nurses are responsible for a patients healthstate. d. Holistic health views the mind, body, and spirit asinterdependent. ANS: D Considerationofthewholepersonistheessenceofholistichealth,whichviewsthemind,body,andspiritas interdependent.Thebasisofdiseaseoriginatesfromboththeexternalenvironmentandfromwithintheperson. Boththeindividualhumanandtheexternalenvironmentareopensystems,continuallychangingandadapting, andeachpersonisresponsibleforhisorherownpersonalhealthstate. DIF: Cognitive Level: Understanding (Comprehension) MSC: Client Needs: Safe and Effective Care Environment: Management of Care 19. Thenurserecognizesthattheconceptofpreventionindescribinghealthisessentialbecause: a. Disease can be prevented by treating the externalenvironment. b. ThemajorityofdeathsamongAmericansunderage65yearsarenotpreventable. c. Preventionplacestheemphasisonthelinkbetweenhealthandpersonalbehavior. d. Themeanstopreventionisthroughtreatmentprovidedbyprimaryhealthcarepractitioners. ANS: C AnaturalprogressiontopreventionroundsouNtUthReSpIrNesGeTnBtc.CoOncMeptofhealth.Guidelinestopreventionplace the emphasis on the link between health and personal behavior. DIF:CognitiveLevel:Understanding(Comprehension) MSC: Client Needs:General 20. Thenurseisperformingaphysicalassessmentonanewlyadmittedpatient.Anexampleofobjective information obtained during the physical assessment includesthe: a. Patients history ofallergies. b. Patients use of medications athome. c. Last menstrual period 1 monthago. d. 2 5 cm scar on the right lowerforearm. ANS: D Objectivedataarethepatientsrecord,laboratorystudies,andconditionthatthehealthprofessionalobservesby inspecting,percussing,palpating,andauscultatingduringthephysicalexamination.Theotherresponsesreflect subjectivedata. DIF: Cognitive Level: Applying (Application) MSC: Client Needs: Safe and Effective Care Environment: Management of Care 21. Avisitingnurseismakinganinitialhomevisitforapatientwhohasmanychronicmedicalproblems. Whichtypeofdatabaseismostappropriatetocollectinthissetting? a. A follow-up data base to evaluate changes at appropriateintervals b. Anepisodicdatabasebecauseofthecontinuing,complexmedicalproblemsofthispatient c. Acompletehealthdatabasebecauseofthenursesprimaryresponsibilityformonitoringthe patientshealth d. Anemergencydatabasebecauseoftheneedtocollectinformationandmakeaccuratediagnoses rapidly ANS: C Thecompletedatabaseiscollectedinaprimarycaresetting,suchasapediatricorfamilypracticeclinic, independentorgroupprivatepractice,collegehealthservice,womenshealthcareagency,visitingnurse agency,orcommunityhealthagency.Inthesesettings,thenurseisthefirsthealthprofessionaltoseethe patientandhastheprimaryresponsibilityformonitoringthepersonshealthcare. DIF: Cognitive Level: Applying (Application) MSC: Client Needs: Safe and Effective Care Environment: Management of Care 22. WhichsituationismostappropriateduringNwUhRiScIhNtGheTBnu.CrsOeMperformsafocusedorproblem-centered history? a. Patient is admitted to a long-term carefacility. b. Patient has a sudden and severe shortness ofbreath. c. Patient is admitted to the hospital for surgery the followingday. d. Patient in an outpatient clinic has cold and influenza-likesymptoms. ANS: D Inafocusedorproblem-centereddatabase,thenursecollectsaminidatabase,whichissmallerinscopethan thecompleteddatabase.Thisminidatabaseprimarilyconcernsoneproblem,onecuecomplex,oronebody system. DIF: Cognitive Level: Applying (Application) MSC: Client Needs: Safe and Effective Care Environment: Management of Care 23. Apatientisattheclinictohaveherbloodpressurechecked.Shehasbeencomingtotheclinicweekly since she changed medications 2 months ago. The nurseshould: a. Collect a follow-up data base and then check her bloodpressure. b. Askhertoreadherhealthrecordandindicateanychangessinceherlastvisit. c. Checkonlyherbloodpressurebecausehercompletehealthhistorywasdocumented2monthsago. d. Obtainacompletehealthhistorybeforecheckingherbloodpressurebecausemuchofherhistory information may havechanged. ANS: A Afollow-updatabaseisusedinallsettingstofollowupshort-termorchronichealthproblems.Theother responses are not appropriate for thesituation. DIF:CognitiveLevel:Applying(Application) MSC: Client Needs: Safe and Effective Care Environment: Management of Care 24. Apatientisbroughtbyambulancetotheemergencydepartmentwithmultipletraumasreceivedinan automobileaccident.Heisalertandcooperative,buthisinjuriesarequitesevere.Howwouldthenurse proceed with datacollection? a. Collecthistoryinformationfirst,thenperformthephysicalexaminationandinstitutelife-saving measures. b. Simultaneouslyaskhistoryquestionswhileperformingtheexaminationandinitiatinglife-saving measures. NURSINGTB.COM c. Collectallinformationonthehistoryform,includingsocialsupportpatterns,strengths,andcoping patterns. d. Performlife-savingmeasuresanddelayaskinganyhistoryquestionsuntilthepatientistransferred to the intensive careunit. ANS: B Theemergencydatabasecallsforarapidcollectionofthedatabase,oftenconcurrentlycompiledwithlife- saving measures. The other responses are not appropriate for thesituation. DIF: Cognitive Level: Analyzing (Analysis) MSC: Client Needs: Safe and Effective Care Environment: Management of Care 25. A42-year-oldpatientofAsiandescentisbeingseenattheclinicforaninitialexamination.Thenurse knowsthatincludingculturalinformationinhishealthassessmentisimportantto: a. Identify the cause of hisillness. b. Make accurate diseasediagnoses. c. Provide cultural health rights for theindividual. d. Provide culturally sensitive and appropriatecare. ANS: D Theinclusionofculturalconsiderationsinthehealthassessmentisofparamountimportancetogatheringdata thatareaccurateandmeaningfulandtointerveningwithculturallysensitiveandappropriatecare. DIF:CognitiveLevel:Understanding(Comprehension) MSC: Client Needs: PsychosocialIntegrity 26. Inthehealthpromotionmodel,thefocusofthehealthprofessionalincludes: a. Changing the patients perceptions ofdisease. b. Identifying biomedical modelinterventions. c. Identifying negative health acts of theconsumer. d. Helping the consumer choose a healthierlifestyle. ANS: D Inthehealthpromotionmodel,thefocusofthehealthprofessionalisonhelpingtheconsumerchoosea healthierlifestyle. NURSINGTB.COM DIF: Cognitive Level: Remembering (Knowledge) MSC:ClientNeeds:HealthPromotionandMaintenance 27. Thenursehasimplementedseveralplannedinterventionstoaddressthenursingdiagnosisofacutepain. Which would be the next appropriateaction? a. Establishpriorities. b. Identify expectedoutcomes. c. Evaluatetheindividualscondition,andcompareactualoutcomeswithexpectedoutcomes. d. Interpret data, and then identify clusters of cues and makeinferences. ANS: C Evaluationisthenextstepaftertheimplementationphaseofthenursingprocess.Duringthisstep,thenurse evaluatestheindividualsconditionandcomparestheactualoutcomeswithexpectedoutcomes. DIF: Cognitive Level: Applying (Application) MSC: Client Needs: Safe and Effective Care Environment: Management of Care 28. Whichstatementbestdescribesaproficientnurse?Aproficientnurseisonewho: a. Haslittleexperiencewithaspecifiedpopulationandusesrulestoguideperformance. b. Hasanintuitivegraspofaclinicalsituationandquicklyidentifiestheaccuratesolution. c. Sees actions in the context of daily plans forpatients. d. Understandsapatientsituationasawholeratherthanalistoftasksandrecognizesthelong-term goals for thepatient. ANS: D The proficient nurse, with more time and experience than the novice nurse, is able to understand a patient situationasawholeratherthanasalistoftasks.Theproficientnurseisabletoseehowtodaysnursingactions canapplytothepointthenursewantsthepatienttoreachatafuturetime. DIF:CognitiveLevel:Applying(Application) MSC: Client Needs:General MULTIPLE RESPONSE 1.Thenurseisreviewingdatacollectedafteranassessment.Ofthedatalistedbelow,whichwouldbe consideredrelatedcuesthatwouldbeclusteredtogetherduringdataanalysis?Selectallthatapply. a. Inspiratory wheezes noted in left lowerlobes NURSINGTB.COM b. Hypoactive bowelsounds c. Nonproductivecough d. Edema, +2, noted on lefthand e. Patient reports dyspnea uponexertion f. Rateofrespirations16breathsperminute ANS: A, C, E,F Clusteringrelatedcueshelpthenurserecognizerelationshipsamongthedata.Thecuesrelatedtothepatients respiratorystatus(e.g.,wheezes,cough,reportofdyspnea,respirationrateandrhythm)areallrelated.Cues relatedtobowelsandperipheraledemaarenotrelatedtotherespiratorycues. DIF: Cognitive Level: Analyzing (Analysis) MSC: Client Needs: Safe and Effective Care Environment: Management of Care MATCHING Put the following patient situations in order according to the level of priority. a. Apatientnewlydiagnosedwithtype2diabetesmellitusdoesnotknowhowtocheckhisown blood glucose levels with aglucometer. b. Ateenagerwhowasstungbyabeeduringasoccermatchishavingtroublebreathing. c. Anolderadultwithaurinarytractinfectionisalsoshowingsignsofconfusionandagitation. 1. a = First-level priorityproblem 2. b = Second-level priorityproblem 3. c = Third-level priorityproblem 1. ANS: B DIF: Cognitive Level: Analyzing(Analysis) MSC: Client Needs: Safe and Effective Care Environment: Management of Care NOT:First-levelpriorityproblemsareimmediatepriorities,suchastroublebreathing(remembertheairway, breathing,circulationpriorities).Second-levelpriorityproblemsarenextinurgency,butnotlife-threatening. Third-levelpriorities(e.g.,patienteducation)areimportanttoapatientshealthbutcanbeaddressedaftermore urgent health problems areaddressed. 2. ANS: C DIF: Cognitive Level: Analyzing(Analysis) MSC: Client Needs: Safe and Effective Care Environment: Management of Care NOT:First-levelpriorityproblemsareimmediatepriorities,suchastroublebreathing(remembertheairway, breathing,circulationpriorities).Second-levelpriorityproblemsarenextinurgency,butnotlife-threatening. Third-levelpriorities(e.g.,patienteducation)NarUeRimSIpNoGrtTanBt.CtoOaMpatientshealthbutcanbeaddressedaftermore urgent health problems are addressed. 3. ANS: A DIF: Cognitive Level: Analyzing(Analysis) MSC: Client Needs: Safe and Effective Care Environment: Management of Care NOT:First-levelpriorityproblemsareimmediatepriorities,suchastroublebreathing(remembertheairway, breathing,circulationpriorities).Second-levelpriorityproblemsarenextinurgency,butnotlife-threatening. Third-levelpriorities(e.g.,patienteducation)areimportanttoapatientshealthbutcanbeaddressedaftermore urgent health problems areaddressed. Chapter 02: Cultural Assessment MULTIPLECHOICE 1. Thenurseisreviewingthedevelopmentofculture.Whichstatementiscorrectregardingthedevelopmentof ones culture? Cultureis: a. Genetically determined on the basis of racialbackground. b. Learned through language acquisition andsocialization. c. A nonspecific phenomenon and is adaptive butunnecessary. d. Biologically determined on the basis of physicalcharacteristics. ANS: B Cultureislearnedfrombirththroughlanguageacquisitionandsocialization.Itisnotbiologicallyor genetically determined and is learned by theindividual. DIF:CognitiveLevel:Understanding(Comprehension) MSC: Client Needs: PsychosocialIntegrity 2. Duringaclassontheaspectsofculture,thenursesharesthatculturehasfourbasiccharacteristics.Which statement correctly reflects one of thesecharacteristics? NURSINGTB.COM a. Cultures are static and unchanging, despite changes aroundthem. b. Cultures are never specific, which makes them hard toidentify. c. Culture is most clearly reflected in a persons language andbehavior. d. Cultureadaptstospecificenvironmentalfactorsandavailablenaturalresources. ANS: D Culture has four basic characteristics. Culture adapts to specific conditions related to environmental and technicalfactorsandtotheavailabilityofnaturalresources,anditisdynamicandeverchanging.Cultureis learned from birth through the process of language acquisition and socialization, but it is not most clearly reflected in ones language andbehavior. DIF: Cognitive Level: Analyzing (Analysis) MSC: Client Needs: Psychosocial Integrity 3. Duringaseminaronculturalaspectsofnursing,thenurserecognizesthatthedefinitionstatingthespecific anddistinctknowledge,beliefs,skills,andcustomsacquiredbymembersofasocietyreflectswhichterm? a. Mores b. Norms c. Culture d. Sociallearning ANS:C Theculturethatdevelopsinanygivensocietyisalwaysspecificanddistinctive,encompassingallofthe knowledge,beliefs,customs,andskillsacquiredbymembersofthesociety.Theothertermsdonotfitthe givendefinition. DIF:CognitiveLevel:Remembering(Knowledge) MSC: Client Needs: PsychosocialIntegrity 4. Whendiscussingtheuseofthetermsubculture,thenurserecognizesthatitisbestdescribedas: a. Fitting as many people into the majority culture aspossible. b. Definingsmallgroupsofpeoplewhodonotwanttobeidentifiedwiththelargerculture. c. Singlingoutgroupsofpeoplewhosufferdifferentialandunequaltreatmentasaresultofcultural variations. NURSINGTB.COM d. Identifyingfairlylargegroupsofpeoplewithsharedcharacteristicsthatarenotcommontoall members of a culture. ANS: D Withincultures,groupsofpeoplesharedifferentbeliefs,values,andattitudes.Differencesoccurbecauseof ethnicity,religion,education,occupation,age,andgender.Whensuchgroupsfunctionwithinalargeculture, they are referred to as subculturalgroups. DIF:CognitiveLevel:Understanding(Comprehension) MSC: Client Needs: PsychosocialIntegrity 5. WhenreviewingthedemographicsofethnicgroupsintheUnitedStates,thenurserecallsthatthelargest and fastest growing populationis: a. Hispanic. b. Black. c. Asian. d. AmericanIndian. ANS: A HispanicsarethelargestandfastestgrowingpopulationintheUnitedStates,followedbyAsians,Blacks, American Indians and Alaska natives, and othergroups. DIF:CognitiveLevel:Remembering(Knowledge) MSC: Client Needs:General 6. Duringanassessment,thenursenoticesthatapatientishandlingasmallcharmthatistiedtoaleatherstrip around his neck. Which action by the nurse isappropriate? a. Ask the patient about the item and itssignificance. b. Askthepatienttolocktheitemwithothervaluablesinthehospitalssafe. c. Tell the patient that a family member should take valuableshome. d. No action isnecessary. ANS: A Thenurseshouldinquireabouttheamuletsmeaning.Amulets,suchascharms,areoftenconsideredan important means of protection from evil spirits by somecultures. DIF:CognitiveLevel:Applying(Application) MSC:ClientNeeds:PsychosocialIntegrity NURSINGTB.COM 7. Thenursemanagerisexplainingculturallycompetentcareduringastaffmeeting.Whichstatement accurately describes the concept of culturally competent care? The caregiver: a. Is able to speak the patients nativelanguage. b. Possesses some basic knowledge of the patients culturalbackground. c. Appliestheproperbackgroundknowledgeofapatientsculturalbackgroundtoprovidethebest possible healthcare. d. Understands and attends to the total context of the patientssituation. ANS: D Culturallycompetentimpliesthatthecaregiverunderstandsandattendstothetotalcontextoftheindividuals situation.Thiscompetencyincludesawarenessofimmigrationstatus,stressfactors,othersocialfactors,and culturalsimilaritiesanddifferences.Itdoesnotrequirethecaregivertospeakthepatientsnativelanguage. DIF: Cognitive Level: Analyzing (Analysis) MSC: Client Needs: Psychosocial Integrity 8. Thenurserecognizesthatanexampleofapersonwhoisheritageconsistentwouldbea: a. Womanwhohasadaptedherclothingtotheclothingstyleofhernewcountry. b. Womanwhofollowsthetraditionsthathermotherfollowedregardingmeals. c. Man who is not sure of his ancestors country oforigin. d. Child who is not able to speak his parents nativelanguage. ANS: B Someonewhoisheritageconsistentlivesalifestylethatreflectshisorhertraditionalheritage,notthenorms and customs of the newcountry. DIF:CognitiveLevel:Understanding(Comprehension) MSC: Client Needs: PsychosocialIntegrity 9. Afteraclassoncultureandethnicity,thenewgraduatenursereflectsacorrectunderstandingoftheconcept of ethnicity with whichstatement? a. Ethnicityisdynamicandeverchanging. b. Ethnicityisthebeliefinahigherpower. c. Ethnicitypertainstoasocialgroupwithinthesocialsystemthatclaimssharedvaluesand traditions. NURSINGTB.COM d. Ethnicityislearnedfrombirththroughtheprocessesoflanguageacquisitionandsocialization. ANS: C Ethnicitypertainstoasocialgroupwithinthesocialsystemthatclaimstohavevariabletraits,suchasa commongeographicorigin,migratorystatus,religion,race,language,values,traditions,symbols,orfood preferences.Cultureisdynamic,everchanging,andlearnedfrombirththroughtheprocessesoflanguage acquisition and socialization. Religion is the belief in a higherpower. DIF:CognitiveLevel:Applying(Application) MSC:ClientNeeds:PsychosocialIntegrity 10. Thenurseiscomparingtheconceptsofreligionandspirituality.Whichofthefollowingisanappropriate component of onesspirituality? a. Belief in and the worship of God orgods b. Attendance at a specific church or place ofworship c. Personal effort made to find purpose and meaning inlife d. Being closely tied to ones ethnicbackground ANS: C Spiritualityreferstoeachpersonsuniquelifeexperiencesandhisorherpersonalefforttofindpurposeand meaning in life. The other responses apply toreligion. DIF:CognitiveLevel:Applying(Application) MSC:ClientNeeds:PsychosocialIntegrity 11. A woman who has lived in the United States for a year after moving from Europe has learned to speak Englishandisalmostfinishedwithhercollegestudies.Shenowdresseslikeherpeersandsaysthatherfamily inEuropewouldhardlyrecognizeher.Thisnurserecognizesthatthissituationillustrateswhichconcept? a. Assimilation b. Heritageconsistency c. Biculturalism d. Acculturation ANS:A Assimilationistheprocessbywhichapersondevelopsanewculturalidentityandbecomeslikemembersof the dominant culture. This concept does not reflect heritage consistency. Biculturalism is a dual pattern of identification;acculturationistheprocessofadaptingtoandacquiringanotherculture. NURSINGTB.COM DIF:CognitiveLevel:Understanding(Comprehension) MSC: Client Needs: PsychosocialIntegrity 12. Thenurseisconductingaheritageassessment.Whichquestionismostappropriateforthisassessment? a. What is yourreligion? b. Do you mostly participate in the religious traditions of yourfamily? c. Do yousmoke? d. Do you have a history of heartdisease? ANS: B Askingquestionsaboutparticipationinthereligioustraditionsoffamilyenablesthenursetoassessapersons heritage.Simplyaskingaboutonesreligion,smokinghistory,orhealthhistorydoesnotreflectheritage. DIF:CognitiveLevel:Applying(Application) MSC:ClientNeeds:PsychosocialIntegrity 13. InthemajoritycultureofAmerica,coughing,sweating,anddiarrheaaresymptomsofanillness.Forsome individuals of Mexican-American origin, however, these symptoms are a normal part of living. The nurse recognizes that this difference is true, probably becauseMexican-Americans: a. Have less efficient immune systems and are oftenill. b. Considerthesesymptomspartofnormalliving,notsymptomsofillhealth. c. Come from Mexico, and coughing is normal and healthythere. d. Areusuallyinalowersocioeconomicgroupandaremorelikelytobesick. ANS: B Thenurseneedstoidentifythemeaningofhealthtothepatient,rememberingthatconceptsarederived,in part,fromthewayinwhichmembersoftheculturalgroupdefinehealth. DIF:CognitiveLevel:Understanding(Comprehension) MSC: Client Needs: PsychosocialIntegrity 14. Thenurseisreviewingtheoriesofillness.Thegermtheory,whichstatesthatmicroscopicorganismssuch as bacteria and viruses are responsible for specific disease conditions, is a basic belief of which theory of illness? a. Holistic b. Biomedical c. Naturalistic d. Magicoreligious NURSINGTB.COM ANS: B Amongthebiomedicalexplanationsfordiseaseisthegermtheory,whichstatesthatmicroscopicorganisms such as bacteria and viruses are responsible for specific disease conditions. The naturalistic, or holistic, perspectiveholdsthattheforcesofnaturemustbekeptinnaturalbalance.Themagicoreligiousperspective holds that supernatural forces dominate and cause illness orhealth. DIF:CognitiveLevel:Understanding(Comprehension) MSC: Client Needs: PsychosocialIntegrity 15. AnAsian-Americanwomanisexperiencingdiarrhea,whichisbelievedtobecoldoryin.Thenurse expects that the woman is likely to try to treat itwith: a. Foods that are hot oryang. b. Readings and Eastern medicinemeditations. c. High doses of medicines believed to becold. d. No treatment is tried because diarrhea is an expected part oflife. ANS: A Yinfoodsarecoldandyangfoodsarehot.Coldfoodsareeatenwithahotillness,andhotfoodsareeatenwith a cold illness. The other explanations do not reflect the yin/yangtheory. DIF:CognitiveLevel:Applying(Application) MSC:ClientNeeds:PsychosocialIntegrity 16. ManyAsiansbelieveintheyin/yangtheory,whichisrootedintheancientChinesephilosophyofTao. WhichstatementmostaccuratelyreflectshealthinanAsianwiththisbelief? a. A person is able to work andproduce. b. A person is happy, stable, and feelsgood. c. All aspects of the person are in perfectbalance. d. A person is able to care for others and functionsocially. ANS: C ManyAsiansbelieveintheyin/yangtheory,inwhichhealthisbelievedtoexistwhenallaspectsoftheperson are in perfect balance. The other statements do not describe thistheory. DIF:CognitiveLevel:Analyzing(Analysis) MSC:ClientNeeds:PsychosocialIntegrity NURSINGTB.COM 17. Illnessisconsideredpartoflifesrhythmiccourseandisanoutwardsignofdisharmonywithin.This statementmostaccuratelyreflectstheviewsaboutillnessfromwhichtheory? a. Naturalistic b. Biomedical c. Reductionist d. Magicoreligious ANS:A The naturalistic perspective states that the laws of nature create imbalances, chaos, and disease. From the perspective of the Chinese, for example, illness is not considered an introducing agent; rather, illness is consideredapartoflifesrhythmiccourseandanoutwardsignofdisharmonywithin.Theotheroptionsarenot correct. DIF:CognitiveLevel:Understanding(Comprehension) MSC: Client Needs: PsychosocialIntegrity 18. Anindividualwhotakesthemagicoreligiousperspectiveofillnessanddiseaseislikelytobelievethathis or her illness was causedby: a. Germs andviruses. b. Supernaturalforces. c. Eating imbalancedfoods. d. An imbalance within his or her spiritualnature. ANS: B Thebasicpremiseofthemagicoreligiousperspectiveisthattheworldisseenasanarenainwhichsupernatura forcesdominate.Thefateoftheworldandthoseinitdependsontheactionsofsupernaturalforcesforgoodor evil. The other answers do not reflect the magicoreligiousperspective. DIF:CognitiveLevel:Understanding(Comprehension) MSC: Client Needs: PsychosocialIntegrity 19. IfanAmericanIndianwomanhascometotheclinictoseekhelpwithregulatingherdiabetes,thenthe nurse can expect thatshe: a. Will comply with the treatmentprescribed. NURSINGTB.COM b. Has obviously given up her belief in naturalistic causes ofdisease. c. May also be seeking the assistance of a shaman or medicineman. d. Willneedextrahelpindealingwithherillnessandmaybeexperiencingacrisisoffaith. ANS: C Whenself-treatmentisunsuccessful,theindividualmayturntothelayorfolkhealingsystems,tospiritualor religioushealing,ortoscientificbiomedicine.Inadditiontoseekinghelpfromabiomedicalorscientifichealth care provider, patients may also seek help from folk or religioushealers. DIF: Cognitive Level: Analyzing (Analysis) MSC: Client Needs: Psychosocial Integrity 20. AnolderMexican-Americanwomanwithtraditionalbeliefshasbeenadmittedtoaninpatientcareunit.A culturally sensitive nursewould: a. Contact the hospital administrator about the best course ofaction. b. Automaticallygetacuranderoforher,becauserequestingoneherselfisnotculturallyappropriate. c. Furtherassessthepatientsculturalbeliefsandofferthepatientassistanceincontactingacurandero or priest if she desires. d. AskthefamilywhattheywouldliketodobecauseMexican-Americanstraditionallygivecontrol of decision making to theirfamilies. ANS: C Inadditiontoseekinghelpfromthebiomedicalorscientifichealthcareprovider,patientsmayalsoseekhelp fromfolkorreligioushealers.Somepeople,suchasthoseofMexican-AmericanorAmerican-Indianorigins, maybelievethatthecureisincompleteunlessthebody,mind,andspiritarealsohealed(althoughthedivision of the person into parts is a Westernconcept). DIF: Cognitive Level: Analyzing (Analysis) MSC: Client Needs: Psychosocial Integrity 21. A63-year-oldChinese-Americanmanentersthehospitalwithcomplaintsofchestpain,shortnessof breath,andpalpitations.Whichstatementmostaccuratelyreflectsthenursesbestcourseofaction? a. The nurse should focus on performing a full cardiacassessment. b. Thenurseshouldfocusonpsychosomaticcomplaintsbecausethepatienthasjustlearnedthathis wife hascancer. c. Thispatientisnotinanydangeratpresent;therefore,thenurseshouldsendhimhomewith instructions to contact hisphysician. NURSINGTB.COM d. Itisunclearwhatishappeningwiththispatient;consequently,thenurseshouldperforman assessment in both the physical and the psychosocialrealms. ANS: D Wideculturalvariationsexistinthemannerinwhichcertainsymptomsanddiseaseconditionsareperceived, diagnosed, labeled, and treated. Chinese-Americans sometimes convert mental experiences or states into bodilysymptoms(e.g.,complainingofcardiacsymptomsbecausethecenterofemotionintheChineseculture is theheart). DIF: Cognitive Level: Analyzing (Analysis) MSC: Client Needs: Psychosocial Integrity 22. Symptoms,suchaspain,areofteninfluencedbyapersonsculturalheritage.Whichofthefollowingisa true statement regarding pain? a. Nursesattitudestowardtheirpatientspainareunrelatedtotheirownexperienceswithpain. b. Nursesneedtorecognizethatmanyculturespracticesilentsufferingasaresponsetopain. c. Anursesareaofclinicalpracticewillmostlikelydeterminehisorherassessmentofapatients pain. d. Anursesyearsofclinicalexperienceandcurrentpositionarestrongindicatorsofhisorher response to patient pain. ANS: B Silentsufferingisapotentialresponsetopaininmanycultures.Thenursesassessmentofpainneedstobe embedded in a cultural context. The other responses are notcorrect. DIF:CognitiveLevel:Understanding(Comprehension) MSC: Client Needs: PsychosocialIntegrity 23. Thenurseisreviewingconceptsofculturalaspectsofpain.Whichstatementistrueregardingpain? a. All patients will behave the same way when inpain. b. Justaspatientsvaryintheirperceptionsofpain,sowilltheyvaryintheirexpressionsofpain. c. Culturalnormshaveverylittletodowithpaintolerance,becausepaintoleranceisalways biologicallydetermined. d. Apatientsexpressionofpainislargelydependentontheamountoftissueinjuryassociatedwith thepain. ANS: B Inadditiontoexpectingvariationsinpainperceptionandtolerance,thenurseshouldexpectvariationsinthe expressionofpain.ItiswellknownthatindiNURSINGTB.COMsocialenvironmentforvalidationand comparison. The other statements are incorrect. DIF:CognitiveLevel:Understanding(Comprehension) MSC: Client Needs: PsychosocialIntegrity 24. Duringaclassonreligionandspirituality,thenurseisaskedtodefinespirituality.Whichansweris correct?Spirituality: a. Is a personal search to discover a supremebeing. b. Isanorganizedsystemofbeliefsconcerningthecause,nature,andpurposeoftheuniverse. c. Isabeliefthateachpersonexistsforeverinsomeform,suchasabeliefinreincarnationorthe afterlife. d. Arisesoutofeachpersonsuniquelifeexperienceandhisorherpersonalefforttofindpurposein life. ANS: D Spiritualityarisesoutofeachpersonsuniquelifeexperienceandhisorherpersonalefforttofindpurposeand meaning in life. The other definitions reflect the concept ofreligion. DIF: Cognitive Level: Understanding (Comprehension) MSC: Client Needs: Psychosocial Integrity 25. Thenurserecognizesthatworkingwithchildrenwithadifferentculturalperspectivemaybeespecially difficultbecause: a. Childrenhavespiritualneedsthatareinfluencedbytheirstagesofdevelopment. b. Childrenhavespiritualneedsthataredirectreflectionsofwhatisoccurringintheirhomes. c. Religious beliefs rarely affect the parents perceptions of theillness. d. Parentsareoftenthedecisionmakers,andtheyhavenoknowledgeoftheirchildrensspiritual needs. ANS: A Illness during childhood may be an especially difficult clinical situation. Children, as well as adults, have spiritualneedsthatvaryaccordingtothechildsdevelopmentallevelandthereligiousclimatethatexistsinthe family. The other statements are notcorrect. DIF:CognitiveLevel:Applying(Application) MSC:ClientNeeds:PsychosocialIntegrity 26. A30-year-oldwomanhasrecentlymovedtotheUnitedStateswithherhusband.Theyarelivingwiththe womanssisteruntiltheycangetahomeoftheirown.Whencompanyarrivestovisitwiththewomanssister, thewomanfeelssuddenlyshyandretreatstotNhUeRbaScINkGbeTdBr.oCoOmMtohideuntilthecompanyleaves.Sheexplains that her reaction to guests is simply because she does not know how to speak perfect English. This woman could beexperiencing: a. Cultureshock. b. Culturaltaboos. c. Culturalunfamiliarity. d. Culturedisorientation. ANS: A Cultureshockisatermusedtodescribethestateofdisorientationorinabilitytorespondtothebehaviorofa different cultural group because of its sudden strangeness, unfamiliarity, and incompatibility with the individuals perceptions and expectations. The other terms are notcorrect. DIF: Cognitive Level: Analyzing (Analysis) MSC: Client Needs: Psychosocial Integrity 27. Afterasymptomisrecognized,thefirsteffortattreatmentisoftenself-care.Whichofthefollowing statements about self-care is true? Self-careis: a. Not recognized as valuable by most health careproviders. b. Usually ineffective and may delay more effectivetreatment. c. Always less expensive than biomedicalalternatives. d. Influenced by the accessibility of over-the-countermedicines. ANS: D Afterasymptomisidentified,thefirsteffortattreatmentisoftenself-care.Theavailabilityofover-the-counter medications, the relatively high literacy level of Americans, and the influence of the mass media in communicatinghealth-relatedinformationtothegeneralpopulationhavecontributedtothehighpercentageof cases ofself-treatment. DIF:CognitiveLevel:Understanding(Comprehension) MSC: Client Needs: PsychosocialIntegrity 28. Thenurseisreviewingthehot/coldtheoryofhealthandillness.Whichstatementbestdescribesthebasic tenets of thistheory? a. Thecausationofillnessisbasedonsupernaturalforcesthatinfluencethehumorsofthebody. b. Herbsandmedicinesareclassifiedontheirphysicalcharacteristicsofhotandcoldandthehumors of thebody. NURSINGTB.COM c. Thefourhumorsofthebodyconsistofblood,yellowbile,spiritualconnectedness,andsocial aspects of theindividual. d. Thetreatmentofdiseaseconsistsofaddingorsubtractingcold,heat,dryness,orwetnesstorestore the balance of the humors of thebody. ANS: D Thehot/coldtheoryofhealthandillnessisbasedonthefourhumorsofthebody:blood,phlegm,blackbile, andyellowbile.Thesehumorsregulatethebasicbodilyfunctions,describedintermsoftemperature,dryness, and moisture. The treatment of disease consists of adding or subtracting cold, heat, dryness, or wetness to restore the balance of the humors. The other statements are notcorrect. DIF:CognitiveLevel:Understanding(Comprehension) MSC: Client Needs: PsychosocialIntegrity 29. Inthehot/coldtheory,illnessesarebelievedtobecausedbyhotorcoldenteringthebody.Whichofthese patient conditions is most consistent with a coldcondition? a. Patient with diabetes and renalfailure b. Teenager with an abscessedtooth c. Child with symptoms of itching and arash d. Older man with gastrointestinaldiscomfort ANS: D Illnessesbelievedtobecausedbycoldenteringthebodyincludeearache,chestcramps,gastrointestinal discomfort,rheumatism,andtuberculosis.Thoseillnessesbelievedtobecausedbyheat,oroverheating, include sore throats, abscessed teeth, rashes, and kidneydisorders. DIF: Cognitive Level: Analyzing (Analysis) MSC: Client Needs: Psychosocial Integrity 30. Whenprovidingculturallycompetentcare,nursesmustincorporateculturalassessmentsintotheirhealth assessments.Whichstatementismostappropriatetousewheninitiatinganassessmentofculturalbeliefswith an older American-Indianpatient? a. Are you of the Christianfaith? b. Do you want to see a medicineman? c. How often do you seek help from medicalproviders? d. What cultural or spiritual beliefs are important toyou? ANS: D NURSINGTB.COM The nurse needs to assess the cultural beliefs and practices of the patient. American Indians may seek assistancefromamedicinemanorshaman,butthenurseshouldnotassumethis.Anopen-endedquestion regardingculturalandspiritualbeliefsisbestusedinitiallywhenperformingaculturalassessment. DIF: Cognitive Level: Analyzing (Analysis) MSC: Client Needs: Psychosocial Integrity 31. Duringaclassonculturalpractices,thenursehearsthetermculturaltaboo.Whichstatementillustratesthe concept of a culturaltaboo? a. Believing that illness is a punishment ofsin b. Trying prayer before seeking medicalhelp c. Refusing to accept blood products as part oftreatment d. Statingthatachildsbirthdefectistheresultoftheparentssins ANS:C Culturaltaboosarepracticesthataretobeavoided,suchasreceivingbloodproducts,eatingpork,and consuming caffeine. The other answers do not reflect culturaltaboos. DIF: Cognitive Level: Applying (Application) MSC: Client Needs: Psychosocial Integrity 32. Thenurserecognizesthatcategoriessuchasethnicity,gender,andreligionillustratetheconceptof: a. Family. b. Cultures. c. Spirituality. d. Subcultures. ANS: D Withincultures,groupsofpeoplesharedifferentbeliefs,values,andattitudes.Differencesoccurbecauseof ethnicity,religion,education,occupation,age,andgender.Whensuchgroupsfunctionwithinalargeculture, they are referred to as subculturalgroups. DIF:CognitiveLevel:Understanding(Comprehension) MSC: Client Needs: PsychosocialIntegrity 33. Thenurseisreviewingconceptsrelatedtoonesheritageandbeliefs.Thebeliefindivineorsuperhuman power(s)tobeobeyedandworshippedasthecreator(s)andruler(s)oftheuniverseisknownas: a. Culture. b. Religion. c. Ethnicity. d. Spirituality. NURSINGTB.COM ANS: B Religionisdefinedasanorganizedsystemofbeliefsconcerningthecause,nature,andpurposeoftheuniverse, especially belief in or the worship of God or gods. Spirituality is born out of each persons unique life experiences and his or her personal efforts to find purpose and meaning in life. Ethnicity pertains to a social group within the social system that claims to possess variable traits, such as a common geographic origin, religion, race, andothers. DIF:CognitiveLevel:Remembering(Knowledge) MSC: Client Needs: PsychosocialIntegrity 34. Whenplanningaculturalassessment,thenurseshouldincludewhichcomponent? a. Familyhistory b. Chiefcomplaint c. Medicalhistory d. Health-related beliefs ANS:D Health-relatedbeliefsandpracticesareonecomponentofaculturalassessment.Theotheritemsreflectother aspects of the patientshistory. DIF:CognitiveLevel:Understanding(Comprehension) MSC: Client Needs: PsychosocialIntegrity 35. WhichofthefollowingreflectsthetraditionalhealthandillnessbeliefsandpracticesofthoseofAfrican heritage? Healthis: a. Beingrewardedforgoodbehavior. b. Thebalanceofthebodyandspirit. c. Maintained by wearing jadeamulets. d. Being in harmony withnature. ANS: D NURSINGTB.COM ThebeliefthathealthisbeinginharmonywithnaturereflectsthehealthbeliefsofthoseofAfricanheritages. TheotherexamplesrepresentIberianandCentralandSouthAmericanheritages,American-Indianheritages, and Asianheritages. DIF: Cognitive Level: Analyzing (Analysis) MSC: Client Needs: Psychosocial Integrity MULTIPLE RESPONSE 1. Thenurseisreviewingaspectsofculturalcare.Whichstatementsillustrateproperculturalcare?Selectall thatapply. a. Examinethepatientwithinthecontextofonesownculturalhealthandillnesspractices. b. Select questions that are notcomplex. c. Ask questionsrapidly. d. Touch patients within the cultural boundaries of theirheritage. e. Pace questions throughout the physicalexamination. ANS: B, D, E Patientsshouldbeexaminedwithinthecontextoftheirownculturalhealthandillnesspractices.Questions should be simply stated and not rapidlyasked. DIF: Cognitive Level: Analyzing (Analysis) MSC: Client Needs: Psychosocial Integrity 2. Thenurseisaskingquestionsaboutapatientshealthbeliefs.Whichquestionsareappropriate?Selectallthat apply. a. What is your definition ofhealth? b. Does your family have a history ofcancer? c. How do you describeillness? d. What did your mother do to keep you from gettingsick? e. Have you ever had anysurgeries? f. How do you keep yourselfhealthy? ANS: A, C, D, F Thequestionslistedareappropriatequestionsforanassessmentofapatientshealthbeliefsandpractices.The questionsregardingfamilyhistoryandsurgerNURSINGTB.COMtientsphysicalhistory,notthepatientshealth beliefs. DIF:CognitiveLevel:Applying(Application) MSC:ClientNeeds:PsychosocialIntegrity Chapter 03: The Interview MULTIPLE CHOICE 1. Thenurseisconductinganinterviewwithawomanwhohasrecentlylearnedthatsheispregnantandwho has come to the clinic today to begin prenatal care. The woman states that she and her husband are excited aboutthepregnancybuthaveafewquestions.Shelooksnervouslyatherhandsduringtheinterviewandsighs loudly.Consideringtheconceptofcommunication,whichstatementdoesthenurseknowtobemostaccurate? The womanis: a. Excited about her pregnancy but nervous about thelabor. b. Exhibiting verbal and nonverbal behaviors that do notmatch. c. Excitedaboutherpregnancy,butherhusbandisnotandthisisupsettingtoher. d. Notexcitedaboutherpregnancybutbelievesthenursewillnegativelyrespondtoherifshestates this. ANS: B Communication is all behaviors, conscious and unconscious, verbal and nonverbal. All behaviors have meaning.Herbehaviordoesnotimplythatsheisnervousaboutlabor,upsetbyherhusband,orworriedabout the nursesresponse. DIF:CognitiveLevel:Analyzing(Analysis) MSC:ClientNeeds:PsychosocialIntegrity NURSINGTB.COM 2. Receivingisapartofthecommunicationprocess.Whichreceiverismostlikelytomisinterpretamessage sent by a health careprofessional? a. Well-adjusted adolescent who came in for a sportsphysical b. Recovering alcoholic who came in for a basic physicalexamination c. Man whose wife has just been diagnosed with lungcancer d. Manwithahearingimpairmentwhousessignlanguagetocommunicateandwhohasan interpreter withhim ANS: C Thereceiverattachesmeaningdeterminedbyhisorherexperiences,culture,self-concept,andcurrentphysical and emotional states. The man whose wife has just been diagnosed with lung cancer may be experiencing emotions that affect hisreceiving. DIF: Cognitive Level: Analyzing (Analysis) MSC: Client Needs: Psychosocial Integrity 3. Thenursemakeswhichadjustmentinthephysicalenvironmenttopromotethesuccessofaninterview? a. Reduces noise by turning off televisions andradios b. Reducesthedistancebetweentheinterviewerandthepatientto2feetorless c. Providesadimlightthatmakestheroomcozyandhelpsthepatientrelax d. Arrangesseatingacrossadeskortabletoallowthepatientsomepersonalspace ANS:A Thenurseshouldreducenoisebyturningoffthetelevision,radio,andotherunnecessaryequipment,because multiplestimuliareconfusing.Theinterviewerandpatientshouldbeapproximately4to5feetapart;theroom should be well-lit, enabling the interviewer and patient to see each other clearly. Having a table or desk in betweenthetwopeoplecreatestheideaofabarrier;equal-statusseating,ateyelevel,isbetter. DIF:CognitiveLevel:Applying(Application) MSC:ClientNeeds:PsychosocialIntegrity 4. Inaninterview,thenursemayfinditnecessarytotakenotestoaidhisorhermemorylater.Which statement is true regardingnote-taking? a. Note-takingmayimpedethenursesobservationofthepatientsnonverbalbehaviors. b. Note-takingallowsthepatienttocNonUtRinSuIeNaGtThBis.CoOrMherownpaceasthenurserecordswhatissaid. c. Note-takingallowsthenursetoshiftattentionawayfromthepatient,resultinginanincreased comfortlevel. d. Note-takingallowsthenursetobreakeyecontactwiththepatient,whichmayincreasehisorher level ofcomfort. ANS: A Theuseofhistoryformsandnote-takingmaybeunavoidable.However,thenursemustbeawarethatnote- taking during the interview has disadvantages. It breaks eye contact too often and shifts the attention away fromthepatient,whichdiminisheshisorhersenseofimportance.Note-takingmayalsointerruptthepatients narrativeflow,anditimpedestheobservationofthepatientsnonverbalbehavior. DIF:CognitiveLevel:Understanding(Comprehension) MSC: Client Needs: PsychosocialIntegrity 5. Thenurseasks,Iwouldliketoaskyousomequestionsaboutyourhealthandyourusualdailyactivitiesso thatwecanbetterplanyourstayhere.Thisquestionisfoundatthe phase of the interview process. a. Summary b. Closing c. Body d. Openingorintroduction ANS:D Whengatheringacompletehistory,thenurseshouldgivethereasonfortheinterviewduringtheopeningor introductionphaseoftheinterview,notduringorattheendoftheinterview. DIF:CognitiveLevel:Understanding(Comprehension) MSC: Client Needs: PsychosocialIntegrity 6. Awomanhasjustenteredtheemergencydepartmentafterbeingbatteredbyherhusband.Thenurseneeds to get some information from her to begin treatment. What is the best choice for an opening phase of the interview with thispatient? a. Hello, Nancy, my name is Mrs.C. b. Hello, Mrs. H., my name is Mrs. C. It sure is coldtoday! c. Mrs. H., my name is Mrs. C. How areyou? d. Mrs.H.,mynameisMrs.C.Illneedtoaskyouafewquestionsaboutwhathappened. NURSINGTB.COM ANS: D Addressthepersonbyusinghisorhersurname.Thenurseshouldintroducehimorherselfandgivethereason for the interview. Friendly small talk is not needed to buildrapport. DIF:CognitiveLevel:Applying(Application) MSC:ClientNeeds:PsychosocialIntegrity 7. Duringaninterview,thenursestates,Youmentionedhavingshortnessofbreath.Tellmemoreaboutthat. Which verbal skill is used with thisstatement? a. Reflection b. Facilitation c. Directquestion d. Open-endedquestion ANS:D Theopen-endedquestionasksfornarrativeinformation.Itstatesthetopictobediscussedbutonlyingeneral terms.Thenurseshoulduseittobegintheinterview,tointroduceanewsectionofquestions,andwheneverth person introduces a newtopic. DIF:CognitiveLevel:Understanding(Comprehension) MSC: Client Needs: PsychosocialIntegrity 8. Apatienthasfinishedgivingthenurseinformationaboutthereasonheisseekingcare.Whenreviewingthe data, the nurse finds that some information about past hospitalizations is missing. At this point, which statement by the nurse would be most appropriate to gather thesedata? a. Mr. Y., at your age, surely you have been hospitalizedbefore! b. Mr.Y.,IjustneedpermissiontogetyourmedicalrecordsfromCountyMedical. c. Mr.Y.,youmentionedthatyouhavebeenhospitalizedonseveraloccasions.Wouldyoutellme more aboutthat? d. Mr.Y.,Ijustneedtogetsomeadditionalinformationaboutyourpasthospitalizations.Whenwas the last time you were admitted for chestpain? ANS: D Thenurseshouldusedirectquestionsafterthepersonsopeningnarrativetofillinanydetailsheorsheleftout. The nurse also should use direct questions when specific facts are needed, such as when asking about past health problems or during the review ofsystems. DIF: Cognitive Level: Applying (Application) MSC:ClientNeeds:PsychosocialIntegrity NURSINGTB.COM 9. Inusingverbalresponsestoassistthepatientsnarrative,someresponsesfocusonthepatientsframeof referenceandsomefocusonthehealthcareprovidersperspective.Anexampleofaverbalresponsethat focuses on the health care providers perspective wouldbe: a. Empathy. b. Reflection. c. Facilitation. d. Confrontation. ANS: D When the health care provider uses the response of confrontation, the frame of reference shifts from the patientsperspectivetotheperspectiveofthehealthcareprovider,andthehealthcareproviderstartstoexpress his or her own thoughts and feelings. Empathy, reflection, and facilitation responses focus on the patients frame ofreference. DIF:CognitiveLevel:Remembering(Knowledge) MSC: Client Needs: PsychosocialIntegrity 10. Whentakingahistoryfromanewlyadmittedpatient,thenursenoticesthatheoftenpausesandexpectantly looks at the nurse. What would be the nurses best response to this behavior? a. Be silent, and allow him to continue when he isready. b. Smileathimandsay,Dontworryaboutallofthis.Imsurewecanfindoutwhyyourehavingthese pains. c. Leanbackinthechairandask,Youarelookingatmekindoffunny;thereisntanythingwrong,is there? d. Standupandsay,Icanseethatthisinterviewisuncomfortableforyou.Wecancontinueitanother time. ANS: A Silentattentivenesscommunicatesthatthepersonhastimetothinkandtoorganizewhatheorshewishesto saywithoutaninterruptionfromthenurse.Healthprofessionalsmostofteninterruptthisthinkingsilence.The other responses are not conducive to idealcommunication. DIF:CognitiveLevel:Applying(Application) MSC:ClientNeeds:PsychosocialIntegrity 11. Awomanisdiscussingtheproblemssheishavingwithher2-year-oldson.Shesays,Hewontgotosleep atnight,andduringthedayhehasseveralfits.Igetsoupsetwhenthathappens.Thenursesbestverbal response would be: NURSINGTB.COM a. Go on, Imlistening. b. Fits? Tell me what you mean bythis. c. Yes, it can be upsetting when a child has afit. d. Dont be upset when he has a fit; every 2 year old hasfits. ANS: B Thenurseshoulduseclarificationwhenthepersonswordchoiceisambiguousorconfusing(e.g.,Tellmewhat youmeanbyfits.).Clarificationisalsousedtosummarizethepersonswordsortosimplifythewordstomake themclearer;thenurseshouldthenaskifheorsheisontherighttrack. DIF:CognitiveLevel:Applying(Application) MSC:ClientNeeds:PsychosocialIntegrity 12. A17-year-oldsinglemotherisdescribinghowdifficultitistoraisea3-year-oldchildbyherself.During thecourseoftheinterviewshestates,Icantbelievemyboyfriendleftmetodothisbymyself!Whataterrible thingtodotome!Whichoftheseresponsesbythenurseusesempathy? a. You feelalone. b. You cant believe he left youalone? c. It must be so hard to face this allalone. d. I would be angry, too; raising a child alone is nopicnic. ANS: C An empathetic response recognizes the feeling and puts it into words. It names the feeling, allows its expression, and strengthens rapport. Other empathetic responses are, This must be very hard for you, I understand,orsimplyplacingyourhandonthepersonsarm.Simplyreflectingthepersonswordsoragreeing with the person is not an empatheticresponse. DIF:CognitiveLevel:Applying(Application) MSC:ClientNeeds:PsychosocialIntegrity 13. A man has been admitted to the observation unit for observation after being treated for a large cut on his forehead. As the nurse works through the interview, one of the standard questions has to do with alcohol, tobacco,anddruguse.Whenthenurseaskshimabouttobaccouse,hestates,Iquitsmokingaftermywifedied 7yearsago.However,thenursenoticesanopenpackofcigarettesinhisshirtpocket.Usingconfrontation,the nurse couldsay: a. Mr. K., I know that you arelying. b. Mr. K., come on, tell me how much yousmoke. NURSINGTB.COM c. Mr.K.,Ididntrealizeyourwifehaddied.Itmustbedifficultforyouatthistime.Pleasetellme more aboutthat. d. Mr.K.,youhavesaidthatyoudontsmoke,butIseethatyouhaveanopenpackofcigarettesin yourpocket. ANS: D Inthecaseofconfrontation,acertainaction,feeling,orstatementhasbeenobserved,andthenursenow focusesthepatientsattentiononit.Thenurseshouldgivehonestfeedbackaboutwhatisseenorfelt. Confrontationmayfocusonadiscrepancy,orthenursemayconfrontthepatientwhenpartsofthestoryare inconsistent. The other statements are notappropriate. DIF:CognitiveLevel:Applying(Application) MSC:ClientNeeds:PsychosocialIntegrity 14. Thenursehasusedinterpretationregardingapatientsstatementoractions.Afterusingthistechnique,it would be best for the nurseto: a. Apologize, because using interpretation can be demeaning for thepatient. b. Allow time for the patient to confirm or correct theinference. c. Continue with the interview as though nothing hashappened. d. Immediatelyrestatethenursesconclusiononthebasisofthepatientsnonverbalresponse. ANS: B Interpretation is not based on direct observation as is confrontation, but it is based on ones inference or conclusion.Thenurserisksmakingthewronginference.Ifthisisthecase,thenthepatientwillcorrectit. However,eveniftheinferenceiscorrect,interpretationhelpspromptfurtherdiscussionofthetopic. DIF: Cognitive Level: Analyzing (Analysis) MSC: Client Needs: Psychosocial Integrity 15. Duringaninterview,awomansays,IhavedecidedthatIcannolongerallowmychildrentolivewiththeir fathersviolence,butIjustcantseemtoleavehim.Usinginterpretation,thenursesbestresponsewouldbe: a. You are going to leavehim? b. If you are afraid for your children, then why cant youleave? c. Itsoundsasifyoumightbeafraidofhowyourhusbandwillrespond. d. Itsoundsasthoughyouhavemadeyourdecision.Ithinkitisagoodone. ANS: C NURSINGTB.COM This statement is not based on ones inference or conclusion. It links events, makes associations, or implies cause.Interpretationalsoascribesfeelingsandhelpsthepersonunderstandhisorherownfeelingsinrelation to the verbal message. The other statements do not reflectinterpretation. DIF:CognitiveLevel:Applying(Application) MSC:ClientNeeds:PsychosocialIntegrity 16. A pregnant woman states, I just know labor will be so painful that I wont be able to stand it. I know it soundsawful,butIreallydreadgoingintolabor.Thenurserespondsbystating,Oh,dontworryaboutlaborso much.Ihavebeenthroughit,andalthoughitispainful,manygoodmedicationsareavailabletodecreasethe pain.Whichstatementistrueregardingthisresponse?Thenursesreplywasa: a. Therapeuticresponse.Bysharingsomethingpersonal,thenursegiveshopetothiswoman. b. Nontherapeuticresponse.Byprovidingfalsereassurance,thenurseactuallycutofffurther discussion of the womansfears. c. Therapeuticresponse.Byprovidinginformationaboutthemedicationsavailable,thenurseis giving information to thewoman. d. Nontherapeuticresponse.Thenurseisessentiallygivingthemessagetothewomanthatlabor cannot be tolerated withoutmedication. ANS: B Byprovidingfalseassuranceorreassurance,thiscouragebuilderrelievesthewomansanxietyandgivesthe nurse the false sense of having provided comfort. However, for the woman, providing false assurance or reassuranceactuallyclosesoffcommunication,trivializesheranxiety,andeffectivelydeniesanyfurthertalk ofit. DIF: Cognitive Level: Analyzing (Analysis) MSC: Client Needs: Psychosocial Integrity 17. Duringavisittotheclinic,apatientstates,ThedoctorjusttoldmehethoughtIoughttostopsmoking.He doesnt understand how hard Ive tried. I just dont know the best way to do it. What should I do? The nurses most appropriate response in this case wouldbe: a. IdquitifIwereyou.Thedoctorreallyknowswhatheistalkingabout. b. Wouldyoulikesomeinformationaboutthedifferentwaysapersoncanquitsmoking? c. Stoppingyourdependenceoncigarettescanbeverydifficult.Iunderstandhowyoufeel. d. Whyareyouconfused?Didntthedoctorgiveyoutheinformationaboutthesmokingcessation program weoffer? ANS: B ClarificationshouldbeusedwhenthepersonsNwUoRrSdIcNhGoTicBe.CisOaMmbiguousorconfusing.Clarificationisalso usedtosummarizethepersonswordsortosimplifythewordstomakethemclearer;thenurseshouldthenask ifheorsheisontherighttrack.Theotherresponsesgiveunwantedadviceordonotofferahelpfulresponse. DIF:CognitiveLevel:Applying(Application) MSC:ClientNeeds:PsychosocialIntegrity 18. Asthenurseentersapatientsroom,thenursefindshercrying.Thepatientstatesthatshehasjustfoundout that the lump in her breast is cancer and says, Im so afraid of, um, you know. The nurses most therapeutic response would be to say in a gentlemanner: a. Youre afraid you might lose yourbreast? b. No, Im not sure what you are talkingabout. c. Illwaithereuntilyougetyourselfundercontrol,andthenwecantalk. d. Icanseethatyouareveryupset.Perhapsweshoulddiscussthislater. ANS: A Reflectionechoesthepatientswords,repeatingpartofwhatthepersonhasjustsaid.Reflectioncanalsohelp express the feelings behind a personswords. DIF: Cognitive Level: Applying (Application) MSC: Client Needs: Psychosocial Integrity 19. A nurse is taking complete health histories on all of the patients attending a wellness workshop. On the historyform,oneofthewrittenquestionsasks,Youdontsmoke,drink,ortakedrugs,doyou?Thisquestionis an exampleof: a. Talking toomuch. b. Usingconfrontation. c. Using biased or leadingquestions. d. Using blunt language to deal with distastefultopics. ANS:C This question is an example of using leading or biased questions. Asking, You dont smoke, do you? implies thatoneanswerisbetterthananother.Ifthepersonwantstopleasesomeone,thenheorsheiseitherforcedto answerinawaythatcorrespondstohisorherimpliedvaluesorismadetofeelguiltywhenadmittingtheother answer. DIF:CognitiveLevel:Understanding(Comprehension) MSC: Client Needs: PsychosocialIntegrity 20. Whenobservingapatientsverbalandnonverbalcommunication,thenursenoticesadiscrepancy.Which statementistrueregardingthissituation?TheNnUurRsSeIsNhGouTlBd.:COM a. Asksomeonewhoknowsthepatientwelltohelpinterpretthisdiscrepancy. b. Focusonthepatientsverbalmessage,andtrytoignorethenonverbalbehaviors. c. Trytointegratetheverbalandnonverbalmessagesandtheninterpretthemasanaverage. d. Focusonthepatientsnonverbalbehaviors,becausetheseareoftenmorereflectiveofapatients truefeelings. ANS: D When nonverbal and verbal messages are congruent, the verbal message is reinforced. When they are incongruent,thenonverbalmessagetendstobethetrueonebecauseitisunderlessconsciouscontrol.Thus studying the nonverbal messages of the patients and examiners and understanding their meanings are important. The other statements are nottrue. DIF:CognitiveLevel:Applying(Application) MSC:ClientNeeds:PsychosocialIntegrity 21. During an interview, a parent of a hospitalized child is sitting in an open position. As the interviewer beginstodiscusshissonstreatment,however,hesuddenlycrosseshisarmsagainsthischestandcrosseshis legs. This changed posture would suggest that the parentis: a. Simply changingpositions. b. More comfortable in thisposition. c. Tired and needs a break from theinterview. d. Uncomfortable talking about his sonstreatment. ANS: D Thepersonspositionisnoted.Anopenpositionwiththeextensionoflargemusclegroupsshowsrelaxation, physical comfort, and a willingness to share information. A closed position with the arms and legs crossed tendstolookdefensiveandanxious.Anychangeinpostureshouldbenoted.Ifapersoninarelaxedposition suddenlytenses,thenthischangeinposturesuggestspossiblediscomfortwiththenewtopic. DIF: Cognitive Level: Analyzing (Analysis) MSC: Client Needs: Psychosocial Integrity 22. Amotherbringsher28-month-olddaughterintotheclinicforawell-childvisit.Atthebeginningofthe visit,thenursefocusesattentionawayfromthetoddler,butastheinterviewprogresses,thetoddlerbeginsto warmupandissmilingshylyatthenurse.Thenursewillbemostsuccessfulininteractingwiththetoddlerif which is donenext? a. Tickle the toddler, and get her tolaugh. NURSINGTB.COM b. Stoopdowntoherlevel,andaskheraboutthetoysheisholding. c. Continue to ignore her until it is time for the physicalexamination. d. Askthemothertoleaveduringtheexaminationofthetoddler,becausetoddlersoftenfusslessif their parent is not inview. ANS: B Although most of the communication is with the parent, the nurse should not completely ignore the child. Makingcontactwillhelpeasethetoddlerlaterduringthephysicalexamination.Thenurseshouldbeginby askingaboutthetoysthechildisplayingwithoraboutaspecialdollorteddybearbroughtfromhome.Does yourdollhaveaname?orWhatcanyourtruckdo?Stoopdowntomeetthechildathisorhereyelevel. DIF:CognitiveLevel:Applying(Application) MSC:ClientNeeds:PsychosocialIntegrity 23. Duringanexaminationofa3-year-oldchild,thenursewillneedtotakeherbloodpressure.Whatmightthe nurse do to try to gain the childs fullcooperation? a. Tellthechildthatthebloodpressurecuffisgoingtogiveherarmabighug. b. Tellthechildthatthebloodpressurecuffisasleepandcannotwakeup. c. Givethebloodpressurecuffanameandrefertoitbythisnameduringtheassessment. d. Tellthechildthatbyusingthebloodpressurecuff,wecanseehowstronghermusclesare. ANS: D Takethetimetogiveashort,simpleexplanationwithaconcreteexplanationforanyunfamiliarequipmentthat willbeusedonthechild.Preschoolersareanimistic;theyimagineinanimateobjectscancomealiveandhave humancharacteristics.Thusabloodpressurecuffcanwakeupandbiteorpinch. DIF:CognitiveLevel:Applying(Application) MSC:ClientNeeds:PsychosocialIntegrity 24. A16-year-oldboyhasjustbeenadmittedtotheunitforovernightobservationafterbeinginanautomobile accident. What is the nurses best approach to communicating withhim? a. Use periods of silence to communicate respect forhim. b. Be totally honest with him, even if the information isunpleasant. c. Tellhimthateverythingthatisdiscussedwillbekepttotallyconfidential. d. Use slang language when possible to help him openup. ANS: B NURSINGTB.COM Successfulcommunicationwithanadolescentispossibleandcanberewarding.Theguidelinesaresimple.The first consideration is ones attitude, which must be one of respect. Second, communication must be totally honest.Anadolescentsintuitionishighlytunedandcandetectphoninessorthewithholdingofinformation. Always tell him or her the truth. DIF:CognitiveLevel:Applying(Application) MSC:ClientNeeds:PsychosocialIntegrity 25. A75-year-oldwomanisattheofficeforapreoperativeinterview.Thenurseisawarethattheinterview maytakelongerthaninterviewswithyoungerpersons.Whatisthereasonforthis? a. An aged person has a longer story totell. b. Anagedpersonisusuallylonelyandlikestohavesomeonewithwhomtotalk. c. Agedpersonslosemuchoftheirmentalabilitiesandrequirelongertimetocompleteaninterview. d. Asapersonages,heorsheisunabletohear;thustheinterviewerusuallyneedstorepeatmuchof what issaid. ANS: A Theinterviewusuallytakeslongerwitholderadultsbecausetheyhavealongerstorytotell.Itisnot necessarilytruethatallolderadultsarelonely,havelostmentalabilities,orarehardofhearing. DIF:CognitiveLevel:Understanding(Comprehension) MSC: Client Needs: PsychosocialIntegrity 26. Thenurseisinterviewingamalepatientwhohasahearingimpairment.Whattechniqueswouldbemost beneficial in communicating with thispatient? a. Determine the communication method heprefers. b. Avoidusingfacialandhandgesturesbecausemosthearing-impairedpeoplefindthisdegrading. c. Requestasignlanguageinterpreterbeforemeetingwithhimtohelpfacilitatethecommunication. d. Speakloudlyandwithexaggeratedfacialmovementwhentalkingwithhimbecausedoingsowill help him lipread. ANS: A The nurse should ask the deaf person the preferred way to communicateby signing, lip reading, or writing. If thepersonpreferslipreading,thenthenurseshouldbesuretofacehimsquarelyandhavegoodlightingonthe nurses face. The nurse should not exaggerate lip movements because this distorts words. Similarly, shouting distortsthereceptionofahearingaidthepersonmaywear.Thenurseshouldspeakslowlyandsupplementhis or her voice with appropriate hand gestures orpantomime. NURSINGTB.COM DIF:CognitiveLevel:Understanding(Comprehension) MSC: Client Needs: PsychosocialIntegrity 27. Duringaprenatalcheck,apatientbeginstocryasthenurseasksheraboutpreviouspregnancies.Shestates thatsheisrememberingherlastpregnancy,whichendedinmiscarriage.Thenursesbestresponsetohercrying wouldbe: a. Im so sorry for making youcry! b. Icanseethatyouaresadrememberingthis.Itisallrighttocry. c. WhydontIstepoutforafewminutesuntilyourefeelingbetter? d. Icanseethatyoufeelsadaboutthis;whydontwetalkaboutsomethingelse? ANS: B A beginning examiner usually feels horrified when the patient starts crying. When the nurse says something thatmakesthepersoncry,thenurseshouldnotthinkheorshehashurttheperson.Thenursehassimplyhiton animportanttopic;therefore,movingontoanewtopicisessential.Thenurseshouldallowthepersontocry andtoexpresshisorherfeelingsfully.Thenursecanofferatissueandwaituntilthecryingsubsidestotalk. DIF: Cognitive Level: Applying (Application) MSC: Client Needs: Psychosocial Integrity 28. Afemalenurseisinterviewingamanwhohasrecentlyimmigrated.Duringthecourseoftheinterview,he leans forward and then finally moves his chair close enough that his knees are nearly touching the nurses knees.Thenursebeginstofeeluncomfortablewithhisproximity.Whichstatementmostcloselyreflectswhat the nurse should donext? a. Thenurseshouldtrytorelax;thesebehaviorsareculturallyappropriateforthisperson. b. Thenurseshoulddiscreetlymovehisorherchairbackuntilthedistanceismorecomfortable,and then continue with theinterview. c. Thesebehaviorsareindicativeofsexualaggression,andthenurseshouldconfrontthisperson about hisbehaviors. d. Thenurseshouldlaughbuttellhimthatheorsheisuncomfortablewithhisproximityandaskhim to moveaway. ANS: A Boththepatientsandthenursessenseofspatialdistancearesignificantthroughouttheinterviewandphysical examination, with culturally appropriate distance zones varying widely. Some cultural groups value close physicalproximityandmayperceiveahealthcareproviderwhoisdistancinghimorherselfasbeingaloofand unfriendly. DIF:CognitiveLevel:Analyzing(Analysis) MSC:ClientNeeds:PsychosocialIntegrity NURSINGTB.COM 29. AfemaleAmericanIndianhascometotheclinicforfollow-updiabeticteaching.Duringtheinterview,the nurse notices that she never makes eye contact and speaks mostly to the floor. Which statement is true regarding thissituation? a. The woman is nervous andembarrassed. b. She has something to hide and isashamed. c. The woman is showing inconsistent verbal and nonverbalbehaviors. d. Sheisshowingthatsheiscarefullylisteningtowhatthenurseissaying. ANS: D Eye contact is perhaps among the most culturally variable nonverbal behaviors. Asian, American Indian, Indochinese, Arabian, and Appalachian people may consider direct eye contact impolite or aggressive, and theymayaverttheireyesduringtheinterview.AmericanIndiansoftenstareatthefloorduringtheinterview, whichisaculturallyappropriatebehavior,indicatingthatthelistenerispayingcloseattentiontothespeaker. DIF: Cognitive Level: Analyzing (Analysis) MSC: Client Needs: Psychosocial Integrity 30. Thenurseisperformingahealthinterviewonapatientwhohasalanguagebarrier,andnointerpreteris available.Whichisthebestexampleofanappropriatequestionforthenursetoaskinthissituation? a. Do you takemedicine? b. Do you sterilize thebottles? c. Do you have nausea andvomiting? d. You have been taking your medicine, haventyou? ANS: A In a situation during which a language barrier exists and no interpreter is available, simple words should be used,avoidingmedicaljargon.Theuseofcontractionsandpronounsshouldalsobeavoided.Nounsshouldbe repeatedly used, and one topic at a time should bediscussed. DIF: Cognitive Level: Analyzing (Analysis) MSC: Client Needs: Psychosocial Integrity 31. Amanarrivesattheclinicforhisannualwellnessphysical.Heisexperiencingnoacutehealthproblems. Whichquestionorstatementbythenurseismostappropriatewhenbeginningtheinterview? a. How is yourfamily? b. How is yourjob? c. Tellmeaboutyourhypertension. NURSINGTB.COM d. How has your health been since your lastvisit? ANS: D Open-endedquestionsareusedforgatheringnarrativeinformation.Thistypeofquestioningshouldbeusedto begintheinterview,tointroduceanewsectionofquestions,andwheneverthepersonintroducesanewtopic. DIF:CognitiveLevel:Applying(Application) MSC:ClientNeeds:PsychosocialIntegrity 32. Thenursemakesthiscommenttoapatient,Iknowitmaybehard,butyoushoulddowhatthedoctor orderedbecausesheistheexpertinthisfield.Whichstatementiscorrectaboutthenursescomment? a. This comment is inappropriate because it shows the nursesbias. b. Thiscommentisappropriatebecausemembersofthehealthcareteamareexpertsintheirareaof patientcare. c. Thistypeofcommentpromotesdependencyandinferiorityonthepartofthepatientandisbest avoided in an interview situation. d. Usingauthoritystatementswhendealingwithpatients,especiallywhentheyareundecidedabout an issue, is necessary attimes. ANS: C Using authority responses promotes dependency and inferiority. Avoiding the use of authority is best. Althoughthehealthcareproviderandpatientdonothaveequalprofessionalknowledge,bothhaveequally worthyrolesinthehealthprocess.Theotherstatementsarenotcorrect. DIF:CognitiveLevel:Applying(Application) MSC:ClientNeeds:PsychosocialIntegrity 33. AfemalepatientdoesnotspeakEnglishwell,andthenurseneedstochooseaninterpreter.Whichofthe following would be the most appropriatechoice? a. Trainedinterpreter b. Male familymember c. Female familymember d. Volunteer college student from the foreign language studiesdepartment NURSINGTB.COM ANS: A Wheneverpossible,thenurseshoulduseatrainedinterpreter,preferablyonewhoknowsmedicalterminology. In general, an older, more mature interpreter is preferred to a younger, less experienced one, and the same gender is preferred whenpossible. DIF:CognitiveLevel:Understanding(Comprehension) MSC: Client Needs: PsychosocialIntegrity 34. During a follow-up visit, the nurse discovers that a patient has not been taking his insulin on a regular basis.Thenurseasks,Whyhaventyoutakenyourinsulin?Whichstatementisanappropriateevaluationofthis question? a. This question may place the patient on thedefensive. b. This question is an innocent search forinformation. c. Discussing his behavior with his wife would have beenbetter. d. Adirectquestionisthebestwaytodiscoverthereasonsforhisbehavior. ANS: A Theadultsuseofwhyquestionsusuallyimpliesblameandcondemnationandplacesthepersononthe defensive. The other statements are notcorrect. DIF: Cognitive Level: Analyzing (Analysis) MSC: Client Needs: Psychosocial Integrity 35. Thenurseisnearingtheendofaninterview.Whichstatementisappropriateatthistime? a. Did we forgetsomething? b. Is there anything else you would like tomention? c. I need to go on to the next patient. Ill beback. d. While Im here, lets talk about your upcomingsurgery. ANS: B Thisquestionoffersthepersonafinalopportunityforself-expression.Nonewtopicshouldbeintroduced.The other questions are notappropriate. DIF:CognitiveLevel:Understanding(Comprehension) MSC: Client Needs: PsychosocialIntegrity 36. DuringtheinterviewportionofdatacollecNtiUoRn,StIhNeGnTuBrs.CeOcoMllects data. a. Physical b. Historical c. Objective d. Subjective ANS:D Theinterviewisthefirst,andreallythemostimportant,partofdatacollection.Duringtheinterview,thenurse collectssubjectivedata;thatis,whatthepersonsaysabouthimorherself. DIF:CognitiveLevel:Remembering(Knowledge) MSC: Client Needs: PsychosocialIntegrity 37. Duringaninterview,thenursewouldexpectthatmostoftheinterviewwilltakeplaceatwhatdistance? a. Intimatezone b. Personaldistance c. Socialdistance d. Publicdistance ANS:C Socialdistance,4to12feet,isusuallythedistancecategoryformostoftheinterview.Publicdistance,over12 feet, is too much distance; the intimate zone is inappropriate, and the personal distance will be used for the physicalassessment. DIF:CognitiveLevel:Understanding(Comprehension) MSC: Client Needs: PsychosocialIntegrity 38. Afemalenurseisinterviewingamalepatientwhoisnearthesameageasthenurse.Duringtheinterview, thepatientmakesanovertlysexualcomment.Thenursesbestreactionwouldbe: a. Stop thatimmediately! b. Oh, you are too funny. Lets keep going with theinterview. c. Do you really think I would beinterested? d. It makes me uncomfortable when you talk that way. Pleasestop. ANS: D NURSINGTB.COM The nurses response must make it clear that she is a health professional who can best care for the person by maintainingaprofessionalrelationship.Atthesametime,thenurseshouldcommunicatethatheorsheaccepts thepersonandunderstandsthepersonsneedtobeself-assertivebutthatsexualadvancescannotbetolerated. DIF: Cognitive Level: Analyzing (Analysis) MSC: Client Needs: Psychosocial Integrity MULTIPLE RESPONSE 1. Thenurseisconductinganinterview.Whichofthesestatementsistrueregardingopen-endedquestions? Select all that apply. a. Open-ended questions elicit coldfacts. b. They allow forself-expression. c. Open-ended questions build and enhancerapport. d. They leave interactionsneutral. e. Open-ended questions call for short one- to two-wordanswers. f. They are used when narrative information isneeded. ANS: B, C, F Open-endedquestionsallowforself-expression,buildandenhancerapport,andobtainnarrativeinformation. Thesefeaturesenhancecommunicationduringaninterview.Theotherstatementsareappropriateforclosedor directquestions. DIF:CognitiveLevel:Applying(Application) MSC:ClientNeeds:PsychosocialIntegrity 2. Thenurseisconductinganinterviewinanoutpatientclinicandisusingacomputertorecorddata.Which arethebestusesofthecomputerinthissituation?Selectallthatapply. a. Collect the patients data in a direct, face-to-facemanner. b. Enter all the data as the patient statesthem. c. Ask the patient to wait as the nurse enters thedata. d. Typethedataintothecomputerafterthenarrativeisfullyexplored. e. Allow the patient to see the monitor duringtyping. ANS: A, D, E NURSINGTB.COM Theuseofacomputercanbecomeabarrier.Thenurseshouldbegintheinterviewasusualbygreetingthe patient,establishingrapport,andcollectingthepatientsnarrativestoryinadirect,face-to-facemanner.Only after the narrative is fully explored should the nurse type data into the computer. When typing, the nurse should position the monitor so that the patient can seeit. DIF:CognitiveLevel:Applying(Application) MSC:ClientNeeds:PsychosocialIntegrity Chapter 04: The Complete Health History MULTIPLE CHOICE 1. Thenurseispreparingtoconductahealthhistory.Whichofthesestatementsbestdescribesthepurposeofa healthhistory? a. Toprovideanopportunityforinteractionbetweenthepatientandthenurse b. To provide a form for obtaining the patients biographicinformation c. Todocumentthenormalandabnormalfindingsofaphysicalassessment d. Toprovideadatabaseofsubjectiveinformationaboutthepatientspastandcurrenthealth ANS:D Thepurposeofthehealthhistoryistocollectsubjectivedatawhatthepersonsaysabouthimorherself.The other options are notcorrect. DIF: Cognitive Level: Understanding (Comprehension) MSC: Client Needs: Safe and Effective Care Environment: Management of Care 2. Whenthenurseisevaluatingthereliabilityofapatientsresponses,whichofthesestatementswouldbe correct? The patient: NURSINGTB.COM a. Has a history of drug abuse and therefore is notreliable. b. Provided consistent information and therefore isreliable. c. Smiled throughout interview and therefore is assumedreliable. d. Wouldnotanswerquestionsconcerningstressandthereforeisnotreliable. ANS: B Areliablepersonalwaysgivesthesameanswers,evenwhenquestionsarerephrasedorarerepeatedlaterin the interview. The other statements are notcorrect. DIF: Cognitive Level: Applying (Application) MSC: Client Needs: Safe and Effective Care Environment: Management of Care 3. A59-year-oldpatienttellsthenursethathehasulcerativecolitis.Hehasbeenhavingblackstoolsforthe last24hours.Howwouldthenursebestdocumenthisreasonforseekingcare? a. J.M. is a 59-year-old man seeking treatment for ulcerativecolitis. b. J.M.cameintothecliniccomplainingofhavingblackstoolsforthepast24hours. c. J.M.isa59-year-oldmanwhostatesthathehasulcerativecolitisandwantsitchecked. d. J.M.isa59-year-oldmanwhostatesthathehasbeenhavingblackstoolsforthepast24hours. ANS: D Thereasonforseekingcareisabriefspontaneousstatementinthepersonsownwordsthatdescribesthe reason for the visit. It states one (possibly two) signs or symptoms and their duration. It is enclosed in quotation marks to indicate the persons exactwords. DIF: Cognitive Level: Applying (Application) MSC: Client Needs: Safe and Effective Care Environment: Management of Care 4. Apatienttellsthenursethatshehashadabdominalpainforthepastweek.Whatwouldbethenursesbest response? a. Can you point to where ithurts? b. Well talk more about that later in theinterview. c. What have you had to eat in the last 24hours? d. Have you ever had any surgeries on yourabdomen? NURSINGTB.COM ANS: A Afinalsummaryofanysymptomthepersonhasshouldinclude,alongwithsevenothercriticalcharacteristics, Location: specific. The person is asked to point to thelocation. DIF: Cognitive Level: Applying (Application) MSC: Client Needs: Safe and Effective Care Environment: Management of Care 5. A29-year-oldwomantellsthenursethatshehasexcruciatingpaininherback.Whichwouldbethenurses appropriate response to the womansstatement? a. How does your family react to yourpain? b. The pain must be terrible. You probably pinched anerve. c. Ive had back pain myself, and it can beexcruciating. d. Howwouldyousaythepainaffectsyourabilitytodoyourdailyactivities? ANS: D Thesymptomofpainisdifficulttoquantifybecauseofindividualinterpretation.Withpain,adjectivesshould beavoidedandthepatientshouldbeaskedhowthepainaffectshisorherdailyactivities.Theotherresponses are notappropriate. DIF: Cognitive Level: Applying (Application) MSC: Client Needs: Safe and Effective Care Environment: Management of Care 6. Inrecordingthechildhoodillnessesofapatientwhodenieshavinghadany,whichnotebythenursewould be mostaccurate? a. Patient denies usual childhoodillnesses. b. Patient states he was a very healthychild. c. Patient states his sister had measles, but hedidnt. d. Patientdeniesmeasles,mumps,rubella,chickenpox,pertussis,andstrepthroat. ANS: D Childhoodillnessesincludemeasles,mumps,rubella,chickenpox,pertussis,andstrepthroat.Avoidrecording usual childhood illnesses because an illness common in the persons childhood may be unusual today (e.g., measles). DIF: Cognitive Level: Remembering (Knowledge) MSC: Client Needs: Safe and Effective Care Environment: Management of Care 7. Afemalepatienttellsthenursethatshehashadsixpregnancies,withfourlivebirthsattermandtwo spontaneousabortions.HerfourchildrenaresNtiUllRliSvIiNngG.THBo.CwOwMouldthenurserecordthisinformation? a. P-6, B-4,(S)Ab-2 b. Grav 6, Term 4, (S)Ab-2, Living4 c. Patient has had four livingbabies. d. Patient has been pregnant sixtimes. ANS: B Obstetric history includes the number of pregnancies (gravidity), number of deliveries in which the fetus reachedterm(term),numberofpretermpregnancies(preterm),numberofincompletepregnancies(abortions), andnumberofchildrenliving(living).Thisisrecorded:Grav Term Preterm Ab Living .Foranyincompletepregnancies,thedurationisrecordedandwhetherthepregnancyresultedin a spontaneous (S) or an induced (I)abortion. DIF: Cognitive Level: Applying (Application) MSC: Client Needs: Safe and Effective Care Environment: Management of Care 8. Apatienttellsthenursethatheisallergictopenicillin.Whatwouldbethenursesbestresponsetothis information? a. Are you allergic to any otherdrugs? b. How often have you receivedpenicillin? c. Illwriteyourallergyonyourchartsoyouwontreceiveanypenicillin. d. Describe what happens to you when you takepenicillin. ANS: D Note both the allergen (medication, food, or contact agent, such as fabric or environmental agent) and the reaction(rash,itching,runnynose,wateryeyes,ordifficultybreathing).Withadrug,thissymptomshouldnot be a side effect but a true allergicreaction. DIF: Cognitive Level: Understanding (Comprehension) MSC: Client Needs: Safe and Effective Care Environment: Management of Care 9. Thenurseistakingafamilyhistory.Importantdiseasesorproblemsaboutwhichthepatientshouldbe specifically askedinclude: a. Emphysema. b. Headtrauma. c. Mentalillness. d. Fracturedbones. NURSINGTB.COM ANS: C Questionsconcerninganyfamilyhistoryofheartdisease,highbloodpressure,stroke,diabetes,obesity,blood disorders, breast and ovarian cancers, colon cancer, sickle cell anemia, arthritis, allergies, alcohol or drug addiction,mentalillness,suicide,seizuredisorder,kidneydisease,andtuberculosisshouldbeasked. DIF: Cognitive Level: Remembering (Knowledge) MSC: Client Needs: Safe and Effective Care Environment: Management of Care 10. The review of systems provides the nursewith: a. Physical findings related to eachsystem. b. Informationregardinghealthpromotionpractices. c. Anopportunitytoteachthepatientmedicalterms. d. Informationnecessaryforthenursetodiagnosethepatientsmedicalproblem. ANS: B The purposes of the review of systems are to: (1) evaluate the past and current health state of each body system,(2)doublecheckfactsincaseanysignificantdatawereomittedinthepresentillnesssection,and(3) evaluate health promotionpractices. DIF: Cognitive Level: Remembering (Knowledge) MSC: Client Needs: Safe and Effective Care Environment: Management of Care 11. Whichofthesestatementsrepresentssubjectivedatathenurseobtainedfromthepatientregardingthe patientsskin? a. Skin appearsdry. b. No lesions areobvious. c. Patient denies any colorchange. d. Lesion is noted on the lateral aspect of the rightarm. ANS: C Thehistoryshouldbelimitedtopatientstatementsorsubjectivedatafactorsthatthepersonsayswereorwere notpresent. DIF: Cognitive Level: Understanding (Comprehension) MSC:ClientNeeds:SafeandEffectiveCareENnUvRirSoInNmGeTnBt:.CMOaMnagementofCare 12. Thenurseisobtainingahistoryfroma30-year-oldmalepatientandisconcernedabouthealthpromotion activities.Whichofthesequestionswouldbeappropriatetousetoassesshealthpromotionactivitiesforthis patient? a. Do you perform testicularself-examinations? b. Have you ever noticed any pain in yourtesticles? c. Have you had any problems with passingurine? d. Do you have any history of sexually transmitteddiseases? ANS: A Healthpromotionforamanwouldincludetheperformanceoftesticularself-examinations.Theother questions are asking about possible disease or illnessissues. DIF: Cognitive Level: Understanding (Comprehension) MSC: Client Needs: Safe and Effective Care Environment: Management of Care 13. Whichoftheseresponsesmightthenurseexpectduringafunctionalassessmentofapatientwhoselegisin acast? a. I broke my right leg in a car accident 2 weeksago. b. The pain is decreasing, but I still need to takeacetaminophen. c. IcheckthecolorofmytoeseveryeveningjustlikeIwastaught. d. Imabletotransfermyselffromthewheelchairtothebedwithouthelp. ANS: D Functionalassessmentmeasuresapersonsself-careabilityintheareasofgeneralphysicalhealthorabsenceof illness. The other statements concern health or illnessissues. DIF: Cognitive Level: Applying (Application) MSC: Client Needs: Safe and Effective Care Environment: Management of Care 14. Inresponsetoaquestionaboutstress,a39-year-oldwomantellsthenursethatherhusbandandmother bothdiedinthepastyear.Whichresponsebythenurseismostappropriate? a. This has been a difficult year foryou. b. I dont know how anyone could handle that much stress in 1year! c. WhatdidyoudotocopewiththelNoUssRoSfINboGthTBy.oCuOrMhusbandandmother? d. Thatisalotofstress;nowletsgoontothenextsectionofyourhistory. ANS: C Questionsaboutcopingandstressmanagementincludequestionsregardingthekindsofstressesinoneslife, especiallyinthelastyear,anychangesinlifestyleoranycurrentstress,methodstriedtorelievestress,and whether these methods have beenhelpful. DIF: Cognitive Level: Applying (Application) MSC: Client Needs: Safe and Effective Care Environment: Management of Care 15. Inresponsetoaquestionregardingtheuseofalcohol,apatientasksthenursewhythenurseneedsto know. What is the reason for needing thisinformation? a. This information is necessary to determine the patientsreliability. b. Alcoholcaninteractwithallmedicationsandcanmakesomediseasesworse. c. Thenurseneedstobeabletoteachthepatientaboutthedangersofalcoholuse. d. Thisinformationisnotnecessaryunlessadrinkingproblemisobvious. ANS: B Alcoholadverselyinteractswithallmedicationsandisafactorinmanysocialproblemssuchaschildorsexual abuse,automobileaccidents,andassaults;alcoholalsocontributestomanyillnessesanddiseaseprocesses. Therefore,assessingforsignsofhazardousalcoholuseisimportant.Theotheroptionsarenotcorrect. DIF: Cognitive Level: Understanding(Comprehension) MSC: Client Needs: Safe and Effective Care Environment: Management of Care 16. Themotherofa16-month-oldtoddlertellsthenursethatherdaughterhasanearache.Whatwouldbean appropriateresponse? a. Maybe she is justteething. b. I will check her ear for an earinfection. c. Are you sure she is really havingpain? d. Describe what she is doing to indicate she is havingpain. ANS: D Withaveryyoungchild,theparentisasked,Howdoyouknowthechildisinpain?Ayoungchildpullingat hisorherearsshouldalertparentstothechildsearpain.Statementsaboutteethingandquestioningwhether thechildisreallyhavingpaindonotexplorethesymptoms,whichshouldbedonebeforeaphysical examination. NURSINGTB.COM DIF: Cognitive Level: Applying (Application) MSC: Client Needs: Safe and Effective Care Environment: Management of Care 17. Duringanassessmentofapatientsfamilyhistory,thenurseconstructsagenogram.Whichstatementbest describes agenogram? a. List of diseases present in a persons nearrelatives b. Graphicfamilytreethatusessymbolstodepictthegender,relationship,andageofimmediate familymembers c. Drawingthatdepictsthepatientsfamilymembersuptofivegenerationsback d. Descriptionofthehealthofapersonschildrenandgrandchildren ANS:B Agenogram(orpedigree)isagraphicfamilytreethatusessymbolstodepictthegender,relationship,andage ofimmediatebloodrelativesinatleastthreegenerations(parents,grandparents,siblings).Theotheroptionsdo not describe agenogram. DIF: Cognitive Level: Applying (Application) MSC: Client Needs: Safe and Effective Care Environment: Management of Care 18. A5-year-oldboyisbeingadmittedtothehospitaltohavehistonsilsremoved.Whichinformationshould the nurse collect before thisprocedure? a. Childs birthweight b. Age at which hecrawled c. Whether the child has had themeasles d. Childsreactionstoprevioushospitalizations ANS:D How the child reacted to previous hospitalizations and any complications should be assessed. If the child reactedpoorly,thenheorshemaybeafraidnowandwillneedspecialpreparationfortheexaminationthatis to follow. The other items are not significant for theprocedure. DIF: Cognitive Level: Analyzing (Analysis) MSC: Client Needs: Safe and Effective Care Environment: Management of Care 19. As part of the health history of a 6-year-old boy at a clinic for a sports physical examination, the nurse reviewshisimmunizationrecordandnotesthathislastmeasles-mumps-rubella(MMR)vaccinationwasat15 months of age. What recommendation should the nursemake? NURSINGTB.COM a. No further MMR immunizations areneeded. b. MMR vaccination needs to be repeated at 4 to 6 years ofage. c. MMRimmunizationneedstoberepeatedevery4yearsuntilage21years. d. A recommendation cannot be made until the physician isconsulted. ANS: B BecauseofrecentoutbreaksofmeaslesacrosstheUnitedStates,theAmericanAcademyofPediatrics(2006) recommendstwodosesoftheMMRvaccine,oneat12to15monthsofageandoneatage4to6years. DIF: Cognitive Level: Analyzing (Analysis) MSC: Client Needs: Safe and Effective Care Environment: Management of Care 20. Inobtainingareviewofsystemsonahealthy7-year-oldgirl,thehealthcareproviderknowsthatitwould be important to includethe: a. Last glaucomaexamination. b. Frequency of breastself-examinations. c. Date of her lastelectrocardiogram. d. Limitations related to her involvement in sportsactivities. ANS: D Whenreviewingthecardiovascularsystem,thehealthcareprovidershouldaskwhetheranyactivityislimited orwhetherthechildcankeepupwithherpeers.Theotheritemsarenotappropriateforachildthisage. DIF: Cognitive Level: Applying (Application) MSC: Client Needs: Safe and Effective Care Environment: Management of Care 21. Whenthenurseasksforadescriptionofwholiveswithachild,themethodofdiscipline,andthesupport system of the child, what part of the assessment is beingperformed? a. Familyhistory b. Review ofsystems c. Functionalassessment d. Reason for seekingcare ANS: C NURSINGTB.COM Functionalassessmentincludesinterpersonalrelationshipsandhomeenvironment.Familyhistoryincludes illnessesinfamilymembers;areviewofsystemsincludesquestionsaboutthevariousbodysystems;andthe reason for seeking care is the rationale for requesting healthcare. DIF: Cognitive Level: Understanding (Comprehension) MSC: Client Needs: Safe and Effective Care Environment: Management of Care 22. Thenurseisobtainingahealthhistoryonan87-year-oldwoman.Whichofthefollowingareasof questioning would be most useful at thistime? a. Obstetrichistory b. Childhoodillnesses c. General health for the past 20years d. Currenthealthpromotionactivities ANS:D Itisimportantforthenursetorecognizepositivehealthmeasures,suchaswhatthepersonhasbeendoingto helphimorherselfstaywellandtolivetoanolderage.Theotherresponsesarenotpertinenttoapatientof thisage. DIF: Cognitive Level: Applying (Application) MSC: Client Needs: Safe and Effective Care Environment: Management of Care 23. Thenurseisperformingareviewofsystemsona76-year-oldpatient.Whichofthesestatementsiscorrect for thissituation? a. The questions asked are identical for allages. b. Theinterviewerwillstartincorporatingdifferentquestionsforpatients70yearsofageandolder. c. Questionsthatarereflectiveofthenormaleffectsofagingareadded. d. Atthisage,areviewofsystemsisnotnecessarythefocusshouldbeoncurrentproblems. ANS: C Thehealthhistoryincludesthesameformatasthatdescribedfortheyoungeradult,aswellassomeadditional questions. These additional questions address ways in which the activities of daily living may have been affectedbythenormalagingprocessesorbytheeffectsofchronicillnessordisability. DIF: Cognitive Level: Understanding (Comprehension) MSC:ClientNeeds:SafeandEffectiveCareEnvironment:ManagementofCare 24. A90-year-oldpatienttellsthenursethathecannotrememberthenamesofthemedicationsheistakingor forwhatreasonheistakingthem.AnapproprNiaUteRrSeIsNpGonTsBe.CfrOomMthenursewouldbe: a. Can you tell me what they looklike? b. Dont worry about it. You are only taking twomedications. c. How long have you been taking each of thepills? d. Would you have a family member bring in yourmedications? ANS: D Thepersonmaynotknowthedrugnameorpurpose.Whenthisoccurs,askthepersonorafamilymemberto bringinthedrugtobeidentified.Theotherresponseswouldnothelptoidentifythemedications. DIF: Cognitive Level: Applying (Application) MSC: Client Needs: Safe and Effective Care Environment: Management of Care 25. Thenurseisperformingafunctionalassessmentonan82-year-oldpatientwhorecentlyhadastroke. Which of these questions would be most important toask? a. Do you wearglasses? b. Are you able to dressyourself? c. Do you have any thyroidproblems? d. How many times a day do you have a bowelmovement? ANS: B Functional assessment measures how a person manages day-to-day activities. For the older person, the meaningofhealthbecomesthoseactivitiesthattheycanorcannotdo.Theotherresponsesdonotrelateto functionalassessment. DIF: Cognitive Level: Applying (Application) MSC: Client Needs: Safe and Effective Care Environment: Management of Care 26. Thenurseispreparingtodoafunctionalassessment.Whichstatementbestdescribesthepurposeofa functionalassessment? a. Thefunctionalassessmentassesseshowtheindividualiscopingwithlifeathome. b. It determines how children are meeting developmentalmilestones. c. Thefunctionalassessmentcanidentifyanyproblemswithmemorytheindividualmaybe experiencing. NURSINGTB.COM d. It helps determine how a person is managing day-to-dayactivities. ANS: D Thefunctionalassessmentmeasureshowapersonmanagesday-to-dayactivities.Theotheranswersdonot reflect the purpose of a functionalassessment. DIF: Cognitive Level: Remembering (Knowledge) MSC: Client Needs: Safe and Effective Care Environment: Management of Care 27. Thenurseisaskingapatientforhisreasonforseekingcareandasksaboutthesignsandsymptomsheis experiencing. Which of these is an example of asymptom? a. Chestpain b. Clammyskin c. Serum potassium level at 4.2mEq/L d. Bodytemperatureof100F ANS:A Asymptomisasubjectivesensation(e.g.,chestpain)thatapersonfeelsfromadisorder.Asignisanobjective abnormalitythattheexaminercandetectonphysicalexaminationorinlaboratoryreports,asillustratedbythe otherresponses. DIF: Cognitive Level: Understanding (Comprehension) MSC: Client Needs: Safe and Effective Care Environment: Management of Care 28. Apatientisdescribinghissymptomstothenurse.Whichofthesestatementsreflectsadescriptionofthe setting of hissymptoms? a. It is a sharp, burning pain in mystomach. b. I also have the sweats and nausea when I feel thispain. c. Ithinkthispainistellingmethatsomethingbadiswrongwithme. d. This pain happens every time I sit down to use thecomputer. ANS: D Thesettingdescribeswherethepersonisorwhatthepersonisdoingwhenthesymptomstarts.Describingthe painassharpandburningreflectsthecharacterorqualityofthepain;statingthatthepainistellingthepatient thatsomethingbadiswrongwithhimreflectsthepatientsperceptionofthepain;anddescribingthesweatsand nausea reflects associated factors that occur with thepain. DIF: Cognitive Level: Analyzing (Analysis) NURSINGTB.COM MSC: Client Needs: Safe and Effective Care Environment: Management of Care 29. Duringanassessment,thenurseusestheCAGEtest.Thepatientanswersyestotwoofthequestions.What could this beindicating? a. The patient is analcoholic. b. The patient is annoyed at thequestions. c. Thepatientshouldbethoroughlyexaminedforpossiblealcoholwithdrawalsymptoms. d. Thenurseshouldsuspectalcoholabuseandcontinuewithamorethoroughsubstanceabuse assessment. ANS: D TheCAGEtestisknownasthecutdown,annoyed,guilty,andeye-openertest.Ifapersonanswersyestotwo or more of the four CAGE questions, then the nurse should suspect alcohol abuse and continue with a more complete substance abuseassessment. DIF: Cognitive Level: Analyzing (Analysis) MSC: Client Needs: Safe and Effective Care Environment: Management of Care 30. Thenurseisincorporatingapersonsspiritualvaluesintothehealthhistory.Whichofthesequestions illustratesthecommunityportionoftheFICA(faithandbelief,importanceandinfluence,community,and addressing or applying in care)questions? a. Do you believe inGod? b. Are you a part of any religious or spiritualcongregation? c. Do you consider yourself to be a religious or spiritualperson? d. Howdoesyourreligiousfaithinfluencethewayyouthinkaboutyourhealth? ANS: B Thecommunityisassessedwhenthenurseaskswhetherapersonispartofareligiousorspiritualcommunity orcongregation.Theotherareasassessedarefaith,influence,andaddressinganyreligiousorspiritualissues orconcerns. DIF: Cognitive Level: Understanding (Comprehension) MSC: Client Needs: Safe and Effective Care Environment: Management of Care 31. Thenurseispreparingtocompleteahealthassessmentona16-year-oldgirlwhoseparentshavebrought hertotheclinic.Whichinstructionwouldbeappropriatefortheparentsbeforetheinterviewbegins? a. Pleasestayduringtheinterview;youcananswerforherifshedoesnotknowtheanswer. NURSINGTB.COM b. It would help to interview the three of youtogether. c. WhileIinterviewyourdaughter,willyoupleasestayintheroomandcompletethesefamilyhealth historyquestionnaires? d. WhileIinterviewyourdaughter,willyoustepouttothewaitingroomandcompletethesefamily health historyquestionnaires? ANS: D Thegirlshouldbeinterviewedalone.Theparentscanwaitoutsideandfilloutthefamilyhealthhistory questionnaires. DIF: Cognitive Level: Analyzing (Analysis) MSC: Client Needs: Safe and Effective Care Environment: Management of Care 32. ThenurseisassessinganewpatientwhohasrecentlyimmigratedtotheUnitedStates.Whichquestionis appropriate to add to the healthhistory? a. Why did you come to the UnitedStates? b. When did you come to the United States and from whatcountry? c. What made you leave your nativecountry? d. Are you planning to return to yourhome? ANS: B Biographicdata,suchaswhenthepersonenteredtheUnitedStatesandfromwhatcountry,areappropriate additionstothehealthhistory.Theotheranswersdonotreflectappropriatequestions. DIF: Cognitive Level: Analyzing (Analysis) MSC: Client Needs: Psychosocial Integrity MULTIPLE RESPONSE 1. Thenurseisassessingapatientsheadachepain.Whichquestionsreflectoneormoreofthecritical characteristicsofsymptomsthatshouldbeassessed?Selectallthatapply. a. Where is the headachepain? b. Did you have these headaches as achild? c. On a scale of 1 to 10, how bad is thepain? d. How often do the headachesoccur? NURSINGTB.COM e. What makes the headaches feelbetter? f. Do you have any family history ofheadaches? ANS: A, C, D, E ThemnemonicPQRSTUmayhelpthenurseremembertoaddressthecriticalcharacteristicsthatneedtobe assessed:(1)P:provocativeorpalliative;(2)Q:qualityorquantity;(3)R:regionorradiation;(4)S:severity scale;(5)T:timing;and(6)U:understandthepatientsperception.Asking,Whereisthepain?reflectsregion. Askingthepatienttoratethepainona1to10scalereflectsseverity.AskingHowoftenreflectstiming. Askingwhatmakesthepainbetterreflectsprovocative.Theotheroptionsreflecthealthhistoryandfamily history. DIF: Cognitive Level: Analyzing (Analysis) MSC: Client Needs: Safe and Effective Care Environment: Management of Care 2. Thenurseisconductingadevelopmentalhistoryona5-year-oldchild.Whichquestionsareappropriateto ask the parents for this part of the assessment? Select all thatapply. a. How much junk food does your childeat? b. How many teeth has he lost, and when did he losethem? c. Is he able to tie hisshoelaces? d. Does he take a childrensvitamin? e. Can he telltime? f. Does he have any foodallergies? ANS: B, C, E Questionsabouttoothloss,abilitytotelltime,andabilitytotieshoelacesareappropriatequestionsfora developmentalassessment.Questionsaboutjunkfoodintakeandvitaminsarepartofanutritionalhistory. Questions about food allergies are not part of a developmentalhistory. DIF: Cognitive Level: Analyzing (Analysis) MSC: Client Needs: Safe and Effective Care Environment: Management of Care NURSINGTB.COM Chapter 05: Mental Status Assessment MULTIPLE CHOICE 1. Duringanexamination,thenursecanassessmentalstatusbywhichactivity? a. Examining the patientselectroencephalogram b. Observingthepatientasheorsheperformsanintelligencequotient(IQ)test c. Observing the patient and inferring health ordysfunction d. Examiningthepatientsresponsetoaspecificsetofquestions ANS:C Mental status cannot be directly scrutinized like the characteristics of skin or heart sounds. Its functioning is inferredthroughanassessmentofanindividualsbehaviors,suchasconsciousness,language,moodandaffect, and otheraspects. PTS: 1 DIF: Cognitive Level: Understanding (Comprehension) 2. Thenurseisassessingthementalstatusofachild.Whichstatementaboutchildrenandmentalstatusistrue? a. All aspects of mental status in children areinterdependent. NURSINGTB.COM b. Children are highly labile and unstable until the age of 2years. c. Childrensmentalstatusislargelyafunctionoftheirparentsleveloffunctioninguntiltheageof7 years. d. Achildsmentalstatusisimpossibletoassessuntilthechilddevelopstheabilitytoconcentrate. ANS: A Separating and tracing the development of only one aspect of mental status is difficult. All aspects are interdependent. For example, consciousness is rudimentary at birth because the cerebral cortex is not yet developed.Theinfantcannotdistinguishtheselffromthemothersbody.Theotherstatementsarenottrue. PTS: 1 DIF: Cognitive Level: Understanding (Comprehension) 3. Thenurseisassessinga75-year-oldman.Asthenursebeginsthementalstatusportionoftheassessment, the nurse expects that thispatient: a. Willhavenodecreaseinanyofhisabilities,includingresponsetime. b. Willhavedifficultyontestsofremotememorybecausethisabilitytypicallydecreaseswithage. c. Maytakealittlelongertorespond,buthisgeneralknowledgeandabilitiesshouldnothave declined. d. Willexhibithadadecreaseinhisresponsetimebecauseofthelossoflanguageandadecreasein generalknowledge. ANS: C Theagingprocessleavestheparametersofmentalstatusmostlyintact.Generalknowledgedoesnotdecrease, andlittleornolossinvocabularyoccurs.Responsetimeisslowerthaninayouth.Ittakesalittlelongerforthe braintoprocessinformationandtoreacttoit.Recentmemory,whichrequiressomeprocessing,issomewhat decreased with aging, but remote memory is notaffected. PTS: 1 DIF: Cognitive Level: Analyzing (Analysis) 4. Whenassessingagingadults,thenurseknowsthatoneofthefirstthingsthatshouldbeassessedbefore making judgments about their mental statusis: a. Presence ofphobias b. Generalintelligence c. Presence of irrational thinkingpatterns d. Sensory-perceptiveabilities ANS: D NURSINGTB.COM Age-relatedchangesinsensoryperceptioncanaffectmentalstatus.Forexample,visionloss(asdetailedin Chapter 15) may result in apathy, social isolation, and depression. Hearing changes are common in older adults,whichproducesfrustration,suspicion,andsocialisolationandmakesthepersonappearconfused. PTS: 1 DIF: Cognitive Level: Analyzing (Analysis) 5. Thenurseispreparingtoconductamentalstatusexamination.Whichstatementistrueregardingthemental statusexamination? a. Apatientsfamilyisthebestresourceforinformationaboutthepatientscopingskills. b. Gatheringmentalstatusinformationduringthehealthhistoryinterviewisusuallysufficient. c. Integratingthementalstatusexaminationintothehealthhistoryinterviewtakesanenormous amount of extratime. d. Togetagoodideaofthepatientsleveloffunctioning,performingacompletementalstatus examination is usuallynecessary. ANS: B The full mental status examination is a systematic check of emotional and cognitive functioning. The steps described,however,rarelyneedtobetakenintheirentirety.Usually,onecanassessmentalstatusthroughthe context of the health history interview. PTS: 1 DIF: Cognitive Level: Applying (Application) 6. Awomanbringsherhusbandtotheclinicforanexamination.Sheisparticularlyworriedbecauseaftera recentfall,heseemstohavelostagreatdealofhismemoryofrecentevents.Whichstatementreflectsthe nurses best course ofaction? a. Perform a complete mental statusexamination. b. Refer him to apsychometrician. c. Plantointegratethementalstatusexaminationintothehistoryandphysicalexamination. d. Reassurehiswifethatmemorylossafteraphysicalshockisnormalandwillsoonsubside. ANS: A Performingacompletementalstatusexaminationisnecessarywhenanyabnormalityinaffectorbehavioris discoveredorwhenfamilymembersareconcernedaboutapersonsbehavioralchanges(e.g.,memoryloss, inappropriate social interaction) or after trauma, such as a headinjury. PTS: 1 DIF: Cognitive Level: Applying (Application) 7. Thenurseisconductingapatientinterview.Whichstatementmadebythepatientshouldthenursemore fully explore during theinterview? NURSINGTB.COM a. I sleep like ababy. b. I have no healthproblems. c. I never did too good inschool. d. I am not currently taking anymedications. ANS: C Ineverymentalstatusexamination,thefollowingfactorsfromthehealthhistorythatcouldaffectthefindings shouldbenoted:anyknownillnessesorhealthproblems,suchasalcoholismorchronicrenaldisease;current medications,thesideeffectsofwhichmaycauseconfusionordepression;theusualeducationalandbehavioral level,notingthislevelasthepatientsnormalbaselineandnotexpectingalevelofperformanceonthemental status examination to exceed it; and responses to personal history questions, indicating current stress, social interaction patterns, and sleephabits. PTS: 1 DIF: Cognitive Level: Analyzing (Analysis) 8. A patient is admitted to the unit after an automobile accident. The nurse begins the mental status examinationandfindsthatthepatienthasdysarthricspeechandislethargic.Thenursesbestapproach regarding this examination isto: a. Plan to defer the rest of the mental statusexamination. b. Skipthelanguageportionoftheexamination,andproceedontoassessingmoodandaffect. c. Conductanin-depthspeechevaluation,anddeferthementalstatusexaminationtoanothertime. d. Proceedwiththeexamination,andassessthepatientforsuicidalthoughtsbecausedysarthriais often accompanied by severedepression. ANS: A In the mental status examination, the sequence of steps forms a hierarchy in which the most basic functions (consciousness, language) are assessed first. The first steps must be accurately assessed to ensure validity of thestepsthatfollow.Forexample,ifconsciousnessisclouded,thenthepersoncannotbeexpectedtohavefull attention and to cooperate with new learning. If language is impaired, then a subsequent assessment of new learningorabstractreasoning(anythingthatrequireslanguagefunctioning)cangiveerroneousconclusions. PTS:1DIF:CognitiveLevel:Analyzing(Analysis) 9. A19-year-oldwomancomestotheclinicattheinsistenceofherbrother.Sheiswearingblackcombatboots and a black lace nightgown over the top of her other clothes. Her hair is dyed pink with black streaks throughout.Shehasseveralpiercedholesinhernaresandearsandiswearinganearringthroughhereyebrow and heavy black makeup. The nurse concludesthat: a. She probably does not have anyproblems. b. Sheisonlytryingtoshockpeopleandthatherdressshouldbeignored. NURSINGTB.COM c. Shehasamanicsyndromebecauseofherabnormaldressandgrooming. d. Moreinformationshouldbegatheredtodecidewhetherherdressisappropriate. ANS: D Grooming and hygiene should be notedthe person is clean and well groomed, hair is neat and clean, women havemoderateornomakeup,andmenareshavedortheirbeardsormoustachesarewellgroomed.Careshould be taken when interpreting clothing that is disheveled, bizarre, or in poor repair because these sometimes reflect the persons economic status or a deliberate fashiontrend. PTS: 1 DIF: Cognitive Level: Applying (Application) 10. Apatienthasbeenintheintensivecareunitfor10days.Hehasjustbeenmovedtothemedical-surgical unit,andtheadmittingnurseisplanningtoperformamentalstatusexamination.Duringthetestsofcognitive function, the nurse would expect thathe: a. May display some disruption in thoughtcontent. b. Will state, I am so relieved to be out of intensivecare. c. Willbeorientedtoplaceandperson,butthepatientmaynotbecertainofthedate. d. May show evidence of some clouding of his level ofconsciousness. ANS: C Thenursecandiscerntheorientationofcognitivefunctionthroughthecourseoftheintervieworcandirectly and tactfully ask, Some people have trouble keeping up with the dates while in the hospital. Do you know todaysdate?Manyhospitalizedpeoplehavetroublewiththeexactdatebutarefullyorientedontheremaining items. PTS: 1 DIF: Cognitive Level: Analyzing (Analysis) 11. Duringamentalstatusexamination,thenursewantstoassessapatientsaffect.Thenurseshouldaskthe patient whichquestion? a. How do you feeltoday? b. Would you please repeat the followingwords? c. Have these medications had any effect on yourpain? d. Has this pain affected your ability to get dressed byyourself? ANS: A Judgemoodandaffectbybodylanguageandfacialexpressionandbydirectlyasking,Howdoyoufeeltoday? orHowdoyouusuallyfeel?ThemoodshouldNbUeRaSpIpNrGopTrBia.CteOtMothepersonsplaceandconditionandshould appropriately change with the topics. PTS: 1 DIF: Cognitive Level: Applying (Application) 12. Thenurseisplanningtoassessnewmemorywithapatient.Thebestwayforthenursetodothiswouldbe to: a. Administer the FACTtest. b. Ask him to describe his firstjob. c. Give him the Four Unrelated WordsTest. d. Askhimtodescribewhattelevisionshowhewaswatchingbeforecomingtotheclinic. ANS: C Askquestionsthatcanbecorroborated,whichscreensfortheoccasionalpersonwhoconfabulatesormakesup answerstofillinthegapsofmemoryloss.TheFourUnrelatedWordsTestteststhepersonsabilitytolaydown new memories and is a highly sensitive and valid memorytest. PTS: 1 DIF: Cognitive Level: Applying (Application) 13. A45-year-oldwomanisattheclinicforamentalstatusassessment.IngivinghertheFourUnrelated WordsTest,thenursewouldbeconcernedifshecouldnot fourunrelatedwords . a. Invent; within 5minutes b. Invent; within 30seconds c. Recall; after a 30-minutedelay d. Recall;aftera60-minutedelay ANS:C TheFourUnrelatedWordsTestteststhepersonsabilitytolaydownnewmemories.Itisahighlysensitiveand validmemorytest.Itrequiresmoreeffortthantherecallofpersonalorhistoricevents.Tothepersonsay,Iam goingtosayfourwords.Iwantyoutorememberthem.InafewminutesIwillaskyoutorecallthem.After5 minutes,askforthefourwords.Thenormalresponseforpersonsunder60yearsisanaccuratethree-orfour- word recall after a 5-, 10-, and 30-minutedelay. PTS: 1 DIF: Cognitive Level: Analyzing (Analysis) 14. Duringamentalstatusassessment,whichquestionbythenursewouldbestassessapersonsjudgment? a. Do you feel that you are being watched, followed, orcontrolled? b. Tellmewhatyouplantodoonceyouaredischargedfromthehospital. NURSINGTB.COM c. Whatdoesthestatement,Peopleinglasshousesshouldntthrowstones,meantoyou? d. Whatwouldyoudoifyoufoundastamped,addressedenvelopelyingonthesidewalk? ANS: B Apersonexercisesjudgmentwhenheorshecancompareandevaluatethealternativesinasituationandreach an appropriate course of action. Rather than testing the persons response to a hypothetical situation (as illustratedintheoptionwiththeenvelope),thenurseshouldbemoreinterestedinthepersonsjudgmentabout dailyorlong-termgoals,thelikelihoodofactinginresponsetodelusionsorhallucinations,andthecapacityfo violent or suicidalbehavior. PTS: 1 DIF: Cognitive Level: Applying (Application) 15. Whichoftheseindividualswouldthenurseconsiderathighestriskforasuicideattempt? a. Man who jokes aboutdeath b. Womanwho,duringapastepisodeofmajordepression,attemptedsuicide c. Adolescentwhojustbrokeupwithherboyfriendandstatesthatshewouldliketokillherself d. Olderadultmanwhotellsthenursethatheisgoingtojoinhiswifeinheaventomorrowandplans to use agun ANS: D When the person expresses feelings of sadness, hopelessness, despair, or grief, assessing any possible risk of physical harm to him or herself is important. The interview should begin with more general questions. If the nursehearsaffirmativeanswers,thenheorsheshouldcontinuewithmorespecificquestions.Aprecisesuicide plantotakeplaceinthenext24to48hourswithuseofalethalmethodconstituteshighrisk. PTS: 1 DIF: Cognitive Level: Applying (Application) 16. Thenurseisperformingamentalstatusassessmentona5-year-oldgirl.Herparentsareundergoingabitter divorce and are worried about the effect it is having on their daughter. Which action or statement might lead the nurse to be concerned about the girls mentalstatus? a. She clings to her mother whenever the nurse is in theroom. b. She appears angry and will not make eye contact with thenurse. c. Hermotherstatesthatshehasbeguntorideatricyclearoundtheiryard. d. Hermotherstatesthatherdaughterpreferstoplaywithtoddlersinsteadofkidsherownagewhile indaycare. ANS: D The mental status assessment of infants and children covers behavioral, cognitive, and psychosocial developmentandexamineshowthechildiscopingwithhisorherenvironment.Essentially,thenurseshould followthesameAssociationforBehavioralanNdUCRoSgInNiGtivTeBT.ChOeMrapies(ABCT)guidelinesasthosefortheadult, withspecialconsiderationfordevelopmentalmilestones.Thebestexaminationtechniquearisesfroma thoroughknowledgeofthedevelopmentalmilestones(describedinChapter2).Abnormalitiesareoften problemsofomission(e.g.,thechilddoesnotachieveamilestoneasexpected). PTS: 1 DIF: Cognitive Level: Applying (Application) 17. Thenurseisassessingorientationina79-year-oldpatient.Whichoftheseresponseswouldleadthenurse to conclude that this patient isoriented? a. IknowmynameisJohn.IcouldnttellyouwhereIam.Ithinkitis2010,though. b. IknowmynameisJohn,buttotellyouthetruth,Igetkindofconfusedaboutthedate. c. IknowmynameisJohn;IguessImatthehospitalinSpokane.No,Idontknowthedate. d. IknowmynameisJohn.IamatthehospitalinSpokane.Icouldnttellyouwhatdateitis,butI know that it is February of a newyear2010. ANS: D Manyagingpersonsexperiencesocialisolation,lossofstructurewithoutajob,achangeinresidence,orsome short-term memory loss. These factors affect orientation, and the person may not provide the precise date or completenameoftheagency.Youmayconsideragingpersonsorientediftheygenerallyknowwheretheyare andthepresentperiod.Theyshouldbeconsideredorientedtotimeiftheyearandmontharecorrectlystated. Orientationtoplaceisacceptedwiththecorrectidentificationofthetypeofsetting(e.g.,hospital)andthe name of the town. PTS: 1 DIF: Cognitive Level: Applying (Application) 18. ThenurseisperformingtheDenverIIscreeningtestona12-month-oldinfantduringaroutinewell-child visit. The nurse should tell the infants parents that the DenverII: a. Tests three areas of development: cognitive, physical, andpsychological b. Willindicatewhetherthechildhasaspeechdisordersothattreatmentcanbegin. c. Isascreeninginstrumentdesignedtodetectchildrenwhoareslowindevelopment. d. Isatesttodetermineintellectualabilityandmayindicatewhetherproblemswilldeveloplaterin school. ANS: C TheDenverIIisascreeninginstrumentdesignedtodetectdevelopmentaldelaysininfantsandpreschoolers.It testsfourfunctions:grossmotor,language,finemotor-adaptive,andpersonal-social.TheDenverIIisnotan intelligencetest;itdoesnotpredictcurrentorfutureintellectualability.Itisnotdiagnostic;itdoesnotsuggest treatmentregimens. PTS: 1 DIF: Cognitive Level: Applying (Application) 19. Apatientdriftsofftosleepwhensheisnotbeingstimulated.Thenursecaneasilyarouseherbycallingher name,butthepatientremainsdrowsyduringtNheUcRoSnIvNeGrsTaBti.oCnO.MThebestdescriptionofthispatientslevelof consciousness would be: a. Lethargic b. Obtunded c. Stuporous d. Semialert ANS:A Lethargic(orsomnolent)iswhenthepersonisnotfullyalert,driftsofftosleepwhennotstimulated,andcan be aroused when called by name in a normal voice but looks drowsy. He or she appropriately responds to questions or commands, but thinking seems slow and fuzzy. He or she is inattentive and loses the train of thought. Spontaneous movements aredecreased. PTS: 1 DIF: Cognitive Level: Understanding (Comprehension) 20. A patient has had a cerebrovascular accident (stroke). He is trying very hard to communicate. He seems driventospeakandsays,Ibuyobiegetspirdingandtakemytrain.Whatisthebestdescriptionofthispatients problem? a. Globalaphasia b. Brocasaphasia c. Echolalia d. Wernickesaphasia ANS:D ThistypeofcommunicationillustratesWernickesorreceptiveaphasia.Thepersoncanhearsoundsandwords but cannot relate them to previous experiences. Speech is fluent, effortless, and well articulated, but it has many paraphasias (word substitutions that are malformed or wrong) and neologisms (made-up words) and oftenlackssubstantivewords.Speechcanbetotallyincomprehensible.Often,agreaturgetospeakispresent. Repetition,reading,andwritingalsoareimpaired.Echolaliaisanimitationortherepetitionofanotherpersons words orphrases. PTS: 1 DIF: Cognitive Level: Applying (Application) 21. Apatientrepeatedlyseemstohavedifficultycomingupwithaword.Hesays,Iwasonmywaytowork, andwhenIgotthere,thethingthatyoustepintothatgoesupintheairwassofullthatIdecidedtotakethe stairs.Thenursewillnoteonhischartthatheisusingorexperiencing: a. Blocking b. Neologism c. Circumlocution d. Circumstantiality NURSINGTB.COM ANS: C Circumlocutionisaroundaboutexpression,substitutingaphrasewhenonecannotthinkofthenameofthe object. PTS: 1 DIF: Cognitive Level: Understanding (Comprehension) 22. Duringanexamination,thenursenotesthatapatientisexhibitingflightofideas.Whichstatementbythe patient is an example of flight ofideas? a. My stomach hurts. Hurts, spurts,burts. b. Kiss, wood, reading, ducks, onto,maybe. c. Takethispill?Thepillisred.Iseered.Redvelvetissoft,softasababysbottom. d. Iwashmyhands,washthem,washthem.Iusuallygotothesinkandwashmyhands. ANS: C Flight of ideas is demonstrated by an abrupt change, rapid skipping from topic to topic, and practically continuousflowofacceleratedspeech.Topicsusuallyhaverecognizableassociationsorareplaysonwords. PTS: 1 DIF: Cognitive Level: Understanding (Comprehension) 23. Apatientdescribesfeelinganunreasonable,irrationalfearofsnakes.Hisfearissopersistentthathecanno longer comfortably look at even pictures of snakes and has made an effort to identify all the places he might encounter a snake and avoids them. The nurse recognizes thathe: a. Has a snakephobia. b. Is a hypochondriac; snakes are usuallyharmless. c. Has an obsession withsnakes. d. Hasadelusionthatsnakesareharmful,whichmuststemfromanearlytraumaticincident involvingsnakes. ANS: A Aphobiaisastrong,persistent,irrationalfearofanobjectorsituation;thepersonfeelsdriventoavoidit. PTS: 1 DIF: Cognitive Level: Applying(Application) 24. Apatienthasbeendiagnosedwithschizophrenia.Duringarecentinterview,heshowsthenurseapicture ofamanholdingadecapitatedhead.Hedescribesthispictureashorrifyingbutthenlaughsloudlyatthe content.Thisbehaviorisadisplayof: a. Confusion b. Ambivalence c. Depersonalization d. Inappropriateaffect NURSINGTB.COM ANS: D Aninappropriateaffectisanaffectclearlydiscordantwiththecontentofthepersonsspeech. PTS: 1 DIF: Cognitive Level: Analyzing(Analysis) 25. Duringreporting,thenursehearsthatapatientisexperiencinghallucinations.Whichisanexampleofa hallucination? a. Man believes that his dead wife is talking tohim. b. Womanhearsthedoorbellringandgoestoanswerit,butnooneisthere. c. Childseesamanstandinginhiscloset.Whenthelightsareturnedon,itisonlyadrycleaningbag. d. Manbelievesthatthedoghascurleduponthebed,butwhenhegetscloserheseesthatitisa blanket. ANS: A Hallucinationsaresensoryperceptionsforwhichnoexternalstimuliexist.Theymaystrikeanysense:visual, auditory, tactile, olfactory, orgustatory. PTS: 1 DIF: Cognitive Level: Remembering (Knowledge) 26. A20-year-oldconstructionworkerhasbeenbroughtintotheemergencydepartmentwithheatstroke.He has delirium as a result of a fluid and electrolyte imbalance. For the mental status examination, the nurse should first assess thepatients: a. Affect andmood b. Memory andaffect c. Languageabilities d. Level of consciousness and cognitiveabilities ANS: D NURSINGTB.COM Deliriumisadisturbanceofconsciousness(i.e.,reducedclarityofawarenessoftheenvironment)withreduced abilitytofocus,sustain,orshiftattention.Deliriumisnotanalterationinmood,affect,orlanguageabilities. PTS: 1 DIF: Cognitive Level: Understanding (Comprehension) 27. Apatientstates,Ifeelsosadallofthetime.IcantfeelhappyevendoingthingsIusedtoliketodo.Healso states that he is tired, sleeps poorly, and has no energy. To differentiate between a dysthymic disorder and a major depressive disorder, the nurse should ask whichquestion? a. Have you had any weightchanges? b. Are you having any thoughts ofsuicide? c. Howlonghaveyoubeenfeelingthisway? d. Areyouhavingfeelingsofworthlessness? ANS: C Majordepressivedisorderischaracterizedbyoneormoremajordepressiveepisodes,thatis,atleast2weeks ofdepressedmoodorlossofinterestaccompaniedbyatleastfouradditionalsymptomsofdepression. Dysthymicdisorderischaracterizedbyatleast2yearsofdepressedmoodformoredaysthannot, accompanied by additional depressivesymptoms. PTS: 1 DIF: Cognitive Level: Analyzing (Analysis) 28. A26-year-oldwomanwasrobbedandbeatenamonthago.Sheisreturningtotheclinictodayforafollow- upassessment.Thenursewillwanttoaskherwhichoneofthesequestions? a. How are things going with thetrial? b. How are things going with yourjob? c. Tellmeaboutyourrecentengagement! d. Areyouhavinganydisturbingdreams? ANS: D In posttraumatic stress disorder, the person has been exposed to a traumatic event. The traumatic event is persistentlyreexperiencedbyrecurrentandintrusive,distressingrecollectionsoftheevent,includingimages, thoughts, or perceptions; recurrent distressing dreams of the event; and acting or feeling as if the traumatic event wererecurring. PTS: 1 DIF: Cognitive Level: Applying (Application) 29. Thenurseisperformingamentalstatusexamination.Whichstatementistrueregardingtheassessmentof mentalstatus? a. Mental status assessment diagnoses specific psychiatricdisorders. NURSINGTB.COM b. Mental disorders occur in response to everyday lifestressors. c. Mentalstatusfunctioningisinferredthroughtheassessmentofanindividualsbehaviors. d. Mentalstatuscanbedirectlyassessed,similartoothersystemsofthebody(e.g.,heartsounds, breathsounds). ANS: C Mentalstatusfunctioningisinferredthroughtheassessmentofanindividualsbehaviors.Itcannotbedirectly assessed like the characteristics of the skin or heartsounds. PTS: 1 DIF: Cognitive Level: Understanding (Comprehension) 30. A23-year-oldpatientintheclinicappearsanxious.Herspeechisrapid,andsheisfidgetyandinconstant motion.Whichofthesequestionsorstatementswouldbemostappropriateforthenursetouseinthissituation to assess attentionspan? a. How do you usually feel? Is this normal behavior foryou? b. Iamgoingtosayfourwords.Inafewminutes,Iwillaskyoutorecallthem. c. Describethemeaningofthephrase,Lookingthroughrose-coloredglasses. d. Pickupthepencilinyourlefthand,moveittoyourrighthand,andplaceitonthetable. ANS: D Attentionspanisevaluatedbyassessingtheindividualsabilitytoconcentrateandcompleteathoughtortask withoutwandering.Givingaseriesofdirectionstofollowisonemethodusedtoassessattentionspan. PTS: 1 DIF: Cognitive Level: Applying (Application) 31. Thenurseisplanninghealthteachingfora65-year-oldwomanwhohashadacerebrovascularaccident (stroke)andhasaphasia.Whichofthesequestionsismostimportanttousewhenassessingmentalstatusin thispatient? a. Please count backward from 100 byseven. b. Iwillnamethreeitemsandaskyoutorepeattheminafewminutes. c. PleasepointtoarticlesintheroomandpartsofthebodyasInamethem. d. Whatwouldyoudoifyoufoundastamped,addressedenvelopeonthesidewalk? ANS: C Additionaltestsforpersonswithaphasiaincludewordcomprehension(askingtheindividualtopointto articlesintheroomorpartsofthebody),reading(askingthepersontoreadavailableprint),andwriting (askingthepersontomakeupandwriteasenNteUncReS).INGTB.COM PTS: 1 DIF: Cognitive Level: Applying (Application) 32. A30-year-oldfemalepatientisdescribingfeelingsofhopelessnessanddepression.Shehasattemptedself- mutilation and has a history of suicide attempts. She describes difficulty sleeping at night and has lost 10 poundsinthepastmonth.Whichofthesestatementsorquestionsisthenursesbestresponseinthissituation? a. Do you have aweapon? b. How do other people treatyou? c. Are you feeling so hopeless that you feel like hurting yourselfnow? d. Peopleoftenfeelhopeless,butthefeelingsresolvewithinafewweeks. ANS: C Whenthepersonexpressesfeelingsofhopelessness,despair,orgrief,assessingtheriskofphysicalharmto himorherselfisimportant.Thisprocessbeginswithmoregeneralquestions.Iftheanswersareaffirmative, then the assessment continues with more specificquestions. PTS: 1 DIF: Cognitive Level: Applying (Application) 33. Thenurseisprovidinginstructionstonewlyhiredgraduatesfortheminimentalstateexamination (MMSE). Which statement best describes thisexamination? a. Scores below 30 indicate cognitiveimpairment. b. The MMSE is a good tool to evaluate mood and thoughtprocesses. c. Thisexaminationisagoodtooltodetectdeliriumanddementiaandtodifferentiatethesefrom psychiatric mentalillness. d. TheMMSEisusefultoolforaninitialevaluationofmentalstatus.Additionaltoolsareneededto evaluate cognition changes overtime. ANS: C The MMSE is a quick, easy test of 11 questions and is used for initial and serial evaluations and can demonstrateaworseningoranimprovementofcognitionovertimeandwithtreatment.Itevaluatescognitive functioning,notmoodorthoughtprocesses.MMSEisagoodscreeningtooltodetectdementiaanddelirium and to differentiate these from psychiatric mentalillness. PTS: 1 DIF: Cognitive Level: Understanding (Comprehension) 34. The nurse discovers speech problems in a patient during an assessment. The patient has spontaneous speech,butitismostlyabsentorisreducedtoafewstereotypicalwordsorsounds.Thisfindingreflectswhich type ofaphasia? a. Global NURSINGTB.COM b. Brocas c. Dysphonic d. Wernickes ANS: A Globalaphasiaisthemostcommonandsevereformofaphasia.Spontaneousspeechisabsentorreducedtoa few stereotyped words or sounds, and prognosis for language recovery is poor. Dysphonic aphasia is not a validcondition. PTS: 1 DIF: Cognitive Level: Understanding (Comprehension) 35. Apatientrepeats,Ifeelhot.Hot,cot,rot,tot,got.Imaspot.Thenursedocumentsthisasanillustrationof: a. Blocking b. Clanging c. Echolalia d. Neologism ANS: B Clangingiswordchoicebasedonsound,notmeaning,andincludesnonsenserhymesandpuns. PTS: 1 DIF: Cognitive Level: Understanding(Comprehension) 36. Duringaninterview,thenursenotesthatthepatientgetsupseveraltimestowashherhandseventhough they are not dirty. This behavior is an exampleof: a. Socialphobia b. Compulsivedisorder c. Generalized anxietydisorder d. Posttraumaticstressdisorder ANS:B Repetitivebehaviors,suchashandwashing,arebehaviorsthatthepersonfeelsdriventoperforminresponseto anobsession.Thebehaviorsareaimedatpreventingorreducingdistressorpreventingsomedreadedeventor situation. PTS: 1 DIF: Cognitive Level: Understanding (Comprehension) 37. ThenurseisadministeringaMini-Cogtesttoanolderadultwoman.Whenaskedtodrawaclockshowing thetimeof10:45,thepatientdrewaclockwitNhUthReSnINumGTbeBr.sCoOuMtoforderandwithanincorrecttime.Thisresul indicates whichfinding? a. Cognitiveimpairment b. Amnesia c. Delirium d. Attention-deficit disorder ANS:A TheMini-Cogisanewerinstrumentthatscreensforcognitiveimpairment,oftenfoundwithdementia.The resultofanabnormaldrawingofaclockandtimeindicatesacognitiveimpairment. PTS: 1 DIF: Cognitive Level: Analyzing (Analysis) 38. Duringmorningrounds,thenurseasksapatient,Howareyoutoday?Thepatientresponds,Youtoday,you today,youtoday!andmumblesthewords.Thisspeechpatternisanexampleof: a. Echolalia b. Clanging c. Wordsalad d. Perseveration ANS:A Echolaliaoccurswhenapersonimitatesorrepeatsanotherswordsorphrases,oftenwithamumbling, mocking, or a mechanicaltone. PTS: 1 DIF: Cognitive Level: Applying (Application) MULTIPLE RESPONSE 1.Thenurseisassessingapatientwhoisadmittedwithpossibledelirium.Whichofthesearemanifestationsof delirium? Select all thatapply. a. Develops over a shortperiod. b. Person is experiencingapraxia. c. Person is exhibiting memory impairment ordeficits. d. Occursasaresultofamedicalcondition,suchassystemicinfection. e. Personisexperiencingagnosia. NURSINGTB.COM ANS: A, C, D Deliriumisadisturbanceofconsciousnessthatdevelopsoverashortperiodandmaybeattributabletoa medicalcondition.Memorydeficitsmayalsooccur.Apraxiaandagnosiaoccurwithdementia. PTS: 1 DIF: Cognitive Level: Applying (Application) Chapter 06: Substance Use Assessment MULTIPLE CHOICE 1. Awomanhascometotheclinictoseekhelpwithasubstanceabuseproblem.Sheadmitstousingcocaine justbeforearriving.Whichoftheseassessmentfindingswouldthenurseexpecttofindwhenexaminingthis woman? a. Dilated pupils, pacing, and psychomotoragitation b. Dilated pupils, unsteady gait, andaggressiveness c. Pupil constriction, lethargy, apathy, anddysphoria d. Constrictedpupils,euphoria,anddecreasedtemperature ANS:A Acocaineusersappearanceincludespupillarydilation,tachycardiaorbradycardia,elevatedorloweredblood pressure, sweating, chills, nausea, vomiting, and weight loss. The persons behavior includes euphoria, talkativeness, hypervigilance, pacing, psychomotor agitation, impaired social or occupational functioning, fighting, grandiosity, and visual or tactilehallucinations. DIF: Cognitive Level: Applying (Application) MSC:ClientNeeds:PhysiologicIntegrity:PhNysUioRlSoIgNicGATBda.CpOtaMtion 2. Thenurseisassessingapatientwhohasbeenadmittedforcirrhosisoftheliver,secondarytochronic alcoholuse.Duringthephysicalassessment,thenurselooksforcardiacproblemsthatareassociatedwith chronic use of alcohol, suchas: a. Hypertension. b. Ventricularfibrillation. c. Bradycardia. d. Mitral valveprolapse. ANS: A Evenmoderatedrinkingleadstohypertensionandcardiomyopathy,withanincreaseinleftventricularmass, dilationofventricles,andwallthinning.Ventricularfibrillation,bradycardia,andmitralvalveprolapsearenot associated with chronic heavy use ofalcohol. DIF: Cognitive Level: Applying (Application) MSC: Client Needs: Physiologic Integrity: Physiologic Adaptation 3. Thenurseisconductingaclassonalcoholandtheeffectsofalcoholonthebody.Howmanystandarddrinks (each containing 14 grams of alcohol) per day in men are associated with increased deaths from cirrhosis, cancers of the mouth, esophagus, andinjuries? a. 2 b. 4 c. 6 d. 8 ANS: B Inmen,alcoholconsumptionofatleastfourstandarddrinksperdayisassociatedwithincreaseddeathsfrom livercirrhosis,cancersofthemouth,esophagusandotherareas,anddeathsfrominjuriesandotherexternal causes. DIF: Cognitive Level: Understanding (Comprehension) MSC: Client Needs: Physiologic Integrity: Reduction of Risk Potential 4. Duringasessiononsubstanceabuse,thenurseisreviewingstatisticswiththeclass.Forpersonsaged12 years and older, which illicit substance was most commonlyused? a. Crackcocaine b. Heroin c. Marijuana d. Hallucinogens NURSINGTB.COM ANS: C Inpersonsage12yearsandolderwhoreportedusingduringthepastmonth,marijuana(hashish)wasthemost commonly used illicit drugreported. DIF: Cognitive Level: Remembering (Knowledge) MSC: Client Needs: Physiologic Integrity: Physiologic Adaptation 5. Awomanwhohasjustdiscoveredthatsheispregnantisintheclinicforherfirstobstetricvisit.Sheasksthe nurse,Howmanydrinksadayissafeformybaby?Thenursesbestresponseis: a. You should limit your drinking to once or twice aweek. b. Its okay to have up to two glasses of wine aday. c. As long as you avoid getting drunk, you should besafe. d. Noamountofalcoholhasbeendeterminedtobesafeduringpregnancy. ANS: D Noamountofalcoholhasbeendeterminedtobesafeforpregnantwomen.Thepotentialadverseeffectsof alcoholuseonthefetusarewellknown;womenwhoarepregnantshouldbescreenedforalcoholuse,and abstinence should berecommended. DIF: Cognitive Level: Analyzing (Analysis) MSC: Client Needs: Safe and Effective Care Environment: Safety and Infection Control 6. When reviewing the use of alcohol by older adults, the nurse notes that older adults have several characteristicsthatcanincreasetheriskofalcoholuse.Whichwouldincreasethebioavailabilityofalcoholin the blood for longer periods in the olderadult? a. Increased musclemass b. Decreased liver and kidneyfunctioning c. Decreased bloodpressure d. Increasedcardiacoutput ANS:B Decreasedliverandkidneyfunctioningincreasesthebioavailabilityofalcoholinthebloodforlongerperiods. Agingpeopleexperiencedecreasedmusclemass(notincreased),whichalsoincreasesthealcohol concentrationinthebloodbecausethealcoholNiUsRdiSsItNriGbuTtBed.CtOoMlesstissueovertime.Bloodpressureand cardiacoutputarenotfactorsregardingbioavailability. DIF: Cognitive Level: Applying(Application) MSC: Client Needs: Physiologic Integrity: Physiologic Adaptation 7. Duringanassessment,thenurseasksafemalepatient,Howmanyalcoholicdrinksdoyouhaveaweek? Which answer by the patient would indicate at-riskdrinking? a. I may have one or two drinks aweek. b. I usually have three or four drinks aweek. c. Ill have a glass or two of wine every now andthen. d. I have seven or eight drinks a week, but I never getdrunk. ANS: D Forwomen,havingsevenormoredrinksaweekorthreeormoredrinksperoccasionisconsideredat-risk drinking,accordingtotheNationalInstituteonAlcoholAbuseandAlcoholism. DIF: Cognitive Level: Applying (Application) MSC: Client Needs: Physiologic Integrity: Reduction of Risk Potential 8. Thenurseisaskinganadolescentaboutillicitsubstanceabuse.Theadolescentanswers,Yes,Iveused marijuanaatpartieswithmyfriends.Whatisthenextquestionthenurseshouldask? a. Who are thesefriends? b. Do your parents know aboutthis? c. When was the last time you usedmarijuana? d. Is this a regularhabit? ANS: C Ifapatientadmitstotheuseofillicitsubstances,thenthenurseshouldask,Whenwasthelasttimeyouused drugs?andHowmuchdidyoutakethattime?Theotherquestionsmaybeconsideredaccusatoryandarenot conducive to gatheringinformation. DIF: Cognitive Level: Applying (Application) MSC: Client Needs: Health Promotion and Maintenance 9. Thenursehascompletedanassessmentonapatientwhocametotheclinicforaleginjury.Asaresultofthe assessment,thenursehasdeterminedthatthepatienthasat-riskalcoholuse.Whichactionbythenurseismost appropriate at thistime? a. RecordtheresultsoftheassessmeNntU,aRnSdINnGotTifBy.CthOeMphysicianoncall. b. State,Youaredrinkingmorethanismedicallysafe.Istronglyrecommendthatyouquitdrinking, and Im willing to helpyou. c. State,Itappearsthatyoumayhaveadrinkingproblem.Hereisthetelephonenumberofourlocal Alcoholics Anonymouschapter. d. Give the patient information about a local rehabilitationclinic. ANS: B Ifanassessmenthasdeterminedthatthepatienthasat-riskdrinkingbehavior,thenthenurseshouldgivea short but clear statement of assistance and concern. Simply giving out a telephone number or referral to agencies may not beenough. DIF: Cognitive Level: Applying (Application) MSC: Client Needs: Safe and Effective Care Environment: Safety and Infection Control 10. A patient is brought to the emergency department. He is restless, has dilated pupils, is sweating, has a runnynoseandtearingeyes,andcomplainsofmuscleandjointpains.Hisgirlfriendthinkshehasinfluenza, butshebecameconcernedwhenhistemperaturewentupto39.4C.Sheadmitsthathehasbeenaheavydrug user,buthehasbeentryingtostoponhisown.Thenursesuspectsthatthepatientisexperiencingwithdrawal symptoms from whichsubstance? a. Alcohol b. Heroin c. Crackcocaine d. Sedatives ANS:B Withdrawalsymptomsofopiates,suchasheroin,aresimilartotheclinicalpictureofinfluenzaandinclude symptomssuchasdilatedpupils,lacrimation,runnynose,tachycardia,fever,restlessness,muscleandjoint pains, and othersymptoms. DIF: Cognitive Level: Applying (Application) MSC: Client Needs: Safe and Effective Care Environment: Safety and Infection Control 11. Thenurseisreviewingaspectsofsubstanceabuseinpreparationforaseminar.Whichofthesestatements illustrates the concept of tolerance to an illicit substance? Theperson: a. Has a physiologic dependence on asubstance. b. Requiresanincreasedamountofthesubstancetoproducethesameeffect. NURSINGTB.COM c. Requiresdailyuseofthesubstancetofunctionandisunabletostopusingit. d. Experiencesasyndromeofphysiologicsymptomsifthesubstanceisnotused. ANS: B Theconceptoftolerancetoasubstanceindicatesthatthepersonrequiresanincreasedamountofthesubstance toproducethesameeffect.Abuseindicatesthatthepersonneedstousethesubstancedailytofunction,andthe personisunabletostopusingit.Dependenceisanactualphysiologicdependenceonthesubstance. Withdrawaloccurswhencessationofthesubstanceproducesasyndromeofphysiologicsymptoms. DIF: Cognitive Level: Understanding(Comprehension) MSC: Client Needs: Physiologic Integrity: Physiologic Adaptation MULTIPLE RESPONSE 1. ApatientwithaknownhistoryofheavyalcoholusehasbeenadmittedtotheICUafterhewasfound unconsciousoutsideabar.Thenursecloselymonitorshimforsymptomsofwithdrawal.Whichofthese symptoms may occur during this time? Select all thatapply. a. Bradycardia b. Coarse tremor of thehands c. Transienthallucinations d. Somnolence e. Sweating ANS: B, C,E Symptomsofuncomplicatedalcoholwithdrawalstartshortlyafterthecessationofdrinking,peakatthesecond day, and improve by the fourth or fifth day. Symptoms include coarse tremors of the hands, tongue, and eyelids; anorexia; nausea and vomiting; autonomic hyperactivity (e.g., tachycardia, sweating, elevated blood pressure); and transient hallucinations, among othersymptoms. DIF: Cognitive Level: Applying (Application) MSC: Client Needs: Safe and Effective Care Environment: Management of Care 2. Apatientvisitstheclinictoaskaboutsmokingcessation.Hehassmokedheavilyfor30yearsandwantsto stopcoldturkey.Heasksthenurse,WhatsymptomscanIexpectifIdothis?Whichofthesesymptomsshould thenursesharewiththepatientaspossiblesymptomsofnicotinewithdrawal?Selectallthatapply. a. Headaches b. Hunger c. Sleepiness d. Restlessness e. Nervousness f. Sweating NURSINGTB.COM ANS: A, B, D, E Symptomsofnicotinewithdrawalincludevasodilation,headaches,anger,irritability,frustration,anxiety, nervousness,awakeningatnight,difficultyconcentrating,depression,hunger,impatience,andthedesireto smoke. DIF: Cognitive Level: Applying (Application) MSC: Client Needs: Physiologic Integrity: Physiologic Adaptation Chapter 07: Domestic and Family Violence Assessment MULTIPLE CHOICE 1. Asamandatoryreporterofelderabuse,whichmustbepresentbeforeanurseshouldnotifytheauthorities? a. Statements from thevictim b. Statements fromwitnesses c. Proof of abuse and/orneglect d. Suspicionofelderabuseand/orneglect ANS:D Manyhealthcareworkersareundertheerroneousassumptionthatproofisrequiredbeforenotificationof suspectedabusecanoccur.Onlythesuspicionofelderabuseorneglectisnecessary. DIF: Cognitive Level: Applying (Application) MSC: Client Needs: Safe and Effective Care Environment: Safety and Infection Control 2. Duringahomevisit,thenursenoticesthatanolderadultwomaniscaringforherbedriddenhusband.The womanstatesthatthisisherduty,shedoesthebestshecan,andherchildrencometohelpwhentheyarein town.HerhusbandisunabletocareforhimseNlfU,aRnSdINshGeTaBp.pCeOaMrsthin,weak,andexhausted.Thenursenotices that several of his prescription medication bottles are empty. This situation is best described by the term: a. Physicalabuse. b. Financialneglect. c. Psychologicalabuse. d. Unintentional physicalneglect. ANS: D Unintentional physical neglect may occur, despite good intentions, and is the failure of a family member or caregivertoprovidebasicgoodsorservices.Physicalabuseisdefinedasviolentactsthatresultorcouldresult ininjury,pain,impairment,ordisease.Financialneglectisdefinedasthefailuretousetheassetsoftheolder person to provide services needed by him or her. Psychological abuse is defined as behaviors that result in mentalanguish. DIF: Cognitive Level: Applying (Application) MSC: Client Needs: Safe and Effective Care Environment: Safety and Infection Control 3. Thenurseisawarethatintimatepartnerviolence(IPV)screeningshouldoccurwithwhichsituation? a. When IPV issuspected b. When a woman has an unexplainedinjury c. As a routine part of each health careencounter d. Whenahistoryofabuseinthefamilyisknown ANS:C Manynursingprofessionalorganizationshavecalledforroutine,universalscreeningforIPVtoassistwomen in getting help for theproblem. DIF: Cognitive Level: Applying (Application) MSC: Client Needs: Safe and Effective Care Environment: Safety and Infection Control 4. WhichstatementisbestforthenursetousewhenpreparingtoadministertheAbuseAssessmentScreen? a. We are required by law to ask thesequestions. b. We need to talk about whether you believe you have beenabused. c. Weareaskingthesequestionsbecausewesuspectthatyouarebeingabused. d. WeneedtoaskthefollowingquesNtiUonRsSbINecGaTuBse.CdOomMesticviolenceissocommoninoursociety. ANS: D Suchanintroductionalertsthewomanthatquestionsaboutdomesticviolencearecomingandensuresthe woman that she is not being singled out for thesequestions. DIF: Cognitive Level: Applying (Application) MSC: Client Needs: Safe and Effective Care Environment: Safety and Infection Control 5. Whichtermreferstoawoundproducedbythetearingorsplittingofbodytissue,usuallyfrombluntimpact over a bonysurface? a. Abrasion b. Contusion c. Laceration d. Hematoma ANS:C The term laceration refers to a wound produced by the tearing or splitting of body tissue. An abrasion is causedbytherubbingoftheskinormucousmembrane.Acontusionisinjurytotissueswithoutbreakageof skin,andahematomaisalocalizedcollectionofextravasatedblood. DIF: Cognitive Level: Remembering(Knowledge) MSC: Client Needs: Safe and Effective Care Environment: Safety and Infection Control 6. Duringanexamination,thenursenoticesapatternedinjuryonapatientsback.Whichofthesewouldcause such aninjury? a. Bluntforce b. Frictionabrasion c. Stabbing from a kitchenknife d. Whippingfromanextensioncord ANS:D Apatternedinjuryisaninjurycausedbyanobjectthatleavesadistinctpatternontheskinororgan.Theother actions do not cause a patternedinjury. DIF: Cognitive Level: Applying (Application) MSC: Client Needs: Safe and Effective Care Environment: Safety and Infection Control 7. WhendocumentingIPVandelderabuse,thNeUnRurSsIeNsGhToBul.dCOinMclude: a. Photographicdocumentationoftheinjuries. b. Summaryoftheabusedpatientsstatements. c. Verbatim documentation of every statementmade. d. General description of injuries in the progressnotes. ANS: A DocumentationofIPVandelderabusemustincludedetailednonbiasedprogressnotes,theuseofinjurymaps, and photographic documentation. Written documentation needs to be verbatim, within reason. Not every statement can bedocumented. DIF: Cognitive Level: Applying (Application) MSC: Client Needs: Safe and Effective Care Environment: Safety and Infection Control 8. AfemalepatienthasdeniedanyabusewhenansweringtheAbuseAssessmentScreen,butthenursehas noticedsomeotherconditionsthatareassociatedwithIPV.Examplesofsuchconditionsinclude: a. Asthma. b. Confusion. c. Depression. d. Frequentcolds. ANS: C Depression is one of the conditions that is particularly associated with IPV. Abused women also have been foundtohavemorechronichealthproblems,suchasneurologic,gastrointestinal,andgynecologicsymptoms; chronic pain; and symptoms of suicidality and posttraumatic stressdisorder. DIF: Cognitive Level: Understanding (Comprehension) MSC: Client Needs: Safe and Effective Care Environment: Safety and Infection Control 9. Thenurseisusingthedangerassessment(DA)tooltoevaluatetheriskofhomicide.Whichofthese statements best describes itsuse? a. The DA tool is to be administered by law enforcementpersonnel. b. TheDAtoolshouldbeusedineveryassessmentofsuspectedabuse. c. Thenumberofyesanswersindicatesthewomansunderstandingofhersituation. NURSINGTB.COM d. Thehigherthenumberofyesanswers,themoreseriousthedangerofthewomanssituation. ANS: D NopredeterminedcutoffscoresexistontheDA.Thehigherthenumberyesanswers,themoreseriousthe dangerofthewomanssituation.Theuseofthistoolisnotlimitedtolawenforcementpersonnelandisnot required in every case of suspectedabuse. DIF: Cognitive Level: Applying (Application) MSC: Client Needs: Safe and Effective Care Environment: Safety and Infection Control 10. Thenurseisassessingbruisingonaninjuredpatient.Whichcolorindicatesanewbruisethatislessthan2 hoursold? a. Red b. Purple-blue c. Greenish-brown d. Brownish-yellow ANS:A Anewbruiseisusuallyredandwilloftendevelopapurpleorpurple-blueappearance12to36hoursafter blunt-forcetrauma.Thecolorofbruises(andecchymoses)generallyprogressesfrompurple-bluetobluish- green to greenish-brown to brownish-yellow before fadingaway. DIF: Cognitive Level: Understanding (Comprehension) MSC: Client Needs: Safe and Effective Care Environment: Safety and Infection Control 11. Thenursesuspectsabusewhena10-year-oldchildistakentotheurgentcarecenterforaleginjury.The best way to document the history and physical findings isto: a. Document what the childs caregiver tells thenurse. b. Usethewordsthechildhassaidtodescribehowtheinjuryoccurred. c. Record what the nurse observes during theconversation. d. Rely on photographs of theinjuries. ANS: B Whendocumentingthehistoryandphysicalfindingsofsuspectedchildabuseandneglect,usethewordsthe childhassaidtodescribehowhisorherinjuryoccurred.Remember,theabusermaybeaccompanyingthe child. DIF: Cognitive Level: Applying (Application) NURSINGTB.COM MSC: Client Needs: Safe and Effective Care Environment: Management of Care 12. Duringaninterview,awomanhasansweredyestotwooftheAbuseAssessmentScreenquestions.What should the nurse saynext? a. I need to report this abuse to theauthorities. b. Tell me about this abuse in yourrelationship. c. So you wereabused? d. Do you know what caused thisabuse? ANS: B IfawomananswersyestoanyoftheAbuseAssessmentScreenquestions,thenthenurseshouldaskquestions designed to assess how recent and how serious the abuse was. Asking the woman an open-ended question, suchastellmeaboutthisabuseinyourrelationshipisagoodwaytostart. DIF: Cognitive Level: Analyzing (Analysis) MSC: Client Needs: Safe and Effective Care Environment: Management of Care 13. Thenurseisexamininga3-year-oldchildwhowasbroughttotheemergencydepartmentafterafall. Which bruise, if found, would be of mostconcern? a. Bruise on theknee b. Bruise on theelbow c. Bruising on theabdomen d. Bruise on theshin ANS:C Studieshaveshownthatchildrenwhoarewalkingoftenhavebruisesoverthebonyprominencesofthefronto their bodies. Other studies have found that bruising in atypical places such as the buttocks, hands, feet, and abdomen were exceedingly rare and should arouseconcern. DIF: Cognitive Level: Analyzing (Analysis) MSC: Client Needs: Safe and Effective Care Environment: Management of Care MULTIPLE RESPONSE 1.Thenurseassessesanolderwomanandsuspectsphysicalabuse.Whichquestionsareappropriatefor screening for abuse? Select all thatapply. a. Hasanyonemadeyouafraid,touchedyouinwaysthatyoudidnotwant,orhurtyouphysically? b. Are you beingabused? NURSINGTB.COM c. Haveyoureliedonpeopleforanyofthefollowing:bathing,dressing,shopping,banking,or meals? d. Haveyoubeenupsetbecausesomeonetalkedtoyouinawaythatmadeyoufeelshamedor threatened? e. Haveyoureliedonpeopleforanyofthefollowing:bathing,dressing,shopping,banking,or meals? ANS: A, C, D, E DirectlyaskingAreyoubeingabused?isnotanappropriatescreeningquestionforabusebecausethewoman could easily say no, and no further information would be obtained. The other questions are among the questionsrecommendedbytheElderAbuseSuspicionIndex(EASI)whenscreeningforelderabuse. DIF: Cognitive Level: Analyzing (Analysis) MSC: Client Needs: Safe and Effective Care Environment: Management of Care Chapter 08: Assessment Techniques and Safety in the Clinical Setting MULTIPLE CHOICE 1. Whenperformingaphysicalassessment,thefirsttechniquethenursewillalwaysuseis: a. Palpation. b. Inspection. c. Percussion. d. Auscultation. ANS: B Theskillsrequisiteforthephysicalexaminationareinspection,palpation,percussion,andauscultation.The skillsareperformedoneatatimeandinthisorder(withtheexceptionoftheabdominalassessment,during whichauscultationtakesplacebeforepalpationandpercussion).Theassessmentofeachbodysystembegins withinspection.Afocusedinspectiontakestimeandyieldsasurprisingamountofinformation. DIF: Cognitive Level: Remembering (Knowledge) MSC: Client Needs: Safe and Effective Care Environment: Management of Care 2. ThenurseispreparingtoperformaphysicaNlUasRseSsIsNmGeTnBt..CWOhMichstatementistrueaboutthephysical assessment? The inspectionphase: a. Usually yields littleinformation. b. Takes time and reveals a surprising amount ofinformation. c. May be somewhat uncomfortable for the expertpractitioner. d. Requiresaquickglanceatthepatientsbodysystemsbeforeproceedingwithpalpation. ANS: B Afocusedinspectiontakestimeandyieldsasurprisingamountofinformation.Initially,theexaminermayfeel uncomfortable,staringatthepersonwithoutalsodoingsomething.Afocusedassessmentissignificantlymore than a quickglance. DIF: Cognitive Level: Understanding (Comprehension) MSC: Client Needs: Safe and Effective Care Environment: Management of Care 3. Thenurseisassessingapatientsskinduringanofficevisit.Whatpartofthehandandtechniqueshouldbe used to best assess the patients skintemperature? a. Fingertips; they are more sensitive to small changes intemperature. b. Dorsalsurfaceofthehand;theskinisthinneronthissurfacethanonthepalms. c. Ulnarportionofthehand;increasedbloodsupplyinthisareaenhancestemperaturesensitivity. d. Palmarsurfaceofthehand;thissurfaceisthemostsensitivetotemperaturevariationsbecauseof its increased nerve supply in thisarea. ANS: B Thedorsa(backs)ofthehandsandfingersarebestfordeterminingtemperaturebecausetheskinisthinneron thedorsalsurfacesthanonthepalms.Fingertipsarebestforfine,tactilediscrimination.Theotherresponses are not useful forpalpation. DIF: Cognitive Level: Applying (Application) MSC: Client Needs: Safe and Effective Care Environment: Management of Care 4. Whichofthesetechniquesusesthesenseoftouchtoassesstexture,temperature,moisture,andswelling when the nurse is assessing apatient? a. Palpation b. Inspection c. Percussion NURSINGTB.COM d. Auscultation ANS: A Palpationusesthesenseoftouchtoassessthepatientforthesefactors.Inspectioninvolvesvision;percussion assessesthroughtheuseofpalpablevibrationsandaudiblesounds;andauscultationusesthesenseofhearing. DIF: Cognitive Level: Remembering (Knowledge) MSC: Client Needs: Safe and Effective Care Environment: Management of Care 5. Thenurseispreparingtoassessapatientsabdomenbypalpation.Howshouldthenurseproceed? a. Palpationofreportedlytenderareasareavoidedbecausepalpationintheseareasmaycausepain. b. Palpatingatenderareaisquicklyperformedtoavoidanydiscomfortthatthepatientmay experience. c. Theassessmentbeginswithdeeppalpation,whileencouragingthepatienttorelaxandtotakedeep breaths. d. Theassessmentbeginswithlightpalpationtodetectsurfacecharacteristicsandtoaccustomthe patient to beingtouched. ANS: D Lightpalpationisinitiallyperformedtodetectanysurfacecharacteristicsandtoaccustomthepersontobeing touched. Tender areas should be palpated last, notfirst. DIF: Cognitive Level: Applying (Application) MSC: Client Needs: Safe and Effective Care Environment: Management of Care 6. The nurse would use bimanual palpation technique in whichsituation? a. Palpatingthethoraxofaninfant b. Palpatingthekidneysanduterus c. Assessing pulsations andvibrations d. Assessingthepresenceoftendernessandpain ANS:B Bimanualpalpationrequirestheuseofbothhandstoenveloporcapturecertainbodypartsororganssuchas thekidneys,uterus,oradnexa.Theothersituationsarenotappropriateforbimanualpalpation. DIF: Cognitive Level: Applying (Application) MSC:ClientNeeds:SafeandEffectiveCareENnUvRirSoInNmGeTnBt:.CMOaMnagementofCare 7. Thenurseispreparingtopercusstheabdomenofapatient.Thepurposeofthepercussionistoassessthe of the underlyingtissue. a. Turgor b. Texture c. Density d. Consistency ANS:C Percussionyieldsasoundthatdepictsthelocation,size,anddensityoftheunderlyingorgan.Turgorand texture are assessed withpalpation. DIF: Cognitive Level: Understanding (Comprehension) MSC: Client Needs: Safe and Effective Care Environment: Management of Care 8. Thenurseisreviewingpercussiontechniqueswithanewlygraduatednurse.Whichtechnique,ifusedbythe new nurse, indicates that more review isneeded? a. Percussing once over eacharea b. Quickly lifting the striking finger after eachstroke c. Striking with the fingertip, not the fingerpad d. Usingthewristtomakethestrikes,notthearm ANS:A Forpercussion,thenurseshouldpercusstwotimesovereachlocation.Thestrikingfingershouldbequickly liftedbecausearestingfingerdampsoffvibrations.Thetipofthestrikingfingershouldmakecontact,notthe padofthefinger.Thewristmustberelaxedandisusedtomakethestrikes,notthearm. DIF: Cognitive Level: Applying (Application) MSC: Client Needs: Safe and Effective Care Environment: Management of Care 9. Whenpercussingovertheliverofapatient,thenursenoticesadullsound.Thenurseshould: a. Consider this a normalfinding. b. Palpate this area for an underlyingmass. c. Reposition the hands, and attempt to percuss in this areaagain. NURSINGTB.COM d. Considerthisfindingasabnormal,andreferthepatientforadditionaltreatment. ANS: A Percussionoverrelativelydenseorgans,suchastheliverorspleen,willproduceadullsound.Theother responses are notcorrect. DIF: Cognitive Level: Analyzing (Analysis) MSC: Client Needs: Safe and Effective Care Environment: Management of Care 10. Thenurseisunabletoidentifyanychangesinsoundwhenpercussingovertheabdomenofanobese patient. What should the nurse donext? a. Askthepatienttotakedeepbreathstorelaxtheabdominalmusculature. b. Considerthisfindingasnormal,andproceedwiththeabdominalassessment. c. Increasetheamountofstrengthusedwhenattemptingtopercussovertheabdomen. d. Decreasetheamountofstrengthusedwhenattemptingtopercussovertheabdomen. ANS: C The thickness of the persons body wall will be a factor. The nurse needs a stronger percussion stroke for personswithobeseorverymuscularbodywalls.Theforceoftheblowdeterminestheloudnessofthenote. The other actions are notcorrect. DIF: Cognitive Level: Applying (Application) MSC: Client Needs: Safe and Effective Care Environment: Management of Care 11. Thenursehearsbilateralloud,long,andlowtoneswhenpercussingoverthelungsofa4-year-oldchild. The nurseshould: a. Palpate over the area for increased pain andtenderness. b. Askthechildtotakeshallowbreaths,andpercussovertheareaagain. c. Immediatelyreferthechildbecauseofanincreasedamountofairinthelungs. d. Considerthisfindingasnormalforachildthisage,andproceedwiththeexamination. ANS: D Percussionnotesthatareloudinamplitude,lowinpitch,ofaboomingquality,andlongindurationarenormal over a childslung. DIF: Cognitive Level: Applying (Application) MSC:ClientNeeds:SafeandEffectiveCareENnUvRirSoInNmGeTnBt:.CMOaMnagementofCare 12. Apatienthassuddenlydevelopedshortnessofbreathandappearstobeinsignificantrespiratorydistress. Aftercallingthephysicianandplacingthepatientonoxygen,whichoftheseactionsisthebestforthenurseto take when further assessing thepatient? a. Count the patientsrespirations. b. Bilaterally percuss the thorax, noting any differences in percussiontones. c. Callforachestx-raystudy,andwaitfortheresultsbeforebeginninganassessment. d. Inspectthethoraxforanynewmassesandbleedingassociatedwithrespirations. ANS: B Percussionisalwaysavailable,portable,andoffersinstantfeedbackregardingchangesinunderlyingtissue density, which may yield clues of the patients physicalstatus. DIF: Cognitive Level: Analyzing (Analysis) MSC: Client Needs: Safe and Effective Care Environment: Management of Care 13. Thenurseisteachingaclassonbasicassessmentskills.Whichofthesestatementsistrueregardingthe stethoscope and itsuse? a. Slope of the earpieces should point posteriorly (toward theocciput). b. Althoughthestethoscopedoesnotmagnifysound,itdoesblockoutextraneousroomnoise. c. Fitandqualityofthestethoscopearenotasimportantasitsabilitytomagnifysound. d. Idealtubinglengthshouldbe22inchestodampenthedistortionofsound. ANS: B The stethoscope does not magnify sound, but it does block out extraneous room sounds. The slope of the earpiecesshouldpointforwardtowardtheexaminersnose.Longtubingwilldistortsound.Thefitandquality of the stethoscope are bothimportant. DIF: Cognitive Level: Understanding (Comprehension) MSC: Client Needs: Safe and Effective Care Environment: Management of Care 14. Thenurseispreparingtouseastethoscopeforauscultation.Whichstatementistrueregardingthe diaphragm of the stethoscope? Thediaphragm: a. Is used to listen for high-pitchedsounds. b. Isusedtolistenforlow-pitchedsounds. NURSINGTB.COM c. Shouldbelightlyheldagainstthepersonsskintoblockoutlow-pitchedsounds. d. Shouldbelightlyheldagainstthepersonsskintolistenforextraheartsoundsandmurmurs. ANS: A The diaphragm of the stethoscope is best for listening to high-pitched sounds such as breath, bowel, and normalheartsounds.Itshouldbefirmlyheldagainstthepersonsskin,firmlyenoughtoleavearing.Thebell ofthestethoscopeisbestforsoft,low-pitchedsoundssuchasextraheartsoundsormurmurs. DIF: Cognitive Level: Understanding (Comprehension) MSC: Client Needs: Safe and Effective Care Environment: Management of Care 15. Beforeauscultatingtheabdomenforthepresenceofbowelsoundsonapatient,thenurseshould: a. Warm the endpiece of the stethoscope by placing it in warmwater. b. Leavethegownonthepatienttoensurethatheorshedoesnotgetchilledduringtheexamination. c. Ensurethatthebellsideofthestethoscopeisturnedtotheonposition. d. Checkthetemperatureoftheroom,andofferblanketstothepatientifheorshefeelscold. ANS: D Theexaminationroomshouldbewarm.Ifthepatientshivers,thentheinvoluntarymusclecontractionscan makeitdifficulttoheartheunderlyingsounds.Theendofthestethoscopeshouldbewarmedbetweenthe examiners hands, not with water. The nurse should never listen through a gown. The diaphragm of the stethoscope should be used to auscultate for bowelsounds. DIF: Cognitive Level: Applying (Application) MSC: Client Needs: Safe and Effective Care Environment: Management of Care 16. Thenursewillusewhichtechniqueofassessmenttodeterminethepresenceofcrepitus,swelling,and pulsations? a. Palpation b. Inspection c. Percussion d. Auscultation ANS:A Palpationappliesthesenseoftouchtoassesstexture,temperature,moisture,organlocationandsize,aswellas anyswelling,vibrationorpulsation,rigidityorspasticity,crepitation,presenceoflumpsormasses,andthe presenceoftendernessorpain. NURSINGTB.COM DIF: Cognitive Level: Understanding (Comprehension) MSC: Client Needs: Safe and Effective Care Environment: Management of Care 17. Thenurseispreparingtouseanotoscopeforanexamination.Whichstatementistrueregardingthe otoscope? Theotoscope: a. Is often used to direct light onto thesinuses. b. Uses a short, broad speculum to help visualize theear. c. Is used to examine the structures of the internalear. d. Directs light into the ear canal and onto the tympanicmembrane. ANS: D Theotoscopedirectslightintotheearcanalandontothetympanicmembranethatdividestheexternaland middle ear. A short, broad speculum is used to visualize thenares. DIF: Cognitive Level: Remembering (Knowledge) MSC: Client Needs: Safe and Effective Care Environment: Management of Care 18. Anexaminerisusinganophthalmoscopetoexamineapatientseyes.Thepatienthasastigmatismandis nearsighted. The use of which of these techniques would indicate that the examination is being correctly performed? a. Usingthelargefullcircleoflightwhenassessingpupilsthatarenotdilated b. Rotatingthelensselectordialtotheblacknumberstocompensateforastigmatism c. Usingthegridonthelensaperturedialtovisualizetheexternalstructuresoftheeye d. Rotatingthelensselectordialtobringtheobjectintofocus ANS:D Theophthalmoscopeisusedtoexaminetheinternaleyestructures.Itcancompensatefornearsightednessor farsightedness,butitwillnotcorrectforastigmatism.Thegridisusedtoassesssizeandlocationoflesionson thefundus.Thelargefullspotoflightisusedtoassessdilatedpupils.Rotatingthelensselectordialbringsthe object intofocus. DIF: Cognitive Level: Analyzing (Analysis) MSC: Client Needs: Safe and Effective Care Environment: Management of Care 19. Thenurseisunabletopalpatetherightradialpulseonapatient.Thebestactionwouldbeto: a. AuscultateovertheareawithafetNosUcRopSeIN.GTB.COM b. Use a goniometer to measure thepulsations. c. Use a Doppler device to check for pulsations over thearea. d. Check for the presence of pulsations with astethoscope. ANS: C Dopplerdevicesareusedtoaugmentpulseorbloodpressuremeasurements.Goniometersmeasurejointrange ofmotion.Afetoscopeisusedtoauscultatefetalhearttones.Stethoscopesareusedtoauscultatebreath,bowel and heartsounds. DIF: Cognitive Level: Analyzing (Analysis) MSC: Client Needs: Safe and Effective Care Environment: Management of Care 20. Thenurseispreparingtoperformaphysicalassessment.Thecorrectactionbythenurseisreflectedby which statement? Thenurse: a. Performs the examination from the left side of thebed. b. Examinestenderorpainfulareasfirsttohelprelievethepatientsanxiety. c. Followsthesameexaminationsequence,regardlessofthepatientsageorcondition. d. Organizestheassessmenttoensurethatthepatientdoesnotchangepositionstoooften. ANS: D Thestepsoftheassessmentshouldbeorganizedtoensurethatthepatientdoesnotchangepositionstoooften. Thesequenceofthestepsoftheassessmentmaydiffer,dependingontheageofthepersonandtheexaminers preference. Tender or painful areas should be assessedlast. DIF: Cognitive Level: Applying (Application) MSC: Client Needs: Safe and Effective Care Environment: Management of Care 21. Amanisattheclinicforaphysicalexamination.Hestatesthatheisveryanxiousaboutthephysical examination.Whatstepscanthenursetaketomakehimmorecomfortable? a. Appear unhurried and confident when examininghim. b. Stay in the room when he undresses in case he needsassistance. c. Askhimtochangeintoanexamininggownandtotakeoffhisundergarments. d. Defermeasuringvitalsignsuntiltheendoftheexamination,whichallowshimtimetobecome comfortable. NURSINGTB.COM ANS: A Anxietycanbereducedbyanexaminerwhoisconfident,self-assured,considerate,andunhurried.Familiar andrelativelynonthreateningactions,suchasmeasuringthepersonsvitalsigns,willgraduallyaccustomthe person to theexamination. DIF: Cognitive Level: Applying (Application) MSC: Client Needs: Safe and Effective Care Environment: Management of Care 22. Whenperformingaphysicalexamination,safetymustbeconsideredtoprotecttheexaminerandthepatien against the spread of infection. Which of these statements describes the most appropriate action the nurse should take when performing a physicalexamination? a. Washingoneshandsafterremovingglovesisnotnecessary,aslongastheglovesarestillintact. b. Hands are washed before and after every physical patientencounter. c. Handsarewashedbeforetheexaminationofeachbodysystemtopreventthespreadofbacteria from one part of the body toanother. d. Glovesarewornthroughouttheentireexaminationtodemonstratetothepatientconcernregarding the spread of infectiousdiseases. ANS: B The nurse should wash his or her hands before and after every physical patient encounter; after contact with blood,bodyfluids,secretions,andexcretions;aftercontactwithanyequipmentcontaminatedwithbodyfluids; andafterremovinggloves.Handsshouldbewashedaftergloveshavebeenremoved,eveniftheglovesappear tobeintact.Glovesshouldbewornwhenpotentialcontactwithanybodyfluidsispresent. DIF: Cognitive Level: Applying (Application) MSC: Client Needs: Safe and Effective Care Environment: Safety and Infection Control 23. Thenurseisexaminingapatientslowerlegandnoticesadrainingulceration.Whichoftheseactionsis most appropriate in thissituation? a. Washing hands, and contacting thephysician b. Continuing to examine the ulceration, and then washinghands c. Washinghands,puttingongloves,andcontinuingwiththeexaminationoftheulceration d. Washinghands,proceedingwithrestofthephysicalexamination,andthencontinuingwiththe examination of the legulceration ANS: C Theexaminershouldweargloveswhenthepotentialcontactwithanybodyfluidsispresent.Inthissituation, thenurseshouldwashhisorherhands,putonNgUloRvSeIsN,GanTdBc.CoOntMinueexaminingtheulceration. DIF: Cognitive Level: Analyzing (Analysis) MSC: Client Needs: Safe and Effective Care Environment: Safety and Infection Control 24. Duringtheexamination,offeringsomebriefteachingaboutthepatientsbodyortheexaminersfindingsis oftenappropriate.Whichoneofthesestatementsbythenurseismostappropriate? a. Your atrial dysrhythmias are undercontrol. b. You have pitting edema and mildvaricosities. c. Yourpulseis80beatsperminute,whichiswithinthenormalrange. d. Imusingmystethoscopetolistenforanycrackles,wheezes,orrubs. ANS: C Thesharingofsomeinformationbuildsrapport,aslongasthepatientisabletounderstandtheterminology. DIF: Cognitive Level: Analyzing(Analysis) MSC: Client Needs: Safe and Effective Care Environment: Management of Care 25. Thenursekeepsinmindthatthemostimportantreasontoshareinformationandtoofferbriefteaching while performing the physical examination is to help the: a. Examiner feel more comfortable and to gain control of thesituation. b. Examinertobuildrapportandtoincreasethepatientsconfidenceinhimorher. c. Patient understand his or her disease process and treatmentmodalities. d. Patientidentifyquestionsabouthisorherdiseaseandthepotentialareasofpatienteducation. ANS: B Sharinginformationbuildsrapportandincreasesthepatientsconfidenceintheexaminer.Italsogivesthe patientalittlemorecontrolinasituationduringwhichfeelingcompletelyhelplessisoftenpresent. DIF: Cognitive Level: Understanding (Comprehension) MSC: Client Needs: Safe and Effective Care Environment: Management of Care 26. ThenurseisexamininganinfantandpreparestoelicittheMororeflexatwhichtimeduringthe examination? a. When the infant issleeping b. Attheendoftheexamination c. Beforeauscultationofthethorax d. Halfwaythroughtheexamination NURSINGTB.COM ANS: B TheMoroorstartlereflexiselicitedattheendoftheexaminationbecauseitmaycausetheinfanttocry. DIF: Cognitive Level: Understanding(Comprehension) MSC: Client Needs: Safe and Effective Care Environment: Management of Care 27. Whenpreparingtoperformaphysicalexaminationonaninfant,thenurseshould: a. Have the parent remove all clothing except the diaper on aboy. b. Instructtheparenttofeedtheinfantimmediatelybeforetheexamination. c. Encouragetheinfanttosuckonapacifierduringtheabdominalexamination. d. Asktheparenttoleavetheroombrieflywhenassessingtheinfantsvitalsigns. ANS: A Theparentshouldalwaysbepresenttoincreasethechildsfeelingofsecurityandtounderstandnormalgrowth anddevelopment.Thetimingoftheexaminationshouldbe1to2hoursafterfeedingwhenthebabyisneither too drowsy nor too hungry. Infants do not object to being nude; clothing should be removed, but a diaper should be left on aboy. DIF: Cognitive Level: Applying (Application) MSC: Client Needs: Safe and Effective Care Environment: Management of Care 28. A6-month-oldinfanthasbeenbroughttothewell-childclinicforacheck-up.Sheiscurrentlysleeping. What should the nurse do first when beginning theexamination? a. Auscultate the lungs and heart while the infant is stillsleeping. b. Examine the infants hips, because this procedure isuncomfortable. c. Beginwiththeassessmentoftheeye,andcontinuewiththeremainderoftheexaminationina head-to-toeapproach. d. Waketheinfantbeforebeginninganyportionoftheexaminationtoobtainthemostaccurate assessment of bodysystems. ANS: A WhentheinfantisquietorsleepingisanideaNltUimReSItNoGasTsBes.CsOthMecardiac,respiratory,andabdominalsystems. Assessmentoftheeye,ear,nose,andthroatareinvasiveproceduresthatshouldbeperformedattheendofthe examination. DIF: Cognitive Level: Applying (Application) MSC: Client Needs: Safe and Effective Care Environment: Management of Care 29. A2-year-oldchildhasbeenbroughttotheclinicforawell-childcheckup.Thebestwayforthenurseto begin the assessment isto: a. Ask the parent to place the child on the examiningtable. b. Havetheparentremoveallofthechildsclothingbeforetheexamination. c. Allowthechildtokeepasecurityobjectsuchasatoyorblanketduringtheexamination. d. Initiallyfocustheinteractionsonthechild,essentiallyignoringtheparentuntilthechildstrusthas beenobtained. ANS: C Thebestplacetoexaminethetoddlerisontheparentslap.Toddlersunderstandsymbols;therefore,asecurity object is helpful. Initially, the focus is more on the parent, which allows the child to adjust gradually and to become familiar with you. A 2-year-old child does not like to take off his or her clothes. Therefore, ask the parent to undress one body part at atime. DIF: Cognitive Level: Applying (Application) MSC: Client Needs: Safe and Effective Care Environment: Management of Care 30. Thenurseisexamininga2-year-oldchildandasks,MayIlistentoyourheartnow?Whichcritiqueofthe nurses technique is mostaccurate? a. Asking questions enhances the childsautonomy b. Asking the child for permission helps develop a sense oftrust c. Thisquestionisanappropriatestatementbecausechildrenatthisageliketohavechoices d. Childrenatthisageliketosay,No.Theexaminershouldnotofferachoicewhennochoiceis available ANS: D Childrenatthisageliketosay,No.Choicesshouldnotbeofferedwhennochoiceisreallyavailable.Ifthe childsays,Noandthenursedoesitanyway,thenthenurselosestrust.Autonomyisenhancedbyofferinga limited option, Shall I listen to your heart next or yourtummy? DIF: Cognitive Level: Analyzing (Analysis) MSC: Client Needs: Safe and Effective Care Environment: Management of Care 31. WithwhichofthesepatientswoulditbemNoUstRaSpINprGoTprBia.CteOfMorthenursetousegamesduringthe assessment,suchashavingthepatientblowoutthelightonthepenlight? a. Infant b. Preschoolchild c. School-agechild d. Adolescent ANS:B Whenassessingpreschoolchildren,usinggamesorallowingthemtoplaywiththeequipmenttoreducetheir fearscanbehelpful.Suchgamesarenotappropriatefortheotheragegroups. DIF: Cognitive Level: Understanding (Comprehension) MSC: Client Needs: Safe and Effective Care Environment: Management of Care 32. Thenurseispreparingtoexaminea4-year-oldchild.Whichactionisappropriateforthisagegroup? a. Explain the procedures in detail to alleviate the childsanxiety. b. Give the child feedback and reassurance during theexamination. c. Donotaskthechildtoremovehisorherclothesbecausechildrenatthisageareusuallyvery private. d. Performanexaminationoftheear,nose,andthroatfirst,andthenexaminethethoraxand abdomen. ANS: B Withpreschoolchildren,short,simpleexplanationsshouldbeused.Childrenatthisageareusuallywillingto undress.Anexaminationoftheheadshouldbeperformedlast.Duringtheexamination,neededfeedbackand reassurance should be given to thepreschooler. DIF: Cognitive Level: Understanding (Comprehension) MSC: Client Needs: Safe and Effective Care Environment: Management of Care 33. When examining a 16-year-old male teenager, the nurseshould: a. Discusshealthteachingwiththeparentbecausetheteenisunlikelytobeinterestedinpromoting wellness. b. Askhisparenttostayintheroomduringthehistoryandphysicalexaminationtoanswerany questions and to alleviate hisanxiety. c. TalktohimthesamemannerasonNeUwRoSuINldGtaTlBk.CtoOaMyoungerchildbecauseateenslevelof understanding may not match his or herspeech. d. Providefeedbackthathisbodyisdevelopingnormally,anddiscussthewidevariationamong teenagers on the rate of growth anddevelopment. ANS: D During the examination, the adolescent needs feedback that his or her body is healthy and developing normally.Theadolescenthasakeenawarenessofbodyimageandoftencompareshimorherselfwithpeers. Apprisetheadolescentofthewidevariationamongteenagersontherateofgrowthanddevelopment. DIF: Cognitive Level: Applying (Application) MSC:ClientNeeds:SafeandEffectiveCareEnvironment:ManagementofCare 34. Whenexamininganolderadult,thenurseshouldusewhichtechnique? a. Avoid touching the patient toomuch. b. Attempt to perform the entire physical examination during onevisit. c. Speakloudlyandslowlybecausemostagingadultshavehearingdeficits. d. Arrangethesequenceoftheexaminationtoallowasfewpositionchangesaspossible. ANS: D Whenexaminingtheolderadult,arrangingthesequenceoftheexaminationtoallowasfewpositionchanges aspossibleisbest.Physicaltouchisespeciallyimportantwiththeolderpersonbecauseothersensesmaybe diminished. DIF: Cognitive Level: Applying (Application) MSC: Client Needs: Safe and Effective Care Environment: Management of Care 35. Themostimportantstepthatthenursecantaketopreventthetransmissionofmicroorganismsinthe hospital setting isto: a. Wear protective eye wear at alltimes. b. Wear gloves during any and all contact withpatients. c. Wash hands before and after contact with eachpatient. d. Clean the stethoscope with an alcohol swab betweenpatients. ANS: C Themostimportantsteptodecreasetheriskofmicroorganismtransmissionistowashhandspromptlyand thoroughlybeforeandafterphysicalcontactwitheachpatient.Stethoscopesshouldalsobecleansedwithan alcoholswabbeforeandaftereachpatientcontact.Thebestroutineistocombinestethoscoperubbingwith handhygieneeachtimehandhygieneisperfoNrmUeRdS.INGTB.COM DIF: Cognitive Level: Understanding (Comprehension) MSC: Client Needs: Safe and Effective Care Environment: Safety and Infection Control 36. WhichofthesestatementsistrueregardingtheuseofStandardPrecautionsinthehealthcaresetting? a. Standard Precautions apply to all body fluids, includingsweat. b. Use alcohol-based hand rub if hands are visiblydirty. c. StandardPrecautionsareintendedforusewithallpatients,regardlessoftheirriskorpresumed infectionstatus. d. StandardPrecautionsaretobeusedonlywhennonintactskin,excretionscontainingvisibleblood, or expected contact with mucous membranes ispresent. ANS: C StandardPrecautionsaredesignedtoreducetheriskoftransmissionofmicroorganismsfrombothrecognized and unrecognized sources and are intended for use for all patients, regardless of their risk or presumed infectionstatus.StandardPrecautionsapplytobloodandallotherbodyfluids,secretionsandexcretionsexcept sweatregardlessofwhethertheycontainvisibleblood,nonintactskin,ormucousmembranes.Handsshouldbe washedwithsoapandwaterifvisiblysoiledwithbloodorbodyfluids.Alcohol-basedhandrubscanbeusedif hands are not visiblysoiled. DIF: Cognitive Level: Understanding (Comprehension) MSC: Client Needs: Safe and Effective Care Environment: Safety and Infection Control 37. Thenurseispreparingtoassessahospitalizedpatientwhoisexperiencingsignificantshortnessofbreath. How should the nurse proceed with theassessment? a. Thepatientshouldliedowntoobtainanaccuratecardiac,respiratory,andabdominalassessment. b. Athoroughhistoryandphysicalassessmentinformationshouldbeobtainedfromthepatients familymember. c. Acompletehistoryandphysicalassessmentshouldbeimmediatelyperformedtoobtainbaseline information. d. Bodyareasappropriatetotheproblemshouldbeexaminedandthentheassessmentcompleted after the problem hasresolved. ANS: D Both altering the position of the patient during the examination and collecting a mini database by examining thebodyareasappropriatetotheproblemmaybenecessaryinthissituation.Anassessmentmaybecompleted later after the distress isresolved. DIF: Cognitive Level: Applying (Application) MSC:ClientNeeds:SafeandEffectiveCareENnUvRirSoInNmGeTnBt:.CMOaMnagementofCare 38. When examining an infant, the nurse should examine which areafirst? a. Ear b. Nose c. Throat d. Abdomen ANS:D Theleast-distressingstepsareperformedfirst,savingtheinvasivestepsoftheexaminationoftheeye,ear, nose, and throat untillast. DIF: Cognitive Level: Understanding (Comprehension) MSC: Client Needs: Safe and Effective Care Environment: Management of Care 39. Whileauscultatingheartsounds,thenursehearsamurmur.Whichoftheseinstrumentsshouldbeusedto assess thismurmur? a. Electrocardiogram b. Bell of thestethoscope c. Diaphragm of thestethoscope d. Palpationwiththenursespalmofthehand ANS:B Thebellofthestethoscopeisbestforsoft,low-pitchedsoundssuchasextraheartsoundsormurmurs.The diaphragmofthestethoscopeisbestusedforhigh-pitchedsoundssuchasbreath,bowel,andnormalheart sounds. DIF: Cognitive Level: Applying (Application) MSC: Client Needs: Safe and Effective Care Environment: Management of Care 40. Duringanexaminationofapatientsabdomen,thenursenotesthattheabdomenisroundedandfirmtothe touch.Duringpercussion,thenursenotesadrumlikequalityofthesoundsacrossthequadrants.Thistypeof soundindicates: a. Constipation. b. Air-filledareas. c. Presence of atumor. d. Presence of denseorgans. NURSINGTB.COM ANS: B Amusicalordrumlikesound(tympany)isheardwhenpercussionoccursoveranair-filledviscus,suchasthe stomach orintestines. DIF: Cognitive Level: Analyzing (Analysis) MSC: Client Needs: Safe and Effective Care Environment: Management of Care 41. Thenurseispreparingtoexaminea6-year-oldchild.Whichactionismostappropriate? a. The thorax, abdomen, and genitalia are examined before thehead. b. Talkingabouttheequipmentbeingusedisavoidedbecausedoingsomayincreasethechilds anxiety. c. Thenurseshouldkeepinmindthatachildatthisagewillhaveasenseofmodesty. d. The child is asked to undress from the waistup. ANS: C A6-year-oldchildhasasenseofmodesty.Thechildshouldundresshimorherself,leavingunderpantsonand usingagownordrape.Aschool-agechildiscurioustoknowhowequipmentworks,andthesequenceshould progress from the childs head to thetoes. DIF: Cognitive Level: Analyzing (Analysis) MSC: Client Needs: Safe and Effective Care Environment: Management of Care 42. Duringauscultationofapatientsheartsounds,thenursehearsanunfamiliarsound.Thenurseshould: a. Document the findings in the patientsrecord. b. Wait 10 minutes, and auscultate the soundagain. c. Ask the patient how he or she isfeeling. d. Ask another nurse to double check thefinding. ANS: D Ifanabnormalfindingisnotfamiliar,thenthenursemayaskanotherexaminertodoublecheckthefinding. The other responses do not help identify the unfamiliarsound. DIF: Cognitive Level: Analyzing (Analysis) MSC: Client Needs: Safe and Effective Care Environment: Management of Care NURSINGTB.COM MULTIPLE RESPONSE 1.Thenurseispreparingtopalpatethethoraxandabdomenofapatient.Whichofthesestatementsdescribes the correct technique for this procedure? Select all thatapply. a. Warm the hands first before touching thepatient. b. Fordeeppalpation,useonelongcontinuouspalpationwhenassessingtheliver. c. Start with light palpation to detect surfacecharacteristics. d. Usethefingertipstoexamineskintexture,swelling,pulsation,andpresenceoflumps. e. Identify any tender areas, and palpate themlast. f. Use the palms of the hands to assess temperature of theskin. ANS: A, C, D, E The hands should always be warmed before beginning palpation. Intermittent pressure rather than one long continuouspalpationisused;anytenderareasareidentifiedandpalpatedlast.Fingertipsareusedtoexamine skintexture,swelling,pulsation,andthepresenceoflumps.Thedorsa(backs)ofthehandsareusedtoassess skintemperaturebecausetheskinonthedorsaisthinnerthanonthepalms. DIF: Cognitive Level: Applying (Application) MSC: Client Needs: Safe and Effective Care Environment: Management of Care NURSINGTB.COM Chapter 09: General Survey and Measurement MULTIPLE CHOICE 1. Thenurseisperformingageneralsurvey.Whichactionisacomponentofthegeneralsurvey? a. Observing the patients body stature and nutritionalstatus b. Interpreting the subjective information the patient hasreported c. Measuring the patients temperature, pulse, respirations, and bloodpressure d. Observingspecificbodysystemswhileperformingthephysicalassessment ANS:A Thegeneralsurveyisastudyofthewholepersonthatincludesobservingthepatientsphysicalappearance, body structure, mobility, andbehavior. DIF: Cognitive Level: Understanding (Comprehension) MSC: Client Needs: Safe and Effective Care Environment: Management of Care 2. Whenmeasuringapatientsweight,thenurseisawareofwhichoftheseguidelines? a. ThepatientisalwaysweighedweaNriUnRgSoInNlyGThBis.CorOhMerundergarments. b. Thetypeofscaledoesnotmatter,aslongastheweightsaresimilarfromdaytoday. c. Thepatientmayleaveonhisorherjacketandshoesaslongasthesearedocumentednexttothe weight. d. Attemptsshouldbemadetoweighthepatientatapproximatelythesametimeofday,ifasequence of weights isnecessary. ANS: D Astandardizedbalancescaleisusedtomeasureweight.Thepatientshouldremovehisorhershoesandheavy outer clothing. If a sequence of repeated weights is necessary, then the nurse should attempt to weigh the patientatapproximatelythesametimeofdayandwiththesametypesofclothingworneachtime. DIF: Cognitive Level: Understanding (Comprehension) MSC: Client Needs: Safe and Effective Care Environment: Management of Care 3. Duringanexaminationofachild,thenurseconsidersthatphysicalgrowthisthebestindexofachilds: a. Generalhealth. b. Geneticmakeup. c. Nutritionalstatus. d. Activity and exercisepatterns. ANS: A Physicalgrowthisthebestindexofachildsgeneralhealth;recordingthechildsheightandweighthelps determine normal growthpatterns. DIF: Cognitive Level: Remembering (Knowledge) MSC:ClientNeeds:HealthPromotionandMaintenance 4. A1-month-oldinfanthasaheadmeasurementof34cmandhasachestcircumferenceof32cm.Basedon the interpretation of these findings, the nursewould: a. Refer the infant to a physician for furtherevaluation. b. Consider these findings normal for a 1-month-oldinfant. c. Expectthechestcircumferencetobegreaterthantheheadcircumference. d. Asktheparenttoreturnin2weekstore-evaluatetheheadandchestcircumferences. ANS: B NURSINGTB.COM The newborns head measures approximately 32 to 38 cm and is approximately 2 cm larger than the chest circumference.Between6monthsand2years,bothmeasurementsareapproximatelythesame,andafterage2 years, the chest circumference is greater than the headcircumference. DIF: Cognitive Level: Analyzing (Analysis) MSC: Client Needs: Health Promotion and Maintenance 5. Thenurseisassessingan80-year-oldmalepatient.Whichassessmentfindingswouldbeconsiderednormal? a. Increase in body weight from his youngeryears b. Additionaldepositsoffatonthethighsandlowerlegs c. Presenceofkyphosisandflexioninthekneesandhips d. Changeinoverallbodyproportion,includingalongertrunkandshorterextremities ANS:C Changes that occur in the aging person include more prominent bony landmarks, decreased body weight (especiallyinmen),adecreaseinsubcutaneousfatfromthefaceandperiphery,andadditionalfatdepositedon theabdomenandhips.Posturalchangesofkyphosisandslightflexioninthekneesandhipsalsooccur. DIF: Cognitive Level: Applying (Application) MSC: Client Needs: Health Promotion and Maintenance 6. The nurse should measure rectal temperatures in which of thesepatients? a. School-agechild b. Olderadult c. Comatoseadult d. Patientreceivingoxygenbynasalcannula ANS:C Rectaltemperaturesshouldbetakenwhentheotherroutesareimpractical,suchasforcomatoseorconfused persons,forthoseinshock,orforthosewhocannotclosethemouthbecauseofbreathingoroxygentubes,a wired mandible, or other facialdysfunctions. DIF: Cognitive Level: Applying (Application) MSC: Client Needs: Safe and Effective Care Environment: Management of Care 7. Thenurseispreparingtomeasurethelength,weight,chest,andheadcircumferenceofa6-month-oldinfant. Which measurement technique iscorrect? NURSINGTB.COM a. Measuring the infants length by using a tapemeasure b. Weighingtheinfantbyplacinghimorheronanelectronicstandingscale c. Measuringthechestcircumferenceatthenipplelinewithatapemeasure d. Measuringtheheadcircumferencebywrappingthetapemeasureoverthenoseandcheekbones ANS:C Tomeasurethechestcircumference,thetapeisencircledaroundthechestatthenippleline.Thelengthshould bemeasuredonahorizontalmeasuringboard.Weightshouldbemeasuredonaplatform-typebalancescale. Headcircumferenceismeasuredwiththetapearoundthehead,alignedattheeyebrows,andattheprominent frontal and occipital bones - the widest span iscorrect. DIF: Cognitive Level: Applying (Application) MSC: Client Needs: Safe and Effective Care Environment: Management of Care 8. Thenurseknowsthatoneadvantageofthetympanicmembranethermometer(TMT)isthat: a. Rapid measurement is useful for uncooperative youngerchildren. b. UsingtheTMTisthemostaccuratemethodformeasuringbodytemperatureinnewborninfants. c. Measuring temperature using the TMT isinexpensive. d. StudiesstronglysupporttheuseoftheTMTinchildrenundertheage6years. ANS: A TheTMTisusefulforyoungchildrenwhomaynotcooperatefororaltemperaturesandfearrectal temperatures.However,theuseaTMTwithnewborninfantsandyoungchildrenisconflicting. DIF: Cognitive Level: Understanding (Comprehension) MSC: Client Needs: Safe and Effective Care Environment: Management of Care 9. Thenurseisexaminingapatientwhoiscomplainingoffeelingcold.Whichisamechanismofheatlossin thebody? a. Exercise b. Radiation c. Metabolism d. Fooddigestion ANS: B NURSINGTB.COM Thebodymaintainsasteadytemperaturethroughathermostatorfeedbackmechanism,whichisregulatedin thehypothalamusofthebrain.Thehypothalamusregulatesheatproductionfrommetabolism,exercise,food digestion, and external factors with heat loss through radiation, evaporation of sweat, convection, and conduction. DIF:CognitiveLevel:Remembering(Knowledge) MSC: Client Needs:General 10. Whenmeasuringapatientsbodytemperature,thenursekeepsinmindthatbodytemperatureisinfluenced by: a. Constipation. b. Patients emotionalstate. c. Diurnalcycle. d. Nocturnalcycle. ANS: C Normaltemperatureisinfluencedbythediurnalcycle,exercise,andage.Theotherresponsesdonotinfluence body temperature. DIF:CognitiveLevel:Remembering(Knowledge) MSC: Client Needs:General 11. Whenevaluatingthetemperatureofolderadults,thenurseshouldrememberwhichaspectaboutanolder adults bodytemperature? a. Thebodytemperatureoftheolderadultislowerthanthatofayoungeradult. b. Anolderadultsbodytemperatureisapproximatelythesameasthatofayoungchild. c. Body temperature depends on the type of thermometerused. d. Intheolderadult,thebodytemperaturevarieswidelybecauseoflesseffectiveheatcontrol mechanisms. ANS: A In older adults, the body temperature is usually lower than in other age groups, with a mean temperature of 36.2 C. DIF: Cognitive Level: Remembering (Knowledge) MSC:ClientNeeds:HealthPromotionandMaintenance NURSINGTB.COM 12. A60-year-oldmalepatienthasbeentreatedforpneumoniaforthepast6weeks.Heisseentodayinthe clinicforanunexplainedweightlossof10poundsoverthelast6weeks.Thenurseknowsthat: a. Weight loss is probably the result of unhealthy eatinghabits. b. Chronic diseases such as hypertension cause weightloss. c. Unexplainedweightlossoftenaccompaniesshort-termillnesses. d. Weightlossisprobablytheresultofamentalhealthdysfunction. ANS: C Anunexplainedweightlossmaybeasignofashort-termillnessorachronicillnesssuchasendocrinedisease, malignancy, depression, anorexia nervosa, orbulimia. DIF: Cognitive Level: Analyzing (Analysis) MSC: Client Needs: Physiologic Integrity: Basic Care and Comfort 13. Whenassessinga75-year-oldpatientwhohasasthma,thenursenotesthatheassumesatripodposition, leaningforwardwitharmsbracedonthechair.Onthebasisofthisobservation,thenurseshould: a. Assumethatthepatientiseagerandinterestedinparticipatingintheinterview. b. Evaluatethepatientforabdominalpain,whichmaybeexacerbatedinthesittingposition. c. Assumethatthepatientishavingdifficultybreathingandassisthimtoasupineposition. d. Recognizethatatripodpositionisoftenusedwhenapatientishavingrespiratorydifficulties. ANS: D Assumingatripodpositionleaningforwardwitharmsbracedonchairarmsoccurswithchronicpulmonary disease. The other actions or assumptions are notcorrect. DIF: Cognitive Level: Analyzing (Analysis) MSC: Client Needs: Physiologic Integrity: Basic Care and Comfort 14. Whichoftheseactionsillustratesthecorrecttechniquethenurseshouldusewhenassessingoral temperature with a mercurythermometer? a. Wait 30 minutes if the patient has ingested hot or icedliquids. b. Leavethethermometerinplace3to4minutesifthepatientisafebrile. c. Placethethermometerinfrontofthetongue,andaskthepatienttoclosehisorherlips. d. Shakethemercury-in-glassthermoNmUeRteSrINdoGwTnB.tCoObMelow36.6Cbeforetakingthetemperature. ANS: B Thethermometershouldbeleftinplace3to4minutesifthepersonisafebrileandupto8minutesifthe personisfebrile.Thenurseshouldwait15minutesifthepersonhasjustingestedhotoricedliquidsand2 minutes if he or she has justsmoked. DIF: Cognitive Level: Applying (Application) MSC: Client Needs: Safe and Effective Care Environment: Management of Care 15. ThenurseistakingtemperaturesinaclinicwithaTMT.Whichstatementistrueregardinguseofthe TMT? a. Atympanictemperatureismoretimeconsumingthanarectaltemperature. b. Thetympanicmethodismoreinvasiveanduncomfortablethantheoralmethod. c. Theriskofcross-contaminationisreduced,comparedwiththerectalroute. d. Thetympanicmembranemostaccuratelyreflectsthetemperatureintheophthalmicartery. ANS: C The TMT is a noninvasive, nontraumatic device that is extremely quick and efficient. The chance of cross- contaminationwiththeTMTisminimalbecausetheearcanalislinedwithskin,notmucousmembranes. DIF: Cognitive Level: Understanding(Comprehension) MSC: Client Needs: Safe and Effective Care Environment: Management of Care 16. To assess a rectal temperature accurately in an adult, the nursewould: a. Use a lubricated blunt tipthermometer. b. Insert the thermometer 2 to 3 inches into therectum. c. Leavethethermometerinplaceupto8minutesifthepatientisfebrile. d. Wait2to3minutesifthepatienthasrecentlysmokedacigarette. ANS: A Alubricatedrectalthermometer(withashort,blunttip)isinsertedonly2to3cm(1inch)intotheadultrectum andleftinplacefor2minutes.Cigarettesmokingdoesnotalterrectaltemperatures. DIF: Cognitive Level: Understanding (Comprehension) MSC: Client Needs: Safe and Effective Care Environment: Management of Care 17. Thenurseisperformingageneralsurveyofapatient.Whichfindingisconsiderednormal? NURSINGTB.COM a. When standing, the patients base isnarrow. b. The patient appears older than his statedage. c. Arm span (fingertip to fingertip) is greater than theheight. d. Arm span (fingertip to fingertip) equals the patientsheight. ANS: D When performing the general survey, the patients arm span (fingertip to fingertip) should equal the patients height.AnarmspanthatisgreaterthanthepersonsheightmayindicateMarfansyndrome.Thebaseshouldbe wide when the patient is standing, and an older appearance than the stated age may indicate a history of a chronic illness or chronicalcoholism. DIF: Cognitive Level: Analyzing (Analysis) MSC: Client Needs: Physiologic Integrity Chapter 10: Vital Signs MULTIPLE CHOICE 1. Apatientsweeklybloodpressurereadingsfor2monthshaverangedbetween124/84mmHgand136/88 mm Hg, with an average reading of 126/86 mm Hg. The nurse knows that this blood pressure falls within which blood pressurecategory? a. Normal bloodpressure b. Prehypertension c. Stage 1hypertension d. Stage2hypertension ANS:B AccordingtotheSeventhReportoftheJointNationalCommittee(JNC7)guidelines,prehypertensionblood pressurereadingsaresystolicreadingsof120to139mmHgordiastolicreadingsof50to89mmHg. DIF: Cognitive Level: Understanding (Comprehension) MSC: Client Needs: Safe and Effective Care Environment: Management of Care 2. Whenassessinganolderadult,whichvitalNsiUgnRSchINanGgTeBs.oCcOcMurwithaging? a. Increase in pulserate b. Widened pulsepressure c. Increase in bodytemperature d. Decreaseindiastolicbloodpressure ANS:B Withaging,thenursekeepsinmindthatthesystolicbloodpressureincreases,leadingtowidenedpulse pressure.Withmanyolderpeople,boththesystolicanddiastolicpressuresincrease.Thepulserateand temperature do notincrease. DIF:CognitiveLevel:Understanding(Comprehension) MSC:ClientNeeds:HealthPromotionandMaintenance 3. Whichtechniqueiscorrectwhenthenurseisassessingtheradialpulseofapatient? The pulse is counted for: a. 1 minute, if the rhythm isirregular. b. 15 seconds and then multiplied by 4, if the rhythm isregular. c. 2 full minutes to detect any variation inamplitude. d. 10secondsandthenmultipliedby6,ifthepatienthasnohistoryofcardiacabnormalities. ANS: A Recent research suggests that the 30-second interval multiplied by 2 is the most accurate and efficient techniquewhenheartratesarenormalorrapidandwhenrhythmsareregular.Iftherhythmisirregular,then the pulse is counted for 1 fullminute. DIF: Cognitive Level: Applying (Application) MSC: Client Needs: Safe and Effective Care Environment: Management of Care 4. Whenassessingapatientspulse,thenurseshouldalsonoticewhichofthesecharacteristics? a. Force b. Pallor c. Capillary refilltime d. Timinginthecardiaccycle NURSINGTB.COM ANS: A The pulse is assessed for rate, rhythm, and force. DIF: Cognitive Level: Understanding (Comprehension) MSC: Client Needs: Safe and Effective Care Environment: Management of Care 5. Whenassessingthepulseofa6-year-oldboy,thenursenoticesthathisheartratevarieswithhisrespiratory cycle, speeding up at the peak of inspiration and slowing to normal with expiration. The nurses next action would beto: a. Immediately notify thephysician. b. Consider this finding normal in children and youngadults. c. Checkthechildsbloodpressure,andnoteanyvariationwithrespiration. d. Documentthatthischildhasbradycardia,andcontinuewiththeassessment. ANS: B Sinusarrhythmiaiscommonlyfoundinchildrenandyoungadults.Duringtherespiratorycycle,theheartrate varies,speedingupatthepeakofinspirationandslowingtonormalwithexpiration. DIF: Cognitive Level: Analyzing (Analysis) MSC: Client Needs: Health Promotion and Maintenance 6. Whenassessingtheforce,orstrength,ofapulse,thenurserecallsthatthepulse: a. Is usually recorded on a 0- to 2-pointscale. b. Demonstrateselasticityofthevesselwall. c. Isareflectionoftheheartsstrokevolume. d. Reflects the blood volume in the arteries duringdiastole. ANS: C Theheartpumpsanamountofblood(thestrokevolume)intotheaorta.Theforceflaresthearterialwallsand generatesapressurewave,whichisfeltintheperipheryasthepulse. DIF:CognitiveLevel:Remembering(Knowledge) MSC: Client Needs:General 7. Thenurseisassessingthevitalsignsofa20-year-oldmalemarathonrunneranddocumentsthefollowing vital signs: temperature36 C; pulse48 beats per minute; respirations14 breaths per minute; blood pressure104/68 mm Hg. Which statement is true concerning theseresults? NURSINGTB.COM a. Thepatientisexperiencingtachycardia. b. These are normal vital signs for a healthy, athleticadult. c. The patients pulse rate is not normalhis physician should benotified. d. Onthebasisofthesereadings,thepatientshouldreturntotheclinicin1week. ANS: B Intheadult,aheartratelessthan50beatsperminuteiscalledbradycardia,whichnormallyoccursinthewell- trainedathletewhoseheartmuscledevelopsalongwiththeskeletalmuscles. DIF: Cognitive Level: Analyzing (Analysis) MSC: Client Needs: Health Promotion and Maintenance 8. Thenurseisassessingthevitalsignsofa3-year-oldpatientwhoappearstohaveanirregularrespiratory pattern. How should the nurse assess this childsrespirations? a. Respirationsshouldbecountedfor1fullminute,noticingrateandrhythm. b. Childspulseandrespirationsshouldbesimultaneouslycheckedfor30seconds. c. Childsrespirationsshouldbecheckedforaminimumof5minutestoidentifyanyvariationsinhis or her respiratorypattern. d. Patientsrespirationsshouldbecountedfor15secondsandthenmultipliedby4toobtainthe number of respirations perminute. ANS:A Respirationsarecountedfor1fullminuteifanabnormalityissuspected.Theotherresponsesarenotcorrect actions. DIF: Cognitive Level: Applying (Application) MSC: Client Needs: Health Promotion and Maintenance 9. Apatientsbloodpressureis118/82mmHg.Heasksthenurse,Whatdothenumbersmean?Thenursesbest replyis: a. Thenumbersarewithinthenormalrangeandarenothingtoworryabout. b. Thebottomnumberisthediastolicpressureandreflectsthestrokevolumeoftheheart. c. Thetopnumberisthesystolicbloodpressureandreflectsthepressureofthebloodagainstthe arteries when the heartcontracts. d. TheconceptofbloodpressureisdNifUficRuSlItNtoGuTnBd.CerOstMand.Theprimarythingtobeconcernedabout is the top number, or the systolic bloodpressure. ANS: C Thesystolicpressureisthemaximumpressurefeltonthearteryduringleftventricularcontraction,orsystole. Thediastolicpressureistheelasticrecoil,orresting,pressurethatthebloodconstantlyexertsinbetweeneach contraction.Thenurseshouldanswerthepatientsquestionandusetermshecanunderstand. DIF: Cognitive Level: Analyzing (Analysis) MSC: Client Needs: Safe and Effective Care Environment: Management of Care 10. Whilemeasuringapatientsbloodpressure,thenurserecallsthatcertainfactors,suchas , help determine bloodpressure. a. Pulserate b. Pulsepressure c. Vascularoutput d. Peripheralvascularresistance ANS:D Thelevelofbloodpressureisdeterminedbyfivefactors:cardiacoutput,peripheralvascularresistance, volumeofcirculatingblood,viscosity,andelasticityofthevesselwalls. DIF:CognitiveLevel:Remembering(Knowledge) MSC: Client Needs:General 11. Anurseishelpingatahealthfairatalocalmall.Whentakingbloodpressuresonavarietyofpeople,the nurse keeps in mindthat: a. Aftermenopause,bloodpressurereadingsinwomenareusuallylowerthanthosetakeninmen. b. ThebloodpressureofaBlackadultisusuallyhigherthanthatofaWhiteadultofthesameage. c. Bloodpressuremeasurementsinpeoplewhoareoverweightshouldbethesameasthoseofpeople who are at a normalweight. d. Ateenagersbloodpressurereadingwillbelowerthanthatofanadult. ANS: B IntheUnitedStates,aBlackadultsbloodpressureisusuallyhigherthanthatofaWhiteadultofthesameage. TheincidenceofhypertensionistwiceashighinBlacksasitisinWhites.Aftermenopause,bloodpressurein womenishigherthaninmen;bloodpressuremeasurementsinpeoplewhoareobeseareusuallyhigherthanin thosewhoarenotoverweight.Normally,agradualriseoccursthroughchildhoodandintotheadultyears. DIF: Cognitive Level: Applying (Application)NURSINGTB.COM MSC: Client Needs: Physiologic Integrity 12. Thenursenoticesacolleagueispreparingtocheckthebloodpressureofapatientwhoisobesebyusinga standard-sized blood pressure cuff. The nurse should expect the readingto: a. Yield a falsely low bloodpressure. b. Yield a falsely high bloodpressure. c. Be the same, regardless of cuffsize. d. Varyasaresultofthetechniqueofthepersonperformingtheassessment. ANS: B Usingacuffthatistoonarrowyieldsafalselyhighbloodpressurebecauseittakesextrapressuretocompress theartery. DIF: Cognitive Level: Applying (Application) MSC: Client Needs: Safe and Effective Care Environment: Management of Care 13. Astudentislateforhisappointmentandhasrushedacrosscampustothehealthclinic.Thenurseshould: a. Allow5minutesforhimtorelaxandrestbeforecheckinghisvitalsigns. b. Checkthebloodpressureinbotharms,expectingadifferenceinthereadingsbecauseofhisrecent exercise. c. Immediatelymonitorhisvitalsignsonhisarrivalattheclinicandthen5minuteslater,recording anydifferences. d. Checkhisbloodpressureinthesupineposition,whichwillprovideamoreaccuratereadingand will allow him to relax at the sametime. ANS: A Acomfortable,relaxedpersonyieldsavalidbloodpressure.Manypeopleareanxiousatthebeginningofan examination;thenurseshouldallowatleasta5-minuterestperiodbeforemeasuringbloodpressure. DIF: Cognitive Level: Analyzing (Analysis) MSC: Client Needs: Safe and Effective Care Environment: Management of Care 14. Thenursewillperformapalpatedpressurebeforeauscultatingbloodpressure.Thereasonforthisisto: a. More clearly hear the Korotkoffsounds. b. Detect the presence of an auscultatorygap. NURSINGTB.COM c. Avoid missing a falsely elevated bloodpressure. d. More readily identify phase IV of the Korotkoffsounds. ANS: B Inflationofthecuff20to30mmHgbeyondthepointatwhichapalpatedpulsedisappearswillavoidmissing anauscultatorygap,whichisaperiodwhentheKorotkoffsoundsdisappearduringauscultation. DIF: Cognitive Level: Understanding (Comprehension) MSC: Client Needs: Safe and Effective Care Environment: Management of Care 15. Thenurseistakinganinitialbloodpressurereadingona72-year-oldpatientwithdocumented hypertension. How should the nurseproceed? a. Cuffshouldbeplacedonthepatientsarmandinflated30mmHgabovethepatientspulserate. b. Cuffshouldbeinflatedto200mmHginanattempttoobtainthemostaccuratesystolicreading. c. Cuffshouldbeinflated30mmHgabovethepointatwhichthepalpatedpulsedisappears. d. Afterconfirmingthepatientspreviousbloodpressurereadings,thecuffshouldbeinflated30mm Hg above the highest systolic readingrecorded. ANS: C An auscultatory gap occurs in approximately 5% of the people, most often in those with hypertension. To checkforthepresenceofanauscultatorygap,thecuffshouldbeinflated20to30mmHgbeyondthepointat which the palpated pulsedisappears. DIF: Cognitive Level: Analyzing (Analysis) MSC: Client Needs: Safe and Effective Care Environment: Management of Care 16. The nurse has collected the following information on a patient: palpated blood pressure180 mm Hg; auscultatedbloodpressure170/100mmHg;apicalpulse60beatsperminute;radialpulse70beatsperminute. What is the patients pulsepressure? a. 10 b. 70 c. 80 d. ANS: B 100 PulsepressureisthedifferencebetweensystoNlicUaRnSdINdGiaTstBo.lCicObMloodpressure(170100=70)andreflectsthe stroke volume. DIF:CognitiveLevel:Applying(Application) MSC: Client Needs: PhysiologicIntegrity 17. Whenauscultatingthebloodpressureofa25-year-oldpatient,thenursenoticesthephaseIKorotkoff soundsbeginat200mmHg.At100mmHg,theKorotkoffsoundsmuffle.At92mmHg,theKorotkoff soundsdisappear.Howshouldthenurserecordthispatientsbloodpressure? a. 200/92 b. 200/100 c. 100/200/92 d. ANS: A 200/100/92 Inadults,thelastaudiblesoundbestindicatesthediastolicpressure.Whenavarianceisgreaterthan10to12 mm Hg between phases IV and V, both phases should be recorded along with the systolic reading (e.g., 142/98/80). DIF: Cognitive Level: Analyzing (Analysis) MSC: Client Needs: Safe and Effective Care Environment: Management of Care 18. Apatientisseenintheclinicforcomplaintsoffaintingepisodesthatstartedlastweek.Howshouldthe nurse proceed with theexamination? a. Blood pressure readings are taken in both the arms and thethighs. b. Thepatientisassistedtoalyingposition,andhisbloodpressureistaken. c. Hisbloodpressureisrecordedinthelying,sitting,andstandingpositions. d. Hisbloodpressureisrecordedinthelyingandsittingpositions;thesenumbersarethenaveraged to obtain a mean bloodpressure. ANS: C Ifthepersonisknowntohavehypertension,istakingantihypertensivemedications,orreportsahistoryof faintingorsyncope,thenthebloodpressurereadingshouldbetakeninthreepositions:lying,sitting,and standing. DIF: Cognitive Level: Applying (Application) MSC: Client Needs: Safe and Effective Care Environment: Management of Care 19. A70-year-oldmanhasabloodpressureof150/90mmHginalyingposition,130/80mmHginasitting position,and100/60mmHginastandingposition.Howshouldthenurseevaluatethesefindings? NURSINGTB.COM a. Thesereadingsareanormalresponseandattributabletochangesinthepatientsposition. b. Thechangeinbloodpressurereadingsiscalledorthostatichypotension. c. Thebloodpressurereadinginthelyingpositioniswithinnormallimits. d. Thechangeinbloodpressurereadingsisconsideredwithinnormallimitsforthepatientsage. ANS: B Orthostatichypotensionisadropinsystolicpressureofmorethan20mmHg,whichoccurswithaquick changetoastandingposition.Agingpeoplehavethegreatestriskofthisproblem. DIF: Cognitive Level: Analyzing (Analysis) MSC: Client Needs: Health Promotion and Maintenance 20. Thenurseishelpinganothernursetotakeabloodpressurereadingonapatientsthigh.Whichactionis correct regarding thigh pressure? a. Eitherthepoplitealorfemoralvesselsshouldbeauscultatedtoobtainathighpressure. b. Thebestpositiontomeasurethighpressureisthesupinepositionwiththekneeslightlybent. c. Ifthebloodpressureinthearmishighinanadolescent,thenitshouldbecomparedwiththethigh pressure. d. Thethighpressureislowerthanthepressureinthearm,whichisattributabletothedistanceaway from the heart and the size of the poplitealvessels. ANS: C Whenbloodpressuremeasuredatthearmisexcessivelyhigh,particularlyinadolescentsandyoungadults,it iscomparedwiththighpressuretocheckforcoarctationoftheaorta.Thepoplitealarteryisauscultatedforthe reading.Generally,thighpressureishigherthanthatofthearm;however,ifcoarctationofthearteryispresent then arm pressures are higher than thighpressures. DIF: Cognitive Level: Understanding (Comprehension) MSC: Client Needs: Safe and Effective Care Environment: Management of Care 21. A4-month-oldchildisattheclinicforawell-babycheck-upandimmunizations.Whichoftheseactionsis most appropriate when the nurse is assessing an infants vitalsigns? a. Theinfantsradialpulseshouldbepalpated,andthenurseshouldnoticeanyfluctuationsresulting from activity orexercise. b. Thenurseshouldauscultateanapicalratefor1minuteandthenassessforanynormal irregularities, such as sinusarrhythmia. c. TheinfantsbloodpressureshouldNbeUaRsSsIeNssGeTdBb.yCOusMingastethoscopewithalargediaphragmpiece to hear the soft muffled Korotkoffsounds. d. Theinfantschestshouldbeobservedandtherespiratoryratecountedfor1minute;therespiratory pattern may varysignificantly. ANS: B The nurse palpates or auscultates an apical rate with infants and toddlers. The pulse should be counted for 1 fullminutetoaccountfornormalirregularities,suchassinusarrhythmia.Childrenyoungerthan3yearsofage havesuchsmallarmvessels;consequently,hearingKorotkoffsoundswithastethoscopeisdifficult.Thenurse shoulduseeitheranelectronicbloodpressuredevicethatusesoscillometryoraDopplerultrasounddeviceto amplify thesounds. DIF: Cognitive Level: Applying (Application) MSC: Client Needs: Safe and Effective Care Environment: Management of Care 22. Thenurseisconductingahealthfairforolderadults.Whichstatementistrueregardingvitalsign measurements in agingadults? a. Thepulseismoredifficulttopalpatebecauseofthestiffnessofthebloodvessels. b. Anincreasedrespiratoryrateandashallowerinspiratoryphaseareexpectedfindings. c. Adecreasedpulsepressureoccursfromchangesinthesystolicanddiastolicbloodpressures. d. Changesinthebodystemperatureregulatorymechanismleavetheolderpersonmorelikelyto develop afever. ANS: B Agingcausesadecreaseinvitalcapacityanddecreasedinspiratoryreservevolume.Theexaminermaynoticea shallower inspiratory phase and an increased respiratory rate. An increase in the rigidity of the arterial walls makes the pulse actually easier to palpate. Pulse pressure is widened in older adults, and changes in the body temperature regulatory mechanism leave the older person less likely to have fever but at a greater risk for hypothermia. DIF:CognitiveLevel:Understanding(Comprehension) MSC:ClientNeeds:HealthPromotionandMaintenance 23. Inapatientwithacromegaly,thenursewillexpecttodiscoverwhichassessmentfindings? a. Heavy, flattened facialfeatures b. Growth retardation and a delayed onset ofpuberty c. Overgrowth of bone in the face, head, hands, andfeet d. Increased height and weight and delayed sexualdevelopment NURSINGTB.COM ANS: C Excessivesecretionsofgrowthhormoneinadulthoodafternormalcompletionofbodygrowthcausesan overgrowthofthebonesintheface,head,hands,andfeetbutnochangeinheight. DIF: Cognitive Level: Understanding (Comprehension) MSC: Client Needs: Safe and Effective Care Environment: Management of Care 24. Thenurseisassessingchildreninapediatricclinic.Whichstatementistrueregardingthemeasurementof blood pressure inchildren? a. Blood pressure guidelines for children are based onage. b. PhaseIIKorotkoffsoundsarethebestindicatorofsystolicbloodpressureinchildren. c. UsingaDopplerdeviceisrecommendedforaccuratebloodpressuremeasurementsuntil adolescence. d. ThedisappearanceofphaseVKorotkoffsoundscanbeusedforthediastolicreadinginchildren. ANS: D ThedisappearanceofphaseVKorotkoffsoundscanbeusedforthediastolicreadinginchildren,aswellasin adults. DIF: Cognitive Level: Remembering (Knowledge) MSC:ClientNeeds:HealthPromotionandMaintenance 25. Whattypeofbloodpressuremeasurementerrorismostlikelytooccurifthenursedoesnotcheckforthe presence of an auscultatorygap? a. Diastolic blood pressure may not beheard. b. Diastolic blood pressure may be falselylow. c. Systolic blood pressure may be falselylow. d. Systolic blood pressure may be falselyhigh. ANS: C Ifanauscultatorygapisundetected,thenafalselylowsystolicorfalselyhighdiastolicreadingmayresult, which is common in patients withhypertension. DIF: Cognitive Level: Understanding (Comprehension) MSC: Client Needs: Safe and Effective Care Environment: Management of Care 26. Whenconsideringtheconceptsrelatedtobloodpressure,thenurseknowsthattheconceptofmeanarterial pressure (MAP) is best described by whichstatement? NURSINGTB.COM a. MAPisthepressureofthearterialpulse. b. MAP reflects the stroke volume of theheart. c. MAPisthepressureforcingbloodintothetissues,averagedoverthecardiaccycle. d. MAPisanaverageofthesystolicanddiastolicbloodpressuresandreflectstissueperfusion. ANS: C MAP is the pressure that forces blood into the tissues, averaged over the cardiac cycle. Stroke volume is reflectedbythebloodpressure.MAPisnotanarithmeticaverageofsystolicanddiastolicpressuresbecause diastolelastslonger;rather,itisavalueclosertodiastolicpressureplusonethirdofthepulsepressure. DIF: Cognitive Level: Remembering (Knowledge) MSC:ClientNeeds:SafeandEffectiveCareEnvironment:ManagementofCare 27. A75-year-oldmanwithahistoryofhypertensionwasrecentlychangedtoanewantihypertensivedrug.He reportsfeelingdizzyattimes.Howshouldthenurseevaluatehisbloodpressure? a. Bloodpressureandpulseshouldberecordedinthesupine,sitting,andstandingpositions. b. Thepatientshouldbedirectedtowalkaroundtheroomandhisbloodpressureassessedafterthis activity. c. Bloodpressureandpulseareassessedatthebeginningandattheendoftheexamination. d. Bloodpressureistakenontherightarmandthen5minuteslaterontheleftarm. ANS: A Orthostatic vital signs should be taken when the person is hypertensive or is taking antihypertensive medications,whenthepersonreportsfaintingorsyncope,orwhenvolumedepletionissuspected.Theblood pressureandpulsereadingsarerecordedinthesupine,sitting,andstandingpositions. DIF: Cognitive Level: Applying (Application) MSC: Client Needs: Safe and Effective Care Environment: Management of Care 28. Whichofthesespecificmeasurementsisthebestindexofachildsgeneralhealth? a. Vitalsigns b. Height andweight c. Headcircumference d. Chestcircumference NURSINGTB.COM ANS: B Physicalgrowth,measuredbyheightandweight,isthebestindexofachildsgeneralhealth. DIF: Cognitive Level: Understanding(Comprehension) MSC: Client Needs: Health Promotion and Maintenance 29. The nurse is assessing an 8-year-old child whose growth rate measures below the third percentile for a childhisage.Heappearssignificantlyyoungerthanhisstatedageandischubbywithinfantilefacialfeatures. Which condition does this childhave? a. Hypopituitarydwarfism b. Achondroplasticdwarfism c. Marfansyndrome d. Acromegaly ANS:A Hypopituitarydwarfismiscausedbyadeficiencyingrowthhormoneinchildhoodandresultsinaretardation ofgrowthbelowthethirdpercentile,delayedpuberty,andotherproblems.Thechildsappearancefitsthis description.Achondroplasticdwarfismisageneticdisorderresultingincharacteristicdeformities;Marfan syndromeisaninheritedconnectivetissuedisordercharacterizedbyatall,thinstatureandotherfeatures. Acromegalyistheresultofexcessivesecretionofgrowthhormoneinadulthood. DIF: Cognitive Level: Applying (Application) MSC: Client Needs: Safe and Effective Care Environment: Management of Care 30. The nurse is counting an infants respirations. Which technique iscorrect? a. Watching the chest rise andfall b. Watching the abdomen formovement c. Placing a hand across the infantschest d. Usingastethoscopetolistentothebreathsounds ANS:B Watchingtheabdomenformovementisthecorrecttechniquebecausetheinfantsrespirationsarenormally morediaphragmaticthanthoracic.Theotherresponsesdonotreflectcorrecttechniques. DIF: Cognitive Level: Analyzing (Analysis) MSC: Client Needs: Health Promotion and Maintenance NURSINGTB.COM 31. Whencheckingforproperbloodpressurecuffsize,whichguidelineiscorrect? a. The standard cuff size is appropriate for allsizes. b. Thelengthoftherubberbladdershouldequal80%ofthearmcircumference. c. Thewidthoftherubberbladdershouldequal80%ofthearmcircumference. d. Thewidthoftherubberbladdershouldequal40%ofthearmcircumference. ANS: D Thewidthoftherubberbladdershouldequal40%ofthecircumferenceofthepersonsarm.Thelengthofthe bladder should equal 80% of thiscircumference. DIF: Cognitive Level: Applying (Application) MSC: Client Needs: Safe and Effective Care Environment: Management of Care 32. Duringanexamination,thenursenoticesthatafemalepatienthasaroundmoonface,centraltrunkobesity and a cervical hump. Her skin is fragile with bruises. The nurse determines that the patient has which condition? a. Marfansyndrome b. Gigantism c. Cushingsyndrome d. Acromegaly ANS:C Cushingsyndromeischaracterizedbyweightgainandedemawithcentraltrunkandcervicalobesity(buffalo hump) and round plethoric face (moon face). Excessive catabolism causes muscle wasting; weakness; thin armsandlegs;reducedheight;andthin,fragileskinwithpurpleabdominalstriae,bruising,andacne. DIF: Cognitive Level: Applying (Application) MSC: Client Needs: Safe and Effective Care Environment: Management of Care 33. Thenurseispreparingtomeasurethevitalsignsofa6-month-oldinfant.Whichactionbythenurseis correct? a. Respirations are measured; then pulse andtemperature. b. Vital signs should be measured more frequently than in anadult. c. Proceduresareexplainedtotheparent,andtheinfantisencouragedtohandletheequipment. NURSINGTB.COM d. Thenurseshouldfirstperformthephysicalexaminationtoallowtheinfanttobecomemore familiar with her and then measure the infants vitalsigns. ANS: A Withaninfant,theorderofvitalsignmeasurementsisreversedtorespiration,pulse,andtemperature.Taking the temperature first, especially if it is rectal, may cause the infant to cry, which will increase the respiratory andpulserate,thusmaskingthenormalrestingvalues.Thevitalsignsaremeasuredwiththesamepurposeand frequency as would be measured in anadult. DIF: Cognitive Level: Applying (Application) MSC: Client Needs: Safe and Effective Care Environment: Management of Care MULTIPLE RESPONSE 1. While measuring a patients blood pressure, the nurse uses the proper technique to obtain an accurate reading.Whichofthesesituationswillresultinafalselyhighbloodpressurereading?Selectallthatapply. a. Thepersonsupportshisorherownarmduringthebloodpressurereading. b. The blood pressure cuff is too narrow for theextremity. c. The arm is held above level of theheart. d. The cuff is loosely wrapped around thearm. e. The person is sitting with his or her legscrossed. f. The nurse does not inflate the cuff highenough. ANS: A, B, D, E Severalfactorscanresultinbloodpressurereadingsthataretoohighortoolow.Havingthepatientsarmheld above the level of the heart is one part of the correcttechnique. DIF: Cognitive Level: Applying (Application) MSC: Client Needs: Safe and Effective Care Environment: Management of Care SHORT ANSWER 1.Whatisthepulsepressureforapatientwhosebloodpressureis158/96mmHgandwhosepulserateis72 beats perminute? ANS: 62 Thepulsepressureisthedifferencebetweenthesystolicanddiastolicandreflectsthestrokevolume.Thepulse rate is not necessary for pulse pressurecalculations. DIF: Cognitive Level: Analyzing (Analysis) MSC:ClientNeeds:SafeandEffectiveCareENnUvRirSoInNmGeTnBt:.CMOaMnagementofCare Chapter 11: Pain Assessment MULTIPLE CHOICE 1. Whenevaluatingapatientspain,thenurseknowsthatanexampleofacutepainwouldbe: a. Arthriticpain. b. Fibromyalgia. c. Kidneystones. d. Low backpain. ANS: C Acutepainisshort-termanddissipatesafteraninjuryheals,suchaswithkidneystones.Theotherconditions areexamplesofchronicpainduringwhichthepaincontinuesfor6monthsorlongeranddoesnotstopwhen the injuryheals. DIF: Cognitive Level: Understanding (Comprehension) MSC: Client Needs: Physiologic Integrity: Basic Care and Comfort 2. Whichstatementindicatesthatthenurseunderstandsthepainexperiencedbyanolderadult? NURSINGTB.COM a. Olderadultsmustlearntotoleratepain. b. Pain is a normal process of aging and is to beexpected. c. Painindicatesapathologicconditionoraninjuryandisnotanormalprocessofaging. d. Olderindividualsperceivepaintoalesserdegreethandoyoungerindividuals. ANS: C Painindicatesapathologicconditionoraninjuryandshouldneverbeconsideredsomethingthatanolderadult shouldexpectortolerate.Painisnotanormalprocessofaging,andnoevidencesuggeststhatpainperception is reduced withaging. DIF: Cognitive Level: Applying (Application) MSC: Client Needs: Physiologic Integrity: Basic Care and Comfort 3. A4-year-oldboyisbroughttotheemergencydepartmentbyhismother.Shesayshepointstohisstomach andsays,Ithurtssobad.Whichpainassessmenttoolwouldbethebestchoicewhenassessingthischildspain? a. DescriptorScale b. Numeric ratingscale c. Brief PainInventory d. FacesPainScaleRevised(FPS-R) ANS:D Ratingscalescanbeintroducedattheageof4or5years.TheFPS-Risdesignedforusebychildrenandasks the child to choose a face that shows how much hurt (or pain) you have now. Young children should not be asked to rate pain by usingnumbers. DIF: Cognitive Level: Applying (Application) MSC: Client Needs: Physiologic Integrity: Basic Care and Comfort 4. Apatientstatesthatthepainmedicationisnotworkingandrateshispostoperativepainata10ona1-to-10 scale.Whichoftheseassessmentfindingsindicatesanacutepainresponsetopoorlycontrolledpain? a. Confusion b. Hyperventilation c. Increased blood pressure andpulse d. Depression ANS: C NURSINGTB.COM Responsestopoorlycontrolledacutepainincludetachycardia,elevatedbloodpressure,andhypoventilation. Confusion and depression are associated with poorly controlled chronicpain. DIF: Cognitive Level: Analyzing (Analysis) MSC: Client Needs: Physiologic Integrity: Basic Care and Comfort 5. A60-year-oldwomanhasdevelopedreflexivesympatheticdystrophyafterarthroscopicrepairofher shoulder. A key feature of this condition is thatthe: a. Affected extremity will eventually regain itsfunction. b. Pain is felt at one site but originates from anotherlocation. c. Patients pain will be associated with nausea, pallor, anddiaphoresis. d. Slightesttouch,suchasasleevebrushingagainstherarm,causessevereandintensepain. ANS: D Akeyfeatureofreflexivesympatheticdystrophyisthatatypicallyinnocuousstimuluscancreateasevere, intenselypainfulresponse.Theaffectedextremitybecomeslessfunctionalovertime. DIF: Cognitive Level: Understanding (Comprehension) MSC: Client Needs: Physiologic Integrity: Basic Care and Comfort 6. Thenurseisassessingapatientspain.Thenurseknowsthatthemostreliableindicatorofpainwouldbethe: a. Patients vitalsigns. b. Physicalexamination. c. Results of a computerized axial tomographicscan. d. Subjectivereport. ANS: D Thesubjectivereportisthemostreliableindicatorofpain.Physicalexaminationfindingscanlendsupport,but thecliniciancannotexclusivelybasethediagnosisofpainonphysicalassessmentfindings. DIF: Cognitive Level: Understanding (Comprehension) MSC: Client Needs: Safe and Effective Care Environment: Management of Care 7. Apatienthashadarthriticpaininherhipsforseveralyearssinceahipfracture.Sheisabletomovearound inherroomandhasnotofferedanycomplaintssofarthismorning.However,whenasked,shestatesthather painisbadthismorningandratesitatan8ona1-to-10scale.Whatdoesthenursesuspect?Thepatient: NURSINGTB.COM a. Is addicted to her pain medications and cannot obtain painrelief. b. Does not want to trouble the nursing staff with hercomplaints. c. Is not in pain but rates it high to receive painmedication. d. Has experienced chronic pain for years and has adapted toit. ANS: D Personswithchronicpaintypicallytrytogivelittleindicationthattheyareinpainand,overtime,adapttothe pain. As a result, they are at risk forunderdetection. DIF: Cognitive Level: Analyzing (Analysis) MSC: Client Needs: Safe and Effective Care Environment: Management of Care 8. Thenurseisreviewingtheprinciplesofpain.Whichtypeofpainisduetoanabnormalprocessingofthe pain impulse through the peripheral or central nervoussystem? a. Visceral b. Referred c. Cutaneous d. Neuropathic ANS:D Neuropathicpainimpliesanabnormalprocessingofthepainmessage.Theothertypesofpainarenamed according to theirsources. DIF: Cognitive Level: Remembering (Knowledge) MSC: Client Needs: Safe and Effective Care Environment: Management of Care 9. Whenassessingthequalityofapatientspain,thenurseshouldaskwhichquestion? a. When did the painstart? b. Is the pain a stabbingpain? c. Is it a sharp pain or dullpain? d. What does your pain feellike? ANS: D NURSINGTB.COM Toassessthequalityofapersonspain,thepatientisaskedtodescribethepaininhisorherownwords. DIF: Cognitive Level: Analyzing(Analysis) MSC: Client Needs: Safe and Effective Care Environment: Management of Care 10. Whenassessingapatientspain,thenurseknowsthatanexampleofvisceralpainwouldbe: a. Hipfracture. b. Cholecystitis. c. Second-degreeburns. d. Pain after a legamputation. ANS: B Visceralpainoriginatesfromthelargerinteriororgans,suchasthegallbladder,liver,orkidneys. DIF: Cognitive Level: Understanding(Comprehension) MSC: Client Needs: Safe and Effective Care Environment: Management of Care 11. Thenurseisreviewingtheprinciplesofnociception.Duringwhichphaseofnociceptiondoestheconscious awareness of a painful sensationoccur? a. Perception b. Modulation c. Transduction d. Transmission ANS:A Perceptionisthethirdphaseofnociceptionandindicatestheconsciousawarenessofapainfulsensation. Duringthisphase,thesensationisrecognizedbyhighercorticalstructuresandidentifiedaspain. DIF: Cognitive Level: Remembering (Knowledge) MSC: Client Needs: Physiologic Integrity: Basic Care and Comfort 12. Whenassessingtheintensityofapatientspain,whichquestionbythenurseisappropriate? a. What makes your pain better orworse? b. How much pain do you havenow? c. How does pain limit youractivities? NURSINGTB.COM d. What does your pain feellike? ANS: B Askingthepatienthowmuchpaindoyouhave?isanassessmentoftheintensityofapatientspain;various intensityscalescanbeused.Askingwhatmakesonespainbetterorworseassessesalleviatingoraggravating factors.Askingwhetherpainlimitsonesactivitiesassessesthedegreeofimpairmentandqualityoflife. Askingwhatdoesyourpainfeellikeassessesthequalityofpain. DIF: Cognitive Level: Analyzing(Analysis) MSC: Client Needs: Physiologic Integrity: Basic Care and Comfort 13. Apatientiscomplainingofseverekneepainaftertwistingitduringabasketballgameandisrequesting pain medication. Which action by the nurse isappropriate? a. Completingthephysicalexaminationfirstandthengivingthepainmedication b. Tellingthepatientthatthepainmedicationmustwaituntilafterthex-rayimagesarecompleted c. Evaluatingthefullrangeofmotionofthekneeandthenmedicatingforpain d. Administering pain medication and then proceeding with theassessment ANS: D According to the American Pain Society (1992), In cases in which the cause of acute pain is uncertain, establishingadiagnosisisapriority,butsymptomatictreatmentofpainshouldbegivenwhiletheinvestigation isproceeding.Withoccasionalexceptions,(e.g.,theinitialexaminationofthepatientwithanacutecondition oftheabdomen),itisrarelyjustifiedtodeferanalgesiauntiladiagnosisismade.Infact,acomfortablepatient is better able to cooperate with diagnosticprocedures. DIF: Cognitive Level: Analyzing (Analysis) MSC: Client Needs: Physiologic Integrity: Basic Care and Comfort 14. Thenurseknowsthatwhichstatementistrueregardingthepainexperiencedbyinfants? a. Painininfantscanonlybeassessedbyphysiologicchanges,suchasanincreasedheartrate. b. The FPS-R can be used to assess pain ininfants. c. Aprocedurethatinducespaininadultswillalsoinducepainintheinfant. d. Infants feel pain less than doadults. ANS: C Ifaprocedureordiseaseprocesscausespaininanadult,thenitwillalsocausepaininaninfant.Physiologic changescannotbeexclusivelyusedtoconfirmordenypainbecauseotherfactors,suchasmedications,fluid status,orstressmaycausephysiologicchangeNs.UTRhSeINFGPST-BR.CcOanMbeusedstartingatage4years. DIF: Cognitive Level: Understanding (Comprehension) MSC: Client Needs: Physiologic Integrity: Basic Care and Comfort 15. Apatienthasbeenadmittedtothehospitalwithvertebralfracturesrelatedtoosteoporosis.Sheisin extreme pain. This type of pain would be classifiedas: a. Referred. b. Cutaneous. c. Visceral. d. Deepsomatic. ANS: D Deepsomaticpaincomesfromsourcessuchasthebloodvessels,joints,tendons,muscles,andbone.Referred painisfeltatonesitebutoriginatesfromanotherlocation.Cutaneouspainisderivedfromtheskinsurfaceand subcutaneous tissues. Visceral pain originates from the larger, interiororgans. DIF: Cognitive Level: Applying (Application) MSC: Client Needs: Physiologic Integrity: Basic Care and Comfort MULTIPLE RESPONSE 1. Duringassessmentofapatientspain,thenurseisawarethatcertainnonverbalbehaviorsareassociatedwith chronicpain.Whichofthesebehaviorsareassociatedwithchronicpain?Selectallthatapply. a. Sleeping b. Moaning c. Diaphoresis d. Bracing e. Restlessness f. Rubbing ANS: A, D,F Behaviorsthathavebeenassociatedwithchronicpainincludebracing,rubbing,diminishedactivity,sighing, andchangesinappetite.Inaddition,thosewithchronicpainmaysleepinanattemptatdistraction.Theother behaviors are associated with acutepain. DIF: Cognitive Level: Applying (Application) NURSINGTB.COM MSC: Client Needs: Physiologic Integrity: Basic Care and Comfort 2. Duringanadmissionassessmentofapatientwithdementia,thenurseassessesforpainbecausethepatient hasrecentlyhadseveralfalls.Whichoftheseareappropriateforthenursetoassessinapatientwithdementia? Select all thatapply. a. Ask the patient, Do you havepain? b. Assess the patients breathing independent ofvocalization. c. Note whether the patient is calling out, groaning, orcrying. d. Have the patient rate pain on a 1-to-10scale. e. Observe the patients body language for pacing andagitation. ANS: B, C, E Patientswithdementiamaysaynowhen,inreality,theyareveryuncomfortablebecausewordshavelosttheir meaning.Patientswithdementiabecomelessabletoidentifyanddescribepainovertime,althoughpainisstill present. People with dementia communicate pain through their behaviors. Agitation, pacing, and repetitive yelling may indicate pain and not a worsening of the dementia. (See the Pain Assessment in Advanced Dementia[PAINAD]scale,whichmayalsobeusedtoassesspaininpersonswithdementia.) DIF: Cognitive Level: Applying (Application) MSC: Client Needs: Physiologic Integrity: Basic Care and Comfort NURSINGTB.COM Chapter 12: Nutrition Assessment MULTIPLE CHOICE 1. Thenurserecognizeswhichofthesepersonsisatgreatestriskforundernutrition? a. 5-month-oldinfant b. 50-year-oldwoman c. 20-year-old collegestudent d. 30-year-oldhospitaladministrator ANS:A Vulnerablegroupsforundernutritionareinfants,children,pregnantwomen,recentimmigrants,personswith low incomes, hospitalized people, and agingadults. DIF: Cognitive Level: Remembering (Knowledge) MSC:ClientNeeds:HealthPromotionandMaintenance 2. Whenassessingapatientsnutritionalstatus,thenurserecallsthatthebestdefinitionofoptimalnutritional status is sufficient nutrientsthat: NURSINGTB.COM a. Are in excess of daily bodyrequirements. b. Provide for the minimum bodyneeds. c. Providefordailybodyrequirementsbutdonotsupportincreasedmetabolicdemands. d. Providefordailybodyrequirementsandsupportincreasedmetabolicdemands. ANS: D Optimalnutritionalstatusisachievedwhensufficientnutrientsareconsumedtosupportday-to-daybodyneeds andanyincreasedmetabolicdemandsresultingfromgrowth,pregnancy,orillness. DIF: Cognitive Level: Remembering (Knowledge) MSC:ClientNeeds:HealthPromotionandMaintenance 3. Thenurseisprovidingnutritioninformationtothemotherofa1-year-oldchild.Whichofthesestatements represents accurate information for this agegroup? a. Maintainingadequatefatandcaloricintakeisimportantforachildinthisagegroup. b. Therecommendeddietaryallowancesforaninfantarethesameasforanadolescent. c. Thebabysgrowthisminimalatthisage;therefore,caloricrequirementsaredecreased. d. Thebabyshouldbeplacedonskimmilktodecreasetheriskofcoronaryarterydiseasewhenheor she growsolder. ANS: A Becauseofrapidgrowth,especiallyofthebrain,bothinfantsandchildrenyoungerthan2yearsofageshould not drink skim or low-fat milk or be placed on low-fat diets. Fats (calories and essential fatty acids) are required for proper growth and central nervous systemdevelopment. DIF: Cognitive Level: Applying (Application) MSC: Client Needs: Health Promotion and Maintenance 4. Apregnantwomanisinterestedinbreastfeedingherbabyandasksseveralquestionsaboutthetopic.Which information is appropriate for the nurse to share withher? a. Breastfeeding is best when also supplemented with bottlefeedings. b. Babies who are breastfed often require supplementalvitamins. c. Breastfeedingisrecommendedforinfantsforthefirst2yearsoflife. d. Breastmilkprovidesthenutrientsnecessaryforgrowth,aswellasnaturalimmunity. NURSINGTB.COM ANS: D Breastfeedingisrecommendedforfull-terminfantsforthefirstyearoflifebecausebreastmilkisideally formulated to promote normal infant growth and development, as well as natural immunity. The other statements are notcorrect. DIF: Cognitive Level: Applying (Application) MSC: Client Needs: Health Promotion and Maintenance 5. Amotherandher13-year-olddaughterexpresstheirconcernrelatedtothedaughtersrecentweightgainand herincreaseinappetite.Whichofthesestatementsrepresentsinformationthenurseshoulddiscusswiththem? a. Dieting and exercising are necessary at thisage. b. Snacks should be high in protein, iron, andcalcium. c. Teenagerswhohaveaweightproblemshouldnotbeallowedtosnack. d. A low-calorie diet is important to prevent the accumulation offat. ANS: B After a period of slow growth in late childhood, adolescence is characterized by rapid physical growth and endocrineandhormonalchanges.Caloricandproteinrequirementsincreasetomeetthisdemand.Becauseof bonegrowthandincreasingmusclemass(and,ingirls,theonsetofmenarche),calciumandironrequirements alsoincrease. DIF: Cognitive Level: Applying (Application) MSC: Client Needs: Health Promotion and Maintenance 6. Thenurseisassessinga30-year-oldunemployedimmigrantfromanunderdevelopedcountrywhohasbeen in the United States for 1 month. Which of these problems related to his nutritional status might the nurse expect tofind? a. Obesity b. Hypotension c. Osteomalacia (softening of thebones) d. Coronaryarterydisease ANS:C Generalundernutrition,hypertension,diarrhea,lactoseintolerance,osteomalacia,scurvy,anddentalcariesare amongthemorecommonnutrition-relatedproblemsofnewimmigrantsfromdevelopingcountries. DIF: Cognitive Level: Applying (Application) NURSINGTB.COM MSC: Client Needs: Health Promotion and Maintenance 7. Forthefirsttime,thenurseisseeingapatientwhohasnohistoryofnutrition-relatedproblems.Theinitial nutritional screening should include whichactivity? a. Caloriecountofnutrients b. Anthropometricmeasures c. Complete physicalexamination d. Measurementofweightandweighthistory ANS:D Parametersusedfornutritionscreeningtypicallyincludeweightandweighthistory,conditionsassociatedwith increasednutritionalrisk,dietinformation,androutinelaboratorydata.Theotherresponsesreflectamorein- depth assessment rather than ascreening. DIF: Cognitive Level: Applying (Application) MSC: Client Needs: Health Promotion and Maintenance 8. Apatientisaskedtoindicateonaformhowmanytimesheeatsaspecificfood.Thismethoddescribes which of these tools for obtaining dietaryinformation? a. Fooddiary b. Caloriecount c. 24-hourrecall d. Food-frequency questionnaire ANS:D Withthistool,informationiscollectedonhowmanytimesperday,week,ormonththeindividualeats particular foods, which provides an estimate of usualintake. DIF: Cognitive Level: Remembering (Knowledge) MSC:ClientNeeds:HealthPromotionandMaintenance 9. Thenurseisprovidingcarefora68-year-oldwomanwhoiscomplainingofconstipation.Whatconcern exists regarding her nutritionalstatus? a. Absorption of nutrients may beimpaired. b. Constipation may represent a foodallergy. NURSINGTB.COM c. The patient may need emergency surgery to correct theproblem. d. Gastrointestinal problems will increase her caloricdemand. ANS: A Gastrointestinalsymptomssuchasvomiting,diarrhea,orconstipationmayinterferewithnutrientintakeor absorption. The other responses are notcorrect. DIF: Cognitive Level: Applying (Application) MSC: Client Needs: Health Promotion and Maintenance 10. Duringanutritionalassessment,whyisitimportantforthenursetoaskapatientwhatmedicationsheor she istaking? a. Certain drugs can affect the metabolism ofnutrients. b. The nurse needs to assess the patient for allergicreactions. c. Medicationsneedtobedocumentedintherecordforthephysiciansreview. d. Medicationscanaffectonesmemoryandabilitytoidentifyfoodeateninthelast24hours. ANS: A Analgesics,antacids,anticonvulsants,antibiotics,diuretics,laxatives,antineoplasticdrugs,steroids,andoral contraceptivesaredrugsthatcaninteractwithnutrients,impairingtheirdigestion,absorption,metabolism,or use. The other responses are notcorrect. DIF:CognitiveLevel:Understanding(Comprehension) MSC:ClientNeeds:HealthPromotionandMaintenance 11. Apatienttellsthenursethathisfoodsimplydoesnothaveanytasteanymore.Thenursesbestresponse wouldbe: a. That must be reallyfrustrating. b. When did you first notice thischange? c. My food doesnt always have a lot of tasteeither. d. Sometimes that happens, but your taste will comeback. ANS: B Withchangesinappetite,taste,smell,orchewingorswallowing,theexaminershouldaskaboutthetypeof change and when the change occurred. These problems interfere with adequate nutrient intake. The other responses are notcorrect. NURSINGTB.COM DIF:CognitiveLevel:Understanding(Comprehension) MSC:ClientNeeds:HealthPromotionandMaintenance 12. Thenurseisperforminganutritionalassessmentona15-year-oldgirlwhotellsthenursethatsheissofat. Assessmentrevealsthatsheis5feet4inchesandweighs110pounds.Thenursesappropriateresponsewould be: a. How much do you think you shouldweigh? b. Dontworryaboutit;yourenotthatoverweight. c. Thebestthingforyouwouldbetogoonadiet. d. I used to always think I was fat when I was yourage. ANS: A Adolescents increased body awareness and self-consciousness may cause eating disorders such as anorexia nervosaorbulimia,conditionsinwhichtherealorperceivedbodyimagedoesnotfavorablycomparewithan idealimage.Thenurseshouldnotbelittletheadolescentsfeelings,provideunsolicitedadvice,oragreewith her. DIF: Cognitive Level: Applying (Application) MSC: Client Needs: Health Promotion and Maintenance 13. Thenurseisdiscussingappropriatefoodswiththemotherofa3-year-oldchild.Whichofthesefoodsare recommended? a. Foods that the child will eat, no matter what theyare b. Foods easy to hold such as hot dogs, nuts, andgrapes c. Anyfoods,aslongastherestofthefamilyisalsoeatingthem d. Fingerfoodsandnutritioussnacksthatcannotcausechoking ANS:D Small portions, finger foods, simple meals, and nutritious snacks help improve the dietary intake of young children.Foodslikelytobeaspiratedshouldbeavoided(e.g.,hotdogs,nuts,grapes,roundcandies,popcorn). DIF: Cognitive Level: Applying (Application) MSC:ClientNeeds:HealthPromotionandMaintenance 14. Thenurseisreviewingthenutritionalassessmentofan82-year-oldpatient.Whichofthesefactorswill most likely affect the nutritional status of an olderadult? a. Increase in taste andsmell b. Livingaloneonafixedincome c. Changeincardiovascularstatus NURSINGTB.COM d. Increase in gastrointestinal motility andabsorption ANS: B Socioeconomic conditions frequently affect the nutritional status of the aging adult; these factors should be closelyevaluated.Physicallimitations,income,andsocialisolationarefrequentproblemsthatinterferewith the acquisition of a balanced diet. A decrease in taste and smell and decreased gastrointestinal motility and absorptionoccurwithaging.Cardiovascularstatusisnotafactorthataffectsanolderadultsnutritionalstatus. DIF:CognitiveLevel:Understanding(Comprehension) MSC:ClientNeeds:HealthPromotionandMaintenance 15. Whenconsideringanutritionalassessment,thenurseisawarethatthemostcommonanthropometric measurementsinclude: a. Height andweight. b. Legcircumference. c. Skinfold thickness of thebiceps. d. Hip and waistmeasurements. ANS: A Themostcommonlyusedanthropometricmeasuresareheight,weight,tricepsskinfoldthickness,elbow breadth, and arm and headcircumferences. DIF: Cognitive Level: Remembering (Knowledge) MSC:ClientNeeds:HealthPromotionandMaintenance 16. Ifa29-year-oldwomanweighs156pounds,andthenursedeterminesheridealbodyweighttobe120 pounds, then how would the nurse classify the womansweight? a. Obese b. Mildlyoverweight c. Suffering frommalnutrition d. Within appropriate range of idealweight ANS: A NURSINGTB.COM Obesity,asaresultofcaloricexcess,referstoweightmorethan20%aboveidealbodyweight.Forthispatient, 20% of her ideal body weight would be 24 pounds, and greater than 20% of her body weight would be over 144pounds.Therefore,havingaweightof156poundswouldbeconsideredobese. DIF: Cognitive Level: Applying (Application) MSC: Client Needs: Health Promotion and Maintenance 17. How should the nurse perform a triceps skinfoldassessment? a. Afterpinchingtheskinandfat,thecalipersareverticallyappliedtothefatfold. b. Theskinandfatonthefrontofthepatientsarmaregentlypinched,andthenthecalipersare applied. c. Afterapplyingthecalipers,thenursewaits3secondsbeforetakingareading.Afterrepeatingthe procedure three times, an average isrecorded. d. Thepatientisinstructedtostandwithhisorherbacktotheexaminerandarmsfoldedacrossthe chest. The skin on the forearm ispinched. ANS: C While holding the skinfold, the lever of the calipers is released. The nurse waits 3 seconds and then takes a reading.Thisprocedureshouldberepeatedthreetimes,andanaverageofthethreeskinfoldmeasurementsis thenrecorded. DIF:CognitiveLevel:Understanding(Comprehension) MSC:ClientNeeds:HealthPromotionandMaintenance 18. Inteachingapatienthowtodeterminetotalbodyfatathome,thenurseincludesinstructionstoobtain measurementsof: a. Height andweight. b. Frame size andweight. c. Waist and hipcircumferences. d. Mid-upper arm circumference and armspan. ANS: A Bodymassindex,calculatedbyusingheightandweightmeasurements,isapracticalmarkerofoptimalweight forheightandanindicatorofobesity.Theotheroptionsarenotcorrect. DIF:CognitiveLevel:Understanding(Comprehension) MSC:ClientNeeds:HealthPromotionandMaintenance NURSINGTB.COM 19. Thenurseisevaluatingpatientsforobesity-relateddiseasesbycalculatingthewaist-to-hipratios.Which one of these patients would be at increasedrisk? a. 29-year-oldwomanwhosewaistmeasures33inchesandhipsmeasure36inches b. 32-year-oldmanwhosewaistmeasures34inchesandhipsmeasure36inches c. 38-year-oldmanwhosewaistmeasures35inchesandhipsmeasure38inches d. 46-year-oldwomanwhosewaistmeasures30inchesandhipsmeasure38inches ANS:A Thewaist-to-hipratioassessesbodyfatdistributionasanindicatorofhealthrisk.Awaist-to-hipratioof1.0or greaterinmenor0.8orgreaterinwomenisindicativeofandroid(upperbodyobesity)andincreasingriskfor obesity-related disease and early death. The 29-year-old woman has a waist-to-hip ratio of 0.92, which is greaterthan0.8.The32-year-oldmanhasawaist-to-hipratioof0.94;the38-year-oldmanhasawaist-to-hip ratio of 0.92; the 46-year-old woman has a waist-to-hip ratio of0.78. DIF: Cognitive Level: Analyzing (Analysis) MSC: Client Needs: Health Promotion and Maintenance 20. A50-year-oldwomanwithelevatedtotalcholesterolandtriglyceridelevelsisvisitingtheclinictofindout aboutherlaboratoryresults.Whatwouldbeimportantforthenursetoincludeinpatientteachinginrelationto these tests? a. The risks of undernutrition should beincluded. b. Offer methods to reduce the stress in herlife. c. Provide information regarding a diet low in saturatedfat. d. Thisconditionishereditary;shecandonothingtochangethelevels. ANS: C Thepatientwithelevatedcholesterolandtriglyceridelevelsshouldbetaughtabouteatingahealthydietthat limitstheintakeoffoodshighinsaturatedfatsortransfats.Reducingdietaryfatsispartofthetreatmentfor this condition. The other responses are not pertinent to hercondition. DIF: Cognitive Level: Applying (Application) MSC: Client Needs: Health Promotion and Maintenance 21. Inperforminganassessmentona49-year-oldwomanwhohasimbalancednutritionasaresultof dysphagia, which data would the nurse expect tofind? a. Increase in hairgrowth b. Inadequatenutrientfoodintake c. Weight10%to20%overideal d. Sore, inflamed buccalcavity NURSINGTB.COM ANS: B Dysphagia,orimpairedswallowing,interfereswithadequatenutrientintake. DIF: Cognitive Level: Applying(Application) MSC: Client Needs: Health Promotion and Maintenance 22. A21-year-oldwomanhasbeenonalow-proteinliquiddietforthepast2months.Shehashadadequate intakeofcaloriesandappearswellnourished.Afterfurtherassessment,whatwouldthenurseexpecttofind? a. Poor skinturgor b. Decreased serumalbumin c. Increased lymphocytecount d. Triceps skinfold less thanstandard ANS: B Kwashiorkor(proteinmalnutrition)isduetodietsthatmaybehighincaloriesbutcontainlittleornoprotein (e.g., low-protein liquid diets, fad diets, and long-term use of dextrose-containing intravenous fluids). The serum albumin would be less than 3.5g/dL. DIF: Cognitive Level: Analyzing (Analysis) MSC: Client Needs: Health Promotion and Maintenance 23. Thenurseisperforminganutritionalassessmentonan80-year-oldpatient.Thenurseknowsthat physiologicchangescandirectlyaffectthenutritionalstatusoftheolderadultandinclude: a. Slowed gastrointestinalmotility. b. Hyperstimulation of the salivaryglands. c. Increased sensitivity to spicy and aromaticfoods. d. Decreased gastrointestinal absorption causing esophagealreflux. ANS: A Normal physiologic changes in aging adults that affect nutritional status include slowed gastrointestinal motility,decreasedgastrointestinalabsorption,diminishedolfactoryandtastesensitivity,decreasedsaliva production, decreased visual acuity, and poorNURSINGTB.COM DIF: Cognitive Level: Remembering (Knowledge) MSC:ClientNeeds:HealthPromotionandMaintenance 24. Whichoftheseinterventionsismostappropriatewhenthenurseisplanningnutritionalinterventionsfora healthy, active 74-year-oldwoman? a. Decreasingtheamountofcarbohydratestopreventleanmusclecatabolism b. Increasingtheamountofsoyandtofuinherdiettopromotebonegrowthandreverseosteoporosis c. Decreasingthenumberofcaloriessheiseatingbecauseofthedecreaseinenergyrequirements from the loss of lean bodymass d. Increasingthenumberofcaloriessheiseatingbecauseoftheincreasedenergyneedsoftheolder adult ANS: C Importantnutritionalfeaturesoftheolderyearsareadecreaseinenergyrequirementsasaresultoflossoflean bodymass,themostmetabolicallyactivetissue,andanincreaseinfatmass. DIF: Cognitive Level: Applying (Application) MSC: Client Needs: Health Promotion and Maintenance 25. A16-year-oldgirlisbeingseenattheclinicforgastrointestinalcomplaintsandweightloss.Thenurse determinesthatmanyofhercomplaintsmayberelatedtoerraticeatingpatterns,eatingpredominantlyfast foods,andhighcaffeineintake.Inthissituation,whichismostappropriatewhencollectingcurrentdietary intakeinformation? a. Schedulingatimefordirectobservationoftheadolescentduringmeals b. Askingthepatientfora24-hourdietrecall,andassumingittobereflectiveofatypicaldayforher c. Havingthepatientcompleteafooddiaryfor3days,including2weekdaysand1weekendday d. Usingthefoodfrequencyquestionnairetoidentifytheamountofintakeofspecificfoods ANS:C Fooddiariesrequiretheindividualtowritedowneverythingconsumedforacertaintimeperiod.Becauseof the erratic eating patterns of this individual, assessing dietary intake over a few days would produce more accurateinformationregardingeatingpatterns.Directobservationisbestusedwithyoungchildrenorolder adults. DIF: Cognitive Level: Analyzing (Analysis) MSC: Client Needs: Health Promotion and Maintenance 26. ThenurseispreparingtomeasurefatandNleUanRbSoINdGyTmBa.sCsOaMndbonemineraldensity.Whichtoolis appropriate? a. Measuringtape b. Skinfoldcalipers c. Bioelectrical impedance analysis(BIA) d. Dual-energyx-rayabsorptiometry(DEXA) ANS:D DEXAmeasuresbothbonemineraldensityandfatandleanbodymass.BIAmeasuresfatandleanbodymass butnotbonemineraldensity.Ameasuringtapemeasuresdistanceorlength,andskinfoldcalipersareusedto determine skinfoldthickness. DIF:CognitiveLevel:Understanding(Comprehension) MSC:ClientNeeds:HealthPromotionandMaintenance 27. Whichoftheseconditionsisduetoaninadequateintakeofbothproteinandcalories? a. Obesity b. Bulimia c. Marasmus d. Kwashiorkor ANS:C Marasmus, protein-calorie malnutrition, is due to an inadequate intake of protein and calories or prolonged starvation.Obesityisduetocaloricexcess;bulimiaisaneatingdisorder.Kwashiorkorisproteinmalnutrition. DIF: Cognitive Level: Remembering (Knowledge) MSC: Client Needs: Physiologic Integrity: Physiologic Adaptation 28. Duringanassessmentofapatientwhohasbeenhomelessforseveralyears,thenursenoticesthathis tongueismagentaincolor,whichisanindicationofadeficiencyinwhatmineraland/orvitamin? a. Iron b. Riboflavin c. Vitamin D andcalcium d. VitaminC NURSINGTB.COM ANS: B Magentatongueisasignofriboflavindeficiency.Incontrast,apaletongueisprobablyattributabletoiron deficiency.VitaminDandcalciumdeficienciescauseosteomalaciainadults,andavitaminCdeficiency causes scorbuticgums. DIF: Cognitive Level: Applying (Application) MSC: Client Needs: Physiologic Integrity: Physiologic Adaptation 29. A 50-year-old patient has been brought to the emergency department after a housemate found that the patientcouldnotgetoutofbedalone.Hehaslivedinagrouphomeforyearsbutforseveralmonthshasnot participatedintheactivitiesandhasstayedinhisroom.Thenurseassessesforsignsofundernutrition,andan x-ray study reveals that he has osteomalacia, which is a deficiencyof: a. Iron. b. Riboflavin. c. Vitamin D andcalcium. d. VitaminC. ANS:C Osteomalacia results from a deficiency of vitamin D and calcium in adults. Iron deficiency would result in anemia,riboflavindeficiencywouldresultinmagentatongue,andvitaminCdeficiencywouldresultinscurvy DIF: Cognitive Level: Applying (Application) MSC: Client Needs: Physiologic Integrity: Physiologic Adaptation 30. Anolderadultpatientinanursinghomehasbeenreceivingtubefeedingsforseveralmonths.Duringan oralexamination,thenursenotesthatpatientsgumsareswollen,ulcerated,andbleedinginsomeareas.The nurse suspects that the patient has whatcondition? a. Rickets b. Vitamin Adeficiency c. Linoleic-aciddeficiency d. Vitamin Cdeficiency ANS:D Vitamin C deficiency causes swollen, ulcerated, and bleeding gums, known as scorbutic gums. Rickets is a conditionrelatedtovitaminDandcalciumdeficienciesininfantsandchildren.Linoleic-aciddeficiencycauses eczematousskin.VitaminAdeficiencycausesBitotspotsandvisualproblems. DIF: Cognitive Level: Applying (Application) NURSINGTB.COM MSC: Client Needs: Physiologic Integrity: Physiologic Adaptation 31. Thenurseisassessingthebodyweightasapercentageofidealbodyweightonanadolescentpatientwho wasadmittedforsuspectedanorexianervosa.Thepatientsusualweightwas125pounds,buttodaysheweighs 98pounds.Thenursecalculatesthepatientsidealbodyweightandconcludesthatthepatientis: a. Experiencing mildmalnutrition. b. Experiencing moderatemalnutrition. c. Experiencing severemalnutrition. d. Still within expected parameters with her currentweight. ANS: B By dividing her current weight by her usual weight and then multiplying by 100, a percentage of 78.4% is obtained,whichmeansthathercurrentweightis78.4%ofheridealbodyweight.Acurrentweightof80%to 90% of ideal weight suggests mild malnutrition; a current weight of 70% to 80% of ideal weight suggests moderatemalnutrition;acurrentweightoflessthan70%ofidealweightsuggestsseveremalnutrition. DIF: Cognitive Level: Analyzing (Analysis) MSC: Client Needs: Physiologic Integrity: Physiologic Adaptation MULTIPLE RESPONSE 1.Thenurseisassessingapatientwhoisobeseforsignsofmetabolicsyndrome.Thisconditionisdiagnosed when three or more certain risk factors are present. Which of these assessment findings are risk factors for metabolic syndrome? Select all thatapply. a. Fasting plasma glucose level less than 100mg/dL b. Fasting plasma glucose level greater than or equal to 110mg/dL c. Bloodpressurereadingof140/90mmHg d. Bloodpressurereadingof110/80mmHg e. Triglyceridelevelof120mg/dL ANS: B,C Metabolic syndrome is diagnosed when three or more of the following risk factors are present: (1) fasting plasma glucose level greater than or equal to 100 mg/dL; (2) blood pressure greater than or equal to 130/85 mmHg;(3)waistcircumferencegreaterthanorequalto40inchesformenand35inchesforwomen;(4)high- density lipoprotein cholesterol less than 40 in men and less than 50 in women; and (5) triglyceride levels greater than or equal to 150 mg/dL (ATP III,2001). DIF: Cognitive Level: Applying (Application) NURSINGTB.COM MSC: Client Needs: Health Promotion and Maintenance SHORT ANSWER 1.Apatienthasbeenunabletoeatsolidfoodfor2weeksandisintheclinictodaycomplainingofweakness, tiredness, and hair loss. The patient states that her usual weight is 175 pounds, but today she weighs 161 pounds.Whatisherrecentweightchangepercentage?Tocalculaterecentweightchangepercentage,usethis formula: Usual weight current weight 100 usual weight ANS: 8% 175 161 = 14 pounds 14 175 = 0.08 0.08 100 = 8% DIF: Cognitive Level: Analyzing (Analysis) MSC: Client Needs: Health Promotion and Maintenance Chapter 13: Skin, Hair, and Nails MULTIPLE CHOICE 1. Thenurseeducatorispreparinganeducationmoduleforthenursingstaffontheepidermallayerofskin. Whichofthesestatementswouldbeincludedinthemodule?Theepidermisis: a. Highlyvascular. b. Thick andtough. c. Thin andnonstratified. d. Replaced every 4weeks. ANS: D Theepidermisisthinyettough,replacedevery4weeks,avascular,andstratifiedintoseveralzones. DIF: Cognitive Level: Understanding(Comprehension) MSC: Client Needs: General 2. Thenurseeducatorispreparinganeducationmoduleforthenursingstaffonthedermislayerofskin.Which of these statements would be included in the module? Thedermis: NURSINGTB.COM a. Contains mostly fatcells. b. Consistsmostlyofkeratin. c. Isreplacedevery4weeks. d. Contains sensoryreceptors. ANS: D Thedermisconsistsmostlyofcollagen,hasresilientelastictissuethatallowstheskintostretch,andcontains nerves,sensoryreceptors,bloodvessels,andlymphaticvessels.Itisnotreplacedevery4weeks. DIF:CognitiveLevel:Understanding(Comprehension) MSC: Client Needs:General 3. Thenurseisexaminingapatientwhotellsthenurse,Isuresweatalot,especiallyonmyfaceandfeetbutit doesnthaveanodor.Thenurseknowsthatthisconditioncouldberelatedto: a. Eccrineglands. b. Apocrineglands. c. Disorder of the stratumcorneum. d. Disorder of the stratumgerminativum. ANS: A The eccrine glands are coiled tubules that directly open onto the skin surface and produce a dilute saline solutioncalledsweat.Apocrineglandsareprimarilylocatedintheaxillae,anogenitalarea,nipples,andnaval areaandmixwithbacterialfloratoproducethecharacteristicmuskybodyodor.Thepatientsstatementisnot related to disorders of the stratum corneum or the stratumgerminativum. DIF: Cognitive Level: Applying (Application) MSC: Client Needs: Physiologic Integrity: Physiologic Adaptation 4. Anewborninfantisintheclinicforawell-babycheckup.Thenurseobservestheinfantforthepossibilityof fluid loss because of which of thesefactors? a. Subcutaneous fat deposits are high in thenewborn. b. Sebaceous glands are overproductive in thenewborn. c. The newborns skin is more permeable than that of theadult. d. The amount of vernix caseosa dramatically rises in thenewborn. NURSINGTB.COM ANS: C The newborns skin is thin, smooth, and elastic and is relatively more permeable than that of the adult; consequently,theinfantisatgreaterriskforfluidloss.Thesubcutaneouslayerintheinfantisinefficient,not thick,andthesebaceousglandsarepresentbutdecreaseinsizeandproduction.Vernixcaseosaisnotproduced afterbirth. DIF: Cognitive Level: Applying (Application) MSC: Client Needs: Health Promotion and Maintenance 5. Thenurseisbathingan80-year-oldmanandnoticesthathisskiniswrinkled,thin,lax,anddry.Thisfinding would be related to which factor in the olderadult? a. Increased vascularity of theskin b. Increased numbers of sweat and sebaceousglands c. An increase in elastin and a decrease in subcutaneousfat d. Anincreasedlossofelastinandadecreaseinsubcutaneousfat ANS:D An accumulation of factors place the aging person at risk for skin disease and breakdown: the thinning of the skin, a decrease in vascularity and nutrients, the loss of protective cushioning of the subcutaneous layer, a lifetimeofenvironmentaltraumatoskin,thesocialchangesofaging,aincreasinglysedentarylifestyle,andthe chance ofimmobility. DIF: Cognitive Level: Applying (Application) MSC: Client Needs: Health Promotion and Maintenance 6. Duringtheagingprocess,thehaircanlookgrayorwhiteandbegintofeelthinandfine.Thenurseknows that this occurs because of a decrease in the number offunctioning: a. Metrocytes. b. Fungacytes. c. Phagocytes. d. Melanocytes. ANS: D Intheaginghairmatrix,thenumberoffunctioningmelanocytesdecreases;asaresult,thehairlooksgrayor white and feels thin and fine. The other options are notcorrect. DIF: Cognitive Level: Understanding (Comprehension) NURSINGTB.COM MSC: Client Needs: Health Promotion and Maintenance 7. Duringanexamination,thenursefindsthatapatienthasexcessivedrynessoftheskin.Thebesttermto describe this conditionis: a. Xerosis. b. Pruritus. c. Alopecia. d. Seborrhea. ANS: A Xerosisisthetermusedtodescribeskinthatisexcessivelydry.Pruritusreferstoitching,alopeciareferstohai loss, and seborrhea refers to oilyskin. DIF: Cognitive Level: Remembering (Knowledge) MSC:ClientNeeds:HealthPromotionandMaintenance 8. A22-year-oldwomancomestotheclinicbecauseofseveresunburnandstates,Iwasoutinthesunforjust couple of minutes. The nurse begins a medication review with her, paying special attention to which medicationclass? a. Nonsteroidal antiinflammatory drugs forpain b. Tetracyclines foracne c. Proton pump inhibitors forheartburn d. Thyroidreplacementhormoneforhypothyroidism ANS:B Drugsthatmayincreasesunlightsensitivityandgiveaburnresponseincludesulfonamides,thiazidediuretics, oral hypoglycemic agents, andtetracycline. DIF: Cognitive Level: Applying (Application) MSC: Client Needs: Health Promotion and Maintenance 9. AwomanisleavingonatriptoHawaiiandhascomeinforacheckup.Duringtheexaminationthenurse learns that she has diabetes and takes oral hypoglycemic agents. The patient needs to be concerned about which possible effect of hermedications? a. Increased possibility ofbruising b. Skin sensitivity as a result of exposure to saltwater NURSINGTB.COM c. Lack of availability of glucose-monitoringsupplies d. Importanceofsunscreenandavoidingdirectsunlight ANS:D Drugsthatmayincreasesunlightsensitivityandgiveaburnresponseincludesulfonamides,thiazidediuretics, oral hypoglycemic agents, andtetracycline. DIF: Cognitive Level: Applying (Application) MSC: Client Needs: Physiologic Integrity: Reduction of Risk Potential 10. A13-year-oldgirlisinterestedinobtaininginformationaboutthecauseofheracne.Thenurseshould share with her thatacne: a. Iscontagious. b. Has no knowncause. c. Is caused by increased sebumproduction. d. Has been found to be related to poorhygiene. ANS: C Approximately90%ofmalesand80%offemaleswilldevelopacne;causesareincreasedsebumproduction and epithelial cells that do not desquamatenormally. DIF:CognitiveLevel:Understanding(Comprehension) MSC:ClientNeeds:HealthPromotionandMaintenance 11. A 75-year-old woman who has a history of diabetes and peripheral vascular disease has been trying to removeacornonthebottomofherfootwithapairofscissors.Thenursewillencouragehertostoptryingto remove the corn with scissorsbecause: a. Thewomancouldbeatincreasedriskforinfectionandlesionsbecauseofherchronicdisease. b. Withherdiabetes,shehasincreasedcirculationtoherfoot,anditcouldcauseseverebleeding. c. Sheis75yearsoldandisunabletosee;consequently,sheplacesherselfatgreaterriskforself- injury with thescissors. d. Withherperipheralvasculardisease,herrangeofmotionislimitedandshemaynotbeableto reach the cornsafely. ANS: A Apersonalhistoryofdiabetesandperipheralvasculardiseaseincreasesapersonsriskforskinlesionsinthe feetorankles.ThepatientneedstoseekaprofNeUssRioSnINalGfTorBa.CssOisMtancewithcornremoval. DIF: Cognitive Level: Applying (Application) MSC: Client Needs: Physiologic Integrity: Reduction of Risk Potential 12. Thenursekeepsinmindthatathoroughskinassessmentisextremelyimportantbecausetheskinholds information about apersons: a. Supportsystems. b. Circulatorystatus. c. Socioeconomicstatus. d. Psychologicalwellness. ANS: B Theskinholdsinformationaboutthebodyscirculation,nutritionalstatus,andsignsofsystemicdiseases,as well as topical data on the integumentary systemitself. DIF: Cognitive Level: Understanding (Comprehension) MSC: Client Needs: Safe and Effective Care Environment: Management of Care 13. Apatientcomesinforaphysicalexaminationandcomplainsoffreezingtodeathwhilewaitingforher examination.Thenursenotesthatherskinispaleandcoolandattributesthisfindingto: a. Venouspooling. b. Peripheralvasodilation. c. Peripheralvasoconstriction. d. Decreased arterialperfusion. ANS: C Achillyorair-conditionedenvironmentcausesvasoconstriction,whichresultsinfalsepallorandcoolness. DIF: Cognitive Level: Applying(Application) MSC: Client Needs: Physiologic Integrity: Basic Care and Comfort 14. A patient comes to the clinic and tells the nurse that he has been confined to his recliner chair for approximately3dayswithhisfeetdownandheasksthenursetoevaluatehisfeet.Duringtheassessment,the nurse might expect tofind: a. Pallor b. Coolness c. Distendedveins d. Prolongedcapillaryfillingtime NURSINGTB.COM ANS: C Keepingthefeetinadependentpositioncausesvenouspooling,resultinginredness,warmth,anddistended veins.Prolongedelevationwouldcausepallorandcoolness.Immobilizationorprolongedinactivitywould cause prolonged capillary fillingtime. DIF: Cognitive Level: Applying (Application) MSC: Client Needs: Physiologic Integrity: Physiologic Adaptation 15. Apatientisespeciallyworriedaboutanareaofskinonherfeetthathasturnedwhite.Thehealthcare providerhastoldherthatherconditionisvitiligo.Thenurseexplainstoherthatvitiligois: a. Caused by an excess of melaninpigment b. Caused by an excess of apocrine glands in herfeet c. Caused by the complete absence of melaninpigment d. Related to impetigo and can be treated with anointment ANS: C Vitiligoisthecompleteabsenceofmelaninpigmentinpatchyareasofwhiteorlightskinontheface,neck, hands,feet,bodyfolds,andaroundorificesotherwise,thedepigmentedskinisnormal. DIF: Cognitive Level: Applying (Application) MSC: Client Needs: Physiologic Integrity: Physiologic Adaptation 16. A patient tells the nurse that he has noticed that one of his moles has started to burn and bleed. When assessinghisskin,thenursepaysspecialattentiontothedangersignsforpigmentedlesionsandisconcerned with which additionalfinding? a. Colorvariation b. Borderregularity c. Symmetry oflesions d. Diameteroflessthan6mm ANS:A AbnormalcharacteristicsofpigmentedlesionsNaUrReSsuINmGmTaBri.CzeOdMinthemnemonicABCD:asymmetryof pigmented lesion, border irregularity, color variation, and diameter greater than 6 mm. DIF:CognitiveLevel:Understanding(Comprehension) MSC:ClientNeeds:HealthPromotionandMaintenance 17. Apatientcomestotheclinicandstatesthathehasnoticedthathisskinisredderthannormal.Thenurse understandsthatthisconditionisduetohyperemiaandknowsthatitcanbecausedby: a. Decreased amounts of bilirubin in theblood b. Excess blood in the underlying bloodvessels c. Decreased perfusion to the surroundingtissues d. Excessbloodinthedilatedsuperficialcapillaries ANS:D Erythemaisanintenserednessoftheskincausedbyexcessblood(hyperemia)inthedilatedsuperficial capillaries. DIF: Cognitive Level: Applying (Application) MSC: Client Needs: Physiologic Integrity: Physiologic Adaptation 18. Duringaskinassessment,thenursenoticesthataMexican-Americanpatienthasskinthatisyellowish- brown;however,theskinonthehardandsoftpalateispinkandthepatientssclerasarenotyellow.Fromthis finding, the nurse could probably ruleout: a. Pallor b. Jaundice c. Cyanosis d. Iron deficiency ANS:B Jaundiceisexhibitedbyayellowcolor,whichindicatesrisinglevelsofbilirubinintheblood.Jaundiceisfirst noticedinthejunctionofthehardandsoftpalateinthemouthandinthescleras. DIF: Cognitive Level: Analyzing (Analysis) MSC: Client Needs: Physiologic Integrity: Physiologic Adaptation 19. Ablackpatientisintheintensivecareunitbecauseofimpendingshockafteranaccident.Thenurse expects to find what characteristics in this patientsskin? a. Ruddyblue. b. Generalizedpallor. c. Ashen, gray, ordull. d. Patchy areas ofpallor. NURSINGTB.COM ANS: C Pallorattributabletoshock,withdecreasedperfusionandvasoconstriction,inblack-skinnedpeoplewillcause the skin to appear ashen, gray, ordull. DIF: Cognitive Level: Analyzing (Analysis) MSC: Client Needs: Physiologic Integrity: Physiologic Adaptation 20. Anolderadultwomanisbroughttotheemergencydepartmentafterbeingfoundlyingonthekitchenfloor for2days;sheisextremelydehydrated.Whatwouldthenurseexpecttoseeduringtheexamination? a. Smooth mucous membranes andlips b. Dry mucous membranes and crackedlips c. Pale mucousmembranes d. White patches on the mucousmembranes ANS: B Withdehydration,mucousmembranesappeardryandthelipslookparchedandcracked.Theotherresponses are not found indehydration. DIF: Cognitive Level: Applying (Application) MSC: Client Needs: Physiologic Integrity: Physiologic Adaptation 21. A42-year-oldwomancomplainsthatshehasnoticedseveralsmall,slightlyraised,brightreddotsonher chest. On examination, the nurse expects that the spots areprobably: a. Anasarca. b. Scleroderma. c. Senileangiomas. d. Latentmyeloma. ANS: C Cherry (senile) angiomas are small, smooth, slightly raised bright red dots that commonly appear on the trunk ofadultsover30yearsold. NURSINGTB.COM DIF: Cognitive Level: Applying (Application) MSC: Client Needs: Physiologic Integrity: Physiologic Adaptation 22. A65-year-oldmanwithemphysemaandbronchitishascometotheclinicforafollow-upappointment.On assessment, the nurse might expect to see whichfinding? a. Anasarca b. Scleroderma c. Pedalerythema d. Clubbingofthenails ANS:D Clubbingofthenailsoccurswithcongenitalcyanoticheartdiseaseandneoplasticandpulmonarydiseases.The other responses are assessment findings not associated with pulmonarydiseases. DIF: Cognitive Level: Analyzing (Analysis) MSC: Client Needs: Physiologic Integrity: Physiologic Adaptation 23. AnewborninfanthasDownsyndrome.Duringtheskinassessment,thenursenoticesatransientmottling inthetrunkandextremitiesinresponsetothecooltemperatureintheexaminationroom.Theinfantsmother alsonoticesthemottlingandaskswhatitis.Thenurseknowsthatthismottlingiscalled: a. Caf aulait. b. Carotenemia. c. Acrocyanosis. d. Cutismarmorata. ANS: D Persistent or pronounced cutis marmorata occurs with infants born with Down syndrome or those born prematurely and is a transient mottling in the trunk and extremities in response to cool room temperatures. A caf au lait spot is a large round or oval patch of light-brown pigmentation. Carotenemia produces a yellow- orangecolorinlight-skinnedpersons.Acrocyanosisisabluishcoloraroundthelips,handsandfingernails,and feet andtoenails. DIF: Cognitive Level: Understanding (Comprehension) MSC: Client Needs: Physiologic Integrity: Physiologic Adaptation 24. A35-year-oldpregnantwomancomestotheclinicforamonthlyappointment.Duringtheassessment,the nurse notices that she has a brown patch of hyperpigmentation on her face. The nurse continues the skin assessment aware that another finding maybe: NURSINGTB.COM a. Keratoses. b. Xerosis. c. Chloasma. d. Acrochordons. ANS: C Inpregnancy,skinchangescanincludestriae,lineanigra(abrownish-blacklinedownthemidline),chloasma (brown patches of hyperpigmentation), and vascular spiders. Keratoses are raised, thickened areas of pigmentationthatlookcrusted,scaly,andwarty.Xerosisisdryskin.Acrochordons,orskintags,occurmore often in the agingadult. DIF: Cognitive Level: Analyzing (Analysis) MSC: Client Needs: Physiologic Integrity: Physiologic Adaptation 25. A man has come in to the clinic for a skin assessment because he is worried he might have skin cancer. Duringtheskinassessmentthenursenoticesseveralareasofpigmentationthatlookgreasy,dark,andstuckon his skin. Which is the bestprediction? a. Senile lentigines, which do not becomecancerous b. Actinic keratoses, which are precursors to basal cellcarcinoma c. Acrochordons, which are precursors to squamous cellcarcinoma d. Seborrheickeratoses,whichdonotbecomecancerous ANS:D Seborrheic keratoses appear like dark, greasy, stuck-on lesions that primarily develop on the trunk. These lesionsdonotbecomecancerous.Senilelentiginesarecommonlycalledliverspotsandarenotprecancerous. Actinic (senile or solar) keratoses are lesions that are red-tan scaly plaques that increase over the years to becomeraisedandroughened.Theymayhaveasilvery-whitescaleadherenttotheplaque.Theyoccuronsun- exposed surfaces and are directly related to sun exposure. They are premalignant and may develop into squamouscellcarcinoma.Acrochordonsareskintagsandarenotprecancerous. DIF: Cognitive Level: Analyzing (Analysis) MSC: Client Needs: Physiologic Integrity: Physiologic Adaptation 26. A70-year-oldwomanwholovestogardenhassmall,flat,brownmaculesoverherarmsandhands.She asks, What causes these liver spots? The nurse tells her, Theyare: a. Signs of decreased hematocrit related toanemia. b. Duetothedestructionofmelanininyourskinfromexposuretothesun. NURSINGTB.COM c. Clusters of melanocytes that appear after extensive sunexposure. d. Areasofhyperpigmentationrelatedtodecreasedperfusionandvasoconstriction. ANS: C Liverspots,orsenilelentigines,areclustersofmelanocytesthatappearontheforearmsanddorsaofthehands after extensive sun exposure. The other responses are notcorrect. DIF: Cognitive Level: Understanding (Comprehension) MSC: Client Needs: Physiologic Integrity: Physiologic Adaptation 27. Thenursenoticesthatapatienthasasolid,elevated,circumscribedlesionthatislessthan1cmin diameter. When documenting this finding, the nurse reports this asa: a. Bulla. b. Wheal. c. Nodule. d. Papule. ANS: D Apapuleissomethingonecanfeel,issolid,elevated,circumscribed,lessthan1cmindiameter,andisdueto superficial thickening in the epidermis. A bulla is larger than 1 cm, superficial, and thin walled. A wheal is superficial, raised, transient, erythematous, and irregular in shape attributable to edema. A nodule is solid, elevated, hard or soft, and larger than 1cm. DIF: Cognitive Level: Understanding (Comprehension) MSC: Client Needs: Physiologic Integrity: Physiologic Adaptation 28. Thenursejustnotedfromthemedicalrecordthatthepatienthasalesionthatisconfluentinnature.On examination, the nurse expects tofind: a. Lesions that runtogether. b. Annular lesions that have growntogether. c. Lesionsarrangedinalinealonganerveroute. d. Lesionsthataregroupedorclusteredtogether. ANS: A Confluentlesions(aswithurticaria[hives])runtogether.Groupedlesionsareclusteredtogether.Annular lesionsarecircularinnature.ZosteriformlesioNnUsRaSreINaGrrTanBg.CedOMalonganerveroute. DIF: Cognitive Level: Understanding (Comprehension) MSC: Client Needs: Physiologic Integrity: Physiologic Adaptation 29. Apatienthashadaterribleitchforseveralmonthsthathehasbeencontinuouslyscratching.On examination, the nurse might expect tofind: a. Akeloid. b. Afissure. c. Keratosis. d. Lichenification. ANS: D Lichenification results from prolonged, intense scratching that eventually thickens the skin and produces tightlypackedsetsofpapules.Akeloidisahypertrophicscar.Afissureisalinearcrackwithabruptedges, whichextendsintothedermis;itcanbedryormoist.Keratosesarelesionsthatareraised,thickenedareasof pigmentation that appear crusted, scaly, andwarty. DIF: Cognitive Level: Understanding (Comprehension) MSC: Client Needs: Physiologic Integrity: Physiologic Adaptation 30. Aphysicianhasdiagnosedapatientwithpurpura.Afterleavingtheroom,anursingstudentasksthenurse whatthephysiciansawthatledtothatdiagnosis.Thenurseshouldsay,Thephysicianisreferringtothe: a. Bluedilationofbloodvesselsinastar-shapedlinearpatternonthelegs. b. Fieryred,star-shapedmarkingonthecheekthathasasolidcircularcenter. c. Confluentandextensivepatchofpetechiaeandecchymosesonthefeet. d. Tinyareasofhemorrhagethatarelessthan2mm,round,discrete,anddarkredincolor. ANS: C Purpura is a confluent and extensive patch of petechiae and ecchymoses and a flat macular hemorrhage observedingeneralizeddisorderssuchasthrombocytopeniaandscurvy.Thebluedilationofbloodvesselsina star-shapedlinearpatternonthelegsdescribesavenouslake.Thefieryred,star-shapedmarkingonthecheek that has a solid circular center describes a spider or star angioma. The tiny areas of hemorrhage that are less than 2 mm, round, discrete, and dark red in color describespetechiae. DIF: Cognitive Level: Understanding (Comprehension) MSC: Client Needs: Physiologic Integrity: Physiologic Adaptation 31. A mother has noticed that her son, who has been to a new babysitter, has some blisters and scabs on his faceandbuttocks.Onexamination,thenurseNnoUtiRcSesINmGoTisBt.,CthOiMn-roofedvesicleswithathinerythematousbase and suspects: a. Eczema. b. Impetigo. c. Herpeszoster. d. Diaperdermatitis. ANS: B Impetigoismoist,thin-roofedvesicleswithathinerythematousbaseandisacontagiousbacterialinfectionof the skin and most common in infants and children. Eczema is characterized by erythematous papules and vesicles with weeping, oozing, and crusts. Herpes zoster (i.e., chickenpox or varicella) is characterized by small,tightvesiclesthatareshinywithanerythematousbase.Diaperdermatitisischaracterizedbyred,moist maculopapular patches with poorly definedborders. DIF: Cognitive Level: Applying (Application) MSC: Client Needs: Physiologic Integrity: Physiologic Adaptation 32. Thenursenoticesthataschool-agedchildhasbluish-white,red-basedspotsinhermouththatareelevated approximately1to3mm.Whatothersignswouldthenurseexpecttofindinthispatient? a. Pink, papular rash on the face andneck b. Pruritic vesicles over her trunk andneck c. Hyperpigmentation on the chest, abdomen, and back of thearms d. Red-purple,maculopapular,blotchyrashbehindtheearsandontheface ANS:D With measles (rubeola), the examiner assesses a red-purple, blotchy rash on the third or fourth day of illness thatappearsfirstbehindtheears,spreadsovertheface,andthenovertheneck,trunk,arms,andlegs.Therash appears coppery and does not blanch. The bluish-white, red-based spots in the mouth are known as Koplik spots. DIF: Cognitive Level: Analyzing (Analysis) MSC: Client Needs: Physiologic Integrity: Physiologic Adaptation 33. Thenurseisassessingtheskinofapatientwhohasacquiredimmunodeficiencysyndrome(AIDS)and notices multiple patchlike lesions on the temple and beard area that are faint pink in color. The nurse recognizes these lesionsas: a. Measles(rubeola). b. Kaposissarcoma. c. Angiomas. d. Herpeszoster. NURSINGTB.COM ANS: B Kaposissarcomaisavasculartumorthat,intheearlystages,appearsasmultiple,patchlike,faintpinklesions overthepatientstempleandbeardareas.Measlesischaracterizedbyared-purplemaculopapularblotchyrash that appears on the third or fourth day of illness. The rash is first observed behind the ears, spreads over the face, and then spreads over the neck, trunk, arms, and legs. Cherry (senile) angiomas are small (1 to 5 mm), smooth,slightlyraisedbrightreddotsthatcommonlyappearonthetrunkinalladultsover30yearsold. Herpeszostercausesvesiclesupto1cminsizethatareelevatedwithacavitycontainingclearfluid. DIF: Cognitive Level: Applying(Application) MSC: Client Needs: Physiologic Integrity: Physiologic Adaptation 34. A45-year-oldfarmercomesinforaskinevaluationandcomplainsofhairlossonhishead.Hishairseems to be breaking off in patches, and he notices some scaling on his head. The nurse begins the examination suspecting: a. Tineacapitis. b. Folliculitis. c. Toxicalopecia. d. Seborrheicdermatitis. ANS: A Tineacapitisisroundedpatchyhairlossonthescalp,leavingbroken-offhairs,pustules,andscalesontheskin, and is caused by a fungal infection. Lesions are fluorescent under a Wood light and are usually observed in children and farmers; tinea capitis is highlycontagious. DIF: Cognitive Level: Analyzing (Analysis) MSC: Client Needs: Physiologic Integrity: Physiologic Adaptation 35. Amotherbringsherchildintotheclinicforanexaminationofthescalpandhair.Shestatesthatthechild hasdevelopedirregularlyshapedpatcheswithbroken-off,stublikehairinsomeplaces;sheisworriedthatthis conditioncouldbesomeformofprematurebaldness.Thenursetellsherthatitis: a. Folliculitis that can be treated with anantibiotic. b. Traumatic alopecia that can be treated with antifungalmedications. c. Tinea capitis that is highly contagious and needs immediateattention. d. Trichotillomania;herchildprobabNlyUhRaSsIaNhGaTbBit.CoOfaMbsentmindedlytwirlingherhair. ANS: D Trichotillomania,self-inducedhairloss,isusuallyduetohabit.Itformsirregularlyshapedpatcheswith broken-off, stublike hairs of varying lengths. A person is never completely bald. It occurs as a child absentmindedlyrubsortwirlstheareawhilefallingasleep,reading,orwatchingtelevision. DIF: Cognitive Level: Applying (Application) MSC: Client Needs: Physiologic Integrity: Physiologic Adaptation 36. Thenursehasdiscovereddecreasedskinturgorinapatientandknowsthatthisfindingisexpectedin whichcondition? a. Severeobesity b. Childhood growthspurts c. Severedehydration d. Connectivetissuedisorderssuchasscleroderma ANS:C Decreasedskinturgorisassociatedwithseveredehydrationorextremeweightloss. DIF: Cognitive Level: Understanding(Comprehension) MSC: Client Needs: Physiologic Integrity: Physiologic Adaptation 37. Whileperforminganassessmentofa65-year-oldmanwithahistoryofhypertensionandcoronaryartery disease,thenursenoticesthepresenceofbilateralpittingedemainthelowerlegs.Theskinispuffyandtight but normal in color. No increased redness or tenderness is observed over his lower legs, and the peripheral pulses are equal and strong. In this situation, the nurse suspects that the likely cause of the edema is which condition? a. Heartfailure b. Venousthrombosis c. Localinflammation d. Blockageoflymphaticdrainage ANS:A Bilateraledemaoredemathatisgeneralizedovertheentirebodyiscausedbyacentralproblemsuchasheart failureorkidneyfailure.Unilateraledemausuallyhasalocalorperipheralcause. DIF: Cognitive Level: Analyzing (Analysis) NURSINGTB.COM MSC: Client Needs: Physiologic Integrity: Physiologic Adaptation 38. A40-year-oldwomanreportsachangeinmolesize,accompaniedbycolorchanges,itching,burning,and bleedingoverthepastmonth.Shehasadarkcomplexionandhasnofamilyhistoryofskincancer,butshehas had many blistering sunburns in the past. The nursewould: a. Tellthepatienttowatchthelesionandreportbackin2months. b. Referthepatientbecauseofthesuggestionofmelanomaonthebasisofhersymptoms. c. Askadditionalquestionsregardingenvironmentalirritantsthatmayhavecausedthiscondition. d. Tellthepatientthatthesesignssuggestacompoundnevus,whichisverycommoninyoungto middle-agedadults. ANS: B The ABCD danger signs of melanoma are asymmetry, border irregularity, color variation, and diameter. In addition,individualsmayreportachangeinsize,thedevelopmentofitching,burning,andbleeding,oranew- pigmentedlesion.Anyoneofthesesignsraisesthesuggestionofmelanomaandwarrantsimmediatereferral. DIF: Cognitive Level: Analyzing (Analysis) MSC: Client Needs: Physiologic Integrity: Physiologic Adaptation 39. Thenurseisassessingforclubbingofthefingernailsandexpectstofind: a. Nail bases that are firm and slightlytender. b. Curved nails with a convex profile and ridges across thenails. c. Nailbasesthatfeelspongywithanangleofthenailbaseof150degrees. d. Nailbaseswithanangleof180degreesorgreaterandnailbasesthatfeelspongy. ANS: D Thenormalnailisfirmatitsbaseandhasanangleof160degrees.Inclubbing,theanglestraightensto180 degrees or greater and the nail base feelsspongy. DIF: Cognitive Level: Understanding (Comprehension) MSC: Client Needs: Physiologic Integrity: Physiologic Adaptation 40. Thenurseisassessingapatientwhohasliverdiseaseforjaundice.Whichoftheseassessmentfindingsis indicative of truejaundice? a. Yellow patches in the outersclera b. YellowcolorofthesclerathatexteNnUdRsSuIpNtGoTthBe.CirOisM c. Skin that appears yellow when examined under lowlight d. Yellowdepositsonthepalmsandsolesofthefeetwherejaundicefirstappears ANS:B The yellow sclera of jaundice extends up to the edge of the iris. Calluses on the palms and soles of the feet oftenappearyellowbutarenotclassifiedasjaundice.Scleraljaundiceshouldnotbeconfusedwiththenormal yellowsubconjunctivalfattydepositsthatarecommonintheouterscleraofdark-skinnedpersons. DIF: Cognitive Level: Understanding (Comprehension) MSC: Client Needs: Physiologic Integrity: Physiologic Adaptation 41. Thenurseisassessingforinflammationinadark-skinnedperson.Whichtechniqueisthebest? a. Assessing the skin for cyanosis andswelling b. Assessing the oral mucosa for generalizederythema c. Palpating the skin for edema and increasedwarmth d. Palpating for tenderness and local areas ofecchymosis ANS: C Becauseinflammationcannotbeseenindark-skinnedpersons,palpatingtheskinforincreasedwarmth,for tautortightlypulledsurfacesthatmaybeindicativeofedema,andforahardeningofdeeptissuesorblood vessels is oftennecessary. DIF: Cognitive Level: Applying (Application) MSC: Client Needs: Physiologic Integrity: Physiologic Adaptation 42. Afewdaysafterasummerhikingtrip,a25-year-oldmancomestotheclinicwitharash.Onexamination, thenursenotesthattherashisred,macular,withabullseyepatternacrosshismidriffandbehindhisknees. The nursesuspects: a. Rubeola. b. Lymedisease. c. Allergy to mosquitobites. d. Rocky Mountain spottedfever. ANS: B LymediseaseoccursinpeoplewhospendtimeoutdoorsinMaythroughSeptember.Thefirstdiseasestate exhibitsthedistinctivebullseyeandaredmaNURSINGTB.COMhthatradiatesfromthesiteofthetickbite withsomecentralclearing.Therashspreads5cmorlarger,andisusuallyintheaxilla,midriff,inguinal,or behindtheknee,withregionallymphadenopathy. DIF: Cognitive Level: Analyzing(Analysis) MSC: Client Needs: Physiologic Integrity: Physiologic Adaptation 43. A52-year-oldwomanhasapapuleonhernosethathasrounded,pearlybordersandacentralredulcer.She said she first noticed it several months ago and that it has slowly grown larger. The nurse suspects which condition? a. Acne b. Basal cellcarcinoma c. Melanoma d. Squamouscellcarcinoma ANS:B Basal cell carcinoma usually starts as a skin-colored papule that develops rounded, pearly borders with a centralredulcer.Itisthemostcommonformofskincancerandgrowsslowly.Thisdescriptiondoesnotfit acnelesions. DIF: Cognitive Level: Applying (Application) MSC: Client Needs: Physiologic Integrity: Physiologic Adaptation 44. A father brings in his 2-month-old infant to the clinic because the infant has had diarrhea for the last 24 hours. He says his baby has not been able to keep any formula down and that the diarrhea has been at least every2hours.Thenursesuspectsdehydration.Thenurseshouldtestskinmobilityandturgorovertheinfants: a. Sternum. b. Forehead. c. Forearms. d. Abdomen. ANS: D Mobilityandturgoraretestedovertheabdomeninaninfant.Poorturgor,ortenting,indicatesdehydrationor malnutrition.Theothersitesarenotappropriateforcheckingskinturgorinaninfant. DIF: Cognitive Level: Analyzing (Analysis) MSC: Client Needs: Physiologic Integrity: Physiologic Adaptation 45. Asemiconsciouswomanisbroughttotheemergencydepartmentaftershewasfoundonthefloorinher kitchen.Herface,nailbeds,lips,andoralmucNoUsaRaSrIeNaGbTrBig.ChOtcMherry-redcolor.Thenursesuspectsthatthis coloring is dueto: a. Polycythemia. b. Carbon monoxidepoisoning. c. Carotenemia. d. Uremia. ANS: B Abrightcherry-redcoloringintheface,uppertorso,nailbeds,lips,andoralmucosaappearsincasesofcarbon monoxidepoisoning. DIF: Cognitive Level: Analyzing (Analysis) MSC: Client Needs: Physiologic Integrity: Physiologic Adaptation 46. Apatienthasbeenadmittedforseverepsoriasis.Thenurseexpectstoseewhatfindinginthepatients fingernails? a. Splinterhemorrhages b. Paronychia c. Pitting d. Beau lines ANS:C Sharplydefinedpittingandcrumblingofthenails,eachwithdistaldetachmentcharacterizepittingnailsand are associated withpsoriasis. DIF: Cognitive Level: Applying (Application) MSC: Client Needs: Physiologic Integrity: Physiologic Adaptation MULTIPLE RESPONSE 1. Thenurseispreparingforacertificationcourseinskincareandneedstobefamiliarwiththevariouslesions thatmaybeidentifiedonassessmentoftheskin.Whichofthefollowingdefinitionsarecorrect?Selectallthat apply. a. Petechiae:Tinypunctatehemorrhages,1to3mm,roundanddiscrete,darkred,purple,orbrownin color b. Bulla: Elevated, circumscribed lesion filled with turbid fluid(pus) c. Papule: Hypertrophicscar NURSINGTB.COM d. Vesicle: Known as a frictionblister e. Nodule:Solid,elevated,andhardorsoftgrowththatislargerthan1cm ANS: A, D,E Apustuleisanelevated,circumscribedlesionfilledwithturbidfluid(pus).Ahypertrophicscarisakeloid.A bullaislargerthan1cmandcontainsclearfluid.Apapuleissolidandelevatedbutmeasureslessthan1cm. DIF: Cognitive Level: Understanding (Comprehension) MSC: Client Needs: Physiologic Integrity: Physiologic Adaptation 2. Apatienthasbeenadmittedtoahospitalafterthestaffinthenursinghomenoticedapressureulcerinhis sacralarea.ThenurseexaminesthepressureulceranddeterminesthatitisastageIIulcer.Whichofthese findingsarecharacteristicofastageIIpressureulcer?Selectallthatapply. a. Intact skin appears red but is notbroken. b. Partialthicknessskinerosionisobservedwithalossofepidermisordermis. c. Ulcer extends into the subcutaneoustissue. d. Localized redness in light skin will blanch with fingertippressure. e. Open blister areas have a red-pink woundbed. f. Patches of eschar cover parts of thewound. ANS: B, E StageIpressureulcershaveintactskinthatappearsredbutisnotbroken,andlocalizedrednessinintactskin willblanchewithfingertippressure.StageIIpressureulcershavepartialthicknessskinerosionwithalossof epidermis or also the dermis; open blisters have a red-pink wound bed. Stage III pressure ulcers are full thickness, extending into the subcutaneous tissue; subcutaneous fat may be seen but not muscle, bone, or tendon. Stage IV pressure ulcers involve all skin layers and extend into supporting tissue, exposing muscle, bone,andtendon.Slough(stringymatterattachedtothewoundbed)oreschar(blackorbrownnecrotictissue) may bepresent. DIF: Cognitive Level: Applying (Application) MSC: Client Needs: Physiologic Integrity: Physiologic Adaptation NURSINGTB.COM Chapter 14: Head, Face, Neck, and Regional Lymphatics MULTIPLE CHOICE 1. Aphysiciantellsthenursethatapatientsvertebraprominensistenderandasksthenursetoreevaluatethe area in 1 hour. The area of the body the nurse will assessis: a. Just above thediaphragm. b. Just lateral to the kneecap. c. At the level of the C7vertebra. d. At the level of the T11vertebra. ANS: C TheC7vertebrahasalongspinousprocess,calledthevertebraprominens,whichispalpablewhentheheadis flexed. DIF: Cognitive Level: Applying (Application) MSC: Client Needs: Physiologic Integrity: Physiologic Adaptation 2. Amotherbringsher2-month-olddaughterinforanexaminationandsays,Mydaughterrolledoveragainst thewall,andnowIhavenoticedthatshehastNhUisRsSpIoNtGthTaBti.CsOsoMftonthetopofherhead.Issomethingterribly wrong? The nurses best response would be: a. Perhapsthatcouldbearesultofyourdietaryintakeduringpregnancy. b. Yourbabymayhavecraniosynostosis,adiseaseofthesuturesofthebrain. c. Thatsoftspotmaybeanindicationofcretinismorcongenitalhypothyroidism. d. Thatsoftspotisnormal,andactuallyallowsforgrowthofthebrainduringthefirstyearofyour babyslife. ANS: D Membrane-coveredsoftspotsallowforgrowthofthebrainduringthefirstyearoflife.Theygraduallyossify; thetriangular-shapedposteriorfontanelisclosedby1to2months,andthediamond-shapedanteriorfontanel closes between 9 months and 2years. DIF: Cognitive Level: Applying (Application) MSC: Client Needs: Health Promotion and Maintenance 3. Thenursenoticesthatapatientspalpebralfissuresarenotsymmetric.Onexamination,thenursemayfind that damage has occurred to which cranial nerve(CN)? a. III b. V c. VII d. VIII ANS: C FacialmusclesaremediatedbyCNVII;asymmetryofpalpebralfissuresmaybeattributabletodamagetoCN VII (Bellpalsy). DIF: Cognitive Level: Applying (Application) MSC: Client Needs: Physiologic Integrity: Physiologic Adaptation 4. Apatientisunabletodifferentiatebetweensharpanddullstimulationtobothsidesofherface.Thenurse suspects: a. Bellpalsy. b. Damage to the trigeminalnerve. c. Frostbite with resultant paresthesia to thecheeks. d. Scleroderma. NURSINGTB.COM ANS: B FacialsensationsofpainortoucharemediatedbyCNV,whichisthetrigeminalnerve.Bellpalsyisassociated withCNVIIdamage.Frostbiteandsclerodermaarenotassociatedwiththisproblem. DIF: Cognitive Level: Applying (Application) MSC: Client Needs: Physiologic Integrity: Physiologic Adaptation 5. Whenexaminingthefaceofapatient,thenurseisawarethatthetwopairsofsalivaryglandsthatare accessible to examinationarethe and glands. a. Occipital;submental b. Parotid;jugulodigastric c. Parotid;submandibular d. Submandibular; occipital ANS:C Two pairs of salivary glands accessible to examination on the face are the parotid glands, which are in the cheeksoverthemandible,anteriortoandbelowtheear;andthesubmandibularglands,whicharebeneaththe mandibleattheangleofthejaw.Theparotidglandsarenormallynonpalpable. DIF: Cognitive Level: Understanding (Comprehension) MSC: Client Needs: Safe and Effective Care Environment: Management of Care 6. Apatientcomestothecliniccomplainingofneckandshoulderpainandisunabletoturnherhead.The nurse suspects damagetoCN andproceedswiththeexaminationby . a. XI; palpating the anterior and posteriortriangles b. XI; asking the patient to shrug her shoulders againstresistance c. XII; percussing the sternomastoid and submandibular neckmuscles d. XII;assessingforapositiveRombergsign ANS:B The major neck muscles are the sternomastoid and the trapezius. They are innervated by CN XI, the spinal accessory.Theinnervatedmusclesassistwithheadrotationandheadflexion,movementoftheshoulders,and extension and turning of thehead. DIF: Cognitive Level: Analyzing (Analysis) MSC:ClientNeeds:SafeandEffectiveCareENnUvRirSoInNmGeTnBt:.CMOaMnagementofCare 7. WhenexaminingapatientsCNfunction,thenurseremembersthatthemusclesintheneckthatare innervated by CN XI arethe: a. Sternomastoid andtrapezius. b. Spinalaccessoryandomohyoid. c. Trapeziusandsternomandibular. d. Sternomandibular and spinalaccessory. ANS: A Themajorneckmusclesarethesternomastoidandthetrapezius.TheyareinnervatedbyCNXI,thespinal accessory. DIF:CognitiveLevel:Remembering(Knowledge) MSC: Client Needs:General 8. Apatientslaboratorydatarevealanelevatedthyroxine(T4)level.Thenursewouldproceedwithan examinationofthe gland. a. Thyroid b. Parotid c. Adrenal d. Parathyroid ANS:A ThethyroidglandisahighlyvascularendocrineglandthatsecretesT4andtriiodothyronine(T3).Theother glands do not secreteT4. DIF: Cognitive Level: Understanding (Comprehension) MSC: Client Needs: Safe and Effective Care Environment: Management of Care 9. A patient says that she has recently noticed a lump in the front of her neck below her Adams apple that seemstobegettingbigger.Duringtheassessment,thefindingthatleadsthenursetosuspectthatthismaynot be a cancerous thyroid nodule is that the lump(nodule): a. Istender. b. Is mobile and nothard. c. DisappearswhenthepatientsmileNs.URSINGTB.COM d. Is hard and fixed to the surroundingstructures. ANS: B Painless,rapidlygrowingnodulesmaybecancerous,especiallytheappearanceofasinglenoduleinayoung person.However,cancerousnodulestendtobehardandfixedtosurroundingstructures,notmobile. DIF: Cognitive Level: Applying (Application) MSC: Client Needs: Physiologic Integrity: Physiologic Adaptation 10. Thenursenoticesthatapatientssubmentallymphnodesareenlarged.Inanefforttoidentifythecauseof the node enlargement, the nurse would assess thepatients: a. Infraclaviculararea. b. Supraclaviculararea. c. Area distal to the enlargednode. d. Area proximal to the enlargednode. ANS: D Whennodesareabnormal,thenurseshouldchecktheareaintowhichtheydrainforthesourceoftheproblem. Theareaproximal(upstream)tothelocationoftheabnormalnodeshouldbeexplored. DIF: Cognitive Level: Analyzing (Analysis) MSC: Client Needs: Safe and Effective Care Environment: Management of Care 11. Thenurseisawarethatthefourareasinthebodywherelymphnodesareaccessiblearethe: a. Head, breasts, groin, andabdomen. b. Arms, breasts, inguinal area, andlegs. c. Head and neck, arms, breasts, andaxillae. d. Head and neck, arms, inguinal area, andaxillae. ANS: D Nodesarelocatedthroughoutthebody,buttheyareaccessibletoexaminationonlyinfourareas:headand neck, arms, inguinal region, andaxillae. DIF: Cognitive Level: Remembering (Knowledge) MSC: Client Needs: Safe and Effective Care Environment: Management of Care 12. AmotherbringshernewborninforanassNesUsRmSeInNtGanTdB.aCskOsM,Istheresomethingwrongwithmybaby?His head seems so big. Which statement is true regarding the relative proportions of the head and trunk of the newborn? a. At birth, the head is one fifth the totallength. b. Headcircumferenceshouldbegreaterthanchestcircumferenceatbirth. c. Theheadsizereaches90%ofitsfinalsizewhenthechildis3yearsold. d. Whentheanteriorfontanelclosesat2months,theheadwillbemoreproportionedtothebody. ANS: B The nurse recognizes that during the fetal period, head growth predominates. Head size is greater than chest circumferenceatbirth,andtheheadsizegrowsduringchildhood,reaching90%ofitsfinalsizewhenthechild is age 6years. DIF:CognitiveLevel:Understanding(Comprehension) MSC:ClientNeeds:HealthPromotionandMaintenance 13. Apatient,an85-year-oldwoman,iscomplainingaboutthefactthatthebonesinherfacehavebecome more noticeable. What explanation should the nurse giveher? a. Dietslowinproteinandhighincarbohydratesmaycauseenhancedfacialbones. b. Bonescanbecomemorenoticeableifthepersondoesnotuseadermatologicallyapproved moisturizer. c. Morenoticeablefacialbonesareprobablyduetoacombinationoffactorsrelatedtoaging,suchas decreased elasticity, subcutaneous fat, and moisture in herskin. d. Facialskinbecomesmoreelasticwithage.Thisincreasedelasticitycausestheskintobemore taught, drawing attention to the facialbones. ANS: C Thefacialbonesandorbitsappearmoreprominentintheagingadult,andthefacialskinsags,whichis attributabletodecreasedelasticity,decreasedsubcutaneousfat,anddecreasedmoistureintheskin. DIF:CognitiveLevel:Understanding(Comprehension) MSC:ClientNeeds:HealthPromotionandMaintenance 14. Apatientreportsexcruciatingheadachepainononesideofhishead,especiallyaroundhiseye,forehead, and cheek that has lasted approximately to 2 hours, occurring once or twice each day. The nurse should suspect: a. Hypertension. b. Clusterheadaches. c. Tensionheadaches. d. Migraineheadaches. NURSINGTB.COM ANS: B Clusterheadachesproducepainaroundtheeye,temple,forehead,andcheekandareunilateralandalwayson thesamesideofthehead.Theyareexcruciatingandoccuronceortwiceperdayandlastto2hourseach. DIF: Cognitive Level: Applying (Application) MSC: Client Needs: Physiologic Integrity: Physiologic Adaptation 15. Apatientcomplainsthatwhilestudyingforanexaminationhebegantonoticeasevereheadacheinthe frontotemporalareaofhisheadthatisthrobbingandissomewhatrelievedwhenheliesdown.Hetellsthe nursethathismotheralsohadtheseheadaches.Thenursesuspectsthathemaybesufferingfrom: a. Hypertension. b. Clusterheadaches. c. Tensionheadaches. d. Migraineheadaches. ANS: D Migraineheadachestendtobesupraorbital,retroorbital,orfrontotemporalwithathrobbingquality.Theyare severeinqualityandarerelievedbylyingdown.Migrainesareassociatedwithafamilyhistoryofmigraine headaches. DIF: Cognitive Level: Applying (Application) MSC: Client Needs: Physiologic Integrity: Physiologic Adaptation 16. A19-year-oldcollegestudentisbroughttotheemergencydepartmentwithasevereheadachehedescribes as,LikenothingIveeverhadbefore.Histemperatureis40C,andhehasastiffneck.Thenurselooksforother signs and symptoms of whichproblem? a. Headinjury b. Clusterheadache c. Migraineheadache d. Meningealinflammation ANS: D NURSINGTB.COM Theacuteonsetofneckstiffnessandpainalongwithheadacheandfeveroccurswithmeningealinflammation. A severe headache in an adult or child who has never had it before is a red flag. Head injury and cluster or migraine headaches are not associated with a fever or stiffneck. DIF: Cognitive Level: Analyzing (Analysis) MSC:ClientNeeds:PhysiologicIntegrity:PhysiologicAdaptation 17. Duringawell-babycheckup,thenursenoticesthata1-week-oldinfantsfacelookssmallcomparedwith his cranium, which seems enlarged. On further examination, the nurse also notices dilated scalp veins and downcast or setting sun eyes. The nurse suspects whichcondition? a. Craniotabes b. Microcephaly c. Hydrocephalus d. Caput succedaneum ANS:C Hydrocephalus occurs with the obstruction of drainage of cerebrospinal fluid that results in excessive accumulation, increasing intracranial pressure, and an enlargement of the head. The face looks small, comparedwiththeenlargedcranium,anddilatedscalpveinsanddowncastorsettingsuneyesarenoted. Craniotabes is a softening of the skulls outer layer. Microcephaly is an abnormally small head. A caput succedaneumisedematousswellingandecchymosisofthepresentingpartoftheheadcausedbybirthtrauma. DIF: Cognitive Level: Applying (Application) MSC: Client Needs: Health Promotion and Maintenance 18. Thenurseneedstopalpatethetemporomandibularjointforcrepitation.Thisjointislocatedjustbelowthe temporal artery and anterior tothe: a. Hyoidbone. b. Vagusnerve. c. Tragus. d. Mandible. ANS: C Thetemporomandibularjointisjustbelowthetemporalarteryandanteriortothetragus. DIF: Cognitive Level: Understanding(Comprehension) MSC: Client Needs: General 19. ApatienthascomeinforanexaminationaNnUdRsStaINteGs,TIBh.aCvOeMthisspotinfrontofmyearlobeonmycheek thatseemstobegettingbiggerandistender.Whatdoyouthinkitis?Thenursenotesswellingbelowtheangle of the jaw and suspects that it could be an inflammation ofhis: a. Thyroidgland. b. Parotidgland. c. Occipital lymphnode. d. Submental lymphnode. ANS: B Swellingoftheparotidglandisevidentbelowtheangleofthejawandismostvisiblewhentheheadis extended.Painfulinflammationoccurswithmumps,andswellingalsooccurswithabscessesortumors. Swelling occurs anterior to the lower earlobe. DIF:CognitiveLevel:Applying(Application) MSC: Client Needs: Physiologic Integrity: Physiologic Adaptation 20. Amalepatientwithahistoryofacquiredimmunodeficiencysyndrome(AIDS)hascomeinforan examinationandhestates,IthinkthatIhavethemumps.Thenursewouldbeginbyexaminingthe: a. Thyroidgland. b. Parotidgland. c. Cervical lymphnodes. d. Mouth and skin forlesions. ANS: B Theparotidglandmaybecomeswollenwiththeonsetofmumps,andparotidenlargementhasbeenfoundwith human immunodeficiency virus(HIV). DIF: Cognitive Level: Applying (Application) MSC: Client Needs: Physiologic Integrity: Physiologic Adaptation 21. Thenursesuspectsthatapatienthashyperthyroidism,andthelaboratorydataindicatethatthepatientsT4andT3hormonelevelsareelevated.Whichofthesefindingswouldthenursemostlikelyfindonexamination? a. Tachycardia b. Constipation c. Rapiddyspnea d. Atrophiednodularthyroidgland NURSINGTB.COM ANS: A T4andT3arethyroidhormonesthatstimulatetherateofcellularmetabolism,resultingintachycardia.Withan enlargedthyroidglandasinhyperthyroidism,thenursemightexpecttofinddiffuseenlargement(goiter)ora nodular lump but not an atrophied gland. Dyspnea and constipation are not findings associated with hyperthyroidism. DIF: Cognitive Level: Analyzing (Analysis) MSC: Client Needs: Physiologic Integrity: Physiologic Adaptation 22. A visitor from Poland who does not speak English seems to be somewhat apprehensive about the nurse examininghisneck.Hewouldprobablybemorecomfortablewiththenurseexamininghisthyroidglandfrom: a. Behind with the nurses hands placed firmly around hisneck. b. Thesidewiththenurseseyesavertedtowardtheceilingandthumbsonhisneck. c. Thefrontwiththenursesthumbsplacedoneithersideofhistracheaandhisheadtiltedforward. d. Thefrontwiththenursesthumbsplacedoneithersideofhistracheaandhisheadtiltedbackward. ANS: C Examiningthispatientsthyroidglandfromthebackmaybeunsettlingforhim.Itwouldbebesttoexaminehis thyroid gland using the anterior approach, asking him to tip his head forward and to the right and then to the left. DIF: Cognitive Level: Applying (Application) MSC: Client Needs: Safe and Effective Care Environment: Management of Care 23. A patients thyroid gland is enlarged, and the nurse is preparing to auscultate the thyroid gland for the presenceofabruit.Abruitisa soundthatisheardbestwiththe of thestethoscope. a. Low gurgling;diaphragm b. Loud, whooshing, blowing;bell c. Soft, whooshing, pulsatile;bell d. High-pitchedtinkling;diaphragm ANS:C Ifthethyroidglandisenlarged,thenthenurseshouldauscultateitforthepresenceofabruit,whichisasoft, pulsatile,whooshing,blowingsoundheardbestwiththebellofthestethoscope. DIF:CognitiveLevel:Understanding(ComprNehUeRnSsiIoNnG)TB.COM MSC: Client Needs: Safe and Effective Care Environment: Management of Care 24. The nurse notices that an infant has a large, soft lump on the side of his head and that his mother is very concerned.Shetellsthenursethatshenoticedthelumpapproximately8hoursafterherbabysbirthandthatit seems to be getting bigger. One possible explanation for thisis: a. Hydrocephalus. b. Craniosynostosis. c. Cephalhematoma. d. Caputsuccedaneum. ANS: C Acephalhematomaisasubperiostealhemorrhagethatistheresultofbirthtrauma.Itissoft,fluctuant,andwell definedoveronecranialbone.Itappearsseveralhoursafterbirthandgraduallyincreasesinsize. DIF: Cognitive Level: Analyzing (Analysis) MSC: Client Needs: Physiologic Integrity: Physiologic Adaptation 25. Amotherbringsinhernewborninfantforanassessmentandtellsthenursethatshehasnoticedthat wheneverhernewbornsheadisturnedtotherightside,shestraightensoutthearmandlegonthesameside andflexestheoppositearmandleg.Afterobservingthisonexamination,thenursetellsherthatthisreflexis: a. Abnormal and is called the atonic neckreflex. b. Normal and should disappear by the first year oflife. c. Normalandiscalledthetonicneckreflex,whichshoulddisappearbetween3and4monthsofage. d. Abnormal.Thebabyshouldbeflexingthearmandlegontherightsideofhisbodywhenthehead is turned to theright. ANS: C By2weeks,theinfantshowsthetonicneckreflexwhensupineandtheheadisturnedtooneside(extensionof samearmandleg,flexionofoppositearmandleg).Thetonicneckreflexdisappearsbetween3and4months ofage. DIF: Cognitive Level: Analyzing (Analysis) MSC: Client Needs: Health Promotion and Maintenance 26. Duringanadmissionassessment,thenursenoticesthatamalepatienthasanenlargedandratherthick skull. The nurse suspects acromegaly and would further assessfor: a. Exophthalmos. b. Bowed longbones. c. Coarse facialfeatures. d. Acorn-shapedcranium. NURSINGTB.COM ANS: C Acromegaly is excessive secretion of growth hormone that creates an enlarged skull and thickened cranial bones. Patients will have elongated heads, massive faces, prominent noses and lower jaws, heavy eyebrow ridges,andcoarsefacialfeatures.Exophthalmosisassociatedwithhyperthyroidism.Bowedlongbonesandan acorn-shaped cranium result from Pagetdisease. DIF: Cognitive Level: Analyzing (Analysis) MSC: Client Needs: Physiologic Integrity: Physiologic Adaptation 27. WhenexaminingchildrenaffectedwithDownsyndrome(trisomy21),thenurselooksforthepossible presenceof: a. Eardysplasia. b. Long, thinneck. c. Protruding thintongue. d. Narrow and raised nasalbridge. ANS: A With the chromosomal aberration trisomy 21, also known as Down syndrome, head and face characteristics may include upslanting eyes with inner epicanthal folds, a flat nasal bridge, a small broad flat nose, a protrudingthicktongue,eardysplasia,ashortbroadneckwithwebbing,andsmallhandswithasinglepalmar crease. DIF: Cognitive Level: Understanding (Comprehension) MSC: Client Needs: Physiologic Integrity: Physiologic Adaptation 28. Apatientvisitstheclinicbecausehehasrecentlynoticedthattheleftsideofhismouthisparalyzed.He statesthathecannotraisehiseyebroworwhistle.Thenursesuspectsthathehas: a. Cushingsyndrome. b. Parkinsondisease. c. Bellpalsy. d. ExperiencedacerebrovascularaccNidUeRntS(ICNVGTAB).oCrOsMtroke. ANS: D Withanuppermotorneuronlesion,aswithaCVA,thepatientwillhaveparalysisoflowerfacialmuscles,but the upper half of the face will not be affected owing to the intact nerve from the unaffected hemisphere. The person is still able to wrinkle the forehead and close theeyes. DIF: Cognitive Level: Applying (Application) MSC: Client Needs: Physiologic Integrity: Physiologic Adaptation 29. Awomancomestotheclinicandstates,Ivebeensickforsolong!Myeyeshavegottensopuffy,andmy eyebrowsandhairhavebecomecoarseanddry.Thenursewillassessforothersignsandsymptomsof: a. Cachexia. b. Parkinsonsyndrome. c. Myxedema. d. Scleroderma. ANS: C Myxedema (hypothyroidism) is a deficiency of thyroid hormone that, when severe, causes a nonpitting edema ormyxedema.Thepatientwillhaveapuffyedematousface,especiallyaroundtheeyes(periorbitaledema); coarse facial features; dry skin; and dry, coarse hair andeyebrows. DIF: Cognitive Level: Applying (Application) MSC: Client Needs: Physiologic Integrity: Physiologic Adaptation 30. Duringanexaminationofafemalepatient,thenursenoteslymphadenopathyandsuspectsanacute infection. Acutely infected lymph nodes wouldbe: a. Clumped. b. Unilateral. c. Firm but freelymovable. d. Firm andnontender. ANS: C Acutelyinfectedlymphnodesarebilateral,enlarged,warm,tender,andfirmbutfreelymovable.Unilaterally enlarged nodes that are firm and nontender may indicatecancer. DIF: Cognitive Level: Understanding (Comprehension) MSC: Client Needs: Physiologic Integrity: Physiologic Adaptation NURSINGTB.COM 31. Thephysicianreportsthatapatientwithanecktumorhasatrachealshift.Thenurseisawarethatthis means that the patients tracheais: a. Pulled to the affectedside. b. Pushed to the unaffectedside. c. Pulleddownward. d. Pulled downward in a rhythmicpattern. ANS: B The trachea is pushed to the unaffected side with an aortic aneurysm, a tumor, unilateral thyroid lobe enlargement, or a pneumothorax. The trachea is pulled to the affected side with large atelectasis, pleural adhesions,orfibrosis.Trachealtugisarhythmicdownwardpullthatissynchronouswithsystoleandoccurs with aortic archaneurysm. DIF: Cognitive Level: Understanding (Comprehension) MSC: Client Needs: Physiologic Integrity: Physiologic Adaptation 32. Duringanassessmentofaninfant,thenursenotesthatthefontanelsaredepressedandsunken.Thenurse suspects whichcondition? a. Rickets b. Dehydration c. Mentalretardation d. Increasedintracranialpressure ANS:B Depressedandsunkenfontanelsoccurwithdehydrationormalnutrition.Mentalretardationandricketshaveno effect on the fontanels. Increased intracranial pressure would cause tense or bulging and possibly pulsating fontanels. DIF: Cognitive Level: Applying (Application) MSC: Client Needs: Physiologic Integrity: Physiologic Adaptation 33. Thenurseisperforminganassessmentona7-year-oldchildwhohassymptomsofchronicwateryeyes, sneezing,andclearnasaldrainage.Thenursenoticesthepresenceofatransverselineacrossthebridgeofthe nose, dark blue shadows below the eyes, and a double crease on the lower eyelids. These findings are characteristicof: a. Allergies. b. Sinusinfection. c. Nasalcongestion. d. Upper respiratoryinfection. NURSINGTB.COM ANS: A Chronicallergiesoftendevelopchronicfacialcharacteristicsandincludeblueshadowsbelowtheeyes,a doubleorsinglecreaseonthelowereyelids,open-mouthbreathing,andatransverselineonthenose. DIF: Cognitive Level: Analyzing (Analysis) MSC: Client Needs: Physiologic Integrity: Physiologic Adaptation 34. Whileperformingawell-childassessmentona5yearold,thenursenotesthepresenceofpalpable, bilateral,cervical,andinguinallymphnodes.Theyareapproximately0.5cminsize,round,mobile,and nontender. The nurse suspects that thischild: a. Has chronicallergies. b. May have aninfection. c. Is exhibiting a normal finding for a well child of thisage. d. Should be referred for additionalevaluation. ANS: C Palpablelymphnodesarenormalinchildrenuntilpubertywhenthelymphoidtissuebeginstoatrophy.Lymph nodesmaybeupto1cminsizeinthecervicalandinguinalareasbutarediscrete,movable,andnontender. DIF: Cognitive Level: Analyzing (Analysis) MSC: Client Needs: Health Promotion and Maintenance 35. Thenursehasjustcompletedalymphnodeassessmentona60-year-oldhealthyfemalepatient.Thenurse knows that most lymph nodes in healthy adults arenormally: a. Shotty. b. Nonpalpable. c. Large, firm, and fixed to thetissue. d. Rubbery, discrete, andmobile. ANS: B Mostlymphnodesarenonpalpableinadults.Thepalpabilityoflymphnodesdecreaseswithage.Normal nodesfeelmovable,discrete,soft,andnontenNdeUr.RSINGTB.COM DIF: Cognitive Level: Applying (Application) MSC: Client Needs: Physiologic Integrity: Physiologic Adaptation 36. Duringanexaminationofapatientinherthirdtrimesterofpregnancy,thenursenoticesthatthepatients thyroidglandisslightlyenlarged.Noenlargementhadbeenpreviouslynoticed.Thenursesuspectsthatthe patient: a. Has an iodinedeficiency. b. Is exhibiting early signs ofgoiter. c. Isexhibitinganormalenlargementofthethyroidglandduringpregnancy. d. Needs further testing for possible thyroidcancer. ANS: C Thethyroidglandenlargesslightlyduringpregnancybecauseofhyperplasiaofthetissueandincreased vascularity. DIF: Cognitive Level: Applying (Application) MSC: Client Needs: Safe and Effective Care Environment: Management of Care 37. Duringanexamination,thenurseknowsthatthebestwaytopalpatethelymphnodesintheneckis described by whichstatement? a. Usinggentlepressure,palpatewithbothhandstocomparethetwosides. b. Usingstrongpressure,palpatewithbothhandstocomparethetwosides. c. Gentlypincheachnodebetweenonesthumbandforefinger,andthenmovedowntheneckmuscle. d. Usingtheindexandmiddlefingers,gentlypalpatebyapplyingpressureinarotatingpattern. ANS: A Usinggentlepressureisrecommendedbecausestrongpressurecanpushthenodesintotheneckmuscles. Palpatingwithbothhandstocomparethetwosidessymmetricallyisusuallymostefficient. DIF: Cognitive Level: Understanding (Comprehension) MSC: Client Needs: Safe and Effective Care Environment: Management of Care 38. Duringawell-babycheckup,amotherisconcernedbecauseher2-month-oldinfantcannotholdherhead upwhensheispulledtoasittingposition.Whichresponsebythenurseisappropriate? a. Head control is usually achieved by 4 months ofage. NURSINGTB.COM b. Youshouldntbetryingtopullyourbabyuplikethatuntilsheisolder. c. Head control should be achieved by thistime. d. This inability indicates possible nerve damage to the neckmuscles. ANS: A Headcontrolisachievedby4monthswhenthebabycanholdtheheaderectandsteadywhenpulledtoa vertical position. The other responses are notappropriate. DIF: Cognitive Level: Analyzing (Analysis) MSC: Client Needs: Health Promotion and Maintenance 39. Duringanexaminationofa3-year-oldchild,thenursenoticesabruitoverthelefttemporalarea.Thenurse should: a. Continuetheexaminationbecauseabruitisanormalfindingforthisage. b. Check for the bruit again in 1hour. c. Notify the parents that a bruit has been detected in theirchild. d. Stop the examination, and notify thephysician. ANS: A Bruitsarecommonintheskullinchildrenunder4or5yearsofageandinchildrenwithanemia.Theyare systolic or continuous and are heard over the temporalarea. DIF: Cognitive Level: Analyzing (Analysis) MSC: Client Needs: Health Promotion and Maintenance 40. Duringanexamination,thenursefindsthatapatientslefttemporalarteryistortuousandfeelshardened andtender,comparedwiththerighttemporalartery.Thenursesuspectswhichcondition? a. Crepitation b. Mastoiditis c. Temporalarteritis d. Bell palsy ANS:C Withtemporalarteritis,thearteryappearsmoretortuousandfeelshardenedandtender.Theseassessment findingsarenotconsistentwiththeotherrespoNnUsResS.INGTB.COM DIF: Cognitive Level: Applying (Application) MSC: Client Needs: Physiologic Integrity: Physiologic Adaptation MULTIPLE RESPONSE 1.Thenurseisassessinga1-month-oldinfantathiswell-babycheckup.Whichassessmentfindingsare appropriate for this age? Select all thatapply. a. Head circumference equal to chestcircumference b. Head circumference greater than chestcircumference c. Head circumference less than chestcircumference d. Fontanels firm and slightlyconcave e. Absent tonic neckreflex f. Nonpalpablecervicallymphnodes ANS: B, D,F Aninfantsheadcircumferenceislargerthanthechestcircumference.Atage2years,bothmeasurementsare thesame.Duringchildhood,thechestcircumferencegrowstoexceedtheheadcircumferenceby5to7cm. Thefontanelsshouldfeelfirmandslightlyconcaveintheinfant,andtheyshouldclosebyage9months.The tonic neck reflex is present until between 3 and 4 months of age, and cervical lymph nodes are normally nonpalpable in aninfant. DIF: Cognitive Level: Applying (Application) MSC: Client Needs: Health Promotion and Maintenance NURSINGTB.COM Chapter 15: Eyes MULTIPLE CHOICE 1. Whenexaminingtheeye,thenursenoticesthatthepatientseyelidmarginsapproximatecompletely.The nurse recognizes that this assessmentfinding: a. Isexpected. b. May indicate a problem with extraocularmuscles. c. May result in problems withtearing. d. Indicates increased intraocularpressure. ANS: A Thepalpebralfissureistheellipticalopenspacebetweentheeyelids,and,whenclosed,thelidmargins approximate completely, which is a normalfinding. DIF: Cognitive Level: Understanding (Comprehension) MSC: Client Needs: Physiologic Integrity: Physiologic Adaptation 2. Duringocularexaminations,thenursekeepsinmindthatmovementoftheextraocularmusclesis: NURSINGTB.COM a. Decreased in the olderadult. b. Impaired in a patient withcataracts. c. Stimulated by cranial nerves (CNs) I andII. d. StimulatedbyCNsIII,IV,andVI. ANS:D MovementoftheextraocularmusclesisstimulatedbythreeCNs:III,IV,andVI. DIF: Cognitive Level: Remembering(Knowledge) MSC: Client Needs: Physiologic Integrity: Physiologic Adaptation 3. Thenurseisperforminganexternaleyeexamination.Whichstatementregardingtheouterlayeroftheeye istrue? a. The outer layer of the eye is very sensitive totouch. b. Theouterlayeroftheeyeisdarklypigmentedtopreventlightfromreflectinginternally. c. Thetrigeminalnerve(CNV)andthetrochlearnerve(CNIV)arestimulatedwhentheouter surface of the eye is stimulated. d. Thevisualreceptivelayeroftheeyeinwhichlightwavesarechangedintonerveimpulsesis located in the outer layer of theeye. ANS: A Thecorneaandthescleramakeuptheouterlayeroftheeye.Thecorneaisverysensitivetotouch.Themiddle layer,thechoroid,hasdarkpigmentationtopreventlightfromreflectinginternally.Thetrigeminalnerve(CN V)andthefacialnerve(CNVII)arestimulatedwhentheoutersurfaceoftheeyeisstimulated.Theretina,in theinnerlayeroftheeye,iswherelightwavesarechangedintonerveimpulses. DIF: Cognitive Level: Understanding (Comprehension) MSC: Client Needs: Physiologic Integrity: Physiologic Adaptation 4. Whenexaminingapatientseyes,thenurserecallsthatstimulationofthesympatheticbranchofthe autonomic nervoussystem: a. Causes pupillaryconstriction. b. Adjusts the eye for nearvision. c. Elevates the eyelid and dilates thepupil. d. CausescontractionoftheciliarybNodUyR.SINGTB.COM ANS: C Stimulation of the sympathetic branch of the autonomic nervous system dilates the pupil and elevates the eyelid.Parasympatheticnervoussystemstimulationcausesthepupiltoconstrict.Themusclefibersoftheiris contractthepupilinbrightlighttoaccommodatefornearvision.Theciliarybodycontrolsthethicknessofthe lens. DIF: Cognitive Level: Understanding (Comprehension) MSC: Client Needs: Physiologic Integrity: Physiologic Adaptation 5. Thenurseisreviewingcausesofincreasedintraocularpressure.Whichofthesefactorsdetermines intraocularpressure? a. Thickness or bulging of thelens b. Posterior chamber as it accommodates increasedfluid c. Contractionoftheciliarybodyinresponsetotheaqueouswithintheeye d. Amountofaqueousproducedandresistancetoitsoutflowattheangleoftheanteriorchamber ANS:D Intraocularpressureisdeterminedbyabalancebetweentheamountofaqueousproducedandtheresistanceto itsoutflowattheangleoftheanteriorchamber.Theotherresponsesareincorrect. DIF: Cognitive Level: Remembering (Knowledge) MSC: Client Needs: Physiologic Integrity: Physiologic Adaptation 6. Thenurseisconductingavisualexamination.Whichofthesestatementsregardingvisualpathwaysand visual fields istrue? a. The right side of the brain interprets the vision for the righteye. b. Theimageformedontheretinaisupsidedownandreversedfromitsactualappearanceinthe outsideworld. c. Lightraysarerefractedthroughthetransparentmediaoftheeyebeforestrikingthepupil. d. Lightimpulsesareconductedthroughtheopticnervetothetemporallobesofthebrain. ANS: B Theimageformedontheretinaisupsidedownandreversedfromitsactualappearanceintheoutsideworld. Thelightraysarerefractedthroughthetransparentmediaoftheeyebeforestrikingtheretina,andthenerve impulsesareconductedthroughtheopticnervetracttothevisualcortexoftheoccipitallobeofthebrain.The left side of the brain interprets vision for the righteye. DIF:CognitiveLevel:Remembering(KnowleNdUgeR)SINGTB.COM MSC: Client Needs: Physiologic Integrity: Physiologic Adaptation 7. Thenurseistestingapatientsvisualaccommodation,whichreferstowhichaction? a. Pupillary constriction when looking at a nearobject b. Pupillary dilation when looking at a farobject c. Changes in peripheral vision in response tolight d. Involuntaryblinkinginthepresenceofbrightlight ANS:A Themusclefibersoftheiriscontractthepupilinbrightlightandaccommodatefornearvision,whichalso results in pupil constriction. The other responses are notcorrect. DIF: Cognitive Level: Remembering (Knowledge) MSC: Client Needs: Physiologic Integrity: Physiologic Adaptation 8. Apatienthasanormalpupillarylightreflex.Thenurserecognizesthatthisreflexindicatesthat: a. The eyes converge to focus on thelight. b. Light is reflected at the same spot in botheyes. c. The eye focuses the image in the center of thepupil. d. Constriction of both pupils occurs in response to brightlight. ANS: D Thepupillarylightreflexisthenormalconstrictionofthepupilswhenbrightlightshinesontheretina.The other responses are notcorrect. DIF: Cognitive Level: Understanding (Comprehension) MSC: Client Needs: Physiologic Integrity: Physiologic Adaptation 9. Amotheraskswhenhernewborninfantseyesightwillbedeveloped.Thenurseshouldreply: a. Vision is not totally developed until 2 years ofage. b. Infantsdeveloptheabilitytofocusonanobjectatapproximately8monthsofage. c. Byapproximately3monthsofage,infantsdevelopmorecoordinatedeyemovementsandcan fixate on anobject. NURSINGTB.COM d. Mostinfantshaveuncoordinatedeyemovementsforthefirstyearoflife. ANS: C Eyemovementsmaybepoorlycoordinatedatbirth,butby3to4monthsofage,theinfantshouldestablish binocularityandshouldbeabletofixatesimultaneouslyonasingleimagewithbotheyes. DIF: Cognitive Level: Applying (Application) MSC: Client Needs: Health Promotion and Maintenance 10. Thenurseisreviewinginage-relatedchangesintheeyeforaclass.Whichofthesephysiologicchangesis responsible forpresbyopia? a. Degeneration of thecornea b. Loss of lenselasticity c. Decreased adaptation todarkness d. Decreaseddistancevisionabilities ANS:B Thelensloseselasticityanddecreasesitsabilitytochangeshapetoaccommodatefornearvision.This condition is calledpresbyopia. DIF:CognitiveLevel:Understanding(Comprehension) MSC:ClientNeeds:HealthPromotionandMaintenance 11. Whichoftheseassessmentfindingswouldthenurseexpecttoseewhenexaminingtheeyesofablack patient? a. Increased nightvision b. Dark retinalbackground c. Increasedphotosensitivity d. Narrowedpalpebralfissures ANS:B Anethnicallybasedvariabilityinthecoloroftheirisandinretinalpigmentationexists,withdarkerirides having darker retinas behindthem. DIF: Cognitive Level: Understanding (Comprehension) MSC: Client Needs: Safe and Effective Care Environment: Management of Care NURSINGTB.COM 12. A52-year-oldpatientdescribesthepresenceofoccasionalfloatersorspotsmovinginfrontofhiseyes. The nurseshould: a. Examine the retina to determine the number offloaters. b. Presume the patient has glaucoma and refer him for furthertesting. c. Considerthesetobeabnormalfindings,andreferhimtoanophthalmologist. d. Knowthatfloatersareusuallyinsignificantandarecausedbycondensedvitreousfibers. ANS: D Floatersareacommonsensationwithmyopiaoraftermiddleageandareattributabletocondensedvitreous fibers. Floaters or spots are not usually significant, but the acute onset of floaters may occur with retinal detachment. DIF: Cognitive Level: Analyzing (Analysis) MSC: Client Needs: Health Promotion and Maintenance 13. Thenurseispreparingtoassessthevisualacuityofa16-year-oldpatient.Howshouldthenurseproceed? a. Perform the confrontationtest. b. Ask the patient to read the print on a handheld Jaegercard. c. Use the Snellen chart positioned 20 feet away from thepatient. d. Determinethepatientsabilitytoreadnewsprintatadistanceof12to14inches. ANS: C The Snellen alphabet chart is the most commonly used and most accurate measure of visual acuity. The confrontationtestisagrossmeasureofperipheralvision.TheJaegercardornewspapertestsareusedtotest nearvision. DIF: Cognitive Level: Analyzing (Analysis) MSC: Client Needs: Safe and Effective Care Environment: Management of Care 14. Apatientsvisionisrecordedas20/30whentheSnelleneyechartisused.Thenurseinterpretstheseresults to indicatethat: a. At 30 feet the patient can read the entirechart. b. Thepatientcanreadat20feetwhatapersonwithnormalvisioncanreadat30feet. c. Thepatientcanreadthechartfrom20feetinthelefteyeand30feetintherighteye. NURSINGTB.COM d. Thepatientcanreadfrom30feetwhatapersonwithnormalvisioncanreadfrom20feet. ANS: B Thetopnumberindicatesthedistancethepersonisstandingfromthechart;thedenominatorgivesthedistance at which a normal eye cansee. DIF: Cognitive Level: Applying (Application) MSC: Client Needs: Physiologic Integrity: Physiologic Adaptation 15. ApatientisunabletoreadeventhelargestlettersontheSnellenchart.Thenurseshouldtakewhichaction next? a. Referthepatienttoanophthalmologistoroptometristforfurtherevaluation. b. Assesswhetherthepatientcancountthenursesfingerswhentheyareplacedinfrontofhisorher eyes. c. AskthepatienttoputonhisorherreadingglassesandattempttoreadtheSnellenchartagain. d. Shortenthedistancebetweenthepatientandthechartuntilthelettersareseen,andrecordthat distance. ANS: D If the person is unable to see even the largest letters when standing 20 feet from the chart, then the nurse should shorten the distance to the chart until the letters are seen, and record that distance (e.g., 10/200). If visualacuityisevenlower,thenthenurseshouldassesswhetherthepersoncancountfingerswhentheyare spreadinfrontoftheeyesorcandistinguishlightperceptionfromapenlight.Ifvisionispoorerthan20/30, then a referral to an ophthalmologist or optometrist is necessary, but the nurse must first assess the visual acuity. DIF: Cognitive Level: Analyzing (Analysis) MSC: Client Needs: Physiologic Integrity: Physiologic Adaptation 16. Apatientsvisionisrecordedas20/80ineacheye.Thenurseinterpretsthisfindingtomeanthatthepatient: a. Has poorvision. b. Has acutevision. c. Has normalvision. d. Ispresbyopic. ANS: A Normalvisualacuityis20/20ineacheye;thelargerthedenominator,thepoorerthevision. DIF: Cognitive Level: Applying(Application) NURSINGTB.COM MSC: Client Needs: Physiologic Integrity: Physiologic Adaptation 17. Whenperformingthecorneallightreflexassessment,thenursenotesthatthelightisreflectedat2oclock in each eye. The nurseshould: a. Consider this a normalfinding. b. Refer the individual for furtherevaluation. c. Document this finding as an asymmetric lightreflex. d. Perform the confrontation test to validate thefindings. ANS: A Reflectionofthelightonthecorneasshouldbeinexactlythesamespotoneacheye,orsymmetric.If asymmetry is noted, then the nurse should administer the covertest. DIF: Cognitive Level: Analyzing (Analysis) MSC: Client Needs: Safe and Effective Care Environment: Management of Care 18. Thenurseisperformingthediagnosticpositionstest.Normalfindingswouldbewhichoftheseresults? a. Convergence of theeyes b. Parallel movement of botheyes c. Nystagmus in extreme superiorgaze d. Slightamountoflidlagwhenmovingtheeyesfromasuperiortoaninferiorposition ANS:B A normal response for the diagnostic positions test is parallel tracking of the object with both eyes. Eye movementthatisnotparallelindicatesaweaknessofanextraocularmuscleordysfunctionoftheCNthat innervatesit. DIF: Cognitive Level: Applying (Application) MSC: Client Needs: Safe and Effective Care Environment: Management of Care 19. Duringanassessmentofthescleraofablackpatient,thenursewouldconsiderwhichoftheseanexpected finding? a. Yellow fatty deposits over thecornea b. Pallor near the outer canthus of the lowerlid c. YellowcolorofthesclerathatexteNnUdRsSuIpNtGoTthBe.CirOisM d. Presence of small brown macules on thesclera ANS: D Normallyindark-skinnedpeople,smallbrownmaculesmaybeobservedinthesclera. DIF: Cognitive Level: Applying(Application) MSC: Client Needs: Safe and Effective Care Environment: Management of Care 20. A60-year-oldmanisattheclinicforaneyeexamination.Thenursesuspectsthathehasptosisofoneeye. How should the nurse check forthis? a. Perform the confrontationtest. b. Assess the individuals nearvision. c. Observe the distance between the palpebralfissures. d. Performthecorneallighttest,andlookforsymmetryofthelightreflex. ANS: C Ptosisisadroopingoftheuppereyelidthatwouldbeapparentbyobservingthedistancebetweentheupper andlowereyelids.Theconfrontationtestmeasuresperipheralvision.Measuringnearvisionorthecorneal light test does not check forptosis. DIF: Cognitive Level: Analyzing (Analysis) MSC: Client Needs: Health Promotion and Maintenance 21. Duringanexaminationoftheeye,thenursewouldexpectwhatnormalfindingwhenassessingthelacrimal apparatus? a. Presence of tears along the innercanthus b. Blocked nasolacrimal duct in a newborninfant c. Slightswellingovertheupperlidandalongthebonyorbitiftheindividualhasacold d. Absenceofdrainagefromthepunctawhenpressingagainsttheinnerorbitalrim ANS:D Noswelling,redness,ordrainagefromthepunctashouldbeobservedwhenitispressed.Regurgitationoffluid fromthepuncta,whenpressed,indicatesductblockage.Thelacrimalglandsarenotfunctionalatbirth. DIF: Cognitive Level: Applying (Application) MSC:ClientNeeds:SafeandEffectiveCareENnUvRirSoInNmGeTnBt:.CMOaMnagementofCare 22. Whenassessingthepupillarylightreflex,thenurseshouldusewhichtechnique? a. Shineapenlightfromdirectlyinfrontofthepatient,andinspectforpupillaryconstriction. b. Askthepatienttofollowthepenlightineightdirections,andobserveforbilateralpupil constriction. c. Shinealightacrossthepupilfromtheside,andobservefordirectandconsensualpupillary constriction. d. Askthepatienttofocusonadistantobject.Thenaskthepatienttofollowthepenlightto approximately 7 cm from thenose. ANS: C Totestthepupillarylightreflex,thenurseshouldadvancealightinfromthesideandnotethedirectand consensual pupillaryconstriction. DIF: Cognitive Level: Applying (Application) MSC: Client Needs: Safe and Effective Care Environment: Management of Care 23. Thenurseisassessingapatientseyesfortheaccommodationresponseandwouldexpecttoseewhich normalfinding? a. Dilation of thepupils b. Consensual lightreflex c. Conjugate movement of theeyes d. Convergenceoftheaxesoftheeyes ANS:D Theaccommodationreactionincludespupillaryconstrictionandconvergenceoftheaxesoftheeyes.Theothe responses are notcorrect. DIF: Cognitive Level: Applying (Application) MSC: Client Needs: Safe and Effective Care Environment: Management of Care 24. Inusingtheophthalmoscopetoassessapatientseyes,thenursenoticesaredglowinthepatientspupils. On the basis of this finding, the nursewould: a. Suspect that an opacity is present in the lens orcornea. b. Checkthelightsourceoftheophthalmoscopetoverifythatitisfunctioning. NURSINGTB.COM c. Considertheredglowanormalreflectionoftheophthalmoscopelightofftheinnerretina. d. Continuewiththeophthalmoscopicexamination,andreferthepatientforfurtherevaluation. ANS: C Theredglowfillingthepersonspupilistheredreflexandisanormalfindingcausedbythereflectionofthe ophthalmoscopelightofftheinnerretina.Theotherresponsesarenotcorrect. DIF: Cognitive Level: Analyzing (Analysis) MSC: Client Needs: Safe and Effective Care Environment: Management of Care 25. Thenurseisexaminingapatientsretinawithanophthalmoscope.Whichfindingisconsiderednormal? a. Optic disc that is a yellow-orangecolor b. Opticdiscmarginsthatareblurredaroundtheedges c. Presenceofpigmentedcrescentsinthemaculararea d. Presenceofthemaculalocatedonthenasalsideoftheretina ANS:A Theopticdiscislocatedonthenasalsideoftheretina.Itscolorisacreamyyellow-orangetoapink,andthe edges are distinct and sharply demarcated, not blurred. A pigmented crescent is black and is due to the accumulation of pigment in thechoroid. DIF: Cognitive Level: Applying (Application) MSC: Client Needs: Safe and Effective Care Environment: Management of Care 26. A2-week-oldinfantcanfixateonanobjectbutcannotfollowalightorbrighttoy.Thenursewould: a. Consider this a normalfinding. b. Assess the pupillary light reflex for possibleblindness. c. Continue with the examination, and assess visualfields. d. Expectthata2-week-oldinfantshouldbeabletofixateandfollowanobject. ANS: A By2to4weeksaninfantcanfixateonanobject.Bytheageof1month,theinfantshouldfixateandfollowa bright light ortoy. DIF: Cognitive Level: Analyzing (Analysis) MSC: Client Needs: Health Promotion and Maintenance NURSINGTB.COM 27. Thenurseisassessingcolorvisionofamalechild.Whichstatementiscorrect?Thenurseshould: a. Check color vision annually until the age of 18years. b. Ask the child to identify the color of his or herclothing. c. Test for color vision once between the ages of 4 and 8years. d. Begin color vision screening at the childs 2-yearcheckup. ANS: C Test boys only once for color vision between the ages of 4 and 8 years. Color vision is not tested in girls becauseitisrareingirls.TestingisperformedwiththeIshiharatest,whichisaseriesofpolychromaticcards. DIF: Cognitive Level: Applying (Application) MSC: Client Needs: Health Promotion and Maintenance 28. Thenurseisperforminganeye-screeningclinicatadaycarecenter.Whenexamininga2-year-oldchild, the nurse suspects that the child has a lazy eye andshould: a. Examine the external structures of theeye. b. Assess visual acuity with the Snellen eyechart. c. Assess the childs visual fields with the confrontationtest. d. Test for strabismus by performing the corneal light reflextest. ANS: D Testingforstrabismusisdonebyperformingthecorneallightreflextestandthecovertest.TheSnelleneye chart and confrontation test are not used to test forstrabismus. DIF: Cognitive Level: Applying (Application) MSC: Client Needs: Health Promotion and Maintenance 29. Thenurseisperforminganeyeassessmentonan80-year-oldpatient.Whichofthesefindingsisconsidered abnormal? a. Decrease in tearproduction b. Unequal pupillary constriction in response tolight c. Presence of arcus senilis observed around thecornea d. LossoftheouterhairontheeyebrNowUsRaStItNriGbuTtBab.CleOtMoadecreaseinhairfollicles ANS:B Pupils are small in the older adult, and the pupillary light reflex may be slowed, but pupillary constriction shouldbesymmetric.Theassessmentfindingsintheotherresponsesareconsiderednormalinolderpersons. DIF:CognitiveLevel:Understanding(Comprehension) MSC:ClientNeeds:HealthPromotionandMaintenance 30. Thenursenoticesthepresenceofperiorbitaledemawhenperforminganeyeassessmentona70-year-old patient. The nurseshould: a. Check for the presence ofexophthalmos. b. Suspect that the patient hashyperthyroidism. c. Ask the patient if he or she has a history of heartfailure. d. Assess for blepharitis, which is often associated with periorbitaledema. ANS: C Periorbitaledemaoccurswithlocalinfections,crying,andsystemicconditionssuchasheartfailure,renal failure,allergy,andhypothyroidism.Periorbitaledemaisnotassociatedwithblepharitis. DIF: Cognitive Level: Analyzing (Analysis) MSC: Client Needs: Physiologic Integrity: Physiologic Adaptation 31. Whenalightisdirectedacrosstheirisofapatientseyefromthetemporalside,thenurseisassessingfor: a. Drainage fromdacryocystitis. b. Presence of conjunctivitis over theiris. c. Presence of shadows, which may indicateglaucoma. d. Scattered light reflex, which may be indicative ofcataracts. ANS: C Thepresenceofshadowsintheanteriorchambermaybeasignofacuteangle-closureglaucoma.Thenormal iris is flat and creates no shadows. This method is not correct for the assessment of dacryocystitis, conjunctivitis, orcataracts. DIF: Cognitive Level: Applying (Application) MSC: Client Needs: Safe and Effective Care Environment: Management of Care 32. In a patient who has anisocoria, the nurse would expect toobserve: a. Dilatedpupils. b. Excessivetearing. c. Pupils of unequalsize. d. Unevencurvatureofthelens. NURSINGTB.COM ANS: C Unequalpupilsizeistermedanisocoria.Itnormallyexistsin5%ofthepopulationbutmayalsobeindicative of central nervous systemdisease. DIF: Cognitive Level: Understanding (Comprehension) MSC: Client Needs: Physiologic Integrity: Physiologic Adaptation 33. Apatientcomestotheemergencydepartmentafteraboxingmatch,andhislefteyeisswollenalmostshut. He has bruises on his face and neck. He says he is worried because he cant see well from his left eye. The physiciansuspectsretinaldamage.Thenurserecognizesthatsignsofretinaldetachmentinclude: a. Loss of centralvision. b. Shadowordiminishedvisioninonequadrantoronehalfofthevisualfield. c. Loss of peripheralvision. d. Sudden loss of pupillary constriction andaccommodation. ANS: B Withretinaldetachment,thepersonhasshadowsordiminishedvisioninonequadrantoronehalfofthevisual field. The other responses are not signs of retinaldetachment. DIF: Cognitive Level: Analyzing (Analysis) MSC: Client Needs: Physiologic Integrity: Physiologic Adaptation 34. Apatientcomesintothecliniccomplainingofpaininherrighteye.Onexamination,thenurseseesa pustuleatthelidmarginthatispainfultotouch,red,andswollen.Thenurserecognizesthatthisisa: a. Chalazion. b. Hordeolum(stye). c. Dacryocystitis. d. Blepharitis. ANS: B NURSINGTB.COM A hordeolum, or stye, is a painful, red, and swollen pustule at the lid margin. A chalazion is a nodule protrudingonthelid,towardtheinside,andisnontender,firm,withdiscreteswelling.Dacryocystitisisan inflammationofthelacrimalsac.Blepharitisisinflammationoftheeyelids. DIF: Cognitive Level: Applying (Application) MSC: Client Needs: Physiologic Integrity: Physiologic Adaptation 35. A68-year-oldwomanisintheeyeclinicforacheckup.Shetellsthenursethatshehasbeenhavingtrouble readingthepaper,sewing,andevenseeingthefacesofhergrandchildren.Onexamination,thenursenotesthat she has some loss of central vision but her peripheral vision is normal. These findings suggest that she may have: a. Maculardegeneration. b. Vision that is normal for someone herage. c. Thebeginningstagesofcataractformation. d. Increasedintraocularpressureorglaucoma. ANS: A Macular degeneration is the most common cause of blindness. It is characterized by the loss of central vision. Cataracts would show lens opacity. Chronic open-angle glaucoma, the most common type of glaucoma, involvesagraduallossofperipheralvision.Thesefindingsarenotconsistentwithvisionthatisconsidered normal at anyage. DIF: Cognitive Level: Applying (Application) MSC: Client Needs: Physiologic Integrity: Physiologic Adaptation 36. Apatientcomesintotheemergencydepartmentafteranaccidentatwork.Amachineblewdustintohis eyes,andhewasnotwearingsafetyglasses.Thenurseexamineshiscorneasbyshiningalightfromtheside acrossthecornea.Whatfindingswouldsuggestthathehassufferedacornealabrasion? a. Smooth and clearcorneas b. Opacity of the lens behind thecornea c. Bleeding from the areas across thecornea d. Shatteredlooktothelightraysreflectingoffthecornea ANS:D Acornealabrasioncausesirregularridgesinreflectedlight,whichproduceashatteredappearancetolight rays.Noopacitiesshouldbeobservedinthecornea.Theotherresponsesarenotcorrect. DIF: Cognitive Level: Applying (Application) NURSINGTB.COM MSC: Client Needs: Physiologic Integrity: Physiologic Adaptation 37. Anophthalmicexaminationrevealspapilledema.Thenurseisawarethatthisfindingindicates: a. Retinaldetachment. b. Diabeticretinopathy. c. Acute-angleglaucoma. d. Increased intracranialpressure. ANS: D Papilledema,orchokeddisk,isaserioussignofincreasedintracranialpressure,whichiscausedbyaspace- occupyingmasssuchasabraintumororhematoma.Thispressurecausesvenousstasisintheglobe,showing redness, congestion, and elevation of the optic disc, blurred margins, hemorrhages, and absent venous pulsations.Papilledemaisnotassociatedwiththeconditionsintheotherresponses. DIF: Cognitive Level: Applying (Application) MSC: Client Needs: Physiologic Integrity: Physiologic Adaptation 38. Duringaphysicaleducationclass,astudentishitintheeyewiththeendofabaseballbat.Whenexamined intheemergencydepartment,thenursenoticesthepresenceofbloodintheanteriorchamberoftheeye.This finding indicates the presence of: a. Hypopyon. b. Hyphema. c. Cornealabrasion. d. Pterygium. ANS: B Hyphemaisthetermforbloodintheanteriorchamberandisaseriousresultofblunttrauma(afistora baseball)orspontaneoushemorrhageandmayindicatescleralruptureormajorintraoculartrauma. DIF: Cognitive Level: Analyzing (Analysis) MSC: Client Needs: Physiologic Integrity: Physiologic Adaptation 39. Duringanassessment,thenursenoticesthatanolderadultpatienthastearsrollingdownhisfacefromhis lefteye.Closerexaminationshowsthatthelowerlidislooseandrollingoutward.Thepatientcomplainsofhis eye feeling dry and itchy. Which action by the nurse iscorrect? a. Assessing the eye for a possible foreignbody NURSINGTB.COM b. Documenting the finding asptosis c. Assessing for other signs ofectropion d. Contactingtheprescriber;thesearesignsofbasalcellcarcinoma ANS:C Theconditiondescribedisknownasectropion,anditoccursinolderadultsandisattributabletoatrophyofthe elasticandfibroustissues.Thelowerliddoesnotapproximatetotheeyeball,and,asaresult,thepunctacanno effectivelysiphontears;excessivetearingresults.Ptosisisadroopingoftheuppereyelid.Thesesignsdonot suggestthepresenceofaforeignbodyintheeyeorbasalcellcarcinoma. DIF: Cognitive Level: Applying (Application) MSC: Client Needs: Physiologic Integrity: Physiologic Adaptation MULTIPLE RESPONSE 1.Duringanexamination,apatientstatesthatshewasdiagnosedwithopen-angleglaucoma2yearsago.The nurse assesses for characteristics of open-angle glaucoma. Which of these are characteristics of open-angle glaucoma?Select all thatapply. a. Patientmayexperiencesensitivitytolight,nausea,andhalosaroundlights. b. Patient experiences tunnel vision in the latestages. c. Immediate treatment isneeded. d. Vision loss begins with peripheralvision. e. Open-angleglaucomacausessuddenattacksofincreasedpressurethatcauseblurredvision. f. Virtually no symptoms areexhibited. ANS: B, D, F Open-angleglaucomaisthemostcommontypeofglaucoma;virtuallynosymptomsareexhibited.Visionloss begins with the peripheral vision, which often goes unnoticed because individuals learn to compensate intuitivelybyturningtheirheads.Theothercharacteristicsarethoseofclosed-angleglaucoma. DIF: Cognitive Level: Understanding (Comprehension) MSC: Client Needs: Physiologic Integrity: Physiologic Adaptation NURSINGTB.COM Chapter 16: Ears MULTIPLE CHOICE 1. Thenurseneedstopulltheportionoftheearthatconsistsofmovablecartilageandskindownandback when administering eardrops. This portion of the ear is calledthe: a. Auricle. b. Concha. c. Outermeatus. d. Mastoidprocess. ANS: A Theexternaleariscalledtheauricleorpinnaandconsistsofmovablecartilageandskin. DIF: Cognitive Level: Remembering(Knowledge) MSC: Client Needs: Physiologic Integrity: Physiologic Adaptation 2. Thenurseisexaminingapatientsearsandnoticescerumenintheexternalcanal.Whichofthesestatements about cerumen iscorrect? NURSINGTB.COM a. Sticky honey-colored cerumen is a sign ofinfection. b. The presence of cerumen is indicative of poorhygiene. c. The purpose of cerumen is to protect and lubricate theear. d. Cerumen is necessary for transmitting sound through the auditorycanal. ANS: C Theearislinedwithglandsthatsecretecerumen,whichisayellowwaxymaterialthatlubricatesandprotects theear. DIF: Cognitive Level: Remembering (Knowledge) MSC: Client Needs: Physiologic Integrity: Physiologic Adaptation 3. Whenexaminingtheearwithanotoscope,thenursenotesthatthetympanicmembraneshouldappear: a. Light pink with a slightbulge. b. Pearly gray and slightlyconcave. c. Pulled in at the base of the cone oflight. d. Whitish with a small fleck of light in the superiorportion. ANS: B Thetympanicmembraneisatranslucentmembranewithapearlygraycolorandaprominentconeoflightin theanteroinferiorquadrant,whichisthereflectionoftheotoscopelight.Thetympanicmembraneisovaland slightlyconcave,pulledinatitscenterbythemalleus,whichisoneofthemiddleearossicles. DIF: Cognitive Level: Remembering (Knowledge) MSC: Client Needs: Physiologic Integrity: Physiologic Adaptation 4. Thenurseisreviewingthestructuresoftheear.Whichofthesestatementsconcerningtheeustachiantubeis true? a. The eustachian tube is responsible for the production ofcerumen. b. It remains open except when swallowing oryawning. c. Theeustachiantubeallowspassageofairbetweenthemiddleandouterear. d. Ithelpsequalizeairpressureonbothsidesofthetympanicmembrane. ANS: D NURSINGTB.COM Theeustachiantubeallowsanequalizationofairpressureoneachsideofthetympanicmembranesothatthe membranedoesnotruptureduring,forexample,altitudechangesinanairplane.Thetubeisnormallyclosed, but it opens with swallowing oryawning. DIF: Cognitive Level: Remembering (Knowledge) MSC: Client Needs: Physiologic Integrity: Physiologic Adaptation 5. Apatientwithamiddleearinfectionasksthenurse,Whatdoesthemiddleeardo?Thenurserespondsby telling the patient that the middle ear functionsto: a. Maintainbalance. b. Interpret sounds as they enter theear. c. Conduct vibrations of sounds to the innerear. d. Increase amplitude of sound for the inner ear tofunction. ANS: C Amongitsotherfunctions,themiddleearconductssoundvibrationsfromtheoutereartothecentralhearing apparatusintheinnerear.Theotherresponsesarenotfunctionsofthemiddleear. DIF: Cognitive Level: Understanding (Comprehension) MSC: Client Needs: Physiologic Integrity: Physiologic Adaptation 6. Thenurseisreviewingthefunctionofthecranialnerves(CNs).WhichCNisresponsibleforconducting nerve impulses to the brain from the organ ofCorti? a. I b. III c. VIII d. XI ANS: C ThenerveimpulsesareconductedbytheauditoryportionofCNVIIItothebrain. DIF: Cognitive Level: Remembering(Knowledge) MSC: Client Needs: Physiologic Integrity: Physiologic Adaptation 7. Thenurseisassessingapatientwhomayhavehearingloss.Whichofthesestatementsistrueconcerningair conduction? a. Air conduction is the normal pathway forhearing. NURSINGTB.COM b. Vibrations of the bones in the skull cause airconduction. c. Amplitude of sound determines the pitch that isheard. d. Loss of air conduction is calleda conductive hearing loss. ANS: A The normal pathway of hearing is air conduction, which starts when sound waves produce vibrations on the tympanicmembrane.Conductivehearinglossresultsfromamechanicaldysfunctionoftheexternalormiddle ear. The other statements are not true concerning airconduction. DIF: Cognitive Level: Understanding (Comprehension) MSC: Client Needs: Physiologic Integrity: Physiologic Adaptation 8. Apatienthasbeenshowntohaveasensorineuralhearingloss.Duringtheassessment,itwouldbeimportant for the nurseto: a. Speak loudly so the patient can hear thequestions. b. Assess for middle ear infection as a possiblecause. c. Ask the patient what medications he is currentlytaking. d. Look for the source of the obstruction in the externalear. ANS: C Asimpleincreaseinamplitudemaynotenablethepersontounderstandspokenwords.Sensorineuralhearing loss may be caused by presbycusis, which is a gradual nerve degeneration that occurs with aging and by ototoxic drugs, which affect the hair cells in thecochlea. DIF: Cognitive Level: Applying (Application) MSC: Client Needs: Safe and Effective Care Environment: Management of Care 9. Duringaninterview,thepatientstateshehasthesensationthateverythingaroundhimisspinning.The nurserecognizesthattheportionoftheearresponsibleforthissensationisthe: a. Cochlea. b. CNVIII. c. Organ ofCorti. d. Labyrinth. ANS: D NURSINGTB.COM Ifthelabyrintheverbecomesinflamed,thenitfeedsthewronginformationtothebrain,creatingastaggering gait and a strong, spinning, whirling sensation calledvertigo. DIF: Cognitive Level: Applying (Application) MSC: Client Needs: Safe and Effective Care Environment: Management of Care 10. Apatientinherfirsttrimesterofpregnancyisdiagnosedwithrubella.Whichofthesestatementsiscorrect regarding the significance of this in relation to the infantshearing? a. Rubella may affect the mothers hearing but not theinfants. b. RubellacandamagetheinfantsorganofCorti,whichwillimpairhearing. c. Rubellaisonlydangeroustotheinfantinthesecondtrimesterofpregnancy. d. RubellacanimpairthedevelopmentofCNVIIIandthusaffecthearing. ANS:B Ifmaternalrubellainfectionoccursduringthefirsttrimester,thenitcandamagetheorganofCortiandimpair hearing. DIF: Cognitive Level: Understanding (Comprehension) MSC: Client Needs: Physiologic Integrity: Physiologic Adaptation 11. Themotherofa2-year-oldisconcernedbecausehersonhashadthreeearinfectionsinthepastyear.What would be an appropriate response by thenurse? a. Itisunusualforasmallchildtohavefrequentearinfectionsunlesssomethingelseiswrong. b. Weneedtochecktheimmunesystemofyoursontodeterminewhyheishavingsomanyear infections. c. Earinfectionsarenotuncommonininfantsandtoddlersbecausetheytendtohavemorecerumen in the externalear. d. Yoursonseustachiantubeisshorterandwiderthanyoursbecauseofhisage,whichallowsfor infections to develop moreeasily. ANS: D The infants eustachian tube is relatively shorter and wider than the adults eustachian tube, and its position is morehorizontal;consequently,pathogensfromthenasopharynxcanmoreeasilymigratethroughtothemiddle ear. The other responses are notappropriate. DIF: Cognitive Level: Applying (Application) MSC: Client Needs: Health Promotion and Maintenance 12. A31-year-oldpatienttellsthenursethathNeUhaRsSnINotGicTeBd.CaOprMogressivelossinhishearing.Hesaysthatit doesseemtohelpwhenpeoplespeaklouderorifheturnsupthevolumeofatelevisionorradio.Themost likely cause of his hearing lossis: a. Otosclerosis. b. Presbycusis. c. Trauma to thebones. d. Frequent earinfections. ANS: A Otosclerosis is a common cause of conductive hearing loss in young adults between the ages of 20 and 40 years.Presbycusisisatypeofhearinglossthatoccurswithaging.Traumaandfrequentearinfectionsarenota likely cause of his hearingloss. DIF: Cognitive Level: Analyzing (Analysis) MSC: Client Needs: Physiologic Integrity: Physiologic Adaptation 13. A70-year-oldpatienttellsthenursethathehasnoticedthatheishavingtroublehearing,especiallyinlarge groups.Hesaysthathecantalwaystellwherethesoundiscomingfromandthewordsoftensoundmixedup. What might the nurse suspect as the cause for thischange? a. Atrophyoftheapocrineglands b. Ciliabecomingcoarseandstiff c. Nerve degeneration in the innerear d. Scarringofthetympanicmembrane ANS:C Presbycusisisatypeofhearinglossthatoccursin60%ofthoseolderthan65yearsofage,eveninthoseliving inaquietenvironment.Thissensorineurallossisgradualandcausedbynervedegenerationintheinnerear. Wordssoundgarbled,andtheabilitytolocalizesoundisalsoimpaired.Thiscommunicationdysfunctionis accentuated when background noise ispresent. DIF: Cognitive Level: Analyzing (Analysis) MSC: Client Needs: Health Promotion and Maintenance 14. Duringanassessmentofa20-year-oldAsianpatient,thenursenoticesthathehasdry,flakycerumeninhis canal. What is the significance of this finding? Thisfinding: a. Is probably the result of lesions from eczema in hisear. b. Represents poorhygiene. NURSINGTB.COM c. Is a normal finding, and no further follow-up isnecessary. d. Couldbeindicativeofchangeincilia;thenurseshouldassessforhearingloss. ANS: C AsiansandNativeAmericansaremorelikelytohavedrycerumen,whereasBlacksandWhitesusuallyhave wetcerumen. DIF: Cognitive Level: Applying (Application) MSC: Client Needs: Safe and Effective Care Environment: Management of Care 15. Thenurseistakingthehistoryofapatientwhomayhaveaperforatedeardrum.Whatwouldbean important question in thissituation? a. Do you ever notice ringing or crackling in yourears? b. When was the last time you had your hearingchecked? c. Have you ever been told that you have any type of hearingloss? d. Isthereanyrelationshipbetweentheearpainandthedischargeyoumentioned? ANS: D Typicallywithperforation,earpainoccursfirst,stoppingwithapoppingsensation,andthendrainageoccurs. DIF: Cognitive Level: Applying(Application) MSC: Client Needs: Safe and Effective Care Environment: Management of Care 16. A31-year-oldpatienttellsthenursethathehasnoticedpaininhisleftearwhenpeoplespeakloudlyto him. The nurse knows that thisfinding: a. Is normal for people of hisage. b. Is a characteristic ofrecruitment. c. May indicate a middle earinfection. d. Indicates that the patient has a cerumenimpaction. ANS: B Recruitmentissignificanthearinglossoccurringwhenspeechisatlowintensity,butsoundactuallybecomes painfulwhenthespeakerrepeatsataloudervolume.Theotherresponsesarenotcorrect. DIF: Cognitive Level: Analyzing (Analysis) MSC:ClientNeeds:SafeandEffectiveCareENnUvRirSoInNmGeTnBt:.CMOaMnagementofCare 17. Whilediscussingthehistoryofa6-month-oldinfant,themothertellsthenursethatshetookasignificant amountofaspirinwhileshewaspregnant.Whatquestionwouldthenursewanttoincludeinthehistory? a. Does your baby seem to startle with loudnoises? b. Has your baby had any surgeries on herears? c. Have you noticed any drainage from herears? d. How many ear infections has your baby had sincebirth? ANS: A Childrenatriskforahearingdeficitincludethoseexposedinuterotoavarietyofconditions,suchasmaternal rubella or to maternal ototoxicdrugs. DIF: Cognitive Level: Applying (Application) MSC: Client Needs: Health Promotion and Maintenance 18. Thenurseisperforminganotoscopicexaminationonanadult.Whichoftheseactionsiscorrect? a. Tilting the persons head forward during theexamination b. Once the speculum is in the ear, releasing thetraction c. Pulling the pinna up and back before inserting thespeculum d. Usingthesmallestspeculumtodecreasetheamountofdiscomfort ANS:C Thepinnaispulledupandbackonanadultorolderchild,whichhelpsstraightentheS-shapeofthecanal. Tractionshouldnotbereleasedontheearuntiltheexaminationiscompletedandtheotoscopeisremoved. DIF: Cognitive Level: Understanding (Comprehension) MSC: Client Needs: Safe and Effective Care Environment: Management of Care 19. Thenurseisassessinga16-year-oldpatientwhohassufferedheadinjuriesfromarecentmotorvehicle accident.Whichofthesestatementsindicatesthemostimportantreasonforassessingforanydrainagefrom the earcanal? a. If the drum has ruptured, then purulent drainage willresult. b. Bloody or clear watery drainage can indicate a basal skullfracture. c. The auditory canal many be occluded from increasedcerumen. NURSINGTB.COM d. Foreign bodies from the accident may cause occlusion of thecanal. ANS: B Frankbloodorclearwaterydrainage(cerebrospinalleak)afteratraumasuggestsabasalskullfractureand warrantsimmediatereferral.Purulentdrainageindicatesotitisexternaorotitismedia. DIF: Cognitive Level: Analyzing (Analysis) MSC: Client Needs: Physiologic Integrity: Physiologic Adaptation 20. Inperformingavoicetesttoassesshearing,whichoftheseactionswouldthenurseperform? a. Shield the lips so that the sound ismuffled. b. Whisperasetofrandomnumbersandletters,andthenaskthepatienttorepeatthem. c. Askthepatienttoplacehisfingerinhiseartooccludeoutsidenoise. d. Standapproximately4feetawaytoensurethatthepatientcanreallyhearatthisdistance. ANS: B Withthehead30to60cm(1to2feet)fromthepatientsear,theexaminerexhalesandslowlywhispersasetof randomnumbersandletters,suchas5,B,6.Normally,thepatientisaskedtorepeateachnumberandletter correctly after hearing the examiner say them. DIF: Cognitive Level: Understanding (Comprehension) MSC: Client Needs: Safe and Effective Care Environment: Management of Care 21. Inperforminganexaminationofa3-year-oldchildwithasuspectedearinfection,thenursewould: a. Omit the otoscopic examination if the child has afever. b. Pull the ear up and back before inserting thespeculum. c. Ask the mother to leave the room while examining thechild. d. Perform the otoscopic examination at the end of theassessment. ANS: D Inadditiontoitsplaceinthecompleteexamination,eardrumassessmentismandatoryforanyinfantorchild requiringcareforanillnessorfever.Fortheinfantoryoungchild,thetimingoftheotoscopicexaminationis best toward the end of the completeexamination. DIF: Cognitive Level: Analyzing (Analysis) MSC: Client Needs: Safe and Effective Care Environment: Management of Care 22. ThenurseispreparingtoperformanotoscNoUpiRcSeIxNaGmTinBa.CtioOnMofanewborninfant.Whichstatementistrue regarding thisexamination? a. Immobility of the drum is a normalfinding. b. An injected membrane would indicate aninfection. c. The normal membrane may appear thick andopaque. d. The appearance of the membrane is identical to that of anadult. ANS:C During the first few days after the birth, the tympanic membrane of a newborn often appears thickened and opaque.Itmaylookinjectedandhaveamildrednessfromincreasedvascularity.Theotherstatementsarenot correct. DIF:CognitiveLevel:Understanding(Comprehension) MSC:ClientNeeds:HealthPromotionandMaintenance 23. Thenurseassessesthehearingofa7-month-oldbyclappinghands.Whatistheexpectedresponse?The infant: a. Turns his or her head to localize thesound. b. Showsnoobviousresponsetothenoise. c. Showsastartleandacousticblinkreflex. d. Stops any movement, and appears to listen for thesound. ANS: A Withaloudsuddennoise,thenurseshouldnoticetheinfantturninghisorherheadtolocalizethesoundandto respondtohisorherownname.Astartlereflexandacousticblinkreflexisexpectedinnewborns;atage3to4 months, the infant stops any movement and appears tolisten. DIF:CognitiveLevel:Understanding(Comprehension) MSC:ClientNeeds:HealthPromotionandMaintenance 24. Thenurseisperforminganearexaminationofan80-year-oldpatient.Whichofthesefindingswouldbe considerednormal? a. High-tone frequencyloss b. Increased elasticity of thepinna c. Thin, translucentmembrane d. Shiny,pinktympanicmembrane NURSINGTB.COM ANS: A Ahigh-tonefrequencyhearinglossisapparentforthoseaffectedwithpresbycusis,thehearinglossthatoccurs withaging.Thepinnaloseselasticity,causingearlobestobependulous.Theeardrummaybewhiterincolor andmoreopaqueanddullerintheolderpersonthanintheyoungeradult. DIF: Cognitive Level: Applying (Application) MSC: Client Needs: Health Promotion and Maintenance 25. Anassessmentofa23-year-oldpatientrevealsthefollowing:anauriclethatistenderandreddish-bluein colorwithsmallvesicles.Thenursewouldneedtoknowadditionalinformationthatincludeswhichofthese? a. Anychangeintheabilitytohear b. Anyrecentdrainagefromtheear c. Recent history of trauma to theear d. Anyprolongedexposuretoextremecold ANS:D Frostbitecausesreddish-bluediscolorationandswellingoftheauricleafterexposuretoextremecold.Vesicles or bullae may develop, and the person feels pain andtenderness. DIF: Cognitive Level: Analyzing (Analysis) MSC: Client Needs: Physiologic Integrity: Physiologic Adaptation 26. Whileperformingtheotoscopicexaminationofa3-year-oldboywhohasbeenpullingonhisleftear,the nurse finds that his left tympanic membrane is bright red and that the light reflex is not visible. The nurse interprets these findings to indicatea(n): a. Fungalinfection. b. Acute otitismedia. c. Perforation of theeardrum. d. Cholesteatoma. ANS: B Absentordistortedlightreflexandabrightredcoloroftheeardrumareindicativeofacuteotitismedia. DIF: Cognitive Level: Analyzing(Analysis) MSC: Client Needs: Physiologic Integrity: Physiologic Adaptation NURSINGTB.COM 27. Themotherofa2-year-oldtoddlerisconcernedabouttheupcomingplacementoftympanostomytubesin hersonsears.Thenursewouldincludewhichofthesestatementsintheteachingplan? a. The tubes are placed in the innerear. b. The tubes are used in children with sensorineuralloss. c. The tubes are permanently inserted during a surgicalprocedure. d. Thepurposeofthetubesistodecreasethepressureandallowfordrainage. ANS: D Polyethylenetubesaresurgicallyinsertedintotheeardrumtorelievemiddleearpressureandtopromote drainageofchronicorrecurrentmiddleearinfections.Tubesspontaneouslyextrudein6monthsto1year. DIF: Cognitive Level: Understanding (Comprehension) MSC: Client Needs: Physiologic Integrity: Physiologic Adaptation 28. Inanindividualwithotitisexterna,whichofthesesignswouldthenurseexpecttofindonassessment? a. Rhinorrhea b. Periorbitaledema c. Pain over the maxillarysinuses d. Enlargedsuperficialcervicalnodes ANS:D Thelymphaticdrainageoftheexternalearflowstotheparotid,mastoid,andsuperficialcervicalnodes.The signsaresevereswellingofthecanal,inflammation,andtenderness.Rhinorrhea,periorbitaledema,andpain over the maxillary sinuses do not occur with otitisexterna. DIF: Cognitive Level: Understanding (Comprehension) MSC: Client Needs: Physiologic Integrity: Physiologic Adaptation 29. Whenperforminganotoscopicexaminationofa5-year-oldchildwithahistoryofchronicearinfections, the nurse sees that his right tympanic membrane is amber-yellow in color and that air bubbles are visible behind the tympanic membrane. The child reports occasional hearing loss and a popping sound with swallowing.Thepreliminaryanalysisbasedonthisinformationisthatthechild: a. Most likely has serous otitismedia. b. Has an acute purulent otitismedia. c. HasevidenceofaresolvingcholesNteUaRtoSmINa.GTB.COM d. Is experiencing the early stages ofperforation. ANS: A An amber-yellow color to the tympanic membrane suggests serum or pus in the middle ear. Air or fluid or bubblesbehindthetympanicmembraneareoftenvisible.Thepatientmayhavefeelingsoffullness,transient hearing loss, and a popping sound with swallowing. These findings most likely suggest that the child has serous otitis media. The other responses are notcorrect. DIF: Cognitive Level: Analyzing (Analysis) MSC: Client Needs: Physiologic Integrity: Physiologic Adaptation 30. Thenurseisperforminganassessmentona65-year-oldman.Hereportsacrustynodulebehindthepinna. Itintermittentlybleedsandhasnothealedoverthepast6months.Onphysicalassessment,thenursefindsan ulceratedcrustednodulewithaninduratedbase.Thepreliminaryanalysisinthissituationisthatthis: a. Is most likely a benign sebaceouscyst. b. Is most likely akeloid. c. Couldbeapotentialcarcinoma,andthepatientshouldbereferredforabiopsy. d. Isatophus,whichiscommonintheolderadultandisasignofgout. ANS: C Anulceratedcrustednodulewithaninduratedbasethatfailstohealischaracteristicofacarcinoma.These lesionsfailtohealandintermittentlybleed.Individualswithsuchsymptomsshouldbereferredforabiopsy. The other responses are notcorrect. DIF: Cognitive Level: Analyzing (Analysis) MSC: Client Needs: Physiologic Integrity: Physiologic Adaptation 31. Thenursesuspectsthatapatienthasotitismedia.Earlysignsofotitismediaincludewhichofthese findings of the tympanicmembrane? a. Red andbulging b. Hypomobility c. Retraction with landmarks clearlyvisible d. Flat,slightlypulledinatthecenter,andmoveswithinsufflation ANS:B AnearlysignofotitismediaishypomobilityoNfUthReSItyNmGpTaBn.iCcOmMembrane.Aspressureincreases,thetympanic membrane begins to bulge. DIF: Cognitive Level: Remembering (Knowledge) MSC: Client Needs: Safe and Effective Care Environment: Management of Care 32. Thenurseisperformingamiddleearassessmentona15-year-oldpatientwhohashadahistoryofchronic ear infections. When examining the right tympanic membrane, the nurse sees the presence of dense white patches.Thetympanicmembraneisotherwiseunremarkable.Itispearly,withthelightreflexat5oclockand landmarks visible. The nurseshould: a. Refer the patient for the possibility of a fungalinfection. b. Know that these are scars caused from frequent earinfections. c. Considerthatthesefindingsmayrepresentthepresenceofbloodinthemiddleear. d. Beconcernedabouttheabilitytohearbecauseofthisabnormalityonthetympanicmembrane. ANS: B Densewhitepatchesonthetympanicmembranearesequelaeofrepeatedearinfections.Theydonot necessarily affecthearing. DIF: Cognitive Level: Analyzing (Analysis) MSC: Client Needs: Safe and Effective Care Environment: Management of Care 33. Thenurseispreparingtodoanotoscopicexaminationona2-year-oldchild.Whichoneofthesereflects the correctprocedure? a. Pulling the pinnadown b. Pulling the pinna up andback c. Slightly tilting the childs head toward theexaminer d. Instructingthechildtotouchhischintohischest ANS:A Foranotoscopicexaminationonaninfantoronachildunder3yearsofage,thepinnaispulleddown.The other responses are not part of the correctprocedure. DIF: Cognitive Level: Applying (Application) MSC: Client Needs: Health Promotion and Maintenance 34. Thenurseisconductingachildsafetyclassfornewmothers.Whichfactorplacesyoungchildrenatriskfor earinfections? a. Familyhistory b. Airconditioning c. Excessivecerumen d. Passive cigarettesmoke NURSINGTB.COM ANS: D Exposuretopassiveandgestationalsmokeisariskfactorforearinfectionsininfantsandchildren. DIF: Cognitive Level: Understanding(Comprehension) MSC: Client Needs: Physiologic Integrity: Reduction of Risk Potential 35. Duringanotoscopicexamination,thenursenoticesanareaofblackandwhitedotsonthetympanic membrane and the ear canal wall. What does this findingsuggest? a. Malignancy b. Viralinfection c. Blood in the middleear d. Yeast or fungalinfection ANS: D Acolonyofblackorwhitedotsonthedrumorcanalwallsuggestsayeastorfungalinfection(otomycosis). DIF: Cognitive Level: Understanding(Comprehension) MSC: Client Needs: Physiologic Integrity: Basic Care and Comfort 36. A17-year-oldstudentisaswimmeronherhighschoolsswimteam.Shehashadthreeboutsofotitis externathisseasonandwantstoknowwhattodotopreventit.Thenurseinstructsherto: a. Useacotton-tippedswabtodrytheearcanalsthoroughlyaftereachswim. b. Use rubbing alcohol or 2% acetic acid eardrops after everyswim. c. Irrigatetheearswithwarmwaterandabulbsyringeaftereachswim. d. Rinsetheearswithawarmedsolutionofmineraloilandhydrogenperoxide. ANS: B Withotitisexterna(swimmersear),swimmingcausestheexternalcanaltobecomewaterloggedandswell; skinfoldsaresetupforinfection.Otitisexternacanbepreventedbyusingrubbingalcoholor2%aceticacid eardrops after every swim. DIF:CognitiveLevel:Analyzing(Analysis) NURSINGTB.COM MSC: Client Needs: Health Promotion and Maintenance 37. Duringanexamination,thepatientstatesheishearingabuzzingsoundandsaysthatitisdrivingmecrazy! The nurse recognizes that this symptomindicates: a. Vertigo. b. Pruritus. c. Tinnitus. d. Cholesteatoma. ANS: C Tinnitusisasoundthatcomesfromwithinaperson;itcanbearinging,crackling,orbuzzingsound.It accompanies some hearing or eardisorders. DIF: Cognitive Level: Understanding (Comprehension) MSC: Client Needs: Physiologic Integrity: Basic Care and Comfort 38. Duringanexamination,thenursenoticesthatthepatientstumblesalittlewhilewalking,and,whenshesits down,sheholdsontothesidesofthechair.Thepatientstates,Itfeelsliketheroomisspinning!Thenurse notices that the patient isexperiencing: a. Objectivevertigo. b. Subjectivevertigo. c. Tinnitus. d. Dizziness. ANS: A Withobjectivevertigo,thepatientfeelsliketheroomspins;withsubjectivevertigo,thepersonfeelslikeheor sheisspinning.Tinnitusisasoundthatcomesfromwithinaperson;itcanbearinging,crackling,orbuzzing sound.Itaccompaniessomehearingoreardisorders.Dizzinessisnotthesameastruevertigo;thepersonwho is dizzy may feel unsteady andlightheaded. DIF: Cognitive Level: Applying (Application) MSC: Client Needs: Physiologic Integrity: Physiologic Adaptation 39. Apatienthasbeenadmittedafteranaccidentatwork.Duringtheassessment,thepatientishavingtrouble hearing and states, I dont know what the matter is. All of a sudden, I cant hear you out of my left ear! What should the nurse donext? NURSINGTB.COM a. Make note of this finding for the report to the nextshift. b. Prepare to remove cerumen from the patientsear. c. Notify the patients health careprovider. d. Irrigate the ear with rubbingalcohol. ANS: C Anysuddenlossofhearinginoneorbothearsthatisnotassociatedwithanupperrespiratoryinfectionneeds tobereportedatoncetothepatientshealthcareprovider.Hearinglossassociatedwithtraumaisoftensudden. Irrigating the ear or removing cerumen is not appropriate at thistime. DIF: Cognitive Level: Applying (Application) MSC: Client Needs: Physiologic Integrity: Basic Care and Comfort MULTIPLE RESPONSE 1.Thenurseistestingthehearingofa78-year-oldmanandisremindedofthechangesinhearingthatoccur with aging that include which of the following? Select all thatapply. a. Hearing loss related to aging begins in the mid40s. b. Progression of hearing loss isslow. c. The aging person has low-frequency toneloss. d. Theagingpersonmayfindithardertohearconsonantsthanvowels. e. Sounds may be garbled and difficult tolocalize. f. Hearing loss reflects nerve degeneration of the middleear. ANS: B, D, E Presbycusisisatypeofhearinglossthatoccurswithagingandisfoundin60%ofthoseolderthan65years.It is a gradual sensorineural loss caused by nerve degeneration in the inner ear or auditory nerve, and it slowly progresses after the age of 50 years. The person first notices a high-frequency tone loss; it is harder to hear consonants(high-pitchedcomponentsofspeech)thanvowels,whichmakeswordssoundgarbled.Theability to localize sound is alsoimpaired. DIF: Cognitive Level: Applying (Application) MSC: Client Needs: Health Promotion and Maintenance NURSINGTB.COM Chapter 17: Nose, Mouth, and Throat MULTIPLE CHOICE 1. Theprimarypurposeoftheciliatedmucousmembraneinthenoseisto: a. Warm the inhaledair. b. Filter out dust andbacteria. c. Filter coarse particles from inhaledair. d. Facilitate the movement of air through thenares. ANS: B Thenasalhairsfilterthecoarsestmatterfrominhaledair,whereasthemucousblanketfiltersoutdustand bacteria.Therichbloodsupplyofthenasalmucosawarmstheinhaledair. DIF:CognitiveLevel:Remembering(Knowledge) MSC: Client Needs:General 2. Theprojectionsinthenasalcavitythatincreasethesurfaceareaarecalledthe: a. Meatus. b. Septum. c. Turbinates. d. Kiesselbachplexus. NURSINGTB.COM ANS: C Thelateralwallsofeachnasalcavitycontainthreeparallelbonyprojections:thesuperior,middle,andinferior turbinates. These increase the surface area, making more blood vessels and mucous membrane available to warm, humidify, and filter the inhaledair. DIF:CognitiveLevel:Remembering(Knowledge) MSC: Client Needs:General 3. Thenurseisreviewingthedevelopmentofthenewborninfant.Regardingthesinuses,whichstatementis true in relation to a newborn infant? a. Sphenoid sinuses are full size atbirth. b. Maxillary sinuses reach full size afterpuberty. c. Frontal sinuses are fairly well developed atbirth. d. Maxillary and ethmoid sinuses are the only sinuses present atbirth. ANS: D Only the maxillary and ethmoid sinuses are present at birth. The sphenoid sinuses are minute at birth and developafterpuberty.Thefrontalsinusesareabsentatbirth,arefairlywelldevelopedatage7to8years,and reach full size afterpuberty. DIF:CognitiveLevel:Remembering(Knowledge) MSC: Client Needs:General 4. Thetissuethatconnectsthetonguetothefloorofthemouthisthe: a. Uvula. b. Palate. c. Papillae. d. Frenulum. ANS: D NURSINGTB.COM Thefrenulumisamidlinefoldoftissuethatconnectsthetonguetothefloorofthemouth.Theuvulaisthefree projectionhangingdownfromthemiddleofthesoftpalate.Thepalateisthearchingroofofthemouth. Papillaearetherough,bumpyelevationsonthetonguesdorsalsurface. DIF: Cognitive Level: Remembering(Knowledge) MSC: Client Needs: General 5. Thesalivaryglandthatisthelargestandlocatedinthecheekinfrontoftheearisthe gland. a. Parotid b. Stensens c. Sublingual d. Submandibular ANS:A Themouthcontainsthreepairsofsalivaryglands.Thelargest,theparotidgland,lieswithinthecheeksinfront of the ear extending from the zygomatic arch down to the angle of the jaw. The Stensens duct (not gland) drains the parotid gland onto the buccal mucosa opposite the second molar. The sublingual gland is located withinthefloorofthemouthunderthetongue.Thesubmandibularglandliesbeneaththemandibleattheangle of the jaw. DIF:CognitiveLevel:Remembering(Knowledge) MSC: Client Needs:General 6. Inassessingthetonsilsofa30yearold,thenursenoticesthattheyareinvoluted,granularinappearance, andappeartohavedeepcrypts.Whatiscorrectresponsetothesefindings? a. Refer the patient to a throatspecialist. b. No response is needed; this appearance is normal for thetonsils. c. Continue with the assessment, looking for any other abnormalfindings. d. Obtainathroatcultureonthepatientforpossiblestreptococcal(strep)infection. ANS: B Thetonsilsarethesamecolorasthesurroundingmucousmembrane,althoughtheylookmoregranularand theirsurfaceshowsdeepcrypts.Tonsillartissueenlargesduringchildhooduntilpubertyandtheninvolutes. DIF: Cognitive Level: Applying (Application) MSC: Client Needs: Safe and Effective Care Environment: Management of Care 7. ThenurseisobtainingahealthhistoryonaN3U-mRoSnINthG-oTlBd.iCnOfaMnt.Duringtheinterview,themotherstates,I thinksheisgettingherfirsttoothbecauseshehasstarteddroolingalot.Thenursesbestresponsewouldbe: a. Youre right, drooling is usually a sign of the firsttooth. b. Itwouldbeunusualfora3montholdtobegettingherfirsttooth. c. This could be the sign of a problem with the salivaryglands. d. Sheisjuststartingtosalivateandhasntlearnedtoswallowthesaliva. ANS: D Intheinfant,salivationstartsat3months.Thebabywilldroolforafewmonthsbeforelearningtoswallowthe saliva.Thisdroolingdoesnotheraldtheeruptionofthefirsttooth,althoughmanyparentsthinkitdoes. DIF:CognitiveLevel:Understanding(Comprehension) MSC:ClientNeeds:HealthPromotionandMaintenance 8. Thenurseisassessingan80-year-oldpatient.Whichofthesefindingswouldbeexpectedforthispatient? a. Hypertrophy of thegums b. Increased production ofsaliva c. Decreased ability to identifyodors d. Finerandlessprominentnasalhair ANS:C Thesenseofsmellmaybereducedbecauseofadecreaseinthenumberofolfactorynervefibers.Nasalhairs growcoarserandstifferwithaging.Thegumsmayrecedewithaging,nothypertrophy,andsalivaproduction decreases. DIF:CognitiveLevel:Understanding(Comprehension) MSC:ClientNeeds:HealthPromotionandMaintenance 9. Thenurseisperforminganoralassessmentona40-year-oldBlackpatientandnoticesthepresenceofa1 cm,nontender,grayish-whitelesionontheleftbuccalmucosa.Whichoneofthesestatementsistrue?This lesionis: a. Leukoedema and is common in dark-pigmentedpersons. b. The result of hyperpigmentation and isnormal. c. Torus palatinus and would normally be found only insmokers. d. IndicativeofcancerandshouldbeNimURmSeIdNiaGteTlBy.CteOstMed. ANS: A Leukoedema,agrayish-whitebenignlesionoccurringonthebuccalmucosa,ismostoftenobservedinBlacks. DIF: Cognitive Level: Understanding(Comprehension) MSC: Client Needs: Physiologic Integrity: Physiologic Adaptation 10. Whileobtainingahealthhistory,apatienttellsthenursethathehasfrequentnosebleedsandasksthebest way to get them to stop. What would be the nurses bestresponse? a. While sitting up, place a cold compress over yournose. b. Sit up with your head tilted forward and pinch yournose. c. Justallowthebleedingtostoponitsown,butdontblowyournose. d. Lieonyourbackwithyourheadtiltedbackandpinchyournose. ANS: B Withanosebleed,thepersonshouldsitupwiththeheadtiltedforwardandpinchthenosebetweenthethumb and forefinger for 5 to 15minutes. DIF: Cognitive Level: Applying (Application) MSC: Client Needs: Physiologic Integrity: Physiologic Adaptation 11. A92-year-oldpatienthashadastroke.Therightsideofhisfaceisdrooping.Thenursemightalsosuspect which of these assessmentfindings? a. Epistaxis b. Rhinorrhea c. Dysphagia d. Xerostomia ANS:C Dysphagiaisdifficultywithswallowingandmayoccurwithavarietyofdisorders,includingstrokeandother neurologicdiseases.Rhinorrheaisarunnynose,epistaxisisabloodynose,andxerostomiaisadrymouth. DIF: Cognitive Level: Analyzing (Analysis) MSC: Client Needs: Physiologic Integrity: Physiologic Adaptation 12. Whileobtainingahealthhistoryfromthemotherofa1-year-oldchild,thenursenoticesthatthebabyhas hadabottleinhismouththeentiretime.Themotherstates,Itmakesagreatpacifier.Thebestresponsebythe nurse would be: NURSINGTB.COM a. Youre right. Bottles make very goodpacifiers. b. Usingabottleasapacifierisbetterfortheteeththanthumb-sucking. c. Itsokaytouseabottleaslongasitcontainsmilkandnotjuice. d. Prolongeduseofabottlecanincreasetheriskfortoothdecayandearinfections. ANS: D Prolongedbottleuseduringthedayorwhengoingtosleepplacestheinfantatriskfortoothdecayandmiddle earinfections. DIF: Cognitive Level: Applying (Application) MSC: Client Needs: Health Promotion and Maintenance 13. A72-year-oldpatienthasahistoryofhypertensionandchroniclungdisease.Animportantquestionforthe nurse to include in the health history wouldbe: a. Do you use a fluoridesupplement? b. Have you had tonsillitis in the lastyear? c. At what age did you get your firsttooth? d. Have you noticed any dryness in yourmouth? ANS: D Xerostomia(drymouth)isasideeffectofmanydrugstakenbyolderpeople,includingantidepressants, anticholinergics, antispasmodics, antihypertensives, antipsychotics, andbronchodilators. DIF: Cognitive Level: Applying (Application) MSC: Client Needs: Physiologic Integrity: Physiologic Adaptation 14. Thenurseisusinganotoscopetoassessthenasalcavity.Whichofthesetechniquesiscorrect? a. Inserting the speculum at least 3 cm into thevestibule b. Avoiding touching the nasal septum with thespeculum c. Gently displacing the nose to the side that is beingexamined d. Keepingthespeculumtipmedialtoavoidtouchingthefloorofthenares ANS: B NURSINGTB.COM Thecorrecttechniqueforusinganotoscopeistoinserttheapparatusintothenasalvestibule,avoidingpressure onthesensitivenasalseptum.Thetipofthenoseshouldbeliftedupbeforeinsertingthespeculum. DIF: Cognitive Level: Understanding (Comprehension) MSC: Client Needs: Safe and Effective Care Environment: Management of Care 15. Thenurseisperforminganassessmentona21-year-oldpatientandnoticesthathisnasalmucosaappears pale,gray,andswollen.Whatwouldbethemostappropriatequestiontoaskthepatient? a. Are you aware of having anyallergies? b. Do you have an elevatedtemperature? c. Have you had any symptoms of acold? d. Have you been having frequentnosebleeds? ANS: A Withchronicallergies,themucosalooksswollen,boggy,pale,andgray.Elevatedbodytemperature,colds,and nosebleeds do not cause these mucosalchanges. DIF: Cognitive Level: Applying (Application) MSC: Client Needs: Safe and Effective Care Environment: Management of Care 16. Thenurseispalpatingthesinusareas.Ifthefindingsarenormal,thenthepatientshouldreportwhich sensation? a. Nosensation b. Firmpressure c. Pain duringpalpation d. Painsensationbehindeyes ANS:B Thepersonshouldfeelfirmpressurebutnopain.Sinusareasaretendertopalpationinpersonswithchronic allergies or an acute infection(sinusitis). DIF: Cognitive Level: Remembering (Knowledge) MSC: Client Needs: Safe and Effective Care Environment: Management of Care 17. Duringanoralassessmentofa30-year-oldBlackpatient,thenursenoticesbluishlipsandadarklinealong thegingivalmargin.Whatactionwouldthenurseperforminresponsetothisfinding? a. CheckthepatientshemoglobinforNaUnRemSIiNa.GTB.COM b. Assess for other signs of insufficient oxygensupply. c. Proceedwiththeassessment,knowingthatthisappearanceisanormalfinding. d. Askifhehasbeenexposedtoanexcessiveamountofcarbonmonoxide. ANS: C SomeBlacksmayhavebluishlipsandadarklineonthegingivalmargin;thisappearanceisanormalfinding. DIF: Cognitive Level: Applying(Application) MSC: Client Needs: Safe and Effective Care Environment: Management of Care 18. Duringanassessmentofa20-year-oldpatientwitha3-dayhistoryofnauseaandvomiting,thenurse noticesdrymucosaanddeepverticalfissuresinthetongue.Thesefindingsarereflectiveof: a. Dehydration. b. Irritationbygastricjuices. c. Anormaloralassessment. d. Side effects from nauseamedication. ANS: A Drymouthoccurswithdehydrationorfever.Thetonguehasdeepverticalfissures. DIF: Cognitive Level: Applying(Application) MSC: Client Needs: Physiologic Integrity: Physiologic Adaptation 19. A32-year-oldwomanisattheclinicforlittlewhitebumpsinmymouth.Duringtheassessment,thenurse notesthatshehasa0.5cmwhite,nontenderpapuleunderhertongueandoneonthemucosaofherrightcheek What would the nurse tell thepatient? a. These spots indicate an infection such as strepthroat. b. Thesebumpscouldbeindicativeofaseriouslesion,soIwillreferyoutoaspecialist. c. Thisconditioniscalledleukoplakiaandcanbecausedbychronicirritationsuchaswithsmoking. d. ThesebumpsareFordycegranules,whicharesebaceouscystsandarenotaseriouscondition. ANS: D Fordyce granules are small, isolated white or yellow papules on the mucosa of the cheek, tongue, and lips. TheselittlesebaceouscystsarepainlessandaNreUnRoStIsNigGnTifBic.CanOtM.Chalky,whiteraisedpatcheswouldindicate leukoplakia.Instrepthroat,theexaminerwouldseetonsilsthatarebrightred,swollen,andmayhaveexudates or whitespots. DIF: Cognitive Level: Applying (Application) MSC: Client Needs: Physiologic Integrity: Physiologic Adaptation 20. A10yearoldisattheclinicforasorethroatthathaslasted6days.Whichofthesefindingswouldbe consistent with an acuteinfection? a. Tonsils 1+/1-4+ and pink; the same color as the oralmucosa b. Tonsils 2+/1-4+ with small plugs of whitedebris c. Tonsils 3+/1-4+ with large whitespots d. Tonsils3+/1-4+withpalecoloring ANS:C Withanacuteinfection,tonsilsarebrightredandswollenandmayhaveexudateorlargewhitespots.Tonsils are enlarged to 2+, 3+, or 4+ with an acuteinfection. DIF: Cognitive Level: Understanding (Comprehension) MSC: Client Needs: Physiologic Integrity: Physiologic Adaptation 21. Immediatelyafterbirth,thenurseisunabletosuctionthenaresofanewborn.Anattemptismadetopassa catheterthroughbothnasalcavitieswithnosuccess.Whatshouldthenursedonext? a. Attempt to suction again with a bulbsyringe. b. Wait a few minutes, and try again once the infant stopscrying. c. Recognize that this situation requires immediateintervention. d. Contactthephysiciantoscheduleanappointmentfortheinfantathisorhernexthospitalvisit. ANS: C Determiningthepatencyofthenaresintheimmediatenewbornperiodisessentialbecausemostnewbornsare obligate nose breathers. Nares blocked with amniotic fluid are gently suctioned with a bulb syringe. If obstructionissuspected,thenasmalllumen(5to10Fr)catheterispasseddowneachnaristoconfirmpatency. The inability to pass a catheter through the nasal cavity indicates choanal atresia, which requires immediate intervention. DIF: Cognitive Level: Analyzing (Analysis) MSC: Client Needs: Physiologic Integrity: Physiologic Adaptation 22. Thenursenoticesthatthemotherofa2-year-oldboybringshimintotheclinicquitefrequentlyforvarious injuriesandsuspectstheremaybesomechildNaUbuRsSeINinGvoTlBv.eCdO.DMuringaninspectionofhismouth,thenurse should look for: a. Swollen, redtonsils. b. Ulcerations on the hardpalate. c. Bruising on the buccal mucosa orgums. d. Small yellow papules along the hardpalate. ANS: C Thenurseshouldnoticeanybruisingorlacerationonthebuccalmucosaorgumsofaninfantoryoungchild. Traumamayindicatechildabusefromaforcedfeedingofabottleorspoon. DIF: Cognitive Level: Applying (Application) MSC: Client Needs: Physiologic Integrity: Reduction of Risk Potential 23. Thenurseisassessinga3yearoldfordrainagefromthenose.Onassessment,apurulentdrainagethathas a very foul odor is noted from the left naris and no drainage is observed from the right naris. The child is afebrile with no other symptoms. What should the nurse donext? a. Refer to the physician for an antibioticorder. b. Have the mother bring the child back in 1week. c. Perform an otoscopic examination of the leftnares. d. Tellthemotherthatthisdrainageisnormalforachildofthisage. ANS: C Childrenarepronetoputanobjectupthenose,producingunilateralpurulentdrainagewithafoulodor. Because some risk for aspiration exists, removal should beprompt. DIF: Cognitive Level: Analyzing (Analysis) MSC: Client Needs: Physiologic Integrity: Physiologic Adaptation 24. Duringanassessmentofa26yearoldattheclinicforaspotonmylipIthinkiscancer,thenursenoticesa group of clear vesicles with an erythematous base around them located at the lip-skin border. The patient mentionsthatshejustreturnedfromHawaii.Whatwouldbethemostappropriateresponsebythenurse? a. Tell the patient she needs to see a skinspecialist. b. Discussthebenefitsofhavingabiopsyperformedonanyunusuallesion. c. TellthepatientthatthesevesiclesareindicativeofherpessimplexIorcoldsoresandthattheywill heal in 4 to 10days. NURSINGTB.COM d. Tellthepatientthatthesevesiclesaremostlikelytheresultofariboflavindeficiencyanddiscuss nutrition. ANS: C Coldsoresaregroupsofclearvesicleswithasurroundingerythematousbase.Theseevolveintopustulesor crustsandhealin4to10days.Themostlikelysiteisthelip-skinjunction.Infectionoftenrecursinthesame site.Recurrentherpesinfectionsmaybeprecipitatedbysunlight,fever,colds,orallergy. DIF: Cognitive Level: Analyzing (Analysis) MSC: Client Needs: Physiologic Integrity: Physiologic Adaptation 25. Whileperforminganassessmentofthemouth,thenursenoticesthatthepatienthasa1-cmulcerationthat iscrustedwithanelevatedborderandlocatedontheouterthirdofthelowerlip.Whatotherinformationwould be most important for the nurse toassess? a. Nutritionalstatus b. When the patient first noticed thelesion c. Whether the patient has had a recentcold d. Whether the patient has had any recent exposure to sickanimals ANS: B Withcarcinoma,theinitiallesionisroundandindurated,butthenitbecomescrustedandulceratedwithan elevatedborder.Mostcancersoccurbetweentheouterandmiddlethirdsofthelip.Anylesionthatisstill unhealed after 2 weeks should bereferred. DIF: Cognitive Level: Applying (Application) MSC: Client Needs: Physiologic Integrity: Reduction of Risk Potential 26. Apregnantwomanstatesthatsheisconcernedabouthergumsbecauseshehasnoticedtheyareswollen andhavestartedbleeding.Whatwouldbeanappropriateresponsebythenurse? a. Your condition is probably due to a vitamin Cdeficiency. b. Imnotsurewhatcausesswollenandbleedinggums,butletmeknowifitsnotbetterinafew weeks. c. Youneedtomakeanappointmentwithyourdentistassoonaspossibletohavethischecked. d. Swollenandbleedinggumscanbecausedbythechangeinhormonalbalanceinyoursystem duringpregnancy. ANS: D NURSINGTB.COM Gummarginsareredandswollenandeasilybleedwithgingivitis.Achanginghormonalbalancemaycause this condition to occur in pregnancy andpuberty. DIF: Cognitive Level: Applying (Application) MSC: Client Needs: Physiologic Integrity: Physiologic Adaptation 27. A 40-year-old patient who has just finished chemotherapy for breast cancer tells the nurse that she is concernedabouthermouth.Duringtheassessmentthenursefindsareasofbuccalmucosathatarerawandred with some bleeding, as well as other areas that have a white, cheesy coating. The nurse recognizes that this abnormalityis: a. Aphthousulcers. b. Candidiasis. c. Leukoplakia. d. Koplikspots. ANS: B Candidiasisisawhite,cheesy,curdlikepatchonthebuccalmucosaandtongue.Itscrapesoff,leavingaraw, redsurfacethateasilybleeds.Italsooccursaftertheuseofantibioticsorcorticosteroidsandinpersonswho areimmunosuppressed. DIF: Cognitive Level: Applying (Application) MSC: Client Needs: Physiologic Integrity: Physiologic Adaptation 28. Thenurseisassessingapatientinthehospitalwhohasreceivednumerousantibioticsandnoticesthathis tongueappearstobeblackandhairy.Inresponsetohisconcern,whatwouldthenursesay? a. We will need to get a biopsy to determine thecause. b. Thisisanovergrowthofhairandwillgoawayinafewdays. c. Black,hairytongueisafungalinfectioncausedbyalltheantibioticsyouhavereceived. d. Thisisprobablycausedbythesamebacteriayouhadinyourlungs. ANS: C A black, hairy tongue is not really hair but the elongation of filiform papillae and painless overgrowth of mycelialthreadsoffungusinfectiononthetongue.Itoccursaftertheuseofantibiotics,whichinhibitnormal bacteria and allow a proliferation offungus. DIF:CognitiveLevel:Analyzing(Analysis) MSC: Client Needs: Physiologic Integrity: Physiologic Adaptation 29. Thenurseisassessingapatientwithahistoryofintravenousdrugabuse.Inassessinghismouth,thenurse noticesadarkredconfluentmaculeontheharNdUpRalSaItNe.GTThBis.CcOoMuldbeanearlysignof: a. Acquired immunodeficiency syndrome(AIDS). b. Measles. c. Leukemia. d. Carcinoma. ANS: A Oral Kaposis sarcoma is a bruiselike, dark red or violet, confluent macule that usually occurs on the hard palate.Itmayappearonthesoftpalateorgingivalmargin.Orallesionsmaybeamongtheearliestlesionsto develop withAIDS. DIF: Cognitive Level: Understanding (Comprehension) MSC: Client Needs: Physiologic Integrity: Physiologic Adaptation 30. Amotherbringsher4-month-oldinfanttotheclinicwithconcernsregardingasmallpadinthemiddleof the upper lip that has been there since 1 month of age. The infant has no health problems. On physical examination,thenursenoticesa0.5-cm,fleshy,elevatedareainthemiddleoftheupperlip.Noevidenceof inflammationordrainageisobserved.Whatwouldthenursetellthismother? a. Thisareaofirritationiscausedfromteethingandisnothingtoworryabout. b. Thisfindingisabnormalandshouldbeevaluatedbyanotherhealthcareprovider. c. Thisareaofirritationistheresultofchronicdroolingandshouldresolvewithinthenextmonthor two. d. Thiselevatedareaisasuckingtuberclecausedfromthefrictionofbreastfeedingorbottle-feeding and isnormal. ANS: D Anormalfindingininfantsisthesuckingtubercle,asmallpadinthemiddleoftheupperlipfromthefriction ofbreastfeedingorbottle-feeding.Thisconditionisnotcausedbyirritation,teething,orexcessivedrooling, and evaluation by another health care provider is notwarranted. DIF: Cognitive Level: Analyzing (Analysis) MSC: Client Needs: Health Promotion and Maintenance 31. Amotherisconcernedbecauseher18-month-oldtoddlerhas12teeth.Sheiswonderingifthisisnormal for a child of this age. The nurses best response wouldbe: a. How many teeth did you have at thisage? b. All 20 deciduous teeth are expected to erupt by age 4years. NURSINGTB.COM c. This is a normal number of teeth for an 18 monthold. d. Normally, by age 2 years, 16 deciduous teeth areexpected. ANS: C Theguidelinesforthenumberofteethforchildrenyoungerthan2yearsoldareasfollows:thechildsagein monthsminusthenumber6shouldbeequaltotheexpectednumberofdeciduousteeth.Normally,all20teeth areinby2yearsold.Inthisinstance,thechildis18monthsold,minus6,equals12deciduousteethexpected. DIF: Cognitive Level: Applying (Application) MSC: Client Needs: Health Promotion and Maintenance 32. Whenexaminingthemouthofanolderpatient,thenurserecognizeswhichfindingisduetotheaging process? a. Teeth appearingshorter b. Tongue that looks smoother inappearance c. Buccal mucosa that is beefy red inappearance d. Small, painless lump on the dorsum of thetongue ANS: B Intheagingadult,thetonguelookssmootherbecauseofpapillaryatrophy.Theteethareslightlyyellowedand appear longer because of the recession of gingivalmargins. DIF: Cognitive Level: Remembering (Knowledge) MSC:ClientNeeds:HealthPromotionandMaintenance 33. Whenexaminingthenaresofa45-year-oldpatientwhohascomplaintsofrhinorrhea,itchingofthenose andeyes,andsneezing,thenursenoticesthefollowing:paleturbinates,swellingoftheturbinates,andclear rhinorrhea. Which of these conditions is most likely thecause? a. Nasalpolyps b. Acutesinusitis c. Allergicrhinitis d. Acute rhinitis ANS:C Rhinorrhea, itching of the nose and eyes, and sneezing are present with allergic rhinitis. On physical examination,serousedemaisnoted,andthetuNrUbiRnSatIeNsGuTsBua.CllOyMappearpalewithasmooth,glisteningsurface. DIF: Cognitive Level: Analyzing (Analysis) MSC: Client Needs: Physiologic Integrity: Physiologic Adaptation 34. Whenassessingthetongueofanadult,thenurseknowsthatanabnormalfindingwouldbe: a. Smooth glossy dorsalsurface. b. Thin white coating over thetongue. c. Raised papillae on the dorsalsurface. d. Visible venous patterns on the ventralsurface. ANS: A Thedorsalsurfaceofthetongueisnormallyroughenedfrompapillae.Athinwhitecoatingmaybepresent. Theventralsurfacemayshowveins.Smooth,glossyareasmayindicateatrophicglossitis. DIF: Cognitive Level: Understanding (Comprehension) MSC: Client Needs: Physiologic Integrity: Physiologic Adaptation 35. Thenurseisperforminganassessment.Whichofthesefindingswouldcausethegreatestconcern? a. Painful vesicle inside the cheek for 2days b. Presence of moist, nontender Stensensducts c. Stippled gingival margins that snugly adhere to theteeth d. Ulcerationonthesideofthetonguewithrollededges ANS:D Ulceration on the side or base of the tongue or under the tongue raises the suspicion of cancer and must be investigated.Theriskofearlymetastasisispresentbecauseofrichlymphaticdrainage.Thevesiclemaybean aphthousulcer,whichispainfulbutnotdangerous.Theotherresponsesarenormalfindings. DIF: Cognitive Level: Applying (Application) MSC: Client Needs: Safe and Effective Care Environment: Management of Care 36. Apatienthasbeendiagnosedwithstrepthroat.Thenurseisawarethatwithouttreatment,which complication mayoccur? a. Rubella b. Leukoplakia c. Rheumaticfever d. Scarletfever NURSINGTB.COM ANS: C Untreatedstrepthroatmayleadtorheumaticfever.Whenperformingahealthhistory,thepatientshouldbe askedwhetherhisorhersorethroathasbeendocumentedasstreptococcal. DIF: Cognitive Level: Understanding (Comprehension) MSC: Client Needs: Physiologic Integrity: Reduction of Risk Potential 37. During a checkup, a 22-year-old woman tells the nurse that she uses an over-the-counter nasal spray becauseofherallergies.Shealsostatesthatitdoesnotworkaswellasitusedtowhenshefirststartedusingit. The best response by the nurse wouldbe: a. Youshouldneveruseover-the-counternasalspraysbecauseoftheriskofaddiction. b. Youshouldtryswitchingtoanotherbrandofmedicationtopreventthisproblem. c. Continuingtousethissprayisimportanttokeepyourallergiesundercontrol. d. Usingthesenasalmedicationsirritatestheliningofthenoseandmaycausereboundswelling. ANS: D Themisuseofover-the-counternasalmedicationsirritatesthemucosa,causingreboundswelling,whichisa commonproblem. DIF: Cognitive Level: Analyzing (Analysis) MSC: Client Needs: Physiologic Integrity: Reduction of Risk Potential 38. Duringanoralexaminationofa4-year-oldNative-Americanchild,thenursenoticesthatheruvulais partially split. Which of these statements isaccurate? a. ThisconditionisacleftpalateandiscommoninNativeAmericans. b. A bifid uvula may occur in some Native-Americangroups. c. Thisconditionisduetoaninjuryandshouldbereportedtotheauthorities. d. Abifiduvulaispalatinus,whichfrequentlyoccursinNativeAmericans. ANS: B Bifiduvula,aconditioninwhichtheuvulaisspliteithercompletelyorpartially,occursinsomeNative- Americangroups. DIF: Cognitive Level: Applying (Application) NURSINGTB.COM MSC: Client Needs: Physiologic Integrity: Physiologic Adaptation 39. Apatientcomesintothecliniccomplainingoffacialpain,fever,andmalaise.Onexamination,thenurse notesswollenturbinatesandpurulentdischargefromthenose.Thepatientalsocomplainsofadull,throbbing paininhischeeksandteethontherightsideandpainwhenthenursepalpatestheareas.Thenurserecognizes that this patienthas: a. Posteriorepistaxis. b. Frontalsinusitis. c. Maxillarysinusitis. d. Nasalpolyps. ANS: C Signsofmaxillarysinusitisincludefacialpainafterupperrespiratoryinfection,redswollennasalmucosa, swollenturbinates,andpurulentdischarge.Thepersonalsohasfever,chills,andmalaise.Withmaxillary sinusitis, dull throbbing pain occurs in the cheeks and teeth on the same side, and pain with palpation is present. With frontal sinusitis, pain is above the supraorbitalridge. DIF: Cognitive Level: Analyzing (Analysis) MSC: Client Needs: Physiologic Integrity: Physiologic Adaptation 40. Awomanwhoisinthesecondtrimesterofpregnancymentionsthatshehashadmorenosebleedsthanever sinceshebecamepregnant.Thenurserecognizesthatthisisaresultof: a. A problem with the patients coagulationsystem. b. Increasedvascularityintheupperrespiratorytractasaresultofthepregnancy. c. Increased susceptibility to colds and nasalirritation. d. Inappropriate use of nasalsprays. ANS: B Nasalstuffinessandepistaxismayoccurduringpregnancyasaresultofincreasedvascularityintheupper respiratorytract. DIF: Cognitive Level: Applying (Application) MSC: Client Needs: Health Promotion and Maintenance MULTIPLE RESPONSE 1. Thenurseisteachingahealthclasstohigh-schoolboys.Whendiscussingthetopicofusingsmokeless tobacco(SLT),whichofthesestatementsareaccurate?Selectallthatapply. a. OnepinchofSLTinthemouthforN3U0RmSIiNnuGtTesBd.CeOlivMerstheequivalentofonecigarette. b. UsingSLThasbeenassociatedwithagreaterriskoforalcancerthansmoking. c. Pain is an early sign of oralcancer. d. Pain is rarely an early sign of oralcancer. e. ToothdecayisanotherriskofSLTbecauseoftheuseofsugarasasweetener. f. SLT is considered a healthy alternative tosmoking. ANS: B, D, E OnepinchofSLTinthemouthfor30minutesdeliverstheequivalentofthreecigarettes.Painisrarelyanearly sign of oral cancer. Many brands of SLT are sweetened with sugars, which promotes tooth decay. SLT is not consideredahealthyalternativetosmoking,andtheuseofSLThasbeenassociatedwithagreaterriskoforal cancer thansmoking. DIF: Cognitive Level: Analyzing (Analysis) MSC: Client Needs: Health Promotion and Maintenance 2. Duringanassessment,apatientmentionsthatIjustcantsmelllikeIusedto.Icanbarelysmelltherosesin my garden. Why is that? For which possible causes of changes in the sense of smell will the nurse assess? Select all thatapply. a. Chronic alcoholuse b. Cigarettesmoking c. Frequent episodes of strepthroat d. Chronicallergies e. Aging f. HerpessimplexvirusI ANS: B, D,E Sen Thesenseofsmelldiminisheswithcigarettesmoking,chronicallergies,andaging.Chronicalcoholuse,a historyofstrepthroat,andherpessimplexvirusIarenotassociatedwithchangesinthesenseofsmell. DIF: Cognitive Level: Applying (Application) MSC: Client Needs: Physiologic Integrity: Physiologic Adaptation NURSINGTB.COM Chapter 18: Breasts, Axillae, and Regional Lymphatics MULTIPLE CHOICE 1. Whichofthefollowingstatementsistrueregardingtheinternalstructuresofthebreast?Thebreastismade upof: a. Primarily muscle with very little fibroustissue. b. Fibrous, glandular, and adiposetissues. c. Primarily milk ducts, known as lactiferousducts. d. Glandulartissue,whichsupportsthebreastbyattachingtothechestwall. ANS: B Thebreastismadeupofglandular,fibrous(includingthesuspensoryligaments),andadiposetissues. DIF: Cognitive Level: Remembering(Knowledge) MSC: Client Needs: General 2. Inperformingabreastexamination,thenurseknowsthatexaminingtheupperouterquadrantofthebreastis especiallyimportant.Thereasonforthisisthattheupperouterquadrantis: NURSINGTB.COM a. The largest quadrant of thebreast. b. The location of most breasttumors. c. Where most of the suspensory ligamentsattach. d. Morepronetoinjuryandcalcificationsthanotherlocationsinthebreast. ANS: B Theupperouterquadrantisthesiteofmostbreasttumors.Intheupperouterquadrant,thenurseshouldnotice the axillary tail of Spence, the cone-shaped breast tissue that projects up into the axilla, close to the pectoral group of axillary lymphnodes. DIF: Cognitive Level: Applying (Application) MSC: Client Needs: Safe and Effective Care Environment: Management of Care 3. Inperforminganassessmentofawomansaxillarylymphsystem,thenurseshouldassesswhichofthese nodes? a. Central, axillary, lateral, andsternal b. Pectoral, lateral, anterior, andsternal c. Central, lateral, pectoral, andsubscapular d. Lateral,pectoral,axillary,andsuprascapular ANS:C Thebreasthasextensivelymphaticdrainage.Fourgroupsofaxillarynodesarepresent:(1)central,(2)pectoral (anterior), (3) subscapular (posterior), and (4)lateral. DIF: Cognitive Level: Applying (Application) MSC: Client Needs: Safe and Effective Care Environment: Management of Care 4. Ifapatientreportsarecentbreastinfection,thenthenurseshouldexpecttofind node enlargement. a. Nonspecific b. Ipsilateralaxillary c. Contralateralaxillary d. Inguinalandcervical ANS: B NURSINGTB.COM Thebreasthasextensivelymphaticdrainage.Mostofthelymph,morethan75%,drainsintotheipsilateral,or same side, axillarynodes. DIF: Cognitive Level: Applying (Application) MSC: Client Needs: Physiologic Integrity: Physiologic Adaptation 5. A9-year-oldgirlisintheclinicforasportphysicalexamination.Aftersomeinitialshynessshefinallyasks, AmInormal?Idontseemtoneedabrayet,butIhavesomefriendswhodo.WhatifInevergetbreasts?The nurses best response wouldbe: a. Dont worry, you still have plenty of time todevelop. b. Iknowjusthowyoufeel,Iwasalatebloomermyself.Justbepatient,andtheywillgrow. c. Youwillprobablygetyourperiodsbeforeyounoticeanysignificantgrowthinyourbreasts. d. Iunderstandthatitishardtofeeldifferentfromyourfriends.Breastsusuallydevelopbetween8 and 10 years ofage. ANS: D Adolescentbreastdevelopmentusuallybeginsbetween8and10yearsofage.Thenurseshouldnotbelittlethe girlsfeelingsbyusingstatementslikedontworryorbysharingpersonalexperiences.Thebeginningofbreast developmentprecedesmenarchebyapproximately2years. DIF: Cognitive Level: Applying(Application) MSC: Client Needs: Health Promotion and Maintenance 6. Apatientcontactstheofficeandtellsthenursethatsheisworriedabouther10-year-olddaughterhaving breast cancer. She describes a unilateral enlargement of the right breast with associated tenderness. She is worriedbecausetheleftbreastisnotenlarged.Whatwouldbethenursesbestresponse?Tellthemotherthat: a. Breastdevelopmentisusuallyfairlysymmetricandthatthedaughtershouldbeexaminedright away. b. Sheshouldbringinherdaughterrightawaybecausebreastcancerisfairlycommonin preadolescentgirls. c. Althoughanexaminationofherdaughterwouldruleoutaproblem,herbreastdevelopmentis most likelynormal. d. Itisunusualforbreaststhatarefirstdevelopingtofeeltenderbecausetheyhaventdevelopedmuch fibroustissue. ANS: C Occasionally,onebreastmaygrowfasterthantheother,producingatemporaryasymmetry,whichmaycause some distress; reassurance is necessary. Tenderness is alsocommon. NURSINGTB.COM DIF: Cognitive Level: Applying (Application) MSC: Client Needs: Physiologic Integrity: Physiologic Adaptation 7. A14-year-oldgirlisanxiousaboutnothavingreachedmenarche.Whentakingthehealthhistory,thenurse should ascertain which of the following? The agethat: a. The girl began to developbreasts. b. Her mother developedbreasts. c. She began to develop pubichair. d. She began to develop axillaryhair. ANS: A Fulldevelopmentfromstage2tostage5takesanaverageof3years,althoughtherangeis1to6years.Pubic hairdevelopsduringthistime,andaxillaryhairappears2yearsaftertheonsetofpubichair.Thebeginningof breast development precedes menarche by approximately 2 years. Menarche occurs in breast development stage3or4,usuallyjustafterthepeakoftheadolescentgrowthspurt,whichoccursaroundage12years. DIF: Cognitive Level: Analyzing (Analysis) MSC: Client Needs: Health Promotion and Maintenance 8. A woman is in the family planning clinic seeking birth control information. She states that her breasts changeallmonthlongandthatsheisworriedthatthisisunusual.Whatisthenursesbestresponse?Thenurse should tell herthat: a. Continualchangesinherbreastsareunusual.Thebreastsofnonpregnantwomenusuallystay pretty much the same all monthlong. b. Breastchangesinresponsetostressareverycommonandthatsheshouldassessherlifefor stressfulevents. c. Becauseofthechanginghormonesduringthemonthlymenstrualcycle,cyclicbreastchangesare common. d. Breastchangesnormallyoccuronlyduringpregnancyandthatapregnancytestisneededatthis time. ANS: C Breasts of the nonpregnant woman change with the ebb and flow of hormones during the monthly menstrual cycle.Duringthe3to4daysbeforemenstruation,thebreastsfeelfull,tight,heavy,andoccasionallysore.The breast volume is smallest on days 4 to 7 of the menstrualcycle. DIF: Cognitive Level: Applying (Application) MSC: Client Needs: Health Promotion and Maintenance 9. AwomanhasjustlearnedthatsheispregnaNnUt.RWSIhNaGtaTrBe.sCoOmMethingsthenurseshouldteachherabouther breasts? a. She can expect her areolae to become larger and darker incolor. b. Breasts may begin secreting milk after the fourth month ofpregnancy. c. Sheshouldinspectherbreastsforvisibleveinsandimmediatelyreportthese. d. Duringpregnancy,breastchangesarefairlyuncommon;mostofthechangesoccurafterthebirth. ANS: A Theareolaebecomelargerandgrowadarkerbrownaspregnancyprogresses,andthetuberclesbecomemore prominent. (The brown color fades after lactation, but the areolae never return to their original color). A venouspatternisanexpectedfindingandprominentovertheskinsurfaceanddoesnotneedtobereported. Afterthefourthmonthofpregnancy,colostrum,athick,yellowfluid(precursortomilk),maybeexpressed from thebreasts. DIF: Cognitive Level: Understanding (Comprehension) MSC: Client Needs: Physiologic Integrity: Physiologic Adaptation 10. Thenurseisteachingapregnantwomanaboutbreastmilk.Whichstatementbythenurseiscorrect? a. Yourbreastmilkisimmediatelypresentafterthedeliveryofyourbaby. b. Breastmilkisrichinproteinandsugars(lactose)buthasverylittlefat. c. Thecolostrum,whichispresentrightafterbirth,doesnotcontainthesamenutrientsasbreastmilk. d. Youmaynoticeathick,yellowfluidexpressedfromyourbreastsasearlyasthefourthmonthof pregnancy. ANS: D Afterthefourthmonth,colostrummaybeexpressed.Thisthickyellowfluidistheprecursorofmilk,andit containsthesameamountofproteinandlactosebutpracticallynofat.Thebreastsproducecolostrumforthe first few days after delivery. It is rich with antibodies that protect the newborn against infection; therefore, breastfeeding isimportant. DIF: Cognitive Level: Applying (Application) MSC: Client Needs: Physiologic Integrity: Physiologic Adaptation 11. A65-year-oldpatientremarksthatshejustcannotbelievethatherbreastssagsomuch.Shestatesitmust befromalackofexercise.Whatexplanationshouldthenurseofferher?Aftermenopause: a. Only women with large breasts experiencesagging. b. SaggingisusuallyduetodecreaseNdUmRuSsIcNleGmTBas.sCOwMithinthebreast. c. Adietthatishighinproteinwillhelpmaintainmusclemass,whichkeepsthebreastsfrom sagging. d. Theglandularandfattissueatrophies,causingbreastsizeandelasticitytodiminish,resultingin breasts thatsag. ANS: D After menopause, the glandular tissue atrophies and is replaced with connective tissue. The fat envelope also atrophies,beginninginthemiddleyearsandbecomingsignificantintheeighthandninthdecadesoflife.These changes decrease breast size and elasticity; consequently, the breasts droop and sag, looking flattened and flabby. DIF:CognitiveLevel:Understanding(Comprehension) MSC:ClientNeeds:HealthPromotionandMaintenance 12. Inexamininga70-year-oldmalepatient,thenursenoticesthathehasbilateralgynecomastia.Whichofthe following describes the nurses best course ofaction? a. Recommendthathemakeanappointmentwithhisphysicianforamammogram. b. Ignore it. Benign breast enlargement in men is notunusual. c. Explainthatthisconditionmaybetheresultofhormonalchanges,andrecommendthatheseehis physician. d. Explainthatgynecomastiainmenisusuallyassociatedwithprostateenlargementandrecommend that he be thoroughlyscreened. ANS: C Gynecomastiamayreappearintheagingmanandmaybeattributabletoatestosteronedeficiency. DIF: Cognitive Level: Analyzing(Analysis) MSC: Client Needs: Safe and Effective Care Environment: Management of Care 13. Duringanexaminationofa7-year-oldgirl,thenursenoticesthatthegirlisshowingbreastbudding.What should the nurse donext? a. Ask the young girl if her periods havestarted. b. Assess the girls weight and body mass index(BMI). c. Ask the girls mother at what age she started to developbreasts. d. Nothing; breast budding is a normalfinding. ANS: B NURSINGTB.COM ResearchhasshownthatgirlswithoverweightorobeseBMIlevelshaveahigheroccurrenceofearlyonsetof breastbudding(beforeage8yearsforblackgirlsandage10yearsforwhitegirls)andearlymenarche. DIF: Cognitive Level: Analyzing (Analysis) MSC: Client Needs: Health Promotion and Maintenance 14. Thenurseisreviewingstatisticsregardingbreastcancer.Whichwoman,aged40yearsintheUnitedStates has the highest risk for developing breastcancer? a. Black b. White c. Asian d. AmericanIndian ANS:A Theincidenceofbreastcancervarieswithindifferentculturalgroups.Whitewomenhaveahigherincidenceof breastcancerthanblackwomenstartingatage45years;butblackwomenhaveahigherincidencebeforeage 45years.Asian,Hispanic,andAmericanIndianwomenhavealowerriskfordevelopmentofbreastcancer (American Cancer Society, 2009-2010). DIF: Cognitive Level: Understanding (Comprehension) MSC: Client Needs: Physiologic Integrity: Reduction of Risk Potential 15. Thenurseispreparingforaclassinearlydetectionofbreastcancer.Whichstatementistruewithregardto breast cancer in black women in the UnitedStates? a. Breast cancer is not a threat to blackwomen. b. Blackwomenhavealowerincidenceofregionalordistantbreastcancerthanwhitewomen. c. Blackwomenaremorelikelytodieofbreastcanceratanyage. d. Breastcancerincidenceinblackwomenishigherthanthatofwhitewomenafterage45. ANS: C Blackwomenhaveahigherincidenceofbreastcancerbeforeage45yearsthanwhitewomenandaremore likely to die of their disease. In addition, black women are significantly more likely to be diagnosed with regionalordistantbreastcancerthanarewhitewomen.Theseracialdifferencesinmortalityratesmaybe relatedtoaninsufficientuseofscreeningmeasuresandalackofaccesstohealthcare. DIF: Cognitive Level: Understanding (Comprehension) MSC:ClientNeeds:PhysiologicIntegrity:ReNduUcRtiSoInNoGfTRBi.sCkOPMotential 16. Duringabreasthealthinterview,apatientstatesthatshehasnoticedpaininherleftbreast.Thenurses most appropriate response to this wouldbe: a. Dont worry about the pain; breast cancer is notpainful. b. Iwouldlikesomemoreinformationaboutthepaininyourleftbreast. c. Oh,Ihadpainlikethataftermysonwasborn;itturnedouttobeablockedmilkduct. d. Breast pain is almost always the result of benign breastdisease. ANS: B Breast pain occurs with trauma, inflammation, infection, or benign breast disease. The nurse will need to gathermoreinformationaboutthepatientspainratherthanmakestatementsthatignorethepatientsconcerns. DIF: Cognitive Level: Analyzing (Analysis) MSC: Client Needs: Safe and Effective Care Environment: Management of Care 17. Duringahealthhistoryinterview,afemalepatientstatesthatshehasnoticedafewdropsofcleardischarge from her right nipple. What should the nurse donext? a. Immediately contact the physician to report thedischarge. b. Ask her if she is possiblypregnant. c. Askthepatientsomeadditionalquestionsaboutthemedicationssheistaking. d. Immediately obtain a sample for culture and sensitivitytesting. ANS: C The use of some medications, such as oral contraceptives, phenothiazines, diuretics, digitalis, steroids, methyldopa, and calcium channel blockers, may cause clear nipple discharge. Bloody or blood-tinged dischargefromthenipple,notclear,issignificant,especiallyifalumpisalsopresent.Inthepregnantfemale, colostrum would be a thick, yellowish liquid, and it would be normally expressed after the fourth month of pregnancy. DIF: Cognitive Level: Applying (Application) MSC: Client Needs: Safe and Effective Care Environment: Management of Care 18. Duringaphysicalexamination,a45-year-oldwomanstatesthatshehashadacrusty,itchyrashonher breastforapproximately2weeks.Intryingtofindthecauseoftherash,whichquestionwouldbeimportant for the nurse toask? a. Is the rash raised andred? b. Doesitappeartobecyclic? NURSINGTB.COM c. Wheredidtherashfirstappearonthenipple,theareola,orthesurroundingskin? d. Whatwasshedoingwhenshefirstnoticedtherash,anddoheractionsmakeitworse? ANS: C Thelocationwheretherashfirstappearedisimportantforthenursetodetermine.Pagetdiseasestartswitha small crust on the nipple apex and then spreads to the areola. Eczema or other dermatitis rarely starts at the nippleunlessitisaresultofbreastfeeding.Itusuallystartsontheareolaorsurroundingskinandthenspreads to thenipple. DIF: Cognitive Level: Analyzing (Analysis) MSC: Client Needs: Physiologic Integrity: Physiologic Adaptation 19. Apatientisnewlydiagnosedwithbenignbreastdisease.Thenurserecognizeswhichstatementabout benign breast disease to be true? The presence of benign breastdisease: a. Makes it hard to examine thebreasts. b. Frequentlyturnsintocancerinawomanslateryears. c. Iseasilyreducedwithhormonereplacementtherapy. d. Is usually diagnosed before a woman reaches childbearingage. ANS: A Thepresenceofbenignbreastdisease(formerlyfibrocysticbreastdisease)makesithardtoexaminethe breasts;thegenerallumpinessofthebreastconcealsanewlump.Theotherstatementsarenottrue. DIF: Cognitive Level: Applying (Application) MSC: Client Needs: Safe and Effective Care Environment: Management of Care 20. During an annual physical examination, a 43-year-old patient states that she does not perform monthly breastself-examinations(BSEs).Shetellsthenursethatshebelievesthatmammogramsdoamuchbetterjob thanIevercouldtofindalump.Thenurseshouldexplaintoherthat: a. BSEs may detect lumps that appear betweenmammograms. b. BSEs are unnecessary until the age of 50years. c. She is correctmammography is a good replacement forBSE. d. ShedoesnotneedtoperformBSEsaslongasaphysicianchecksherbreastsannually. ANS: A NURSINGTB.COM ThemonthlypracticeofBSE,alongwithclinicalbreastexaminationandmammograms,arecomplementary screeningmeasures.Mammographycanrevealcancerstoosmalltobedetectedbythewomanorbythemost experiencedexaminer.However,intervallumpsmaybecomepalpablebetweenmammograms. DIF: Cognitive Level: Understanding (Comprehension) MSC: Client Needs: Physiologic Integrity: Reduction of Risk Potential 21. Duringaninterview,apatientrevealsthatsheispregnant.Shestatesthatsheisnotsurewhethershewill breastfeed her baby and asks for some information about this. Which of these statements by the nurse is accurate? a. Breastfed babies tend to be morecolicky. b. Breastfeeding provides the perfect food and antibodies for yourbaby. c. Breastfed babies eat more often than infants onformula. d. Breastfeeding is second nature, and every woman can doit. ANS: B Exclusivelybreastfeedingfor6monthsprovidestheperfectfoodandantibodiesforthebaby,decreasesthe risk of ear infections, promotes bonding, and providesrelaxation. DIF: Cognitive Level: Applying (Application) MSC: Client Needs: Physiologic Integrity: Reduction of Risk Potential 22. Thenurseisreviewingriskfactorsforbreastcancer.Whichofthesewomenhaveriskfactorsthatplace them at a higher risk for breastcancer? a. 37 year old who is slightlyoverweight b. 42 year old who has had ovariancancer c. 45 year old who has never beenpregnant d. 65yearoldwhosemotherhadbreastcancer ANS:D Riskfactorsforbreastcancerincludehavingafirst-degreerelativewithbreastcancer(mother,sister,or daughter) and being older than 50 years ofage. DIF: Cognitive Level: Applying (Application) MSC: Client Needs: Physiologic Integrity: Reduction of Risk Potential 23. Duringanexaminationofawoman,thenursenoticesthatherleftbreastisslightlylargerthanherright breast. Which of these statements is true about thisfinding? a. BreastsshouldalwaysbesymmetrNicU.RSINGTB.COM b. Asymmetryofbreastsizeandshapeisprobablyduetobreastfeedingandisnothingtoworryabout. c. Asymmetryisnotunusual,butthenurseshouldverifythatthischangeisnotnew. d. Asymmetryofbreastsizeandshapeisveryunusualandmeansshemayhaveaninflammationor growth. ANS: C Thenurseshouldnoticesymmetryofsizeandshape.Itiscommontohaveaslightasymmetryinsize;oftenthe leftbreastisslightlylargerthantheright.Asuddenincreaseinthesizeofonebreastsignifiesinflammationor newgrowth. DIF: Cognitive Level: Analyzing (Analysis) MSC: Client Needs: Physiologic Integrity: Physiologic Adaptation 24. ThenurseisassistingwithaBSEclinic.Whichofthesewomenreflectabnormalfindingsduringthe inspection phase of breastexamination? a. Womanwhosenipplesareindifferentplanes(deviated). b. Womanwhoseleftbreastisslightlylargerthanherright. c. Nonpregnant woman whose skin is marked with linearstriae. d. Pregnantwomanwhosebreastshaveafinebluenetworkofveinsvisibleundertheskin. ANS: A Thenipplesshouldbesymmetricallyplacedonthesameplaneonthetwobreasts.Withdeviationinpointing, an underlying cancer may cause fibrosis in the mammary ducts, which pulls the nipple angle toward it. The other examples are normalfindings. DIF: Cognitive Level: Understanding (Comprehension) MSC: Client Needs: Physiologic Integrity: Physiologic Adaptation 25. Duringthephysicalexamination,thenursenoticesthatafemalepatienthasaninvertedleftnipple.Which statement regarding this is mostaccurate? a. Normal nipple inversion is usuallybilateral. b. Unilateral inversion of a nipple is always a serioussign. c. Whether the inversion is a recent change should bedetermined. d. Nippleinversionisnotsignificantunlessaccompaniedbyanunderlyingpalpablemass. ANS: C NURSINGTB.COM Thenurseshoulddistinguishbetweenarecentlyretractednipplefromonethathasbeeninvertedformany yearsorsincepuberty.Normalnippleinversionmaybeunilateralorbilateralandusuallycanbepulledout; thatis,ifitisnotfixed.Recentnippleretractionsignifiesacquireddisease. DIF: Cognitive Level: Analyzing (Analysis) MSC: Client Needs: Physiologic Integrity: Physiologic Adaptation 26. The nurse is performing a breast examination. Which of these statements best describes the correct proceduretousewhenscreeningfornippleandskinretractionduringabreastexamination?Havethewoman: a. Bend over and touch hertoes. b. Lie down on her left side and notice anyretraction. c. Shiftfromasupinepositiontoastandingposition,andnoteanylagorretraction. d. Slowlyliftherarmsaboveherhead,andnoteanyretractionorlaginmovement. ANS: D Thewomanshouldbedirectedtochangepositionwhilecheckingthebreastsforsignsofskinretraction. Initially,sheshouldbeaskedtoliftherarmsslowlyoverherhead.Bothbreastsshouldmoveup symmetrically.Retractionsignsareduetofibrosisinthebreasttissue,usuallycausedbygrowingneoplasms. The nurse should notice whether movement of one breast islagging. DIF: Cognitive Level: Applying (Application) MSC: Client Needs: Safe and Effective Care Environment: Management of Care 27. Thenurseispalpatingafemalepatientsbreastsduringanexamination.Whichofthesepositionsismost likely to make significant lumps more distinct during breastpalpation? a. Supine with the arms raised over herhead b. Sittingwiththearmsrelaxedathersides c. Supinewiththearmsrelaxedathersides d. Sittingwiththearmsflexedandfingertipstouchinghershoulders ANS:A Thenurseshouldhelpthewomantoasupineposition,tuckasmallpadunderthesidetobepalpated,andhelp thewomanraiseherarmoverherhead.Thesemaneuverswillflattenthebreasttissueandmediallydisplaceit. Any significant lumps will then feel moredistinct. DIF: Cognitive Level: Applying (Application) MSC:ClientNeeds:SafeandEffectiveCareENnUvRirSoInNmGeTnBt:.CMOaMnagementofCare 28. Whichoftheseclinicalsituationswouldthenurseconsidertobeoutsidenormallimits? a. Apatienthashadonepregnancyandstatesthatshebelievesshemaybeenteringmenopause.Her breast examination reveals breasts that are soft and slightlysagging. b. Apatienthasneverbeenpregnant.Herbreastexaminationrevealslargependulousbreaststhat haveafirm,transverseridgealongthelowerquadrantinbothbreasts. c. A patient has never been pregnant and reports that she should begin her period tomorrow. Her breastexaminationrevealsbreasttissuethatisnodularandsomewhatengorged.Shestatesthatthe examination was slightlypainful. d. A patient has had two pregnancies, and she breastfed both of her children. Her youngest child is now10yearsold.Herbreastexaminationrevealsbreasttissuethatissomewhatsoft,andshehasa small amount of thick yellow discharge from bothnipples. ANS: D Ifanydischargeappears,thenurseshouldnoteitscolorandconsistency.Exceptinpregnancyandlactation, anydischargeisabnormal.Innulliparouswomen,normalbreasttissuefeelsfirm,smooth,andelastic;after pregnancy, the tissue feels soft and loose. Premenstrual engorgement is normal, and consists of a slight enlargement,tendernesstopalpation,andageneralizednodularity.Afirm,transverseridgeofcompressed tissueinthelowerquadrants,knownastheinframammaryridge,isespeciallynoticeableinlargebreasts. DIF: Cognitive Level: Analyzing (Analysis) MSC: Client Needs: Safe and Effective Care Environment: Management of Care 29. Apatientstatesduringtheinterviewthatshenoticedanewlumpintheshowerafewdaysago.Itwason her left breast near her axilla. The nurse should planto: a. Palpate the lumpfirst. b. Palpate the unaffected breastfirst. c. Avoidpalpatingthelumpbecauseitcouldbeacyst,whichmightrupture. d. Palpatethebreastwiththelumpfirstbutplantopalpatetheaxillalast. ANS: B Ifthewomanmentionsabreastlumpshehasdiscoveredherself,thenthenurseshouldexaminetheunaffected breast first to learn a baseline of normal consistency for thisindividual. DIF: Cognitive Level: Applying (Application) MSC: Client Needs: Safe and Effective Care Environment: Management of Care 30. Thenursehaspalpatedalumpinafemalepatientsrightbreast.Thenursedocumentsthisasasmall,round, firm, distinct, lump located at 2 oclock, 2 cm from the nipple. It is nontender and fixed. No associated retractionoftheskinornipple,noerythema,aNnUdRnSoIaNxGilTlaBr.yClOyMmphadenopathyareobserved.Whatinformation is missing from thedocumentation? a. Shape of thelump b. Consistency of thelump c. Size of thelump d. Whetherthelumpissolitaryormultiple ANS:C Ifthenursefeelsalumpormass,thenheorsheshouldnotethesecharacteristics:(1)location,(2)sizejudgein centimeters in three dimensions: width length thickness, (3) shape, (4) consistency, (5) motility, (6) distinctness,(7)nipple,(8)theskinoverthelump,(9)tenderness,and(10)lymphadenopathy. DIF: Cognitive Level: Applying (Application) MSC: Client Needs: Safe and Effective Care Environment: Management of Care 31. ThenurseisconductingaclassonBSE.WhichofthesestatementsindicatestheproperBSEtechnique? a. ThebesttimetoperformBSEisinthemiddleofthemenstrualcycle. b. ThewomanneedstoperformBSEonlybimonthlyunlessshehasfibrocysticbreasttissue. c. ThebesttimetoperformaBSEis4to7daysafterthefirstdayofthemenstrualperiod. d. Ifshesuspectsthatsheispregnant,thenthewomanshouldnotperformaBSEuntilherbabyis born. ANS: C Thenurseshouldhelpeachwomanestablisharegularscheduleofself-care.ThebesttimetoconductaBSEis rightafterthemenstrualperiod,orthefourththroughseventhdayofthemenstrualcycle,whenthebreastsare the smallest and least congested. The pregnant or menopausal woman who is not having menstrual periods shouldbeadvisedtoselectafamiliardatetoexamineherbreastseachmonthforexample,herbirthdateorthe day the rent isdue. DIF: Cognitive Level: Applying (Application) MSC: Client Needs: Physiologic Integrity: Reduction of Risk Potential 32. ThenurseispreparingtoteachawomanaboutBSE.Whichstatementbythenurseiscorrect? a. BSEismoreimportantthaneverforyoubecauseyouhaveneverhadanychildren. b. BSEissoimportantbecauseoneoutofninewomenwilldevelopbreastcancerinherlifetime. c. BSEonamonthlybasiswillhelpyNoUuRbSeIcNoGmTeBf.aCmOiMliarwithyourownbreastsandfeeltheirnormal variations. d. BSEwillsaveyourlifebecauseyouarelikelytofindacancerouslumpbetweenmammograms. ANS: C ThenurseshouldstressthataregularmonthlyBSEwillfamiliarizethewomanwithherownbreastsandtheir normalvariations.BSEisapositivestepthatwillreassureherofherhealthystate.Whileteaching,thenurse should focus on the positive aspects of BSE and avoid citing frightening mortality statistics about breast cancer,whichmaygenerateexcessivefearanddenialthatcanobstructawomansself-careactions. DIF: Cognitive Level: Applying (Application) MSC: Client Needs: Physiologic Integrity: Reduction of Risk Potential 33. A 55-year-old postmenopausal woman is being seen in the clinic for her annual examination. She is concernedaboutchangesinherbreaststhatshehasnoticedoverthepast5years.Shestatesthatherbreasts havedecreasedinsizeandthattheelasticityhaschangedsothatherbreastsseemflatandflabby.Thenurses best reply wouldbe: a. Thischangeoccursmostoftenbecauseoflong-termuseofbrasthatdonotprovideenoughsupport to the breasttissues. b. Thisisanormalchangethatoccursaswomengetolderandisduetotheincreasedlevelsof progesterone during the agingprocess. c. Decreasesinhormonesaftermenopausecausesatrophyoftheglandulartissueinthebreastandisa normal process ofaging. d. Posturalchangesinthespinemakeitappearthatyourbreastshavechangedinshape.Exercisesto strengthenthemusclesoftheupperbackandchestwallwillhelppreventthechangesinelasticity andsize. ANS: C Thehormonalchangesofmenopausecausethebreastglandulartissuetoatrophy,makingthebreastsmore pendulous, flattened, andsagging. DIF: Cognitive Level: Applying (Application) MSC: Client Needs: Health Promotion and Maintenance 34. A43-year-oldwomanisattheclinicforaroutineexamination.Shereportsthatshehashadabreastlump in her right breast for years. Recently, it has begun to change in consistency and is becoming harder. She reportsthat5yearsagoherphysicianevaluatedthelumpanddeterminedthatitwasnothingtoworryabout. Theexaminationvalidatesthepresenceofamassintherightupperouterquadrantat1oclock,approximately 5 cm from the nipple. It is firm, mobile, and nontender, with borders that are not well defined. The nurse replies: a. Becauseofthechangeinconsistencyofthelump,itshouldbefurtherevaluatedbyaphysician. b. Thechangescouldberelatedtoyourmenstrualcycles.Keeptrackofthechangesinthemasseach month. NURSINGTB.COM c. Thelumpisprobablynothingtoworryaboutbecauseithasbeenpresentforyearsandwas determined to be noncancerous 5 yearsago. d. Becauseyouareexperiencingnopainandthesizehasnotchanged,youshouldcontinueto monitor the lump and return to the clinic in 3months. ANS: A Alumpthathasbeenpresentforyearsandisnotexhibitingchangesmaynotbeseriousbutshouldstillbe explored.Anyrecentchangeoranewlumpshouldbeevaluated.Theotherresponsesarenotcorrect. DIF: Cognitive Level: Analyzing (Analysis) MSC: Client Needs: Safe and Effective Care Environment: Management of Care 35. DuringadiscussionaboutBSEswitha30-year-oldwoman,whichofthesestatementsbythenurseismost appropriate? a. The best time to examine your breasts is duringovulation. b. Examine your breasts every month on the same day of themonth. c. Examine your breasts shortly after your menstrual period eachmonth. d. Thebesttimetoexamineyourbreastsisimmediatelybeforemenstruation. ANS: C ThebesttimetoconductaBSEisshortlyafterthemenstrualperiodwhenthebreastsarethesmallestandleast congested. DIF: Cognitive Level: Applying (Application) MSC: Client Needs: Safe and Effective Care Environment: Management of Care 36. ThenurseisdiscussingBSEswithapostmenopausalwoman.Thebesttimeforpostmenopausalwomento perform BSEsis: a. On the same day everymonth. b. Daily, during the shower orbath. c. One week after her menstrualperiod. d. Every year with her annual gynecologicexamination. ANS: A Postmenopausalwomenarenolongerexperiencingregularmenstrualcyclesbutneedtocontinuetoperform BSEsonamonthlybasis.ChoosingthesameNdaUyRoSfINthGeTmBo.CntOhMisahelpfulremindertoperformthe examination. DIF: Cognitive Level: Understanding (Comprehension) MSC: Client Needs: Safe and Effective Care Environment: Management of Care 37. Whileinspectingapatientsbreasts,thenursefindsthattheleftbreastisslightlylargerthantherightwith the bilateral presence of Montgomery glands. The nurseshould: a. Palpate over the Montgomery glands, checking fordrainage. b. Consider these findings as normal, and proceed with theexamination. c. Askextensivehealthhistoryquestionsregardingthewomansbreastasymmetry. d. Continuewiththeexamination,andthenreferthepatientforfurtherevaluationoftheMontgomery glands. ANS: B Normalfindingsofthebreastincludeonebreast(mostoftentheleft)slightlylargerthantheotherandthe presence of Montgomery glands across theareola. DIF: Cognitive Level: Analyzing (Analysis) MSC: Client Needs: Safe and Effective Care Environment: Management of Care 38. Duringanexamination,thenursenotesasupernumerarynipplejustunderthepatientsleftbreast.The patienttellsthenursethatshealwaysthoughtitwasamole.Whichstatementaboutthisfindingiscorrect? a. This variation is normal and not a significantfinding. b. This finding is significant and needs furtherinvestigation. c. Asupernumerarynipplealsocontainsglandulartissueandmayleakmilkduringpregnancyand lactation. d. Thepatientiscorrectasupernumerarynippleisactuallyamolethathappenstobelocatedunderthe breast. ANS: A Asupernumerarynipplelookslikeamole,butcloseexaminationrevealsatinynippleandareola;itisnota significantfinding. DIF: Cognitive Level: Analyzing (Analysis) MSC: Client Needs: Safe and Effective Care Environment: Management of Care 39. Whileexamininga75-year-oldwoman,thenursenoticesthattheskinoverherrightbreastisthickened and the hair follicles are exaggerated. This condition is knownas: NURSINGTB.COM a. Dimpling. b. Retraction. c. Peaudorange. d. Benign breastdisease. ANS: C Thisconditionisknownaspeaudorange.Lymphaticobstructionproducesedema,whichthickenstheskinand exaggerates the hair follicles. The skin has a pig-skin or orange-peel appearance, and this condition suggests cancer. DIF: Cognitive Level: Understanding (Comprehension) MSC: Client Needs: Physiologic Integrity: Physiologic Adaptation 40. Whenabreastfeedingmotherisdiagnosedwithabreastabscess,whichoftheseinstructionsfromthenurse is correct? The mother needsto: a. Continue to nurse on both sides to encourage milkflow. b. Immediately discontinue nursing to allow forhealing. c. Temporarilydiscontinuenursingontheaffectedbreast,andmanuallyexpressmilkanddiscardit. d. Temporarilydiscontinuenursingonaffectedbreast,butmanuallyexpressmilkandgiveittothe baby. ANS: C Withabreastabscess,thepatientmusttemporarilydiscontinuenursingontheaffectedbreast,manually expressthemilk,andthendiscardit.Nursingcancontinueontheunaffectedside. DIF: Cognitive Level: Applying (Application) MSC: Client Needs: Physiologic Integrity: Physiologic Adaptation 41. Anewmothercallstheclinictoreportthatpartofherleftbreastisred,swollen,tender,veryhot,andhard. Shehasafeverof38.3C.Shealsohashadsymptomsofinfluenza,suchaschills,sweating,andfeelingtired. Thenursenoticesthatshehasbeenbreastfeedingfor1month.Fromherdescription,whatconditiondoesthe nursesuspect? a. Mastitis b. Pagetdisease c. Plugged milkduct d. Mammary ductectasia NURSINGTB.COM ANS: A Thesymptomsdescribemastitis,whichstemsfromaninfectionorstasiscausedbyapluggedduct.Aplugged duct does not have infectionpresent. DIF: Cognitive Level: Understanding (Comprehension) MSC: Client Needs: Physiologic Integrity: Physiologic Adaptation 42. Duringabreastexaminationonafemalepatient,thenursenoticesthatthenippleisflat,broad,andfixed. Thepatientstatesitstarteddoingthatafewmonthsago.Thisfindingsuggests: a. Dimpling. b. Retractednipple. c. Nippleinversion. d. Deviation in nipplepointing. ANS: B The retracted nipple looks flatter and broader, similar to an underlying crater. A recent retraction suggests cancer, which causes fibrosis of the whole duct system and pulls in the nipple. It also may occur with benign lesionssuchasectasiaoftheducts.Thenurseshouldnotconfuseretractionwiththenormallong-standingtype of nipple inversion, which has no broadening and is notfixed. DIF: Cognitive Level: Analyzing (Analysis) MSC: Client Needs: Physiologic Integrity: Physiologic Adaptation 43. A 54-year-old man comes to the clinic with a horrible problem. He tells the nurse that he has just discoveredalumponhisbreastandisfearfulofcancer.Thenurseknowswhichstatementaboutbreastcancer in men istrue? a. Breastmassesinmenaredifficulttodetectbecauseofminimalbreasttissue. b. Breast cancer in men rarely spreads to the lymphnodes. c. One percent of all breast cancers occurs inmen. d. Most breast masses in men are diagnosed asgynecomastia. ANS: C Onepercentofallbreastcancersoccursinmen.Theearlyspreadingtoaxillarylymphnodesisattributableto minimal breasttissue. DIF: Cognitive Level: Applying (Application)NURSINGTB.COM MSC: Client Needs: Health Promotion and Maintenance MULTIPLE RESPONSE 1. Thenurseisassessingthebreastsofa68-year-oldwomananddiscoversamassintheupperouterquadrant oftheleftbreast.Whenassessingthismass,thenurseisawarethatcharacteristicsofacancerousmassinclude which of the following? Select all thatapply. a. Nontendermass b. Dull, heavy pain onpalpation c. Rubbery texture andmobile d. Hard, dense, andimmobile e. Regularborder f. Irregular,poorlydelineatedborder ANS: A, D,F Cancerous breast masses are solitary, unilateral, and nontender. They are solid, hard, dense, and fixed to underlyingtissuesorskinascancerbecomesinvasive.Theirbordersareirregularandpoorlydelineated.They are often painless, although the person may experience pain. They are most common in the upper outer quadrant. A dull, heavy pain on palpation and a mass with a rubbery texture and a regular border are characteristics of benign breastdisease. DIF: Cognitive Level: Analyzing (Analysis) MSC: Client Needs: Physiologic Integrity: Physiologic Adaptation 2. Thenurseisexamininga62-year-oldmanandnotesthathehasbilateralgynecomastia.Thenurseshould explorehishealthhistoryforwhichrelatedconditions?Selectallthatapply. a. Malnutrition b. Hyperthyroidism c. Type 2 diabetesmellitus d. Liverdisease e. Historyofalcoholabuse ANS: B, D,E GynecomastiaoccurswithCushingsyndrome,livercirrhosis,adrenaldisease,hyperthyroidism,andnumerous drugs,suchasalcoholandmarijuanause,estrNogUeRnStIrNeaGtmTBen.CtOfoMrprostatecancer,antibiotics(metronidazole, isoniazid),digoxin,angiotensin-convertingenzyme(ACE)inhibitors,diazepam,andtricyclicantidepressants. DIF: Cognitive Level: Applying (Application) MSC: Client Needs: Physiologic Integrity: Physiologic Adaptation Chapter 19: Thorax and Lungs MULTIPLE CHOICE 1. Whichofthesestatementsistrueregardingthevertebraprominens?Thevertebraprominensis: a. The spinous process ofC7. b. Usually nonpalpable in mostindividuals. c. Opposite the interior border of thescapula. d. Located next to the manubrium of thesternum. ANS: A The spinous process of C7 is the vertebra prominens and is the most prominent bony spur protruding at the baseoftheneck.Countingribsandintercostalspacesontheposteriorthoraxisdifficultbecauseofthemuscles andsofttissue.Thevertebraprominensiseasiertoidentifyandisusedasastartingpointincountingthoracic processes and identifying landmarks on the posteriorchest. DIF:CognitiveLevel:Remembering(Knowledge) MSC: Client Needs:General 2. WhenperformingarespiratoryassessmentNonUaRSpIaNtiGenTtB,t.hCeOnMursenoticesacostalangleofapproximately90 degrees. This characteristicis: a. Observed in patients withkyphosis. b. Indicative of pectusexcavatum. c. A normal finding in a healthyadult. d. An expected finding in a patient with a barrelchest. ANS: C Therightandleftcostalmarginsformananglewheretheymeetatthexiphoidprocess.Usually,thisangleis 90degreesorless.Theangleincreaseswhentheribcageischronicallyoverinflated,asinemphysema. DIF: Cognitive Level: Understanding (Comprehension) MSC: Client Needs: Safe and Effective Care Environment: Management of Care 3. When assessing a patients lungs, the nurse recalls that the leftlung: a. Consists of twolobes. b. Is divided by the horizontalfissure. c. Primarily consists of an upper lobe on the posteriorchest. d. Is shorter than the right lung because of the underlyingstomach. ANS: A The left lung has two lobes, and the right lung has three lobes. The right lung is shorter than the left lung becauseoftheunderlyingliver.Theleftlungisnarrowerthantherightlungbecausetheheartbulgestothe left. The posterior chest is almost all lowerlobes. DIF:CognitiveLevel:Remembering(Knowledge) MSC: Client Needs:General 4. Whichstatementabouttheapicesofthelungsistrue?Theapicesofthelungs: a. Are at the level of the second ribanteriorly. b. Extend 3 to 4 cm above the inner third of theclavicles. c. Are located at the sixth rib anteriorly and the eighth riblaterally. d. Restonthediaphragmatthefifthintercostalspaceinthemidclavicularline(MCL). ANS: B NURSINGTB.COM Theapexofthelungontheanteriorchestis3to4cmabovetheinnerthirdoftheclavicles.Ontheposterior chest, the apices are at the level ofC7. DIF:CognitiveLevel:Remembering(Knowledge) MSC: Client Needs:General 5. Duringanexaminationoftheanteriorthorax,thenurseisawarethatthetracheabifurcatesanteriorlyatthe: a. Costalangle. b. Sternalangle. c. Xiphoidprocess. d. Suprasternalnotch. ANS: B Thesternalanglemarksthesiteoftrachealbifurcationintotherightandleftmainbronchi;itcorrespondswith theupperbordersoftheatriaoftheheart,anditliesabovethefourththoracicvertebraontheback. DIF: Cognitive Level: Remembering (Knowledge) MSC: Client Needs: Safe and Effective Care Environment: Management of Care 6. Duringanassessment,thenurseknowsthatexpectedassessmentfindingsinthenormaladultlunginclude the presenceof: a. Adventitious sounds and limited chestexpansion. b. Increased tactile fremitus and dull percussiontones. c. Muffled voice sounds and symmetric tactilefremitus. d. Absent voice sounds and hyperresonant percussiontones. ANS: C Normallungfindingsincludesymmetricchestexpansion,resonantpercussiontones,vesicularbreathsounds overtheperipherallungfields,muffledvoicesounds,andnoadventitioussounds. DIF: Cognitive Level: Understanding (Comprehension) MSC: Client Needs: Safe and Effective Care Environment: Management of Care 7. The primary muscles of respiration includethe: a. Diaphragmandintercostals. b. Sternomastoidsandscaleni. c. Trapeziiandrectusabdominis. NURSINGTB.COM d. External obliques and pectoralismajor. ANS: A Themajormuscleofrespirationisthediaphragm.Theintercostalmusclesliftthesternumandelevatetheribs duringinspiration,increasingtheanteroposteriordiameter.Expirationisprimarilypassive.Forcedinspiration involves the use of other muscles, such as the accessory neck musclessternomastoid, scaleni, and trapezii muscles. Forced expiration involves the abdominalmuscles. DIF:CognitiveLevel:Remembering(Knowledge) MSC: Client Needs:General 8. A65-year-oldpatientwithahistoryofheartfailurecomestotheclinicwithcomplaintsofbeingawakened fromsleepwithshortnessofbreath.Whichactionbythenurseismostappropriate? a. Obtainingadetailedhealthhistoryofthepatientsallergiesandahistoryofasthma b. Tellingthepatienttosleeponhisorherrightsidetofacilitateeaseofrespirations c. Assessing for other signs and symptoms of paroxysmal nocturnaldyspnea d. Assuringthepatientthatparoxysmalnocturnaldyspneaisnormalandwillprobablyresolvewithin the nextweek ANS: C Thepatientisexperiencingparoxysmalnocturnaldyspneabeingawakenedfromsleepwithshortnessofbreath and the need to be upright to achievecomfort. DIF:CognitiveLevel:Applying(Application) MSC: Client Needs: Safe and Effective Care Environment: Management of Care 9. Whenassessingtactilefremitus,thenurserecallsthatitisnormaltofeeltactilefremitusmostintenselyover whichlocation? a. Between thescapulae b. Third intercostal space,MCL c. Fifth intercostal space, midaxillary line(MAL) d. Over the lower lobes, posteriorside ANS: A NURSINGTB.COM Normally,fremitusismostprominentbetweenthescapulaeandaroundthesternum.Thesesitesarewherethe major bronchi are closest to the chest wall. Fremitus normally decreases as one progresses down the chest because more tissue impedes soundtransmission. DIF: Cognitive Level: Understanding (Comprehension) MSC: Client Needs: Safe and Effective Care Environment: Management of Care 10. Thenurseisreviewingthetechniqueofpalpatingfortactilefremituswithanewgraduate.Whichstatement bythegraduatenursereflectsacorrectunderstandingoftactilefremitus?Tactilefremitus: a. Is caused by moisture in thealveoli. b. Indicates that air is present in the subcutaneoustissues. c. Is caused by sounds generated from thelarynx. d. Reflects the blood flow through the pulmonaryarteries. ANS: C Fremitusisapalpablevibration.Soundsgeneratedfromthelarynxaretransmittedthroughpatentbronchiand thelungparenchymatothechestwallwheretheyarefeltasvibrations.Crepitusisthetermforairinthe subcutaneous tissues. DIF: Cognitive Level: Understanding (Comprehension) MSC: Client Needs: Safe and Effective Care Environment: Management of Care 11. Duringpercussion,thenurseknowsthatadullpercussionnoteelicitedoveralunglobemostlikelyresults from: a. Shallowbreathing. b. Normal lungtissue. c. Decreased adiposetissue. d. Increased density of lungtissue. ANS: D Adullpercussionnoteindicatesanabnormaldensityinthelungs,aswithpneumonia,pleuraleffusion, atelectasis,oratumor.Resonanceistheexpectedfindinginnormallungtissue. DIF:CognitiveLevel:Understanding(Comprehension) MSC: Client Needs:General 12. ThenurseisobservingtheauscultationtecNhUniRqSuIeNoGfTaBno.CthOeMrnurse.Thecorrectmethodtousewhen progressingfromoneauscultatorysiteonthethoraxtoanotheris comparison. a. Side-to-side b. Top-to-bottom c. Posterior-to-anterior d. Interspace-by-interspace ANS:A Side-to-sidecomparisonismostimportantwhenauscultatingthechest.Thenurseshouldlistentoatleastone full respiration in each location. The other techniques are notcorrect. DIF: Cognitive Level: Understanding (Comprehension) MSC: Client Needs: Safe and Effective Care Environment: Management of Care 13. Whenauscultatingthelungsofanadultpatient,thenursenotesthatlow-pitched,softbreathsoundsare heardovertheposteriorlowerlobes,withinspirationbeinglongerthanexpiration.Thenurseinterpretsthat these soundsare: a. Normally auscultated over thetrachea. b. Bronchialbreathsoundsandnormalinthatlocation. c. Vesicularbreathsoundsandnormalinthatlocation. d. Bronchovesicular breath sounds and normal in thatlocation. ANS: C Vesicularbreathsoundsarelow-pitched,softsoundswithinspirationbeinglongerthanexpiration.These breathsoundsareexpectedovertheperipherallungfieldswhereairflowsthroughsmallerbronchiolesand alveoli. DIF: Cognitive Level: Applying (Application) MSC: Client Needs: Safe and Effective Care Environment: Management of Care 14. Thenurseisauscultatingthechestinanadult.Whichtechniqueiscorrect? a. Instructing the patient to take deep, rapidbreaths b. Instructingthepatienttobreatheinandoutthroughhisorhernose c. Firmly holding the diaphragm of the stethoscope against thechest d. LightlyholdingthebellofthestetNhoUsRcoSpINeGagTaBin.CstOtMhechesttoavoidfriction ANS:C Firmly holding the diaphragm of the stethoscope against the chest is the correct way to auscultate breath sounds.Thepatientshouldbeinstructedtobreathethroughhisorhermouth,alittledeeperthanusual,butnot tohyperventilate. DIF: Cognitive Level: Understanding (Comprehension) MSC: Client Needs: Safe and Effective Care Environment: Management of Care 15. Thenurseispercussingoverthelungsofapatientwithpneumonia.Thenurseknowsthatpercussionover an area of atelectasis in the lungs willreveal: a. Dullness. b. Tympany. c. Resonance. d. Hyperresonance. ANS: A A dull percussion note signals an abnormal density in the lungs, as with pneumonia, pleural effusion, atelectasis, or a tumor. DIF: Cognitive Level: Understanding (Comprehension) MSC: Client Needs: Physiologic Integrity: Physiologic Adaptation 16. Duringauscultationofthelungs,thenurseexpectsdecreasedbreathsoundstobeheardinwhichsituation? a. When the bronchial tree isobstructed b. When adventitious sounds arepresent c. In conjunction with whisperedpectoriloquy d. Inconditionsofconsolidation,suchaspneumonia ANS:A Decreasedorabsentbreathsoundsoccurwhenthebronchialtreeisobstructed,asinemphysema,andwhen soundtransmissionisobstructed,asinpleurisy,pneumothorax,orpleuraleffusion. DIF: Cognitive Level: Understanding (Comprehension) MSC: Client Needs: Physiologic Integrity: Physiologic Adaptation 17. Thenurseknowsthatanormalfindingwhenassessingtherespiratorysystemofanolderadultis: NURSINGTB.COM a. Increased thoracicexpansion. b. Decreased mobility of thethorax. c. Decreased anteroposteriordiameter. d. Bronchovesicular breath sounds throughout thelungs. ANS: B Thecostalcartilagesbecomecalcifiedwithaging,resultinginalessmobilethorax.Chestexpansionmaybe somewhatdecreased,andthechestcagecommonlyshowsanincreasedanteroposteriordiameter. DIF:CognitiveLevel:Understanding(Comprehension) MSC:ClientNeeds:HealthPromotionandMaintenance 18. Amotherbringsher3-month-oldinfanttotheclinicforevaluationofacold.Shetellsthenursethathehas hadarunnynoseforaweek.Whenperformingthephysicalassessment,thenursenotesthatthechildhasnasal flaringandsternalandintercostalretractions.Thenursesnextactionshouldbeto: a. Assure the mother that these signs are normal symptoms of acold. b. Recognize that these are serious signs, and contact thephysician. c. Ask the mother if the infant has had trouble withfeedings. d. Performacompletecardiacassessmentbecausethesesignsareprobablyindicativeofearlyheart failure. ANS: B Theinfantisanobligatorynosebreatheruntiltheageof3months.Normally,noflaringofthenostrilsandno sternalorintercostalretractionoccurs.Significantretractionsofthesternumandintercostalmusclesandnasal flaringindicateincreasedinspiratoryeffort,asinpneumonia,acuteairwayobstruction,asthma,andatelectasis; therefore, immediate referral to the physician is warranted. These signs do not indicate heart failure, and an assessment of the infants feeding is not a priority at thistime. DIF: Cognitive Level: Analyzing (Analysis) MSC: Client Needs: Physiologic Integrity: Physiologic Adaptation 19. Whenassessingtherespiratorysystemofa4-year-oldchild,whichofthesefindingswouldthenurse expect? a. Crepitus palpated at the costochondraljunctions b. Nodiaphragmaticexcursionasaresultofachildsdecreasedinspiratoryvolume c. PresenceofbronchovesicularbreaNthUsRoSuInNdGsTinBt.hCeOpMeripherallungfields d. Irregularrespiratorypatternandarespiratoryrateof40breathsperminuteatrest ANS:C Bronchovesicular breath sounds in the peripheral lung fields of the infant and young child up to age 5 or 6 yearsarenormalfindings.Theirthinchestwallswithunderdevelopedmusculaturedonotdampenthesound, asdothethickerchestwallsofadults;therefore,breathsoundsareloudandharsh. DIF: Cognitive Level: Applying (Application) MSC: Client Needs: Safe and Effective Care Environment: Management of Care 20. Wheninspectingtheanteriorchestofanadult,thenurseshouldincludewhichassessment? a. Diaphragmaticexcursion b. Symmetric chestexpansion c. Presence of breathsounds d. Shapeandconfigurationofthechestwall ANS:D Inspectionoftheanteriorchestincludesshapeandconfigurationofthechestwall;assessmentofthepatients levelofconsciousnessandthepatientsskincolorandcondition;qualityofrespirations;presenceorabsenceof retraction and bulging of the intercostal spaces; and use of accessory muscles. Symmetric chest expansion is assessedbypalpation.Diaphragmaticexcursionisassessedbypercussionoftheposteriorchest.Breathsounds are assessed byauscultation. DIF: Cognitive Level: Understanding (Comprehension) MSC: Client Needs: Safe and Effective Care Environment: Management of Care 21. Thenurseknowsthatauscultationoffinecrackleswouldmostlikelybenoticedin: a. A healthy 5-year-oldchild. b. A pregnantwoman. c. The immediate newbornperiod. d. Association with apneumothorax. ANS: C Finecracklesarecommonlyheardintheimmediatenewbornperiodasaresultoftheopeningoftheairways andaclearingoffluid.Persistentfinecrackleswouldbenoticedwithpneumonia,bronchiolitis,oratelectasis. DIF: Cognitive Level: Applying (Application) NURSINGTB.COM MSC: Client Needs: Safe and Effective Care Environment: Management of Care 22. Duringanassessmentofanadult,thenursehasnotedunequalchestexpansionandrecognizesthatthis occurs in whichsituation? a. In an obesepatient b. When part of the lung is obstructed orcollapsed c. Whenbulgingoftheintercostalspacesispresent d. Whenaccessorymusclesareusedtoaugmentrespiratoryeffort ANS:B Unequalchestexpansionoccurswhenpartofthelungisobstructedorcollapsed,aswithpneumonia,orwhen guarding to avoid postoperative incisionalpain. DIF: Cognitive Level: Understanding (Comprehension) MSC: Client Needs: Physiologic Integrity: Physiologic Adaptation 23. Duringauscultationofthelungsofanadultpatient,thenursenoticesthepresenceofbronchophony.The nurse should assess for signs of whichcondition? a. Airwayobstruction b. Emphysema c. Pulmonaryconsolidation d. Asthma ANS:C Pathologicconditionsthatincreaselungdensity,suchaspulmonaryconsolidation,willenhancethe transmission of voice sounds, such asbronchophony. DIF: Cognitive Level: Understanding (Comprehension) MSC: Client Needs: Physiologic Integrity: Physiologic Adaptation 24. Thenurseisreviewingthecharacteristicsofbreathsounds.Whichstatementaboutbronchovesicularbreath sounds is true? Bronchovesicular breath soundsare: a. Musical inquality. b. Usually caused by a pathologicdisease. c. Expected near the major airways.NURSINGTB.COM d. Similar to bronchial sounds except shorter induration. ANS: C Bronchovesicular breath sounds are heard over major bronchi where fewer alveoli are located posteriorlybetweenthescapulae,especiallyontheright;andanteriorly,aroundtheuppersternuminthefirst and second intercostal spaces. The other responses are notcorrect. DIF: Cognitive Level: Understanding (Comprehension) MSC: Client Needs: Safe and Effective Care Environment: Management of Care 25. Thenurseislisteningtothebreathsoundsofapatientwithsevereasthma.Airpassingthroughnarrowed bronchioles would produce which of these adventitioussounds? a. Wheezes b. Bronchialsounds c. Bronchophony d. Whisperedpectoriloquy ANS: A Wheezesarecausedbyairsqueezedorcompressedthroughpassagewaysnarrowedalmosttoclosureby collapsing,swelling,secretions,ortumors,suchaswithacuteasthmaorchronicemphysema. DIF: Cognitive Level: Understanding (Comprehension) MSC: Client Needs: Physiologic Integrity: Physiologic Adaptation 26. Apatienthasalonghistoryofchronicobstructivepulmonarydisease(COPD).Duringtheassessment,the nurse will most likely observe which ofthese? a. Unequalchestexpansion b. Increasedtactilefremitus c. Atrophied neck and trapeziusmuscles d. Anteroposterior-to-transversediameterratioof1:1 ANS:D An anteroposterior-to-transverse diameter ratio of 1:1 or barrel chest is observed in individuals with COPD because of hyperinflation of the lungs. The ribs are more horizontal, and the chest appears as if held in continuous inspiration. Neck muscles are hypertrophied from aiding in forced respiration. Chest expansion maybedecreasedbutsymmetric.Decreasedtactilefremitusoccursfromdecreasedtransmissionofvibrations. NURSINGTB.COM DIF: Cognitive Level: Applying (Application) MSC: Client Needs: Physiologic Integrity: Physiologic Adaptation 27. A teenage patient comes to the emergency department with complaints of an inability to breathe and a sharppainintheleftsideofhischest.Theassessmentfindingsincludecyanosis,tachypnea,trachealdeviation totheright,decreasedtactilefremitusontheleft,hyperresonanceontheleft,anddecreasedbreathsoundson theleft.Thenurseinterpretsthattheseassessmentfindingsareconsistentwith: a. Bronchitis. b. Pneumothorax. c. Acutepneumonia. d. Asthmaticattack. ANS: B With a pneumothorax, free air in the pleural space causes partial or complete lung collapse. If the pneumothoraxislarge,thentachypneaandcyanosisareevident.Unequalchestexpansion,decreasedorabsent tactilefremitus,trachealdeviationtotheunaffectedside,decreasedchestexpansion,hyperresonantpercussion tones,anddecreasedorabsentbreathsoundsarefoundwiththepresenceofpneumothorax. DIF: Cognitive Level: Analyzing (Analysis) MSC: Client Needs: Physiologic Integrity: Physiologic Adaptation 28. Anadultpatientwithahistoryofallergiescomestothecliniccomplainingofwheezinganddifficultyin breathingwhenworkinginhisyard.Theassessmentfindingsincludetachypnea,theuseofaccessoryneck muscles,prolongedexpiration,intercostalretractions,decreasedbreathsounds,andexpiratorywheezes.The nurse interprets that these assessment findings are consistentwith: a. Asthma. b. Atelectasis. c. Lobarpneumonia. d. Heartfailure. ANS: A Asthmaisallergichypersensitivitytocertaininhaledparticlesthatproducesinflammationandareactionof bronchospasm,whichincreasesairwayresistance,especiallyduringexpiration.Anincreasedrespiratoryrate, theuseofaccessorymuscles,aretractionoftheintercostalmuscles,prolongedexpiration,decreasedbreath sounds, and expiratory wheezing are all characteristics ofasthma. DIF: Cognitive Level: Analyzing (Analysis) MSC: Client Needs: Physiologic Integrity: Physiologic Adaptation 29. ThenurseisassessingthelungsofanoldeNrUadRuSlItN.WGThBic.hCOofMthesechangesarenormalintherespiratory system of the olderadult? a. Severedyspneaisexperiencedonexertion,resultingfromchangesinthelungs. b. Respiratorymusclestrengthincreasestocompensateforadecreasedvitalcapacity. c. Decreaseinsmallairwayclosureoccurs,leadingtoproblemswithatelectasis. d. Lungsarelesselasticanddistensible,whichdecreasestheirabilitytocollapseandrecoil. ANS: D Intheagingadult,thelungsarelesselasticanddistensible,whichdecreasestheirabilitytocollapseandrecoil. Vital capacity is decreased, and a loss of intra-alveolar septa occurs, causing less surface area for gas exchange. The lung bases become less ventilated, and the older person is at risk for dyspnea with exertion beyond his or her usualworkload. DIF:CognitiveLevel:Understanding(Comprehension) MSC:ClientNeeds:HealthPromotionandMaintenance 30. Awomaninher26thweekofpregnancystatesthatsheisnotreallyshortofbreathbutfeelsthatsheis awareofherbreathingandtheneedtobreathe.Whatisthenursesbestreply? a. Thediaphragmbecomesfixedduringpregnancy,makingitdifficulttotakeinadeepbreath. b. Theincreaseinestrogenlevelsduringpregnancyoftencausesadecreaseinthediameteroftherib cage and makes it difficult tobreathe. c. Whatyouareexperiencingisnormal.Somewomenmayinterpretthisasshortnessofbreath,butit is a normal finding and nothing iswrong. d. This increased awareness of the need to breathe is normal as the fetus grows because of the increasedoxygendemandonthemothersbody,whichresultsinanincreasedrespiratoryrate. ANS: C Duringpregnancy,thewomanmaydevelopanincreasedawarenessoftheneedtobreathe.Somewomenmay interpret this as dyspnea, although structurally nothing is wrong. Increases in estrogen relax the chest cage ligaments, causing an increase in the transverse diameter. Although the growing fetus increases the oxygen demand on the mothers body, this increased demand is easily met by the increasing tidal volume (deeper breathing). Little change occurs in the respiratoryrate. DIF: Cognitive Level: Applying (Application) MSC: Client Needs: Health Promotion and Maintenance 31. A35-year-oldrecentimmigrantisbeingseenintheclinicforcomplaintsofacoughthatisassociatedwith rust-colored sputum, low-grade afternoon fevers, and night sweats for the past 2 months. The nurses preliminaryanalysis,basedonthishistory,isthatthispatientmaybesufferingfrom: NURSINGTB.COM a. Bronchitis. b. Pneumonia. c. Tuberculosis. d. Pulmonaryedema. ANS: C Sputumisnotdiagnosticalone,butsomeconditionshavecharacteristicsputumproduction.Tuberculosisoften producesrust-coloredsputuminadditiontoothersymptomsofnightsweatsandlow-gradeafternoonfevers. DIF: Cognitive Level: Analyzing (Analysis) MSC: Client Needs: Physiologic Integrity: Physiologic Adaptation 32. A70-year-oldpatientisbeingseenintheclinicforsevereexacerbationofhisheartfailure.Whichofthese findings is the nurse most likely to observe in thispatient? a. Shortnessofbreath,orthopnea,paroxysmalnocturnaldyspnea,andankleedema b. Rasping cough, thick mucoid sputum, wheezing, andbronchitis c. Productive cough, dyspnea, weight loss, anorexia, andtuberculosis d. Fever,drynonproductivecough,anddiminishedbreathsounds ANS:A Apersonwithheartfailureoftenexhibitsincreasedrespiratoryrate,shortnessofbreathonexertion,orthopnea, paroxysmal nocturnal dyspnea, nocturia, ankle edema, and pallor in light-skinned individuals. A patient with rasping cough, thick mucoid sputum, and wheezing may have bronchitis. Productive cough, dyspnea, weight loss, and dyspnea indicate tuberculosis; fever, dry nonproductive cough, and diminished breath sounds may indicate Pneumocystis jiroveci (P. carinii)pneumonia. DIF: Cognitive Level: Applying (Application) MSC: Client Needs: Physiologic Integrity: Physiologic Adaptation 33. Apatientcomestothecliniccomplainingofacoughthatisworseatnightbutnotasbadduringtheday. The nurse recognizes that this cough mayindicate: a. Pneumonia. b. Postnasal drip orsinusitis. c. Exposure to irritants atwork. d. ChronicbronchialirritationfromsNmUoRkiSnIgN.GTB.COM ANS: B A cough that primarily occurs at night may indicate postnasal drip or sinusitis. Exposure to irritants at work causesanafternoonoreveningcough.Smokersexperienceearlymorningcoughing.Coughingassociatedwith acute illnesses such as pneumonia is continuous throughout theday. DIF: Cognitive Level: Applying (Application) MSC: Client Needs: Physiologic Integrity: Physiologic Adaptation 34. Duringamorningassessment,thenursenoticesthatthepatientssputumisfrothyandpink.Which condition could this findingindicate? a. Croup b. Tuberculosis c. Viralinfection d. Pulmonaryedema ANS:D Sputum, alone, is not diagnostic, but some conditions have characteristic sputum production. Pink, frothy sputum indicates pulmonary edema or it may be a side effect of sympathomimetic medications. Croup is associatedwithabarkingcough,notsputumproduction.Tuberculosismayproducerust-coloredsputum.Viral infections may produce white or clear mucoidsputum. DIF: Cognitive Level: Understanding (Comprehension) MSC: Client Needs: Physiologic Integrity: Physiologic Adaptation 35. Duringauscultationofbreathsounds,thenurseshouldcorrectlyusethestethoscopeinwhichofthe followingways? a. Listening to at least one full respiration in eachlocation b. Listeningasthepatientinhalesandthengoingtothenextsiteduringexhalation c. Instructingthepatienttobreatheinandoutrapidlywhilelisteningtothebreathsounds d. Ifthepatientismodest,listeningtosoundsoverhisorherclothingorhospitalgown ANS:A Duringauscultationofbreathsoundswithastethoscope,listeningtoonefullrespirationineachlocationis important.Duringtheexamination,thenurseshouldmonitorthebreathingandoffertimesforthepersonto breathe normally to prevent possibledizziness. DIF: Cognitive Level: Applying (Application)NURSINGTB.COM MSC: Client Needs: Safe and Effective Care Environment: Management of Care 36. Apatienthasbeenadmittedtotheemergencydepartmentwithapossiblemedicaldiagnosisofpulmonary embolism.Thenurseexpectstoseewhichassessmentfindingsrelatedtothiscondition? a. Absent or decreased breathsounds b. Productive cough with thin, frothysputum c. Chest pain that is worse on deep inspiration anddyspnea d. Diffuseinfiltrateswithareasofdullnessuponpercussion ANS:C Findingsforpulmonaryembolismincludechestpainthatisworseondeepinspiration,dyspnea,apprehension, anxiety,restlessness,partialarterialpressureofoxygen(PaO2)lessthan80mmHg,diaphoresis,hypotension, crackles, andwheezes. DIF: Cognitive Level: Analyzing (Analysis) MSC: Client Needs: Physiologic Integrity: Physiologic Adaptation 37. Duringpalpationoftheanteriorchestwall,thenursenoticesacoarse,cracklingsensationovertheskin surface. On the basis of these findings, the nurse suspects: a. Tactilefremitus. b. Crepitus. c. Frictionrub. d. Adventitioussounds. ANS:B Crepitusisacoarse,cracklingsensationpalpableovertheskinsurface.Itoccursinsubcutaneousemphysema when air escapes from the lung and enters the subcutaneous tissue, such as after open thoracic injury or surgery. DIF: Cognitive Level: Applying (Application) MSC: Client Needs: Physiologic Integrity: Physiologic Adaptation 38. Thenurseisauscultatingthelungsofapatientwhohadbeensleepingandnoticesshort,popping,crackling soundsthatstopafterafewbreaths.Thenurserecognizesthatthesebreathsoundsare: a. Atelectatic crackles that do not have a pathologiccause. NURSINGTB.COM b. Fine crackles and may be a sign ofpneumonia. c. Vesicular breathsounds. d. Finewheezes. ANS: A Onetypeofadventitioussound,atelectaticcrackles,doesnothaveapathologiccause.Theyareshort,popping, crackling sounds that sound similar to fine crackles but do not last beyond a few breaths. When sections of alveoli are not fully aerated (as in people who are asleep or in older adults), they deflate slightly and accumulatesecretions.Cracklesareheardwhenthesesectionsareexpandedbyafewdeepbreaths.Atelectatic cracklesareheardonlyintheperiphery,usuallyindependentportionsofthelungs,anddisappearafterthefirst few breaths or after acough. DIF: Cognitive Level: Analyzing (Analysis) MSC: Client Needs: Safe and Effective Care Environment: Management of Care 39. Apatienthasbeenadmittedtotheemergencydepartmentforasuspecteddrugoverdose.Hisrespirations are shallow, with an irregular pattern, with a rate of 12 respirations per minute. The nurse interprets this respiration pattern as which of thefollowing? a. Bradypnea b. Cheyne-Stokesrespirations c. Hypoventilation d. Chronicobstructivebreathing ANS:C Hypoventilationischaracterizedbyanirregular,shallowpattern,andcanbecausedbyanoverdoseof narcoticsoranesthetics.Bradypneaisslowbreathing,witharatelessthan10respirationsperminute. DIF: Cognitive Level: Analyzing (Analysis) MSC: Client Needs: Safe and Effective Care Environment: Management of Care 40. Apatientwithpleuritishasbeenadmittedtothehospitalandcomplainsofpainwithbreathing.Whatother key assessment finding would the nurse expect to find uponauscultation? a. Stridor b. Frictionrub c. Crackles d. Wheezing ANS: B NURSINGTB.COM A patient with pleuritis will exhibit a pleural friction rub upon auscultation. This sound is made when the pleurae become inflamed and rub together during respiration. The sound is superficial, coarse, and low- pitched,asiftwopiecesofleatherarebeingrubbedtogether.Stridorisassociatedwithcroup,acuteepiglottitis in children, and foreign body inhalation. Crackles are associated with pneumonia, heart failure, chronic bronchitis,andotherdiseases.Wheezesareassociatedwithdiffuseairwayobstructioncausedbyacuteasthma or chronicemphysema. DIF: Cognitive Level: Applying (Application) MSC: Client Needs: Safe and Effective Care Environment: Management of Care MULTIPLE RESPONSE 1.Thenurseisassessingvoicesoundsduringarespiratoryassessment.Whichofthesefindingsindicatesa normal assessment? Select all thatapply. a. Voicesoundsarefaint,muffled,andalmostinaudiblewhenthepatientwhispersone,two,threein a very softvoice. b. Asthepatientrepeatedlysaysninety-nine,theexaminerclearlyhearsthewordsninety-nine. c. Whenthepatientspeaksinanormalvoice,theexaminercanhearasoundbutcannotexactly distinguish what is beingsaid. d. Asthepatientsaysalongee-ee-eesound,theexamineralsohearsalongee-ee-eesound. e. Asthepatientsaysalongee-ee-eesound,theexaminerhearsalongaaaaaasound. ANS: A, C, D As a patient repeatedly says ninety-nine, normally the examiner hears voice sounds but cannot distinguish what is being said. If a clear ninety-nine is auscultated, then it could indicate increased lung density, which enhancesthetransmissionofvoicesounds,whichisameasureofbronchophony.Whenapatientsaysalong ee-ee-ee sound, normally the examiner also hears a long ee-ee-ee sound through auscultation, which is a measure of egophony. If the examiner hears a long aaaaaa sound instead, this sound could indicate areas of consolidationorcompression.Withwhisperedpectoriloquy,aswhenapatientwhispersaphrasesuchasone- two-three,thenormalresponsewhenauscultatingvoicesoundsistohearsoundsthatarefaint,muffled,and almostinaudible.Iftheexaminerclearlyhearsthewhisperedvoice,asifthepatientisspeakingthroughthe stethoscope, then consolidation of the lung fields mayexist. DIF: Cognitive Level: Applying (Application) MSC: Client Needs: Safe and Effective Care Environment: Management of Care NURSINGTB.COM Chapter 20: Heart and Neck Vessels MULTIPLE CHOICE 1. The sac that surrounds and protects the heart is calledthe: a. Pericardium. b. Myocardium. c. Endocardium. d. Pleuralspace. ANS: A Thepericardiumisatough,fibrousdouble-walledsacthatsurroundsandprotectstheheart.Ithastwolayers that contain a few milliliters of serous pericardialfluid. DIF:CognitiveLevel:Remembering(Knowledge) MSC: Client Needs:General 2. Thedirectionofbloodflowthroughtheheartisbestdescribedbywhichofthese? a. VenacavarightatriumrightventriNcUleRluSnINgsGpTuBl.mCoOnMaryarteryleftatriumleftventricle b. Rightatriumrightventriclepulmonaryarterylungspulmonaryveinleftatriumleftventricle c. Aortarightatriumrightventriclelungspulmonaryveinleftatriumleftventriclevenacava d. Rightatriumrightventriclepulmonaryveinlungspulmonaryarteryleftatriumleftventricle ANS:B Returningbloodfromthebodyemptiesintotherightatriumandflowsintotherightventricleandthengoesto thelungsthroughthepulmonaryartery.Thelungsoxygenatetheblood,anditisthenreturnedtotheleftatrium through the pulmonary vein. The blood goes from there to the left ventricle and then out to the body through theaorta. DIF:CognitiveLevel:Remembering(Knowledge) MSC: Client Needs:General 3. Thenurseisreviewingtheanatomyandphysiologicfunctioningoftheheart.Whichstatementbest describes what is meant by atrialkick? a. Theatriacontractduringsystoleandattempttopushagainstclosedvalves. b. Contractionoftheatriaatthebeginningofdiastolecanbefeltasapalpitation. c. Atrialkickisthepressureexertedagainsttheatriaastheventriclescontractduringsystole. d. Theatriacontracttowardtheendofdiastoleandpushtheremainingbloodintotheventricles. ANS: D Towardtheendofdiastole,theatriacontractandpushthelastamountofblood(approximately25%ofstroke volume) into the ventricles. This active filling phase is called presystole, or atrial systole, or sometimes theatrialkick. DIF:CognitiveLevel:Remembering(Knowledge) MSC: Client Needs:General 4. Whenlisteningtoheartsounds,thenurseknowsthevalveclosuresthatcanbeheardbestatthebaseofthe heartare: a. Mitralandtricuspid. b. Tricuspidandaortic. c. Aortic andpulmonic. d. Mitral andpulmonic. ANS: C NURSINGTB.COM Thesecondheartsound(S2)occurswiththeclosureofthesemilunar(aorticandpulmonic)valvesandsignals theendofsystole.Althoughitisheardoveralltheprecordium,theS2 isloudestatthebaseoftheheart. DIF:CognitiveLevel:Understanding(Comprehension) MSC: Client Needs:General 5. Whichofthesestatementsdescribestheclosureofthevalvesinanormalcardiaccycle? a. The aortic valve closes slightly before the tricuspidvalve. b. The pulmonic valve closes slightly before the aorticvalve. c. The tricuspid valve closes slightly later than the mitralvalve. d. Both the tricuspid and pulmonic valves close at the sametime. ANS: C Eventsoccurjustslightlylaterintherightsideoftheheartbecauseoftherouteofmyocardialdepolarization. As a result, two distinct components to each of the heart sounds exist, and sometimes they can be heard separately.Inthefirstheartsound,themitralcomponent(M1)closesjustbeforethetricuspidcomponent(T1). DIF:CognitiveLevel:Understanding(Comprehension) MSC: Client Needs:General 6. Thecomponentoftheconductionsystemreferredtoasthepacemakeroftheheartisthe: a. Atrioventricular (AV)node. b. Sinoatrial (SA)node. c. Bundle ofHis. d. Bundlebranches. ANS: B SpecializedcellsintheSAnodenearthesuperiorvenacavainitiateanelectricalimpulse.BecausetheSA nodehasanintrinsicrhythm,itiscalledthepacemakeroftheheart. DIF:CognitiveLevel:Remembering(Knowledge) MSC: Client Needs:General 7. The electrical stimulus of the cardiac cycle follows whichsequence? a. AVnodeSAnodebundleofHis b. BundleofHisAVnodeSAnode NURSINGTB.COM c. SA node AV node bundle of His bundlebranches d. AVnodeSAnodebundleofHisbundlebranches ANS:D SpecializedcellsintheSAnodenearthesuperiorvenacavainitiateanelectricalimpulse.Thecurrentflowsin anorderlysequence,firstacrosstheatriatotheAVnodelowintheatrialseptum.Thereitisdelayedslightly, allowing the atria the time to contract before the ventricles are stimulated. Then the impulse travels to the bundleofHis,therightandleftbundlebranches,andthenthroughtheventricles. DIF:CognitiveLevel:Understanding(Comprehension) MSC: Client Needs:General 8. Thefindingsfromanassessmentofa70-year-oldpatientwithswellinginhisanklesincludejugularvenous pulsations5cmabovethesternalanglewhentheheadofhisbediselevated45degrees.Thenurseknowsthat this findingindicates: a. Decreased fluidvolume. b. Increased cardiacoutput. c. Narrowing of jugularveins. d. Elevated pressure related to heartfailure. ANS: D Becausenocardiacvalveexiststoseparatethesuperiorvenacavafromtherightatrium,thejugularveinsgive informationabouttheactivityontherightsideoftheheart.Theyreflectfillingpressuresandvolumechanges. Normaljugularvenouspulsationis2cmorlessabovethesternalangle.Elevatedpressureismorethan3cm above the sternal angle at 45 degrees and occurs with heartfailure. DIF: Cognitive Level: Analyzing (Analysis) MSC: Client Needs: Physiologic Integrity: Physiologic Adaptation 9. Whenassessinganewborninfantwhois5minutesold,thenurseknowswhichofthesestatementstobe true? a. Theleftventricleislargerandweighsmorethantherightventricle. b. The circulation of a newborn is identical to that of anadult. c. Bloodcanflowintotheleftsideoftheheartthroughanopeningintheatrialseptum. NURSINGTB.COM d. Theforamenovaleclosesjustminutesbeforebirth,andtheductusarteriosusclosesimmediately after. ANS: C First, approximately two thirds of the blood is shunted through an opening in the atrial septum, the foramen ovale,intotheleftsideoftheheart,whereitispumpedoutthroughtheaorta.Theforamenovalecloseswithin thefirsthourafterbirthbecausethepressureintherightsideoftheheartisnowlowerthanintheleftside. DIF: Cognitive Level: Applying (Application) MSC: Client Needs: Health Promotion and Maintenance 10. A25-year-oldwomaninherfifthmonthofpregnancyhasabloodpressureof100/70mmHg.Inreviewing herpreviousexamination,thenursenotesthatherbloodpressureinhersecondmonthwas124/80mmHg.In evaluating this change, what does the nurse know to betrue? a. Thisdeclineinbloodpressureistheresultofperipheralvasodilatationandisanexpectedchange. b. Becauseofincreasedcardiacoutput,thebloodpressureshouldbehigheratthistime. c. Thischangeinbloodpressureisnotanexpectedfindingbecauseitmeansadecreaseincardiac output. d. Thisdeclineinbloodpressuremeansadecreaseincirculatingbloodvolume,whichisdangerous for thefetus. ANS: A Despitetheincreasedcardiacoutput,arterialbloodpressuredecreasesinpregnancybecauseofperipheral vasodilatation.Thebloodpressuredropstoitslowestpointduringthesecondtrimesterandthenrisesafter that. DIF: Cognitive Level: Analyzing (Analysis) MSC: Client Needs: Physiologic Integrity: Physiologic Adaptation 11. Inassessinga70-year-oldman,thenursefindsthefollowing:bloodpressure140/100mmHg;heartrate 104 beats per minute and slightly irregular; and the split S2 heart sound. Which of these findings can be explained by expected hemodynamic changes related toage? a. Increase in resting heartrate b. Increase in systolic bloodpressure c. Decrease in diastolic bloodpressure d. Increaseindiastolicbloodpressure ANS:B Withaging,anincreaseinsystolicbloodpressNuUreRoScINcuGrsT.BN.CoOsMignificantchangeindiastolicpressureandno change in the resting heart rate occur with aging. Cardiac output at rest is does not changed with aging. DIF: Cognitive Level: Analyzing (Analysis) MSC: Client Needs: Health Promotion and Maintenance 12. A45-year-oldmanisintheclinicforaroutinephysicalexamination.Duringtherecordingofhishealth history,thepatientstatesthathehasbeenhavingdifficultysleeping.Illbesleepinggreat,andthenIwakeup andfeellikeIcantgetmybreath.Thenursesbestresponsetothiswouldbe: a. When was your lastelectrocardiogram? b. Its probably because its been so hot atnight. c. Do you have any history of problems with yourheart? d. Have you had a recent sinus infection or upper respiratoryinfection? ANS: C Paroxysmal nocturnal dyspnea (shortness of breath generally occurring at night) occurs with heart failure. Lyingdownincreasesthevolumeofintrathoracicblood,andtheweakenedheartcannotaccommodatethe increasedload.Classically,thepersonawakensafter2hoursofsleep,arises,andflingsopenawindowwith the perception of needing freshair. DIF: Cognitive Level: Applying (Application) MSC: Client Needs: Physiologic Integrity: Physiologic Adaptation 13. Inassessingapatientsmajorriskfactorsforheartdisease,whichwouldthenursewanttoincludewhen taking ahistory? a. Family history, hypertension, stress, andage b. Personality type, high cholesterol, diabetes, andsmoking c. Smoking, hypertension, obesity, diabetes, and highcholesterol d. Alcoholconsumption,obesity,diabetes,stress,andhighcholesterol ANS:C To assess for major risk factors of coronary artery disease, the nurse should collect data regarding elevated serumcholesterol,elevatedbloodpressure,bloodglucoselevelsabove100mg/dLorknowndiabetesmellitus, obesity,anylengthofhormonereplacementtherapyforpostmenopausalwomen,cigarettesmoking,andlow activitylevel. DIF: Cognitive Level: Applying (Application) MSC: Client Needs: Physiologic Integrity: Reduction of Risk Potential 14. Themotherofa3-month-oldinfantstatesNthUaRthSeINrGbaTbBy.ChaOsMnotbeengainingweight.Withfurther questioning,thenursefindsthattheinfantfallsasleepafternursingandwakesupafterashorttime,hungry again. What other information would the nurse want tohave? a. Infants sleepingposition b. Sibling history of eatingdisorders c. Amount of background noise wheneating d. Presenceofdyspneaordiaphoresiswhensucking ANS:D To screen for heart disease in an infant, the focus should be on feeding. Fatigue during feeding should be noted.Aninfantwithheartfailuretakesfewerounceseachfeeding,becomesdyspneicwithsucking,maybe diaphoretic,andthenfallsintoexhaustedsleepandawakensafterashorttimehungryagain. DIF: Cognitive Level: Analyzing (Analysis) MSC: Client Needs: Physiologic Integrity: Physiologic Adaptation 15. Inassessingthecarotidarteriesofanolderpatientwithcardiovasculardisease,thenursewould: a. Palpate the artery in the upper one third of theneck. b. Listen with the bell of the stethoscope to assess forbruits. c. Simultaneously palpate both arteries to compareamplitude. d. Instruct the patient to take slow deep breaths duringauscultation. ANS: B Ifcardiovasculardiseaseissuspected,thenthenurseshouldauscultateeachcarotidarteryforthepresenceofa bruit. The nurse should avoid compressing the artery, which could create an artificial bruit and compromise circulationifthecarotidarteryisalreadynarrowedbyatherosclerosis.Excessivepressureonthecarotidsinus area high in the neck should be avoided, and excessive vagal stimulation could slow down the heart rate, especiallyinolderadults.Palpatingonlyonecarotidarteryatatimewillavoidcompromisingarterialbloodto thebrain. DIF: Cognitive Level: Applying (Application) MSC: Client Needs: Safe and Effective Care Environment: Management of Care 16. Duringanassessmentofa68-year-oldmanwitharecentonsetofright-sidedweakness,thenursehearsa blowing, swishing sound with the bell of the stethoscope over the left carotid artery. This finding would indicate: a. Valvulardisorder. b. Blood flowturbulence. c. Fluid volumeoverload. d. Ventricularhypertrophy. NURSINGTB.COM ANS: B Abruitisablowing,swishingsoundindicatingbloodflowturbulence;normally,noneispresent. DIF: Cognitive Level: Analyzing(Analysis) MSC: Client Needs: Safe and Effective Care Environment: Management of Care 17. Duringaninspectionoftheprecordiumofanadultpatient,thenursenoticesthechestmovinginaforceful manner along the sternal border. This finding most likely suggestsa(n): a. Normalheart. b. Systolicmurmur. c. Enlargement of the leftventricle. d. Enlargement of the rightventricle. ANS: D Normally,theexaminermayormaynotseeanapicalimpulse;whenvisible,itoccupiesthefourthorfifth intercostalspaceatorinsidethemidclavicularline.Aheaveorliftisasustainedforcefulthrustingofthe ventricleduringsystole.Itoccurswithventricularhypertrophyasaresultofincreasedworkload.Aright ventricularheaveisseenatthesternalborder;aleftventricularheaveisseenattheapex. DIF: Cognitive Level: Applying (Application) MSC: Client Needs: Safe and Effective Care Environment: Management of Care 18. Duringanassessmentofahealthyadult,wherewouldthenurseexpecttopalpatetheapicalimpulse? a. Third left intercostal space at the midclavicularline b. Fourth left intercostal space at the sternalborder c. Fourth left intercostal space at the anterior axillaryline d. Fifthleftintercostalspaceatthemidclavicularline ANS:D Theapicalimpulseshouldoccupyonlyoneintercostalspace,thefourthorfifth,anditshouldbeatormedial to the midclavicularline. DIF:CognitiveLevel:Remembering(KnowleNdUgeR)SINGTB.COM MSC: Client Needs: Safe and Effective Care Environment: Management of Care 19. Thenurseisexaminingapatientwhohaspossiblecardiacenlargement.Whichstatementaboutpercussion of the heart istrue? a. Percussion is a useful tool for outlining the heartsborders. b. Percussion is easier in patients who areobese. c. Studiesshowthatpercussedcardiacbordersdonotcorrelatewellwiththetruecardiacborder. d. Onlyexperthealthcareprovidersshouldattemptpercussionoftheheart. ANS: C Numerouscomparisonstudieshaveshownthatthepercussedcardiacbordercorrelatesonlymoderatelywith thetruecardiacborder.Percussionisoflimitedusefulnesswiththefemalebreasttissue,inapersonwhois obese,orinapersonwithamuscularchestwall.Chestx-rayimagesorechocardiographicexaminationsare significantly more accurate in detecting heartenlargement. DIF: Cognitive Level: Understanding (Comprehension) MSC: Client Needs: Safe and Effective Care Environment: Management of Care 20. Thenurseispreparingtoauscultateforheartsounds.Whichtechniqueiscorrect? a. Listeningtothesoundsattheaortic,tricuspid,pulmonic,andmitralareas b. ListeningbyinchingthestethoscopeinaroughZpattern,fromthebaseoftheheartacrossand down, then over to theapex c. Listeningtothesoundsonlyatthesitewheretheapicalpulseisfelttobethestrongest d. Listeningforallpossiblesoundsatatimeateachspecifiedarea ANS:B Auscultationofbreathsoundsshouldnotbelimitedtoonlyfourlocations.Soundsproducedbythevalvesmay be heard all over the precordium. The stethoscope should be inched in a rough Z pattern from the base of the heartacrossanddown,thenovertotheapex;or,startingattheapex,itshouldbeslowlyworkedup.Listening selectively to one sound at a time isbest. DIF: Cognitive Level: Understanding (Comprehension) MSC: Client Needs: Safe and Effective Care Environment: Management of Care 21. Whilecountingtheapicalpulseona16-year-oldpatient,thenursenoticesanirregularrhythm.Hisrate speedsuponinspirationandslowsonexpiration.Whatwouldbethenursesresponse? a. TalkwiththepatientabouthisintaNkUeRoSfIcNafGfeTiBn.eC.OM b. Perform an electrocardiogram after theexamination. c. Nofurtherresponseisneededbecausesinusarrhythmiacanoccurnormally. d. Refer the patient to a cardiologist for furthertesting. ANS: C Therhythmshouldberegular,althoughsinusarrhythmiaoccursnormallyinyoungadultsandchildren.With sinus arrhythmia, the rhythm varies with the persons breathing, increasing at the peak of inspiration and slowing withexpiration. DIF: Cognitive Level: Analyzing (Analysis) MSC: Client Needs: Health Promotion and Maintenance 22. Whenlisteningtoheartsounds,thenurseknowsthattheS1: a. IslouderthantheS2atthebaseoftheheart. b. Indicates the beginning ofdiastole. c. Coincides with the carotid arterypulse. d. Is caused by the closure of the semilunarvalves. ANS: C TheS1coincideswiththecarotidarterypulse,isthestartofsystole,andislouderthantheS2attheapexofthe heart; the S2 is louder than the S1 at the base. The nurse should gently feel the carotid artery pulse while auscultatingattheapex;thesoundheardaseachpulseisfeltistheS1. DIF:CognitiveLevel:Understanding(Comprehension) MSC: Client Needs:General 23. Duringthecardiacauscultation,thenursehearsasoundimmediatelyoccurringaftertheS2atthesecond leftintercostalspace.Tofurtherassessthissound,whatshouldthenursedo? a. Havethepatientturntotheleftsidewhilethenurselistenswiththebellofthestethoscope. b. Askthepatienttoholdhisorherbreathwhilethenurselistensagain. c. NofurtherassessmentisneededbecausethenurseknowsthissoundisanS3. d. Watchthepatientsrespirationswhilelisteningfortheeffectonthesound. ANS: D NURSINGTB.COM AsplitS2isanormalphenomenonthatoccurstowardtheendofinspirationinsomepeople.AsplitS2isheard onlyinthepulmonicvalvearea,thesecondleftinterspace.WhenthesplitS2isfirstheard,thenurseshouldno be tempted to ask the person to hold his or her breath so that the nurse can concentrate on the sounds. Breath holdingwillonlyequalizeejectiontimesintherightandleftsidesoftheheartandcausethesplittogoaway. Rather, the nurse should concentrate on the split while watching the persons chest rise up and down with breathing. DIF: Cognitive Level: Analyzing (Analysis) MSC: Client Needs: Safe and Effective Care Environment: Management of Care 24. Whichofthesefindingswouldthenurseexpecttonoticeduringacardiacassessmentona4-year-old child? a. S3 when sittingup b. Persistent tachycardia above 150 beats perminute c. Murmur at the second left intercostal space whensupine d. Palpableapicalimpulseinthefifthleftintercostalspacelateraltomidclavicularline ANS:C Some murmurs are common in healthy children or adolescents and are termed innocent or functional. The innocentmurmurisheardatthesecondorthirdleftintercostalspaceanddisappearswithsitting,andtheyoung person has no associated signs of cardiacdysfunction. DIF: Cognitive Level: Applying (Application) MSC: Client Needs: Health Promotion and Maintenance 25. Whileauscultatingheartsoundsona7-year-oldchildforaroutinephysicalexamination,thenursehearsan S3,asoftmurmurattheleftmidsternalborder,andavenoushumwhenthechildisstanding.Whatwouldbea correct interpretation of thesefindings? a. S3isindicativeofheartdiseaseinchildren. b. Thesefindingscanallbenormalinachild. c. These findings are indicative of congenitalproblems. d. The venous hum most likely indicates ananeurysm. ANS: B AphysiologicS3iscommoninchildren.Avenoushum,causedbyturbulenceofbloodflowinthejugular venoussystem,iscommoninhealthychildrenandhasnopathologicsignificance.Heartmurmursthatare innocent (or functional) in origin are very common throughchildhood. DIF:CognitiveLevel:Analyzing(Analysis) NURSINGTB.COM MSC: Client Needs: Health Promotion and Maintenance 26. Duringtheprecordialassessmentonanpatientwhois8monthspregnant,thenursepalpatestheapical impulseatthefourthleftintercostalspacelateraltothemidclavicularline.Thisfindingwouldindicate: a. Right ventricularhypertrophy. b. Increased volume and size of the heart as a result ofpregnancy. c. Displacementoftheheartfromelevationofthediaphragm. d. Increasedbloodflowthroughtheinternalmammaryartery. ANS: C Palpation of the apical impulse is higher and more lateral, compared with the normal position, because the enlarginguteruselevatesthediaphragmanddisplacestheheartupandtotheleftandrotatesitonitslongaxis. DIF: Cognitive Level: Analyzing (Analysis) MSC: Client Needs: Health Promotion and Maintenance 27. InassessingforanS4heartsoundwithastethoscope,thenursewouldlistenwiththe: a. Bellofthestethoscopeatthebasewiththepatientleaningforward. b. Bellofthestethoscopeattheapexwiththepatientintheleftlateralposition. c. Diaphragmofthestethoscopeintheaorticareawiththepatientsitting. d. Diaphragmofthestethoscopeinthepulmonicareawiththepatientsupine. ANS: B The S4 is a ventricular filling sound that occurs when the atria contract late in diastole and is heard immediatelybeforetheS1.TheS4isaverysoftsoundwithaverylowpitch.Thenurseneedsagoodbelland mustlistenforthissound.AnS4isheardbestattheapex,withthepersonintheleftlateralposition. DIF: Cognitive Level: Understanding (Comprehension) MSC: Client Needs: Safe and Effective Care Environment: Management of Care 28. A70-year-oldpatientwithahistoryofhypertensionhasabloodpressureof180/100mmHgandaheart rateof90beatsperminute.ThenursehearsanextraheartsoundattheapeximmediatelybeforetheS1.The soundisheardonlywiththebellofthestethoscopewhilethepatientisintheleftlateralposition.Withthese findingsandthepatientshistory,thenurseknowsthatthisextraheartsoundismostlikelya(n): a. SplitS1. NURSINGTB.COM b. Atrialgallop. c. Diastolicmurmur. d. Summationsound. ANS: B ApathologicS4istermedanatrialgalloporanS4gallop.Itoccurswithdecreasedcomplianceoftheventricle and with systolic overload (afterload), including outflow obstruction to the ventricle (aortic stenosis) and systemic hypertension. A left-sided S4 occurs with these conditions and is heard best at the apex with the patient in the left lateralposition. DIF: Cognitive Level: Analyzing (Analysis) MSC: Client Needs: Health Promotion and Maintenance 29. The nurse is performing a cardiac assessment on a 65-year-old patient 3 days after her myocardial infarction(MI).Heartsoundsarenormalwhensheissupine,butwhensheissittingandleaningforward,the nursehearsahigh-pitched,scratchysoundwiththediaphragmofthestethoscopeattheapex.Itdisappearson inspiration. The nursesuspects: a. Increased cardiacoutput. b. AnotherMI. c. Inflammation of theprecordium. d. Ventricular hypertrophy resulting from muscledamage. ANS: C Inflammationoftheprecordiumgivesrisetoafrictionrub.Thesoundishighpitchedandscratchy,similarto sandpaper being rubbed. A friction rub is best heard with the diaphragm of the stethoscope, with the person sittingupandleaningforward,andwiththebreathheldinexpiration.Africtionrubcanbeheardanyplaceon theprecordium.Usually,however,thesoundisbestheardattheapexandleftlowersternalborder,whichare placeswherethepericardiumcomesinclosecontactwiththechestwall. DIF: Cognitive Level: Analyzing (Analysis) MSC: Client Needs: Physiologic Integrity: Physiologic Adaptation 30. Themotherofa10-month-oldinfanttellsthenursethatshehasnoticedthathersonbecomesbluewhenhe is crying and that the frequency of this is increasing. He is also not crawling yet. During the examination the nursepalpatesathrillattheleftlowersternalborderandauscultatesaloudsystolicmurmurinthesamearea. What would be the most likely cause of thesefindings? a. Tetralogy ofFallot b. Atrial septaldefect c. Patent ductusarteriosus d. Ventricular septaldefect NURSINGTB.COM ANS: A ThecauseofthesefindingsistetralogyofFallot.Itssubjectivefindingsinclude:(1)severecyanosis,notinthe firstmonthsoflifebutdevelopingastheinfantgrows,andrightventricleoutflow(i.e.,pulmonic)stenosisthat gets worse; (2) cyanosis with crying and exertion at first and then at rest; and (3) slowed development. Its objectivefindingsinclude:(1)thrillpalpableattheleftlowersternalborder;(2)theS1isnormal,theS2hasa loudA2,andtheP2isdiminishedorabsent;and(3)themurmurissystolic,loud,andcrescendo-decrescendo. DIF: Cognitive Level: Analyzing (Analysis) MSC: Client Needs: Health Promotion and Maintenance 31. A30-year-oldwomanwithahistoryofmitralvalveproblemsstatesthatshehasbeenverytired.Shehas started waking up at night and feels like her heart is pounding. During the assessment, the nurse palpates a thrillandliftatthefifthleftintercostalspacemidclavicularline.Inthesamearea,thenursealsoauscultatesa blowing,swishingsoundrightaftertheS1.Thesefindingswouldbemostconsistentwith: a. Heartfailure. b. Aorticstenosis. c. Pulmonaryedema. d. Mitralregurgitation. ANS: D Thesefindingsareconsistentwithmitralregurgitation.Itssubjectivefindingsincludefatigue,palpitation,and orthopnea, and its objective findings are: (1) a thrill in systole at the apex; (2) a lift at the apex; (3) the apical impulsedisplaceddownandtotheleft;(4)theS1isdiminished,theS2 isaccentuated,andtheS3attheapexis often present; and (5) a pansystolic murmur that is often loud, blowing, best heard at the apex, and radiating well to the leftaxilla. DIF: Cognitive Level: Analyzing (Analysis) MSC: Client Needs: Physiologic Integrity: Physiologic Adaptation 32. During a cardiac assessment on a 38-year-old patient in the hospital for chest pain, the nurse finds the following: jugular vein pulsations 4 cm above the sternal angle when the patient is elevated at 45 degrees, bloodpressure98/60mmHg,heartrate130beatsperminute,ankleedema,difficultybreathingwhensupine, andanS3onauscultation.Whichoftheseconditionsbestexplainsthecauseofthesefindings? a. Fluidoverload b. Atrial septaldefect c. MI d. Heartfailure ANS: D Heart failure causes decreased cardiac output when the heart fails as a pump and the circulation becomes backed up and congested. Signs and symptoms include dyspnea, orthopnea, paroxysmal nocturnal dyspnea, decreased blood pressure, dependent and pitting edema; anxiety; confusion; jugular vein distention; and fatigue.TheS3isassociatedwithheartfailureandisalwaysabnormalafter35yearsofage.The S3maybethe earliest sign of heartfailure. DIF: Cognitive Level: Analyzing (Analysis) MSC: Client Needs: Physiologic Integrity: Physiologic Adaptation 33. The nurse knows that normal splitting of the S2 is associatedwith: a. Expiration. b. Inspiration. c. Exercisestate. d. Low resting heartrate. ANS: B NormalorphysiologicsplittingoftheS2 isassociatedwithinspirationbecauseoftheincreasedbloodreturnto the right side of the heart, delaying closure of the pulmonicvalve. DIF: Cognitive Level: Understanding (Comprehension) MSC: Client Needs: Safe and Effective Care Environment: Management of Care 34. Duringacardiovascularassessment,thenurseknowsthatathrillis: a. Vibration that ispalpable. b. Palpated in the right epigastricarea. c. Associated with ventricularhypertrophy. d. Murmur auscultated at the third intercostalspace. ANS: A Athrillisapalpablevibrationthatsignifiesturbulentbloodflowandaccompaniesloudmurmurs.Theabsence of a thrill does not rule out the presence of amurmur. DIF: Cognitive Level: Remembering (Knowledge) NURSINGTB.COM MSC: Client Needs: Safe and Effective Care Environment: Management of Care 35. During a cardiovascular assessment, the nurse knows that an S4 heart soundis: a. Heard at the onset of atrialdiastole. b. Usually a normal finding in the olderadult. c. Heard at the end of ventriculardiastole. d. Heardbestoverthesecondleftintercostalspacewiththeindividualsittingupright. ANS: C AnS4heartsoundisheardattheendofdiastolewhentheatriacontract(atrialsystole)andwhentheventricles are resistant to filling. The S4 occurs just before theS1. DIF:CognitiveLevel:Understanding(Comprehension) MSC: Client Needs: Safe and Effective Care Environment: Management of Care 36. Duringanassessment,thenursenotesthatthepatientsapicalimpulseislaterallydisplacedandispalpable over a wide area. This findingindicates: a. Systemichypertension. b. Pulmonichypertension. c. Pressure overload, as in aorticstenosis. d. Volume overload, as in heartfailure. ANS: D Withvolumeoverload,asinheartfailureandcardiomyopathy,cardiacenlargementlaterallydisplacesthe apicalimpulseandispalpableoverawiderareawhenleftventricularhypertrophyanddilationarepresent. DIF: Cognitive Level: Applying (Application) MSC: Client Needs: Safe and Effective Care Environment: Management of Care 37. Whenthenurseisauscultatingthecarotidarteryforbruits,whichofthesestatementsreflectsthecorrect technique? a. Whilelisteningwiththebellofthestethoscope,thepatientisaskedtotakeadeepbreathandhold it. b. Whileauscultatingonesidewiththebellofthestethoscope,thecarotidarteryispalpatedonthe other side to checkpulsations. NURSINGTB.COM c. Whilelightlyapplyingthebellofthestethoscopeoverthecarotidarteryandlistening,thepatientis asked to take a breath, exhale, and briefly holdit. d. Whilefirmlyplacingthebellofthestethoscopeoverthecarotidarteryandlistening,thepatientis asked to take a breath, exhale, and briefly holdit. ANS: C Thecorrecttechniqueforauscultatingthecarotidarteryforbruitsinvolvesthenurselightlyapplyingthebell of the stethoscope over the carotid artery at three levels. While listening, the nurse asks the patient take a breath, exhale, and briefly hold it. Holding the breath on inhalation will also tense the levator scapulae muscles, which makes it hard to hear the carotid arteries. Examining only one carotid artery at a time will avoid compromising arterial blood flow to the brain. Pressure over the carotid sinus, which may lead to decreasedheartrate,decreasedbloodpressure,andcerebralischemiawithsyncope,shouldbeavoided. DIF: Cognitive Level: Applying (Application) MSC: Client Needs: Safe and Effective Care Environment: Management of Care 38. Thenurseispreparingforaclassonriskfactorsforhypertensionandreviewsrecentstatistics.Which racial group has the highest prevalence of hypertension in theworld? a. Blacks b. Whites c. AmericanIndians d. Hispanics ANS:A AccordingtotheAmericanHeartAssociation,theprevalenceofhypertensionishigheramongBlacksthanin other racialgroups. DIF: Cognitive Level: Understanding (Comprehension) MSC: Client Needs: Physiologic Integrity: Reduction of Risk Potential 39. Thenurseisassessingapatientwithpossiblecardiomyopathyandassessesthehepatojugularreflux.If heart failure is present, then the nurse should recognize which finding while pushing on the right upper quadrant of the patients abdomen, just below the ribcage? a. Thejugularveinswillriseforafewsecondsandthenrecedebacktothepreviousleveliftheheart is properlyworking. b. Thejugularveinswillremainelevatedaslongaspressureontheabdomenismaintained. c. Animpulsewillbevisibleatthefourthorfifthintercostalspaceatorinsidethemidclavicularline. d. ThejugularveinswillnotbedetecNteUdRdSuIrNinGgTtBh.iCsOmManeuver. ANS: B Whenperforminghepatojugularreflux,thejugularveinswillriseforafewsecondsandthenrecedebackto thepreviousleveliftheheartisabletopumptheadditionalvolumecreatedbythepushing.However,with heartfailure,thejugularveinsremainelevatedaslongaspressureontheabdomenismaintained. DIF: Cognitive Level: Analyzing (Analysis) MSC: Client Needs: Safe and Effective Care Environment: Management of Care 40. Thenurseisassessingtheapicalpulseofa3-month-oldinfantandfindsthattheheartrateis135beatsper minute. The nurse interprets this resultas: a. Normal for thisage. b. Lower thanexpected. c. Higher than expected, probably as a result ofcrying. d. Higher than expected, reflecting persistenttachycardia. ANS: A The heart rate may range from 100 to 180 beats per minute immediately after birth and then stabilize to an averageof120to140beatsperminute.Infantsnormallyhavewidefluctuationswithactivity,from170beats per minute or more with crying or being active to 70 to 90 beats per minute with sleeping. Persistent tachycardiaisgreaterthan200beatsperminuteinnewbornsorgreaterthan150beatsperminuteininfants. DIF: Cognitive Level: Analyzing (Analysis) MSC: Client Needs: Safe and Effective Care Environment: Management of Care MULTIPLE RESPONSE 1.Thenurseispresentingaclassonriskfactorsforcardiovasculardisease.Whichoftheseareconsidered modifiable risk factors for MI? Select all thatapply. a. Ethnicity b. Abnormallipids c. Smoking d. Gender e. Hypertension f. Diabetes g. Familyhistory NURSINGTB.COM ANS: B, C, E, F Nine modifiable risk factors for MI, as identified by a recent study, include abnormal lipids, smoking, hypertension,diabetes,abdominalobesity,psychosocialfactors,consumptionoffruitsandvegetables,alcohol use, and regular physicalactivity. DIF: Cognitive Level: Applying (Application) MSC: Client Needs: Health Promotion and Maintenance SHORT ANSWER 1.Thenurseisassessingapatientspulsesandnoticesadifferencebetweenthepatientsapicalpulseandradial pulse.Theapicalpulsewas118beatsperminute,andtheradialpulsewas105beatsperminute.Whatisthe pulsedeficit? ANS: 13 Thenurseshouldcountaserialmeasurement(oneaftertheother)oftheapicalpulseandthentheradialpulse. Normally,everybeatheardattheapexshouldperfusetotheperipheryandbepalpable.Thetwocountsshould beidentical.Iftheyaredifferent,thenthenurseshouldsubtracttheradialratefromtheapicalpulseandrecord the remainder as the pulsedeficit. DIF: Cognitive Level: Analyzing (Analysis) MSC: Client Needs: Safe and Effective Care Environment: Management of Care NURSINGTB.COM Chapter 21: Peripheral Vascular System and LymphaticSystem MULTIPLE CHOICE 1. Which statement is true regarding the arterialsystem? a. Arteries are large-diametervessels. b. The arterial system is a high-pressuresystem. c. The walls of arteries are thinner than those of theveins. d. Arteriescangreatlyexpandtoaccommodatealargebloodvolumeincrease. ANS: B Thepumpingheartmakesthearterialsystemahigh-pressuresystem. DIF: Cognitive Level: Remembering(Knowledge) MSC: Client Needs: General 2. Thenurseisreviewingthebloodsupplytothearm.Themajorarterysupplyingthearmisthe artery. a. Ulnar b. Radial c. Brachial d. Deeppalmar NURSINGTB.COM ANS: C The major artery supplying the arm is the brachial artery. The brachial artery bifurcates into the ulnar and radialarteriesimmediatelybelowtheelbow.Inthehand,theulnarandradialarteriesformtwoarchesknown as the superficial and deep palmararches. DIF:CognitiveLevel:Remembering(Knowledge) MSC: Client Needs:General 3. Thenurseispreparingtoassessthedorsalispedisartery.Whereisthecorrectlocationforpalpation? a. Behind theknee b. Over the lateralmalleolus c. In the groove behind the medialmalleolus d. Lateral to the extensor tendon of the greattoe ANS: D The dorsalis pedis artery is located on the dorsum of the foot. The nurse should palpate just lateral to and parallelwiththeextensortendonofthebigtoe.Thepoplitealarteryispalpatedbehindtheknee.Theposterior tibialpulseispalpatedinthegroovebetweenthemalleolusandtheAchillestendon.Nopulseispalpatedatthe lateralmalleolus. DIF:CognitiveLevel:Understanding(Comprehension) MSC: Client Needs:General 4. A65-year-oldpatientisexperiencingpaininhisleftcalfwhenheexercisesthatdisappearsafterrestingfor afewminutes.Thenurserecognizesthatthisdescriptionismostconsistentwith the leftleg. a. Venous obstructionof b. Claudication due to venous abnormalitiesin c. Ischemia caused by a partial blockage of an arterysupplying d. Ischemia caused by the complete blockage of an arterysupplying ANS: C NURSINGTB.COM Ischemiaisadeficientsupplyofoxygenatedarterialbloodtoatissue.Apartialblockagecreatesaninsufficien supply,andtheischemiamaybeapparentonlyduringexercisewhenoxygenneedsincrease. DIF: Cognitive Level: Analyzing (Analysis) MSC: Client Needs: Physiologic Integrity: Physiologic Adaptation 5. Thenurseisreviewingvenousbloodflowpatterns.Whichofthesestatementsbestdescribesthe mechanism(s) by which venous blood returns to theheart? a. Intraluminal valves ensure unidirectional flow toward theheart. b. Contracting skeletal muscles milk blood distally toward theveins. c. High-pressure system of the heart helps facilitate venousreturn. d. Increasedthoracicpressureanddecreasedabdominalpressurefacilitatevenousreturntotheheart. ANS: A Bloodmovesthroughtheveinsby(1)contractingskeletalmusclesthatproximallymilktheblood;(2)pressure gradients caused by breathing, during which inspiration makes the thoracic pressure decrease and the abdominal pressure increase; and (3) the intraluminal valves, which ensure unidirectional flow toward the heart. DIF:CognitiveLevel:Understanding(Comprehension) MSC: Client Needs:General 6. Whichvein(s)is(are)responsibleformostofthevenousreturninthearm? a. Deep b. Ulnar c. Subclavian d. Superficial ANS:D Thesuperficialveinsofthearmsareinthesubcutaneoustissueandareresponsibleformostofthevenous return. DIF:CognitiveLevel:Remembering(Knowledge) MSC: Client Needs:General 7. A70-year-oldpatientisscheduledforopen-heartsurgery.Thesurgeonplanstousethegreatsaphenousvein for the coronary bypass grafts. The patient asks, What happens to my circulation when this vein is removed? The nurse shouldreply: NURSINGTB.COM a. Venous insufficiency is a common problem after this type ofsurgery. b. Oh,youhavelotsofveinsyouwontevennoticethatithasbeenremoved. c. Youwillprobablyexperiencedecreasedcirculationaftertheveinisremoved. d. Thisveincanberemovedwithoutharmingyourcirculationbecausethedeeperveinsinyourleg are in goodcondition. ANS: D Aslongasthefemoralandpoplitealveinsremainintact,thesuperficialveinscanbeexcisedwithoutharming circulation. The other responses are notcorrect. DIF: Cognitive Level: Analyzing (Analysis) MSC: Client Needs: Physiologic Integrity: Physiologic Adaptation 8. Thenurseisreviewingtheriskfactorsforvenousdisease.Whichofthesesituationsbestdescribesaperson at highest risk for the development of venousdisease? a. Woman in her second month ofpregnancy b. Person who has been on bed rest for 4days c. Person with a 30-year, 1 pack per day smokinghabit d. Olderadulttakinganticoagulantmedication ANS:B Peoplewhoundergoprolongedstanding,sitting,orbedrestareatriskforvenousdisease.Hypercoagulable (notanticoagulated)statesandvein-walltraumaalsoplacethepersonatriskforvenousdisease.Obesityand the late months of pregnancy are also riskfactors. DIF: Cognitive Level: Applying (Application) MSC: Client Needs: Physiologic Integrity: Reduction of Risk Potential 9. Thenurseisteachingareviewclassonthelymphaticsystem.Aparticipantshowscorrectunderstandingof the material with whichstatement? a. Lymph flow is propelled by the contraction of theheart. b. The flow of lymph is slow, compared with that of theblood. c. Oneofthefunctionsofthelymphistoabsorblipidsfromthebiliarytract. d. Lymphvesselshavenovalves;therefore,lymphfluidflowsfreelyfromthetissuespacesintothe bloodstream. NURSINGTB.COM ANS: B The flow of lymph is slow, compared with flow of the blood. Lymph flow is not propelled by the heart but ratherbycontractingskeletalmuscles,pressurechangessecondarytobreathing,andcontractionofthevessel walls.Lymphdoesnotabsorblipidsfromthebiliarytract.Thevesselsdohavevalves;therefore,flowisone way from the tissue spaces to thebloodstream. DIF: Cognitive Level: Understanding (Comprehension) MSC: Client Needs: Physiologic Integrity: Physiologic Adaptation 10. Whenperforminganassessmentofapatient,thenursenoticesthepresenceofanenlargedright epitrochlear lymph node. What should the nurse donext? a. Assess the patients abdomen, and notice anytenderness. b. Carefullyassessthecervicallymphnodes,andcheckforanyenlargement. c. Askadditionalhealthhistoryquestionsregardinganyrecentearinfectionsorsorethroats. d. Examinethepatientslowerarmandhand,andcheckforthepresenceofinfectionorlesions. ANS: D Theepitrochlearnodesarelocatedintheantecubitalfossaanddrainthehandandlowerarm.Theotheractions are not correct for this assessmentfinding. DIF: Cognitive Level: Applying (Application) MSC: Client Needs: Safe and Effective Care Environment: Management of Care 11. A35-year-oldmanisseenintheclinicforaninfectioninhisleftfoot.Whichofthesefindingsshouldthe nurse expect to see during an assessment of thispatient? a. Hard and fixed cervicalnodes b. Enlarged and tender inguinalnodes c. Bilateral enlargement of the poplitealnodes d. Pelletlikenodesinthesupraclavicularregion ANS:B Theinguinalnodesinthegroindrainmostofthelymphofthelowerextremities.Withlocalinflammation,the nodes in that area become swollen andtender. DIF:CognitiveLevel:Applying(Application) MSC: Client Needs: Physiologic Integrity: Physiologic Adaptation NURSINGTB.COM 12. Thenurseisexaminingthelymphaticsystemofahealthy3-year-oldchild.Whichfindingshouldthenurse expect? a. Excessive swelling of the lymphnodes b. Presence of palpable lymphnodes c. Nopalpablenodesbecauseoftheimmatureimmunesystemofachild d. Fewernumbersandasmallersizeoflymphnodescomparedwiththoseofanadult ANS:B Lymphnodesarerelativelylargeinchildren,andthesuperficialonesareoftenpalpableevenwhenthechildis healthy. DIF: Cognitive Level: Applying (Application) MSC: Client Needs: Health Promotion and Maintenance 13. Duringanassessmentofanolderadult,thenurseshouldexpecttonoticewhichfindingasanormal physiologic change associated with the agingprocess? a. Hormonalchangescausingvasodilationandaresultingdropinbloodpressure b. Progressiveatrophyoftheintramuscularcalfveins,causingvenousinsufficiency c. Peripheralbloodvesselsgrowingmorerigidwithage,producingariseinsystolicbloodpressure d. Narrowingoftheinferiorvenacava,causinglowbloodflowandincreasesinvenouspressure resulting invaricosities ANS:C Peripheralbloodvesselsgrowmorerigidwithage,resultinginariseinsystolicbloodpressure.Aging producesprogressiveenlargementoftheintramuscularcalfveins,notatrophy.Theotheroptionsarenot correct. DIF:CognitiveLevel:Understanding(Comprehension) MSC:ClientNeeds:HealthPromotionandMaintenance 14. A67-year-oldpatientstatesthatherecentlybegantohavepaininhisleftcalfwhenclimbingthe10stairs to his apartment. This pain is relieved by sitting for approximately 2 minutes; then he is able to resume his activities. The nurse interprets that this patient is most likelyexperiencing: a. Claudication. b. Soremuscles. c. Musclecramps. d. Venousinsufficiency. NURSINGTB.COM ANS: A Intermittentclaudicationfeelslikeacrampandisusuallyrelievedbyrestwithin2minutes.Theother responses are notcorrect. DIF: Cognitive Level: Analyzing (Analysis) MSC: Client Needs: Physiologic Integrity: Physiologic Adaptation 15. Apatientcomplainsoflegpainthatwakeshimatnight.Hestatesthathehasbeenhavingproblemswith his legs. He has pain in his legs when they are elevated that disappears when he dangles them. He recently noticedasoreontheinneraspectoftherightankle.Onthebasisofthishealthhistoryinformation,thenurse interprets that the patient is most likelyexperiencing: a. Pain related to lymphaticabnormalities. b. Problemsrelatedtoarterialinsufficiency. c. Problemsrelatedtovenousinsufficiency. d. Pain related to musculoskeletalabnormalities. ANS: B Night leg pain is common in aging adults and may indicate the ischemic rest pain of peripheral vascular disease.Alterationsinarterialcirculationcausepainthatbecomesworsewithlegelevationandiseasedwhen the extremity isdangled. DIF: Cognitive Level: Analyzing (Analysis) MSC: Client Needs: Physiologic Integrity: Physiologic Adaptation 16. Duringanassessment,thenurseusestheprofilesigntodetect: a. Pittingedema. b. Earlyclubbing. c. Symmetry of thefingers. d. Insufficient capillaryrefill. ANS: B Thenurseshouldusetheprofilesign(viewingthefingerfromtheside)todetectearlyclubbing. DIF: Cognitive Level: Understanding(Comprehension) NURSINGTB.COM MSC: Client Needs: Physiologic Integrity: Physiologic Adaptation 17. Thenurseisperforminganassessmentonanadult.Theadultsvitalsignsarenormal,andcapillaryrefill time is 5 seconds. What should the nurse donext? a. Ask the patient about a history offrostbite. b. Suspect that the patient has venousinsufficiency. c. Consider this a delayed capillary refill time, and investigatefurther. d. Considerthisanormalcapillaryrefilltimethatrequiresnofurtherassessment. ANS: C Normalcapillaryrefilltimeislessthan1to2seconds.Thefollowingconditionscanskewthefindings:acool room,decreasedbodytemperature,cigarettesmoking,peripheraledema,andanemia. DIF: Cognitive Level: Analyzing (Analysis) MSC: Client Needs: Health Promotion and Maintenance 18. Whenassessingapatient,thenursenotesthattheleftfemoralpulseasdiminished,1+/4+.Whatshouldthe nurse donext? a. Document thefinding. b. Auscultate the site for abruit. c. Check for calfpain. d. Check capillary refill in thetoes. ANS: B Ifapulseisweakordiminishedatthefemoralsite,thenthenurseshouldauscultateforabruit.Thepresenceo abruit,orturbulentbloodflow,indicatespartialocclusion.Theotherresponsesarenotcorrect. DIF: Cognitive Level: Analyzing (Analysis) MSC: Client Needs: Safe and Effective Care Environment: Management of Care 19. Whenperformingaperipheralvascularassessmentonapatient,thenurseisunabletopalpatetheulnar pulses.Thepatientsskiniswarmandcapillaryrefilltimeisnormal.Next,thenurseshould: a. Check for the presence ofclaudication. b. Refer the individual for furtherevaluation. c. Considerthisfindingasnormal,anNdUpRrSoIcNeeGdTwBi.CthOtMheperipheralvascularevaluation. d. Askthepatientifheorshehasexperiencedanyunusualcrampingortinglinginthearm. ANS: C Palpatingtheulnarpulsesisnotusuallynecessary.Theulnarpulsesarenotoftenpalpableinthenormal person. The other responses are notcorrect. DIF: Cognitive Level: Analyzing (Analysis) MSC: Client Needs: Safe and Effective Care Environment: Management of Care 20. Thenurseisassessingthepulsesofapatientwhohasbeenadmittedforuntreatedhyperthyroidism.The nurse should expect tofinda(n) pulse. a. Normal b. Absent c. Bounding d. Weak,thready ANS:C A full, bounding pulse occurs with hyperkinetic states (e.g., exercise, anxiety, fever), anemia, and hyperthyroidism.Anabsentpulseoccurswithocclusion.Weak,threadypulsesoccurwithshockandperiphera arterydisease. DIF: Cognitive Level: Understanding (Comprehension) MSC: Client Needs: Physiologic Integrity: Physiologic Adaptation 21. ThenurseispreparingtoperformamodifiedAllentest.Whichisanappropriatereasonforthistest? a. To measure the rate of lymphaticdrainage b. Toevaluatetheadequacyofcapillarypatencybeforevenousblooddraws c. Toevaluatetheadequacyofcollateralcirculationbeforecannulatingtheradialartery d. Toevaluatethevenousrefillratethatoccursaftertheulnarandradialarteriesaretemporarily occluded ANS: C AmodifiedAllentestisusedtoevaluatetheadequacyofcollateralcirculationbeforetheradialarteryis cannulated.TheotherresponsesarenotreasonsforamodifiedAllentest. DIF: Cognitive Level: Understanding (Comprehension) MSC:ClientNeeds:PhysiologicIntegrity:PhNysUioRlSoIgNicGATBda.CpOtaMtion 22. Apatienthasbeendiagnosedwithvenousstasis.Whichofthesefindingswouldthenursemostlikely observe? a. Unilateral coolfoot b. Thin, shiny, atrophicskin c. Pallor of the toes and cyanosis of the nailbeds d. Brownishdiscolorationtotheskinofthelowerleg ANS:D A brown discoloration occurs with chronic venous stasis as a result of hemosiderin deposits (a by-product of redbloodcelldegradation).Pallor,cyanosis,atrophicskin,andunilateralcoolnessareallsignsassociatedwith arterialproblems. DIF: Cognitive Level: Applying (Application) MSC: Client Needs: Physiologic Integrity: Physiologic Adaptation 23. Thenurseisattemptingtoassessthefemoralpulseinapatientwhoisobese.Whichoftheseactionswould be mostappropriate? a. The patient is asked to assume a proneposition. b. Thepatientisaskedtobendhisorherkneestothesideinafroglikeposition. c. Thenursefirmlypressesagainstthebonewiththepatientinasemi-Fowlerposition. d. Thenurselistenswithastethoscopeforpulsations;palpatingthepulseinanobesepersonis extremelydifficult. ANS: B Tohelpexposethefemoralarea,particularlyinobesepeople,thenurseshouldaskthepersontobendhisor her knees to the side in a froglikeposition. DIF: Cognitive Level: Applying (Application) MSC: Client Needs: Safe and Effective Care Environment: Management of Care 24. Whenauscultatingoverapatientsfemoralarteries,thenursenoticesthepresenceofabruitontheleftside. The nurse knows thatbruits: a. Are often associated with venousdisease. b. Occur in the presence oflymphadenopathy. NURSINGTB.COM c. In the femoral arteries are caused by hypermetabolicstates. d. Occur with turbulent blood flow, indicating partialocclusion. ANS: D Abruitoccurswithturbulentbloodflowandindicatespartialocclusionoftheartery.Theotherresponsesare notcorrect. DIF: Cognitive Level: Understanding (Comprehension) MSC: Client Needs: Safe and Effective Care Environment: Management of Care 25. Howshouldthenursedocumentmild,slightpittingedematheanklesofapregnantpatient? a. 1+/0-4+ b. 3+/0-4+ c. 4+/0-4+ d. Brawny edema ANS: A Ifpittingedemaispresent,thenthenurseshouldgradeitonascaleof1+(mild)to4+(severe).Brawnyedema appears as nonpitting edema and feels hard to thetouch. DIF: Cognitive Level: Applying (Application) MSC: Client Needs: Safe and Effective Care Environment: Management of Care 26. Apatienthashard,nonpittingedemaoftheleftlowerlegandankle.Therightleghasnoedema.Basedon these findings, the nurse recallsthat: a. Nonpitting, hard edema occurs with lymphaticobstruction. b. Alterations in arterial function will causeedema. c. Phlebitis of a superficial vein will cause bilateraledema. d. Long-standing arterial obstruction will cause pittingedema. ANS: A Unilateraledemaoccurswithocclusionofadeepveinandwithunilaterallymphaticobstruction.Withthese factors,theedemaisnonpittingandfeelshardtothetouch(brawnyedema). DIF: Cognitive Level: Applying (Application) MSC: Client Needs: Physiologic Integrity: Physiologic Adaptation NURSINGTB.COM 27. When assessing a patients pulse, the nurse notes that the amplitude is weaker during inspiration and strongerduringexpiration.Whenthenursemeasuresthebloodpressure,thereadingdecreases20mmHg duringinspirationandincreaseswithexpiration.Thispatientisexperiencingpulsus: a. Alternans. b. Bisferiens. c. Bigeminus. d. Paradoxus. ANS: D Inpulsusparadoxus,beatshaveweakeramplitudewithinspirationandstrongeramplitudewithexpirationand isbestdeterminedduringbloodpressuremeasurement;readingdecreases(>10mmHg)duringinspirationand increases withexpiration. DIF: Cognitive Level: Analyzing (Analysis) MSC: Client Needs: Safe and Effective Care Environment: Management of Care 28. Duringanassessment,thenursehaselevatedapatientslegs12inchesoffthetableandhashadhimwag hisfeettodrainoffvenousblood.Afterhelpinghimsitupanddanglehislegsoverthesideofthetable,the nurse should expect that a normal finding at this point wouldbe: a. Significant elevationalpallor. b. Venous filling within 15seconds. c. No change in the coloration of theskin. d. Colorreturningtothefeetwithin20secondsofassumingasittingposition. ANS: B Inthistest,itnormallytakes10secondsorlessforthecolortoreturntothefeetand15secondsfortheveins of the feet to fill. Significant elevational pallor, as well as delayed venous filling, occurs with arterial insufficiency. DIF: Cognitive Level: Applying (Application) MSC: Client Needs: Safe and Effective Care Environment: Management of Care 29. Duringavisittotheclinic,awomaninherseventhmonthofpregnancycomplainsthatherlegsfeelheavy inthecalfandthatsheoftenhasfootcrampsatnight.Thenursenoticesthatthepatienthasdilated,tortuous veinsapparentinherlowerlegs.Whichconditionisreflectedbythesefindings? a. Deep-veinthrombophlebitis b. Varicoseveins c. Lymphedema d. Raynaudphenomenon NURSINGTB.COM ANS: B Superficialvaricoseveinsarecausedbyincompetentdistantvalvesintheveins,whichresultsintherefluxof blood,producingdilated,tortuousveins.Varicoseveinsaremorecommoninwomen,andpregnancycanalso beacause.Symptomsincludeaching,heavinessinthecalf,easyfatigability,andnightlegorfootcramps. Dilated,tortuousveinsareobservedonassessment. DIF: Cognitive Level: Applying(Application) MSC: Client Needs: Physiologic Integrity: Physiologic Adaptation 30. Duringanassessment,thenursenoticesthatapatientsleftarmisswollenfromtheshoulderdowntothe fingers,withnonpittingbrawnyedema.Therightarmisnormal.Thepatienthadaleft-sidedmastectomy1 year ago. The nurse suspects whichproblem? a. Venousstasis b. Lymphedema c. Arteriosclerosis d. Deep-veinthrombosis ANS: B Lymphedemaafterbreastcancercausesunilateralswellingandnonpittingbrawnyedema,withoverlyingskin indurated. It is caused by the removal of lymph nodes with breast surgery or damage to lymph nodes and channelswithradiationtherapyforbreastcancer,andlymphedemacanimpededrainageoflymph.Theother responses are notcorrect. DIF: Cognitive Level: Applying (Application) MSC: Client Needs: Safe and Effective Care Environment: Management of Care 31. Thenurseispreparingtoassesstheankle-brachialindex(ABI)ofapatient.Whichstatementaboutthe ABI istrue? a. Normal ABI indices are from 0.5 to1.0. b. Normal ankle pressure is slightly lower than the brachialpressure. c. TheABIisareliablemeasurementofperipheralvasculardiseaseinindividualswithdiabetes. d. AnABIof0.9to0.7indicatesthepresenceofperipheralvasculardiseaseandmildclaudication. ANS: D NURSINGTB.COM Use of the Doppler stethoscope is a noninvasive way to determine the extent of peripheral vascular disease. The normal ankle pressure is slightly greater than or equal to the brachial pressure. An ABI of 0.9 to 0.7 indicatesthepresenceofperipheralvasculardiseaseandmildclaudication.TheABIislessreliableinpatients withdiabetesmellitusbecauseofclaudication,whichmakesthearteriesnoncompressibleandmaygiveafalse high-anklepressure. DIF: Cognitive Level: Applying (Application) MSC: Client Needs: Safe and Effective Care Environment: Management of Care 32. Thenurseisperformingawell-childcheckupona5-year-oldboy.Hehasnocurrentconditionthatwould leadthenursetosuspectanillness.Hishealthhistoryisunremarkable,andhereceivedimmunizations1week ago.Whichofthesefindingsshouldbeconsiderednormalinthispatient? a. Enlarged, warm, and tendernodes b. Lymphadenopathy of the cervicalnodes c. Palpable firm, small, shotty, mobile, and nontender lymphnodes d. Firm,rubbery,andlargenodes,somewhatfixedtotheunderlyingtissue ANS:C Palpable lymph nodes are often normal in children and infants. They are small, firm, shotty, mobile, and nontender.Vaccinationscanproducelymphadenopathy.Enlarged,warm,andtendernodesindicateacurrent infection. DIF: Cognitive Level: Analyzing (Analysis) MSC: Client Needs: Health Promotion and Maintenance 33. WhenusingaDopplerultrasonicstethoscope,thenurserecognizesvenousflowwhenwhichsoundis heard? a. Low hummingsound b. Regular lub, dubpattern c. Swishing, whooshingsound d. Steady,even,flowingsound ANS:C WhenusingtheDopplerultrasonicstethoscope,thepulsesiteisfoundwhenonehearsaswishing,whooshing sound. DIF: Cognitive Level: Understanding (Comprehension) MSC:ClientNeeds:SafeandEffectiveCareENnUvRirSoInNmGeTnBt:.CMOaMnagementofCare 34. Thenurseisdescribingaweak,threadypulseonthedocumentationflowsheet.Whichstatementis correct? a. Is easily palpable; pounds under thefingertips. b. Has greater than normal force, then suddenlycollapses. c. Ishardtopalpate,mayfadeinandout,andiseasilyobliteratedbypressure. d. Rhythmisregular,butforcevarieswithalternatingbeatsoflargeandsmallamplitude. ANS: C Aweak,threadypulseishardtopalpate,mayfadeinandout,andiseasilyobliteratedbypressure.Itis associated with decreased cardiac output and peripheral arterialdisease. DIF: Cognitive Level: Understanding (Comprehension) MSC: Client Needs: Safe and Effective Care Environment: Management of Care 35. Duringanassessment,apatienttellsthenursethatherfingersoftenchangecolorwhenshegoesoutincold weather. She describes these episodes as her fingers first turning white, then blue, then red with a burning, throbbing pain. The nurse suspects that she isexperiencing: a. Lymphedema. b. Raynauddisease. c. Deep-veinthrombosis. d. Chronic arterialinsufficiency. ANS: B Theconditionwithepisodesofabrupt,progressivetricolorchangesofthefingersinresponsetocold, vibration, or stress is known as Raynauddisease. DIF: Cognitive Level: Analyzing (Analysis) MSC: Client Needs: Physiologic Integrity: Physiologic Adaptation 36. Duringaroutineofficevisit,apatienttakesoffhisshoesandshowsthenursethisawfulsorethatwont heal.Oninspection,thenursenotesa3-cmroundulcerontheleftgreattoe,withapaleischemicbase,well- definededges,andnodrainage.Thenurseshouldassessforothersignsandsymptomsof: a. Varicosities. b. Venous stasisulcer. c. Arterial ischemiculcer. d. Deep-veinthrombophlebitis. NURSINGTB.COM ANS: C Arterialischemiculcersoccuratthetoes,metatarsalheads,heels,andlateralankleandarecharacterizedbya pale ischemic base, well-defined edges, and nobleeding. DIF: Cognitive Level: Analyzing (Analysis) MSC: Client Needs: Physiologic Integrity: Physiologic Adaptation 37. Thenurseisreviewinganassessmentofapatientsperipheralpulsesandnoticesthatthedocumentation statesthattheradialpulsesare2+.Thenurserecognizesthatthisreadingindicateswhattypeofpulse? a. Bounding b. Normal c. Weak d. Absent ANS: B Whendocumentingtheforce,oramplitude,ofpulses,3+indicatesanincreased,full,orboundingpulse,2+ indicatesanormalpulse,1+indicatesaweakpulse,and0indicatesanabsentpulse. DIF: Cognitive Level: Applying (Application) MSC: Client Needs: Safe and Effective Care Environment: Management of Care MULTIPLE RESPONSE 1. Apatientisrecoveringfromseveralhoursoforthopedicsurgery.Duringanassessmentofthepatientslower legs, the nurse will monitor for signs of acute venous symptoms. Signs of acute venous symptoms include which of the following? Select all thatapply. a. Intense, sharp pain, with the deep muscle tender to thetouch b. Aching, tired pain, with a feeling offullness c. Pain that is worse at the end of theday d. Suddenonset e. Warm, red, and swollencalf f. PainthatisrelievedwithelevationNoUfRthSeINleGgTB.COM ANS: A, D, E Signs and symptoms of acute venous problems include pain in the calf that has a sudden onset and that is intenseandsharpwithtendernessinthedeepmusclewhentouched.Thecalfiswarm,red,andswollen.The other options are symptoms of chronic venousproblems. DIF: Cognitive Level: Analyzing (Analysis) MSC: Client Needs: Physiologic Integrity: Reduction of Risk Potential 2. Apatienthasbeenadmittedwithchronicarterialsymptoms.Duringtheassessment,thenurseshouldexpect which findings? Select all thatapply. a. Patient has a history of diabetes and cigarettesmoking. b. Skin of the patient is pale andcool. c. His ankles have two small, weepingulcers. d. Patient works long hours sitting at a computerdesk. e. He states that the pain gets worse whenwalking. f. Patient states that the pain is worse at the end of theday. ANS: A, B, E Patients with chronic arterial symptoms often have a history of smoking and diabetes (among other risk factors).Thepainhasagradualonsetwithexertionandisrelievedwithrestordangling.Theskinappearscool and pale. The other responses reflect chronic venousproblems. DIF: Cognitive Level: Analyzing (Analysis) MSC: Client Needs: Physiologic Integrity: Reduction of Risk Potential NURSINGTB.COM Chapter 22: Abdomen MULTIPLE CHOICE 1. Thenurseispercussingtheseventhrightintercostalspaceatthemidclavicularlineovertheliver.Which sound should the nurse expect tohear? a. Dullness b. Tympany c. Resonance d. Hyperresonance ANS:A Theliverislocatedintherightupperquadrantandwouldelicitadullpercussionnote. DIF: Cognitive Level: Understanding(Comprehension) MSC: Client Needs: Safe and Effective Care Environment: Management of Care 2. Which structure is located in the left lower quadrant of theabdomen? a. Liver b. Duodenum c. Gallbladder d. Sigmoidcolon NURSINGTB.COM ANS: D Thesigmoidcolonislocatedintheleftlowerquadrantoftheabdomen. DIF: Cognitive Level: Remembering(Knowledge) MSC: Client Needs: General 3. Apatientishavingdifficultyswallowingmedicationsandfood.Thenursewoulddocumentthatthispatient has: a. Aphasia. b. Dysphasia. c. Dysphagia. d. Anorexia. ANS: C Dysphagiaisaconditionthatoccurswithdisordersofthethroatoresophagusandresultsindifficulty swallowing.Aphasiaanddysphasiaarespeechdisorders.Anorexiaisalossofappetite. DIF: Cognitive Level: Applying (Application) MSC: Client Needs: Safe and Effective Care Environment: Management of Care 4. Thenursesuspectsthatapatienthasadistendedbladder.Howshouldthenurseassessforthiscondition? a. Percuss and palpate in the lumbarregion. b. Inspect and palpate in the epigastricregion. c. Auscultate and percuss in the inguinalregion. d. Percuss and palpate the midline area above the suprapubicbone. ANS: D Dullpercussionsoundswouldbeelicitedoveradistendedbladder,andthehypogastricareawouldseemfirm topalpation. NURSINGTB.COM DIF: Cognitive Level: Applying (Application) MSC: Client Needs: Safe and Effective Care Environment: Management of Care 5. Thenurseisawarethatonechangethatmayoccurinthegastrointestinalsystemofanagingadultis: a. Increasedsalivation. b. Increased liversize. c. Increased esophagealemptying. d. Decreased gastric acidsecretion. ANS: D Gastricacidsecretiondecreaseswithaging.Asoneages,salivationdecreases,esophagealemptyingisdelayed, and liver sizedecreases. DIF:CognitiveLevel:Understanding(Comprehension) MSC:ClientNeeds:HealthPromotionandMaintenance 6. A22-year-oldmancomestotheclinicforanexaminationafterfallingoffhismotorcycleandlandingonhis leftsideonthehandlebars.Thenursesuspectsthathemayhaveinjuredhisspleen.Whichofthesestatements is trueregarding assessment of the spleen in this situation? a. The spleen can be enlarged as a result oftrauma. b. The spleen is normally felt on routinepalpation. c. Ifanenlargedspleenisnoted,thenthenurseshouldthoroughlypalpatetodetermineitssize. d. Anenlargedspleenshouldnotbepalpatedbecauseitcaneasilyrupture. ANS: D Ifanenlargedspleenisfelt,thenthenurseshouldreferthepersonandshouldnotcontinuetopalpateit.An enlarged spleen is friable and can easily rupture withoverpalpation. DIF: Cognitive Level: Applying (Application) MSC: Client Needs: Physiologic Integrity: Physiologic Adaptation 7. Apatientsabdomenisbulgingandstretchedinappearance.Thenurseshoulddescribethisfindingas: a. Obese. b. Herniated. c. Scaphoid. d. Protuberant. NURSINGTB.COM ANS: D Aprotuberantabdomenisrounded,bulging,andstretched.Ascaphoidabdomencavesinward. DIF: Cognitive Level: Remembering(Knowledge) MSC: Client Needs: Physiologic Integrity: Physiologic Adaptation 8. Thenurseisdescribingascaphoidabdomen.Tothehorizontalplane,ascaphoidcontouroftheabdomen depictsa profile. a. Flat b. Convex c. Bulging d. Concave ANS: D Contourdescribestheprofileoftheabdomenfromtheribmargintothepubicbone;ascaphoidcontourisone that is concave from a horizontalplane. DIF: Cognitive Level: Understanding (Comprehension) MSC: Client Needs: Physiologic Integrity: Physiologic Adaptation 9. Whileexaminingapatient,thenurseobservesabdominalpulsationsbetweenthexiphoidprocessand umbilicus. The nurse would suspect that theseare: a. Pulsations of the renalarteries. b. Pulsations of the inferior venacava. c. Normal abdominal aorticpulsations. d. Increased peristalsis from a bowelobstruction. ANS: C Normally,thepulsationsfromtheaortaareobservedbeneaththeskinintheepigastricarea,particularlyinthin persons who have good muscle wallrelaxation. DIF: Cognitive Level: Applying (Application) NURSINGTB.COM MSC: Client Needs: Safe and Effective Care Environment: Management of Care 10. Apatienthashypoactivebowelsounds.Thenurseknowsthatapotentialcauseofhypoactivebowelsounds is: a. Diarrhea. b. Peritonitis. c. Laxativeuse. d. Gastroenteritis. ANS: B Diminishedorabsentbowelsoundssignaldecreasedmotilityfrominflammationasexhibitedwithperitonitis, with paralytic ileus after abdominal surgery, or with late bowelobstruction. DIF: Cognitive Level: Understanding (Comprehension) MSC: Client Needs: Physiologic Integrity: Physiologic Adaptation 11. Thenurseiswatchinganewgraduatenurseperformauscultationofapatientsabdomen.Whichstatement by the new graduate shows a correct understanding of the reason auscultation precedes percussion and palpation of theabdomen? a. Weneedtodeterminetheareasoftendernessbeforeusingpercussionandpalpation. b. Auscultationpreventsdistortionofbowelsoundsthatmightoccurafterpercussionandpalpation. c. Auscultationallowsthepatientmoretimetorelaxandthereforebemorecomfortablewiththe physicalexamination. d. Auscultationpreventsdistortionofvascularsounds,suchasbruitsandhums,thatmightoccurafter percussion andpalpation. ANS: B Auscultationisperformedfirst(afterinspection)becausepercussionandpalpationcanincreaseperistalsis, which would give a false interpretation of bowelsounds. DIF: Cognitive Level: Applying (Application) MSC: Client Needs: Safe and Effective Care Environment: Management of Care 12. Thenurseislisteningtobowelsounds.Whichofthesestatementsistrueofbowelsounds?Bowelsounds: a. Are usually loud, high-pitched, rushing, and tinklingsounds. b. Are usually high-pitched, gurgling, and irregularsounds. NURSINGTB.COM c. Sound like two pieces of leather being rubbedtogether. d. Originatefromthemovementofairandfluidthroughthelargeintestine. ANS: B Bowelsoundsarehigh-pitched,gurgling,andcascadingsoundsthatirregularlyoccurfrom5to30timesper minute.Theyoriginatefromthemovementofairandfluidthroughthesmallintestine. DIF: Cognitive Level: Understanding (Comprehension) MSC: Client Needs: Safe and Effective Care Environment: Management of Care 13. Thephysiciancommentsthatapatienthasabdominalborborygmi.Thenurseknowsthatthistermrefers to: a. Loud continualhum. b. Peritoneal frictionrub. c. Hypoactive bowelsounds. d. Hyperactive bowelsounds. ANS: D Borborygmiisthetermusedforhyperperistalsiswhenthepersonactuallyfeelshisorherstomachgrowling. DIF: Cognitive Level: Understanding(Comprehension) MSC: Client Needs: Safe and Effective Care Environment: Management of Care 14. Duringanabdominalassessment,thenursewouldconsiderwhichofthesefindingsasnormal? a. Presence of a bruit in the femoralarea b. Tympanic percussion note in the umbilicalregion c. Palpablespleenbetweentheninthandeleventhribsintheleftmidaxillaryline d. Dullpercussionnoteintheleftupperquadrantatthemidclavicularline ANS:B Tympany should predominate in all four quadrants of the abdomen because air in the intestines rises to the surfacewhenthepersonissupine.Vascularbruitsarenotusuallypresent.Normally,thespleenisnotpalpable. Dullnesswouldnotbefoundintheareaoflungresonance(leftupperquadrantatthemidclavicularline). DIF: Cognitive Level: Understanding (Comprehension) MSC:ClientNeeds:SafeandEffectiveCareENnUvRirSoInNmGeTnBt:.CMOaMnagementofCare 15. Thenurseisassessingtheabdomenofapregnantwomanwhoiscomplainingofhavingacidindigestionall thetime.Thenurseknowsthatesophagealrefluxduringpregnancycancause: a. Diarrhea. b. Pyrosis. c. Dysphagia. d. Constipation. ANS: B Pyrosis,orheartburn,iscausedbyesophagealrefluxduringpregnancy.Theotheroptionsarenotcorrect. DIF: Cognitive Level: Applying(Application) MSC: Client Needs: Physiologic Integrity: Physiologic Adaptation 16. Thenurseisperformingpercussionduringanabdominalassessment.Percussionnotesheardduringthe abdominal assessment mayinclude: a. Flatness, resonance, anddullness. b. Resonance, dullness, andtympany. c. Tympany, hyperresonance, anddullness. d. Resonance, hyperresonance, andflatness. ANS: C Percussion notes normally heard during the abdominal assessment may include tympany, which should predominatebecauseairintheintestinesrisestothesurfacewhenthepersonissupine;hyperresonance,which maybepresentwithgaseousdistention;anddullness,whichmaybefoundoveradistendedbladder,adipose tissue, fluid, or amass. DIF: Cognitive Level: Understanding (Comprehension) MSC: Client Needs: Safe and Effective Care Environment: Management of Care 17. Anolderpatienthasbeendiagnosedwithperniciousanemia.Thenurseknowsthatthisconditioncouldbe relatedto: a. Increased gastric acidsecretion. b. Decreased gastric acidsecretion. c. DelayedgastrointestinalemptyingNtiUmReS.INGTB.COM d. Increased gastrointestinal emptyingtime. ANS: B Gastricacidsecretiondecreaseswithagingandmaycauseperniciousanemia(becauseitinterfereswith vitamin B12 absorption), iron-deficiency anemia, and malabsorption ofcalcium. DIF: Cognitive Level: Applying (Application) MSC: Client Needs: Physiologic Integrity: Physiologic Adaptation 18. Apatientiscomplainingofasharppainalongthecostovertebralangles.Thenurseisawarethatthis symptom is most often indicativeof: a. Ovaryinfection. b. Liverenlargement. c. Kidneyinflammation. d. Spleenenlargement. ANS: C Sharppainalongthecostovertebralanglesoccurswithinflammationofthekidneyorparanephricarea.The other options are notcorrect. DIF: Cognitive Level: Applying (Application) MSC: Client Needs: Physiologic Integrity: Physiologic Adaptation 19. Anursenoticesthatapatienthasascites,whichindicatesthepresenceof: a. Fluid. b. Feces. c. Flatus. d. Fibroidtumors. ANS: A Ascitesisfreefluidintheperitonealcavityandoccurswithheartfailure,portalhypertension,cirrhosis, hepatitis, pancreatitis, andcancer. DIF: Cognitive Level: Understanding (Comprehension) MSC: Client Needs: Physiologic Integrity: Physiologic Adaptation 20. ThenurseknowsthatduringanabdominaNlaUsRseSsIsNmGeTnBt,.dCeOeMppalpationisusedtodetermine: a. Bowelmotility. b. Enlargedorgans. c. Superficialtenderness. d. Overall impression of skin surface and superficialmusculature. ANS: B Withdeeppalpation,thenurseshouldnoticethelocation,size,consistency,andmobilityofanypalpable organsandthepresenceofanyabnormalenlargement,tenderness,ormasses. DIF: Cognitive Level: Understanding (Comprehension) MSC: Client Needs: Safe and Effective Care Environment: Management of Care 21. Thenursenoticesthatapatienthashadablack,tarrystoolandrecallsthatapossiblecausewouldbe: a. Gallbladderdisease. b. Overuse oflaxatives. c. Gastrointestinalbleeding. d. Localized bleeding around theanus. ANS: C Blackstoolsmaybetarryasaresultofoccultblood(melena)fromgastrointestinalbleeding.Redbloodin stools occurs with localized bleeding around theanus. DIF: Cognitive Level: Understanding (Comprehension) MSC: Client Needs: Physiologic Integrity: Physiologic Adaptation 22. Duringanabdominalassessment,thenurseelicitstendernessonlightpalpationintherightlowerquadrant. Thenurseinterpretsthatthisfindingcouldindicateadisorderofwhichofthesestructures? a. Spleen b. Sigmoid c. Appendix d. Gallbladder ANS: C NURSINGTB.COM Theappendixislocatedintherightlowerquadrant.Whentheiliopsoasmuscleisinflamed,whichoccurswith aninflamedorperforatedappendix,painisfeltintherightlowerquadrant. DIF: Cognitive Level: Applying (Application) MSC: Client Needs: Physiologic Integrity: Physiologic Adaptation 23. Thenurseisassessingtheabdomenofanolderadult.Whichstatementregardingtheolderadultand abdominal assessment istrue? a. Abdominal tone isincreased. b. Abdominal musculature isthinner. c. Abdominal rigidity with an acute abdominal condition is morecommon. d. Theolderadultwithanacuteabdominalconditioncomplainsmoreaboutpainthantheyounger person. ANS: B In the older adult, the abdominal musculature is thinner and has less tone than that of the younger adult, and abdominalrigiditywithanacuteabdominalconditionislesscommonintheagingperson.Theolderadultwith anacuteabdominalconditionoftencomplainslessaboutpainthantheyoungerperson. DIF: Cognitive Level: Applying(Application) MSC: Client Needs: Health Promotion and Maintenance 24. Duringanassessmentofanewborninfant,thenurserecallsthatpyloricstenosiswouldbeexhibitedby: a. Projectilevomiting. b. Hypoactive bowelactivity. c. Palpable olive-sized mass in the right lowerquadrant. d. Pronounced peristaltic waves crossing from right toleft. ANS: A Significantperistalsis,togetherwithprojectilevomiting,inthenewbornsuggestspyloricstenosis.After feeding,pronouncedperistalticwavescrossfromlefttoright,leadingtoprojectilevomiting.Onecanalso palpate an olive-sized mass in the right upperquadrant. DIF: Cognitive Level: Applying (Application) MSC: Client Needs: Health Promotion and Maintenance 25. ThenurseisreviewingtheassessmentofaNnUaRoSrtIiNcGanTeBu.rCyOsmM.Whichofthesestatementsistrueregarding an aorticaneurysm? a. A bruit isabsent. b. Femoral pulses areincreased. c. A pulsating mass is usuallypresent. d. Most are located below theumbilicus. ANS: C Mostaorticaneurysmsarepalpableduringroutineexaminationandfeellikeapulsatingmass.Abruitwillbe audible,andfemoralpulsesarepresentbutdecreased.Suchaneurysmsarelocatedintheupperabdomenjustto the left ofmidline. DIF: Cognitive Level: Understanding (Comprehension) MSC: Client Needs: Physiologic Integrity: Physiologic Adaptation 26. Duringanabdominalassessment,thenurseisunabletohearbowelsoundsinapatientsabdomen.Before reportingthisfindingassilentbowelsounds,thenurseshouldlistenforatleast: a. 1minute. b. 5minutes. c. 10minutes. d. 2 minutes in eachquadrant. ANS: B Absentbowelsoundsarerare.Thenursemustlistenfor5minutesbeforedecidingthatbowelsoundsare completelyabsent. DIF: Cognitive Level: Applying (Application) MSC: Client Needs: Safe and Effective Care Environment: Management of Care 27. Apatientissuspectedofhavinginflammationofthegallbladder,orcholecystitis.Thenurseshould conduct which of these techniques to assess for thiscondition? a. Obturatortest b. Test for Murphysign c. Assess for reboundtenderness d. Iliopsoas muscletest NURSINGTB.COM ANS: B Normally,palpatingthelivercausesnopain.Inapersonwithinflammationofthegallbladder,orcholecystitis, painoccursasthedescendingliverpushestheinflamedgallbladderontotheexamininghandduringinspiration (Murphytest).Thepersonfeelssharppainandabruptlystopsmidwayduringinspiration. DIF: Cognitive Level: Understanding (Comprehension) MSC: Client Needs: Physiologic Integrity: Physiologic Adaptation 28. Justbeforegoinghome,anewmotherasksthenurseabouttheinfantsumbilicalcord.Whichofthese statements iscorrect? a. It should fall off in 10 to 14days. b. It will soften before it fallsoff. c. It contains two veins and oneartery. d. Skin will cover the area within 1week. ANS: A Atbirth,theumbilicalcordiswhiteandcontainstwoumbilicalarteriesandoneveininsidetheWhartonjelly. Theumbilicalstumpdrieswithinaweek,hardens,andfallsoffin10to14days.Skinwillcovertheareain3 to 4weeks. DIF: Cognitive Level: Applying (Application) MSC: Client Needs: Health Promotion and Maintenance 29. Whichofthesepercussionfindingswouldthenurseexpecttofindinapatientwithalargeamountof ascites? a. Dullness across theabdomen b. Flatness in the right upperquadrant c. Hyperresonance in the left upperquadrant d. Tympanyintherightandleftlowerquadrants ANS:A Alargeamountofasciticfluidproducesadullsoundtopercussion. DIF: Cognitive Level: Understanding(Comprehension) MSC: Client Needs: Physiologic Integrity: Physiologic Adaptation NURSINGTB.COM 30. A40-year-oldmanstatesthathisphysiciantoldhimthathehasahernia.Heasksthenursetoexplainwhat a hernia is. Which response by the nurse isappropriate? a. No need to worry. Most men your age develophernias. b. Aherniaisaloopofbowelprotrudingthroughaweakspotintheabdominalmuscles. c. Aherniaistheresultofprenatalgrowthabnormalitiesthatarejustnowcausingproblems. d. Ill have to have your physician explain this toyou. ANS: B Thenurseshouldexplainthataherniaisaprotrusionoftheabdominalviscerathroughanabnormalopeningin the musclewall. DIF: Cognitive Level: Applying (Application) MSC: Client Needs: Physiologic Integrity: Physiologic Adaptation 31. A45-year-oldmanisintheclinicforaphysicalexamination.Duringtheabdominalassessment,thenurse percussestheabdomenandnoticesanareaofdullnessabovetherightcostalmarginofapproximately11cm. The nurseshould: a. Document the presence ofhepatomegaly. b. Ask additional health history questions regarding his alcoholintake. c. Describethisdullnessasindicativeofanenlargedliver,andreferhimtoaphysician. d. Consider this finding as normal, and proceed with theexamination. ANS: D Aliverspanof10.5cmisthemeanformalesand7cmforfemales.Menandtallerindividualsareattheupper end of this range. Women and shorter individuals are at the lower end of this range. A liver span of 11 cm is within normal limits for thisindividual. DIF: Cognitive Level: Analyzing (Analysis) MSC: Client Needs: Safe and Effective Care Environment: Management of Care 32. Whenpalpatingtheabdomenofa20-year-oldpatient,thenursenoticesthepresenceoftendernessinthe leftupperquadrantwithdeeppalpation.Whichofthesestructuresismostlikelytobeinvolved? a. Spleen b. Sigmoidcolon c. Appendix d. Gallbladder NURSINGTB.COM ANS: A Thespleenislocatedintheleftupperquadrantoftheabdomen.Thegallbladderisintherightupperquadrant, thesigmoidcolonisintheleftlowerquadrant,andtheappendixisintherightlowerquadrant. DIF: Cognitive Level: Applying (Application) MSC: Client Needs: Physiologic Integrity: Physiologic Adaptation 33. Thenurseisreviewingstatisticsforlactoseintolerance.IntheUnitedStates,theincidenceoflactose intolerance is higher in adults of which ethnicgroup? a. Blacks b. Hispanics c. Whites d. Asians ANS: A Arecentstudyfoundestimatesoflactose-intoleranceprevalenceasfollows:19.5%forBlacks,10%for Hispanics, and 7.72% forWhites. DIF:CognitiveLevel:Understanding(Comprehension) MSC:ClientNeeds:HealthPromotionandMaintenance 34. Thenurseisassessingapatientforpossiblepepticulcerdisease.Whichconditionorhistoryoftencauses thisproblem? a. Hypertension b. Streptococcalinfections c. Recurrent constipation with frequent laxativeuse d. Frequentuseofnonsteroidalantiinflammatorydrugs ANS:D Pepticulcerdiseaseoccurswiththefrequentuseofnonsteroidalantiinflammatorydrugs,alcoholuse,smoking and Helicobacter pyloriinfection. DIF: Cognitive Level: Applying (Application) MSC: Client Needs: Physiologic Integrity: Reduction of Risk Potential NURSINGTB.COM 35. Duringreporting,thestudentnursehearsthatapatienthashepatomegalyandrecognizesthatthisterm refersto: a. Enlargedliver. b. Enlargedspleen. c. Distendedbowel. d. Excessivediarrhea. ANS: A Thetermhepatomegalyreferstoanenlargedliver.Thetermsplenomegalyreferstoanenlargedspleen.The other responses are notcorrect. DIF: Cognitive Level: Remembering (Knowledge) MSC: Client Needs: Physiologic Integrity: Physiologic Adaptation 36. Duringanassessment,thenursenoticesthatapatientsumbilicusisenlargedandeverted.Itispositioned midlinewithnochangeinskincolor.Thenurserecognizesthatthepatientmayhavewhichcondition? a. Intra-abdominalbleeding b. Constipation c. Umbilicalhernia d. Abdominaltumor ANS:C Theumbilicusisnormallymidlineandinvertedwithnosignsofdiscoloration.Withanumbilicalhernia,the mass is enlarged and everted. The other responses areincorrect. DIF: Cognitive Level: Applying (Application) MSC: Client Needs: Physiologic Integrity: Physiologic Adaptation 37. Duringanabdominalassessment,thenursetestsforafluidwave.Apositivefluidwavetestoccurswith: a. Splenomegaly. b. Distendedbladder. c. Constipation. d. Ascites. NURSINGTB.COM ANS: D Ifascites(fluidintheabdomen)ispresent,thentheexaminerwillfeelafluidwavewhenassessingthe abdomen.Afluidwaveisnotpresentwithsplenomegaly,adistendedbladder,orconstipation. DIF: Cognitive Level: Applying (Application) MSC: Client Needs: Physiologic Integrity: Physiologic Adaptation 38. Thenurseispreparingtoexamineapatientwhohasbeencomplainingofrightlowerquadrantpain.Which technique is correct during theassessment? The nurse should: a. Examine the tender areafirst. b. Examine the tender arealast. c. Avoid palpating the tenderarea. d. Palpate the tender area first, and then auscultate for bowelsounds. ANS: B Thenurseshouldsavetheexaminationofanyidentifiedtenderareasuntillast.Thismethodavoidspainand the resulting muscle rigidity that would obscure deep palpation later in the examination. Auscultation is performedbeforepercussionandpalpationbecausepercussionandpalpationcanincreaseperistalsis,which would give a false interpretation of bowelsounds. DIF: Cognitive Level: Analyzing (Analysis) MSC: Client Needs: Safe and Effective Care Environment: Management of Care 39. Duringahealthhistory,thepatienttellsthenurse,Ihavepainallthetimeinmystomach.Itsworse2hours after I eat, but it gets better if I eat again! Based on these symptoms, the nurse suspects that the patient has whichcondition? a. Appendicitis b. Gastriculcer c. Duodenalulcer d. Cholecystitis ANS:C Painassociatedwithduodenalulcersoccurs2to3hoursafterameal;itmayrelievedbymorefood.Chronic painassociatedwithgastriculcersusuallyoccursonanemptystomach.Severe,acutepainwouldoccurwith appendicitis andcholecystitis. NURSINGTB.COM DIF: Cognitive Level: Applying (Application) MSC: Client Needs: Safe and Effective Care Environment: Management of Care MULTIPLE RESPONSE 1.Thenursesuspectsthatapatienthasappendicitis.Whichoftheseproceduresareappropriateforusewhen assessing for appendicitis or a perforated appendix? Select all thatapply. a. Test for the Murphysign b. Test for the Blumbergsign c. Test for shiftingdullness d. Perform the iliopsoas muscletest e. Test for fluidwave ANS: B,D TestingfortheBlumbergsign(reboundtenderness)andperformingtheiliopsoasmuscletestshouldbeused when assessing for appendicitis. The Murphy sign is used when assessing for an inflamed gallbladder or cholecystitis.Testingforafluidwaveandshiftingdullnessisperformedwhenassessingforascites. DIF: Cognitive Level: Applying (Application) MSC: Client Needs: Physiologic Integrity: Physiologic Adaptation NURSINGTB.COM Chapter 23: Musculoskeletal System MULTIPLE CHOICE 1. Apatientisbeingassessedforrange-of-jointmovement.Thenurseaskshimtomovehisarmintowardthe center of his body. This movement iscalled: a. Flexion. b. Abduction. c. Adduction. d. Extension. ANS: C Movingalimbtowardthemidlineofthebodyiscalledadduction;movingalimbawayfromthemidlineof thebodyiscalledabduction.Flexionisbendingalimbatajoint;andextensionisstraighteningalimbata joint. DIF: Cognitive Level: Understanding (Comprehension) MSC: Client Needs: Physiologic Integrity: Physiologic Adaptation 2. ApatienttellsthenursethatsheishavingaNhUarRdStIiNmGeTbBri.CngOiMngherhandtohermouthwhensheeatsortries tobrushherteeth.Thenurseknowsthatforhertomoveherhandtohermouth,shemustperformwhich movement? a. Flexion b. Abduction c. Adduction d. Extension ANS:A Flexion,orbendingalimbatajoint,isrequiredtomovethehandtothemouth.Extensionisstraighteninga limbatajoint.Movingalimbtowardthemidlineofthebodyiscalledadduction;abductionismovingalimb away from the midline of thebody. DIF: Cognitive Level: Understanding (Comprehension) MSC: Client Needs: Physiologic Integrity: Physiologic Adaptation 3. The functional units of the musculoskeletal system arethe: a. Joints. b. Bones. c. Muscles. d. Tendons. ANS: A Joints are the functional units of the musculoskeletal system because they permit the mobility needed to performtheactivitiesofdailyliving.Theskeleton(bones)istheframeworkofthebody.Theotheroptionsare notcorrect. DIF:CognitiveLevel:Remembering(Knowledge) MSC: Client Needs:General 4. Whenreviewingthemusculoskeletalsystem,thenurserecallsthathematopoiesistakesplaceinthe: a. Liver. b. Spleen. c. Kidneys. d. Bonemarrow. NURSINGTB.COM ANS: D The musculoskeletal system functions to encase and protect the inner vital organs, to support the body, to produceredbloodcellsinthebonemarrow(hematopoiesis),andtostoreminerals.Theotheroptionsarenot correct. DIF:CognitiveLevel:Remembering(Knowledge) MSC: Client Needs:General 5. Fibrousbandsrunningdirectlyfromonebonetoanotherthatstrengthenthejointandhelpprevent movement in undesirable directions arecalled: a. Bursa. b. Tendons. c. Cartilage. d. Ligaments. ANS: D Fibrous bands running directly from one bone to another that strengthen the joint and help prevent movement inundesirabledirectionsarecalledligaments.Theotheroptionsarenotcorrect. DIF: Cognitive Level: Remembering(Knowledge) MSC: Client Needs: General 6. Thenursenoticesthatawomaninanexerciseclassisunabletojumprope.Thenurseisawarethattojump rope, ones shoulder has to be capableof: a. Inversion. b. Supination. c. Protraction. d. Circumduction. ANS: D Circumductionisdefinedasmovingthearminacirclearoundtheshoulder.Theotheroptionsarenotcorrect. DIF: Cognitive Level: Applying(Application) MSC: Client Needs: Physiologic Integrity: Physiologic Adaptation 7. Thearticulationofthemandibleandthetemporalboneisknownasthe: NURSINGTB.COM a. Intervertebralforamen. b. Condyle of themandible. c. Temporomandibularjoint. d. Zygomatic arch of the temporalbone. ANS: C Thearticulationofthemandibleandthetemporalboneisthetemporomandibularjoint.Theotherresponsesare notcorrect. DIF:CognitiveLevel:Remembering(Knowledge) MSC: Client Needs:General 8. Topalpatethetemporomandibularjoint,thenursesfingersshouldbeplacedinthedepression of the ear. a. Distal to thehelix b. Proximal to thehelix c. Anterior to thetragus d. Posteriortothetragus ANS:C Thetemporomandibularjointcanbefeltinthedepressionanteriortothetragusoftheear.Theotherlocations are notcorrect. DIF: Cognitive Level: Understanding (Comprehension) MSC: Client Needs: Safe and Effective Care Environment: Management of Care 9. Of the 33 vertebrae in the spinal column, thereare: a. 5lumbar. b. 5thoracic. c. 7sacral. d. 12cervical. ANS: A NURSINGTB.COM Thereare7cervical,12thoracic,5lumbar,5sacral,and3to4coccygealvertebraeinthespinalcolumn. DIF: Cognitive Level: Remembering(Knowledge) MSC: Client Needs: General 10. Animaginarylineconnectingthehighestpointoneachiliaccrestwouldcrossthe vertebra. a. Firstsacral b. Fourthlumbar c. Seventhcervical d. Twelfththoracic ANS:B Animaginarylineconnectingthehighestpointoneachiliaccrestcrossesthefourthlumbarvertebra.Theother options are notcorrect. DIF:CognitiveLevel:Remembering(Knowledge) MSC: Client Needs:General 11. Thenurseisexplainingtoapatientthatthereareshockabsorbersinhisbacktocushionthespineandto help it move. The nurse is referring to his: a. Vertebralcolumn. b. Nucleuspulposus. c. Vertebralforamen. d. Intervertebraldisks. ANS: D Intervertebraldisksareelasticfibrocartilaginousplatesthatcushionthespinesimilartoshockabsorbersand helpitmove.Thevertebralcolumnisthespinalcolumnitself.Thenucleuspulposusislocatedinthecenterof eachdisk.Thevertebralforamenisthechannel,oropening,forthespinalcordinthevertebrae. DIF: Cognitive Level: Understanding (Comprehension) MSC: Client Needs: Physiologic Integrity: Physiologic Adaptation 12. Thenurseisprovidingpatienteducationforamanwhohasbeendiagnosedwitharotatorcuffinjury.The nurse knows that a rotator cuff injury involvesthe: a. Nucleuspulposus. b. Articularprocesses. c. Medialepicondyle. d. Glenohumeraljoint. NURSINGTB.COM ANS: D Arotatorcuffinjuryinvolvestheglenohumeraljoint,whichisenclosedbyagroupoffourpowerfulmuscles andtendonsthatsupportandstabilizeit.Thenucleuspulposusislocatedinthecenterofeachintervertebral disk. The articular processes are projections in each vertebral disk that lock onto the next vertebra, thereby stabilizingthespinalcolumn.Themedialepicondyleislocatedattheelbow. DIF: Cognitive Level: Applying (Application) MSC: Client Needs: Physiologic Integrity: Physiologic Adaptation 13. Duringaninterviewthepatientstates,Icanfeelthisbumponthetopofbothofmyshouldersitdoesnthurt butIamcuriousaboutwhatitmightbe.Thenurseshouldtellthepatientthatitishis: a. Subacromialbursa. b. Acromionprocess. c. Glenohumeraljoint. d. Greater tubercle of thehumerus. ANS:B Thebumpofthescapulasacromionprocessisfeltattheverytopoftheshoulder.Theotheroptionsarenot correct. DIF: Cognitive Level: Applying (Application) MSC: Client Needs: Physiologic Integrity: Physiologic Adaptation 14. Thenurseischeckingtherangeofmotioninapatientskneeandknowsthatthekneeiscapableofwhich movement(s)? a. Flexion andextension b. Supination andpronation c. Circumduction d. Inversion andeversion ANS: A NURSINGTB.COM Thekneeisahingejoint,permittingflexionandextensionofthelowerlegonasingleplane.Thekneeisnot capable of the other movementslisted. DIF: Cognitive Level: Understanding (Comprehension) MSC: Client Needs: Physiologic Integrity: Physiologic Adaptation 15. A patient is visiting the clinic for an evaluation of a swollen, painful knuckle. The nurse notices that the knuckleabovehisringonthelefthandisswollenandthatheisunabletoremovehisweddingring.Thisjoint iscalledthe joint. a. Interphalangeal b. Tarsometatarsal c. Metacarpophalangeal d. Tibiotalar ANS:C Thejointlocatedjustabovetheringonthefingeristhemetacarpophalangealjoint.Theinterphalangealjointis located distal to the metacarpophalangeal joint. The tarsometatarsal and tibiotalar joints are found in the foot andankle. DIF: Cognitive Level: Understanding (Comprehension) MSC: Client Needs: Physiologic Integrity: Physiologic Adaptation 16. Thenurseisassessingapatientsischialtuberosity.Topalpatetheischialtuberosity,thenurseknowsthatit is best to have thepatient: a. Standing. b. Flexing thehip. c. Flexing theknee. d. Lying in the supineposition. ANS: B Theischialtuberosityliesunderthegluteusmaximusmuscleandispalpablewhenthehipisflexed.Theother options are notcorrect. DIF: Cognitive Level: Understanding (Comprehension) MSC: Client Needs: Safe and Effective Care Environment: Management of Care 17. Thenurseisexaminingthehipareaofapatientandpalpatesaflatdepressionontheupper,lateralsideof thethighwhenthepatientisstanding.Thenurseinterpretsthisfindingasthe: a. Ischialtuberosity. b. Greatertrochanter. c. Iliaccrest. d. Gluteus maximusmuscle. NURSINGTB.COM ANS: B Thegreatertrochanterofthefemurispalpatedwhenthepersonisstanding,anditappearsasaflatdepression ontheupperlateralsideofthethigh.Theiliaccrestistheupperpartofthehipbone;theischialtuberositylies underthegluteusmaximusmuscleandispalpablewhenthehipisflexed;andthegluteusmuscleispartofthe buttocks. DIF: Cognitive Level: Understanding (Comprehension) MSC: Client Needs: Physiologic Integrity: Physiologic Adaptation 18. The ankle joint is the articulation of the tibia, fibula,and: a. Talus. b. Cuboid. c. Calcaneus. d. Cuneiformbones. ANS: A Theankleortibiotalarjointisthearticulationofthetibia,fibula,andtalus.Theotherboneslistedarefoot bones and not part of the anklejoint. DIF:CognitiveLevel:Remembering(Knowledge) MSC: Client Needs:General 19. Thenurseisexplainingthemechanismofthegrowthoflongbonestoamotherofatoddler.Wheredoes lengthening of the bonesoccur? a. Bursa b. Calcaneus c. Epiphyses d. Tuberosities ANS: C NURSINGTB.COM Lengtheningoccursattheepiphyses,orgrowthplates.Theotheroptionsarenotcorrect. DIF: Cognitive Level: Understanding(Comprehension) MSC: Client Needs: Health Promotion and Maintenance 20. Awomanwhois8monthspregnantcommentsthatshehasnoticedachangeinherpostureandishaving lowerbackpain.Thenursetellsherthatduringpregnancy,womenhaveapostureshifttocompensateforthe enlarging fetus. This shift in posture is knownas: a. Lordosis. b. Scoliosis. c. Ankylosis. d. Kyphosis. ANS: A Lordosis compensates for the enlarging fetus, which would shift the center of balance forward. This shift in balance,inturn,createsastrainonthelowbackmuscles,feltaslowbackpainduringlatepregnancybysome women. Scoliosis is lateral curvature of portions of the spine; ankylosis is extreme flexion of the wrist, as observedwithsevererheumatoidarthritis;andkyphosisisanenhancedthoraciccurvatureofthespine. DIF:CognitiveLevel:Understanding(Comprehension) MSC:ClientNeeds:HealthPromotionandMaintenance 21. An85-year-oldpatientcommentsduringhisannualphysicalexaminationthatheseemstobegetting shorterasheages.Thenurseshouldexplainthatdecreasedheightoccurswithagingbecause: a. Long bones tend to shorten withage. b. The vertebral columnshortens. c. A significant loss of subcutaneous fatoccurs. d. A thickening of the intervertebral disksdevelops. ANS: B Posturalchangesareevidentwithaging;decreasedheightismostnoticeableandisduetoshorteningofthe vertebralcolumn.Longbonesdonotshortenwithage.Intervertebraldisksactuallygetthinnerwithage. Subcutaneousfatisnotlostbutisredistributedtotheabdomenandhips. DIF: Cognitive Level: Applying(Application) MSC: Client Needs: Health Promotion and Maintenance 22. Apatienthasbeendiagnosedwithosteoporosisandasksthenurse,Whatisosteoporosis?Thenurse explains that osteoporosis is defined as: a. Increased bonematrix. b. Loss of bonedensity. c. New, weaker bonegrowth. d. Increasedphagocyticactivity. NURSINGTB.COM ANS: B Afterage40years,alossofbonematrix(resorption)occursmorerapidlythannewboneformation.Thenet effectisagraduallossofbonedensity,orosteoporosis.Theotheroptionsarenotcorrect. DIF: Cognitive Level: Remembering (Knowledge) MSC: Client Needs: Physiologic Integrity: Physiologic Adaptation 23. Thenurseisteachingaclassonpreventingosteoporosistoagroupofperimenopausalwomen.Whichof theseactionsisthebestwaytopreventordelaybonelossinthisgroup? a. TakingcalciumandvitaminDsupplements b. Takingmedicationstopreventosteoporosis c. Performing physical activity, such as fastwalking d. Assessingbonedensityannually ANS:C Physicalactivity,suchasfastwalking,delaysorpreventsbonelossinperimenopausalwomen.Thefasterthe paceofwalking,thehigherthepreventiveeffectisontheriskofhipfracture.Theotheroptionsarenotcorrect DIF: Cognitive Level: Applying (Application) MSC: Client Needs: Health Promotion and Maintenance 24. Ateenagegirlhasarrivedcomplainingofpaininherleftwrist.Shewasplayingbasketballwhenshefell andlandedonherlefthand.Thenurseexaminesherhandandwouldexpectafractureifthegirlcomplainsof a: a. Dullache. b. Deep pain in herwrist. c. Sharp pain that increases withmovement. d. Dull throbbing pain that increases withrest. NURSINGTB.COM ANS:C Afracturecausessharppainthatincreaseswithmovement.Theothertypesofpaindonotoccurwitha fracture. DIF: Cognitive Level: Analyzing (Analysis) MSC: Client Needs: Physiologic Integrity: Physiologic Adaptation 25. Apatientiscomplainingofpaininhisjointsthatisworseinthemorning,betterafterhemovesaroundfor awhile,andthengetsworseagainifhesitsforlongperiods.Thenurseshouldassessforothersignsofwhat problem? a. Tendinitis b. Osteoarthritis c. Rheumatoidarthritis d. Intermittent claudication ANS:C Rheumatoidarthritisisworseinthemorningwhenapersonarises.Movementincreasesmostjointpain,excep thepainwithrheumatoidarthritis,whichdecreaseswithmovement.Theotheroptionsarenotcorrect. DIF: Cognitive Level: Analyzing(Analysis) MSC: Client Needs: Physiologic Integrity: Physiologic Adaptation 26. Apatientstates,IcanhearacrunchingorgratingsoundwhenIkneel.Shealsostatesthatitisverydifficult togetoutofbedinthemorningbecauseofstiffnessandpaininmyjoints.Thenurseshouldassessforsignsof whatproblem? a. Crepitation b. Bonespur c. Loosetendon d. Fluidinthekneejoint ANS:A Crepitation is an audible and palpable crunching or grating that accompanies movement and occurs when articularsurfacesinthejointsareroughened,aswithrheumatoidarthritis.Theotheroptionsarenotcorrect. DIF: Cognitive Level: Analyzing (Analysis) MSC: Client Needs: Physiologic Integrity: Physiologic Adaptation NURSINGTB.COM 27. Apatientisabletoflexhisrightarmforwardwithoutdifficultyorpainbutisunabletoabducthisarm because of pain and muscle spasms. The nurse shouldsuspect: a. Crepitation. b. Rotator cufflesions. c. Dislocatedshoulder. d. Rheumatoidarthritis. ANS: B Rotatorcufflesionsmaylimitrangeofmotionandcausepainandmusclespasmsduringabduction,whereas forward flexion remains fairly normal. The other options are notcorrect. DIF: Cognitive Level: Analyzing (Analysis) MSC: Client Needs: Physiologic Integrity: Physiologic Adaptation 28. Aprofessionaltennisplayercomesintothecliniccomplainingofasoreelbow.Thenursewillassessfor tenderness atthe: a. Olecranonbursa. b. Annularligament. c. Base of theradius. d. Medial and lateralepicondyle. ANS: D Theepicondyles,theheadoftheradius,andthetendonsarecommonsitesofinflammationandlocal tenderness,commonlyreferredtoastenniselbow.Theotherlocationsarenotaffected. DIF: Cognitive Level: Analyzing (Analysis) MSC: Client Needs: Physiologic Integrity: Physiologic Adaptation 29. ThenursesuspectsthatapatienthascarpaltunnelsyndromeandwantstoperformthePhalentest.To perform this test, the nurse should instruct the patientto: a. Dorsiflex thefoot. b. Plantarflex thefoot. c. Holdbothhandsbacktobackwhileflexingthewrists90degreesfor60seconds. d. HyperextendthewristswiththepaNlmURarSsINurGfaTcBe.oCfObMothhandstouching,andwaitfor60seconds. ANS: C ForthePhalentest,thenurseshouldaskthepersontoholdbothhandsbacktobackwhileflexingthewrists90 degrees.Acuteflexionofthewristfor60secondsproducesnosymptomsinthenormalhand.ThePhalentest reproduces numbness and burning in a person with carpal tunnel syndrome. The other actions are not correct when testing for carpal tunnelsyndrome. DIF: Cognitive Level: Applying (Application) MSC: Client Needs: Physiologic Integrity: Physiologic Adaptation 30. An80-year-oldwomanisvisitingtheclinicforacheckup.Shestates,IcantwalkasmuchasIusedto.The nurseisobservingformotordysfunctioninherhipandshouldaskherto: a. Internally rotate her hip while she issitting. b. Abductherhipwhilesheislyingonherback. c. Adductherhipwhilesheislyingonherback. d. Externallyrotateherhipwhilesheisstanding. ANS: B Limitedabductionofthehipwhilesupineisthemostcommonmotiondysfunctionfoundinhipdisease.The other options are notcorrect. DIF: Cognitive Level: Analyzing (Analysis) MSC: Client Needs: Physiologic Integrity: Physiologic Adaptation 31. Thenursehascompletedthemusculoskeletalexaminationofapatientskneeandhasfoundapositivebulge sign. The nurse interprets this finding toindicate: a. Irregular bonymargins. b. Soft-tissue swelling in thejoint. c. Swelling from fluid in theepicondyle. d. Swelling from fluid in the suprapatellarpouch. ANS: D Apositivebulgesignconfirmsthepresenceofswellingcausedbyfluidinthesuprapatellarpouch.Theother options are notcorrect. DIF: Cognitive Level: Analyzing (Analysis) MSC: Client Needs: Physiologic Integrity: Physiologic Adaptation NURSINGTB.COM 32. Duringanexamination,thenurseasksapatienttobendforwardfromthewaistandnoticesthatthepatient has lateral tilting. When his leg is raised straight up, the patient complains of a pain going down his buttock into his leg. The nursesuspects: a. Scoliosis. b. Meniscustear. c. Herniated nucleuspulposus. d. Spasm of paravertebralmuscles. ANS: C Lateraltiltingandsciaticpainwithstraightlegraisingarefindingsthatoccurwithaherniatednucleus pulposus. The other options are notcorrect. DIF: Cognitive Level: Analyzing (Analysis) MSC: Client Needs: Physiologic Integrity: Physiologic Adaptation 33. Thenurseisexamininga3-month-oldinfant.Whilethenurseholdshisorherthumbsontheinfantsinner midthighsandthefingersontheoutsideoftheinfantships,touchingthegreatertrochanter,thenurseadducts thelegsuntilthehisorherthumbstouchandthenabductsthelegsuntiltheinfantskneestouchthetable.The nursedoesnotnoticeanyclunkingsoundsandisconfidenttorecorda: a. Positive Allistest. b. Negative Allistest. c. Positive Ortolanisign. d. Negative Ortolanisign. ANS: D Normally,thismaneuverfeelssmoothandhasnosound.WithapositiveOrtolanisign,however,thenursewill feelandhearaclunk,astheheadofthefemurpopsbackintoplace.ApositiveOrtolanisignalsoreflectship instability.TheAllistestalsotestsforhipdislocationbutisperformedbycomparingleglengths. DIF: Cognitive Level: Analyzing (Analysis) MSC: Client Needs: Physiologic Integrity: Physiologic Adaptation 34. Duringaneonatalexamination,thenursenoticesthatthenewborninfanthassixtoes.Thisfindingis documentedas: a. Unidactyly. b. Syndactyly. c. Polydactyly. d. Multidactyly. NURSINGTB.COM ANS: C Polydactylyisthepresenceofextrafingersortoes.Syndactylyiswebbingbetweenadjacentfingersortoes. The other terms are notcorrect. DIF: Cognitive Level: Understanding (Comprehension) MSC: Client Needs: Physiologic Integrity: Physiologic Adaptation 35. A mother brings her newborn baby boy in for a checkup; she tells the nurse that he does not seem to be movinghisrightarmasmuchashisleftandthatheseemstohavepainwhensheliftshimupunderthearms. The nurse suspects a fractured clavicle and would observefor: a. Negative Allistest. b. Positive Ortolanisign. c. Limited range of motion during the Mororeflex. d. Limited range of motion during Lasguetest. ANS: C Forafracturedclavicle,thenurseshouldobserveforlimitedarmrangeofmotionandunilateralresponseto theMororeflex.Theothertestsarenotappropriateforthistypeoffracture. DIF: Cognitive Level: Analyzing (Analysis) MSC: Client Needs: Health Promotion and Maintenance 36. A40-year-oldmanhascomeintotheclinicwithcomplaintsofextremepaininhistoes.Thenursenotices thathistoesareslightlyswollen,reddened,andwarmtothetouch.Hiscomplaintswouldsuggest: a. Osteoporosis. b. Acutegout. c. Ankylosingspondylitis. d. Degenerative jointdisease. ANS: B Clinical findings for acute gout consist of redness, swelling, heat, and extreme pain like a continuous throbbing.Goutisametabolicdisorderofdisturbedpurinemetabolism,associatedwithelevatedserumuric acid. DIF:CognitiveLevel:Analyzing(Analysis) NURSINGTB.COM MSC: Client Needs: Physiologic Integrity: Physiologic Adaptation 37. Ayoungswimmercomestothesportscliniccomplainingofaverysoreshoulder.Hewasrunningatthe pool,slippedonsomewetconcrete,andtriedtocatchhimselfwithhisoutstretchedhand.Helandedonhis outstretchedhandandhasnotbeenabletomovehisshouldersince.Thenursesuspects: a. Jointeffusion. b. Tearofrotatorcuff. c. Adhesivecapsulitis. d. Dislocatedshoulder. ANS: D Adislocatedshoulderoccurswithtraumainvolvingabduction,extension,andexternalrotation(e.g.,fallingon an outstretched arm or diving into apool). DIF: Cognitive Level: Analyzing (Analysis) MSC: Client Needs: Physiologic Integrity: Physiologic Adaptation 38. A 68-year-old woman has come in for an assessment of her rheumatoid arthritis, and the nurse notices raised,firm,nontendernodulesattheolecranonbursaandalongtheulna.Thesenodulesaremostcommonly diagnosedas: a. Epicondylitis. b. Goutyarthritis. c. Olecranonbursitis. d. Subcutaneousnodules. ANS: D Subcutaneousnodulesareraised,firm,andnontenderandoccurwithrheumatoidarthritisintheolecranon bursa and along the extensor surface of theulna. DIF: Cognitive Level: Analyzing (Analysis) MSC: Client Needs: Physiologic Integrity: Physiologic Adaptation 39. Awomanwhohashadrheumatoidarthritisforyearsisstartingtonoticethatherfingersaredriftingtothe side. The nurse knows that this condition is commonly referred toas: a. Radialdrift. b. Ulnardeviation. c. Swan-neckdeformity. d. Dupuytrencontracture. NURSINGTB.COM ANS: B Fingersdrifttotheulnarsidebecauseofstretchingofthearticularcapsuleandmuscleimbalancecausedby chronic rheumatoid arthritis. A radial drift is notobserved. DIF: Cognitive Level: Applying (Application) MSC: Client Needs: Physiologic Integrity: Physiologic Adaptation 40. Apatientwhohashadrheumatoidarthritisforyearscomestotheclinictoaskaboutchangesinherfingers. The nurse will assess for signs of whatproblems? a. Heberdennodes b. Bouchardnodules c. Swan-neckdeformities d. Dupuytrencontractures ANS: C Changesinthefingerscausedbychronicrheumatoidarthritisincludeswan-neckandboutonnieredeformities. HeberdennodesandBouchardnodulesareassociatedwithosteoarthritis.Dupuytrencontracturesofthedigits occur because of chronic hyperplasia of the palmarfascia. DIF: Cognitive Level: Applying (Application) MSC: Client Needs: Physiologic Integrity: Physiologic Adaptation 41. Apatientsannualphysicalexaminationrevealsalateralcurvatureofthethoracicandlumbarsegmentsof hisspine;however,thiscurvaturedisappearswithforwardbending.Thenurseknowsthatthisabnormalityof the spine iscalled: a. Structuralscoliosis. b. Functionalscoliosis. c. Herniated nucleuspulposus. d. Dislocatedhip. ANS: B NURSINGTB.COM Functional scoliosis is flexible and apparent with standing but disappears with forward bending. Structural scoliosisisfixed;thecurvatureshowsbothwhenstandingandwhenbendingforward.Thesefindingsarenot indicative of a dislocatedhip. DIF: Cognitive Level: Analyzing (Analysis) MSC: Client Needs: Physiologic Integrity: Physiologic Adaptation 42. A14-year-oldboywhohasbeendiagnosedwithOsgood-Schlatterdiseasereportspainfulswellingjust belowthekneeforthepast5months.Whichresponsebythenurseisappropriate? a. If these symptoms persist, you may need arthroscopicsurgery. b. Youareexperiencingdegenerationofyourknee,whichmaynotresolve. c. Yourdiseaseisduetorepeatedstressonthepatellartendon.Itisusuallyself-limited,andyour symptoms should resolve withrest. d. Increasingyouractivityandperformingknee-strengtheningexerciseswillhelpdecreasethe inflammation and maintain mobility in theknee. ANS: C Osgood-Schlatterdiseaseisapainfulswellingofthetibialtuberclejustbelowthekneeandmostlikelydueto repeatedstressonthepatellartendon.Itisusuallyself-limited,occurringduringrapidgrowthandmostoften inboys.Thesymptomsresolvewithrest.Theotherresponsesarenotappropriate. DIF: Cognitive Level: Applying(Application) MSC: Client Needs: Physiologic Integrity: Physiologic Adaptation 43. When assessing muscle strength, the nurse observes that a patient has complete range of motion against gravitywithfullresistance.Whatgradeofmusclestrengthshouldthenurserecordusinga0-to5-pointscale? a. 2 b. 3 c. 4 d. 5 ANS: D Completerangeofmotionagainstgravityisnormalmusclestrengthandisrecordedasgrade5muscle strength. The other options are notcorrect. DIF: Cognitive Level: Applying (Application) MSC: Client Needs: Physiologic Integrity: Physiologic Adaptation 44. Thenurseisexamininga6-month-oldinfaNnUtRanSdINpGlaTcBes.CthOeMinfantsfeetflatonthetableandflexeshis kneesup.Thenursenotesthattherightkneeissignificantlylowerthantheleft.Whichofthesestatementsis true of thisfinding? a. This finding is a positive Allis sign and suggests hipdislocation. b. The infant probably has a dislocated patella on the rightknee. c. ThisfindingisanegativeAllissignandnormalforaninfantofthisage. d. Theinfantshouldreturntotheclinicin2weekstoseeifhisconditionhaschanged. ANS: A FindingonekneesignificantlylowerthantheotherisapositiveAllissignandsuggestshipdislocation. Normally,thetopsofthekneesareatthesameelevation.Theotherstatementsarenotcorrect. DIF: Cognitive Level: Analyzing (Analysis) MSC: Client Needs: Health Promotion and Maintenance 45. Thenurseisassessinga1-week-oldinfantandistestinghismusclestrength.Thenurseliftstheinfantwith handsundertheaxillaeandnoticesthattheinfantstartstoslipbetweenthehands.Thenurseshould: a. Suspect a fracturedclavicle. b. Suspect that the infant may have a deformity of thespine. c. Suspect that the infant may have weakness of the shouldermuscles. d. Concludethatthisisanormalfindingbecausethemusculatureofaninfantatthisageis undeveloped. ANS: C Aninfantwhostartstoslipbetweenthenurseshandsshowsweaknessoftheshouldermuscles.Aninfantwith normalmusclestrengthwedgessecurelybetweenthenurseshands.Theotherresponsesarenotcorrect. DIF: Cognitive Level: Analyzing (Analysis) MSC: Client Needs: Health Promotion and Maintenance 46. Thenurseisexamininga2-month-oldinfantandnoticesasymmetryoftheinfantsglutealfolds.Thenurse should assess for other signs of whatdisorder? a. Fracturedclavicle b. Downsyndrome c. Spinabifida d. Hipdislocation NURSINGTB.COM ANS: D Unequalglutealfoldsmayaccompanyhipdislocationafter2to3monthsofage,butsomeasymmetrymay occurinhealthychildren.Furtherassessmentisneeded.Theotherresponsesarenotcorrect. DIF: Cognitive Level: Applying (Application) MSC: Client Needs: Safe and Effective Care Environment: Management of Care 47. Thenurseshouldusewhichtesttocheckforlargeamountsoffluidaroundthepatella? a. Ballottement b. Tinelsign c. Phalentest d. McMurraytest ANS:A Ballottementofthepatellaisreliablewhenlargeamountsoffluidarepresent.TheTinelsignandthePhalen testareusedtocheckforcarpaltunnelsyndrome.TheMcMurraytestisusedtotestthekneeforatorn meniscus. DIF: Cognitive Level: Understanding (Comprehension) MSC: Client Needs: Safe and Effective Care Environment: Management of Care 48. Apatienttellsthenursethat,AllmylifeIvebeencalledknockknees.Thenurseknowsthatanotherterm for knock kneesis: a. Genuvarum. b. Genuvalgum. c. Pesplanus. d. Metatarsusadductus. ANS: B Genuvalgumisalsoknownasknockkneesandispresentwhenmorethan2.5cmisbetweenthemedial malleoli when the knees aretogether. DIF: Cognitive Level: Understanding (Comprehension) MSC: Client Needs: Physiologic Integrity: Physiologic Adaptation 49. AmanwhohashadgoutforseveralyearsNcUomRSesINtoGTthBe.CclOinMicwithaproblemwithhistoe.Onexamination, the nurse notices the presence of hard, painless nodules over the great toe; one has burst open with a chalky discharge. This finding is knownas: a. Callus. b. Plantarwart. c. Bunion. d. Tophi. ANS: D Tophiarecollectionsofmonosodiumuratecrystalsresultingfromchronicgoutinandaroundthejointthat cause extreme swelling and joint deformity. They appear as hard, painless nodules (tophi) over the metatarsophalangealjointofthefirsttoeandtheysometimesburstwithachalkydischarge. DIF: Cognitive Level: Applying (Application) MSC: Client Needs: Physiologic Integrity: Physiologic Adaptation 50. Whenperformingamusculoskeletalassessment,thenurseknowsthatthecorrectapproachforthe examination shouldbe: a. Proximal todistal. b. Distal toproximal. c. Posterior toanterior. d. Anterior toposterior. ANS: A Themusculoskeletalassessmentshouldbeperformedinanorderlyapproach,headtotoe,proximaltodistal, from the midline outward. The other options are notcorrect. DIF: Cognitive Level: Applying (Application) MSC: Client Needs: Safe and Effective Care Environment: Management of Care MULTIPLE RESPONSE 1.Thenurseisassessingthejointsofawomanwhohasstated,Ihavealongfamilyhistoryofarthritis,andmy jointshurt.Thenursesuspectsthatshehasosteoarthritis.Whichofthesearesymptomsofosteoarthritis?Select all thatapply. a. Symmetric jointinvolvement b. Asymmetricjointinvolvement NURSINGTB.COM c. Pain with motion of affectedjoints d. Affected joints are swollen with hard, bonyprotuberances e. Affectedjointsmayhaveheat,redness,andswelling ANS: B, C,D Inosteoarthritis,asymmetricjointinvolvementcommonlyaffectshands,knees,hips,andlumbarandcervical segmentsofthespine.Affectedjointshavestiffness,swellingwithhardbonyprotuberances,painwithmotion, andlimitationofmotion.Theotheroptionsreflectthesignsofrheumatoidarthritis. DIF: Cognitive Level: Applying (Application) MSC: Client Needs: Physiologic Integrity: Physiologic Adaptation Chapter 24: Neurologic System MULTIPLE CHOICE 1. The two parts of the nervous system arethe: a. Motor andsensory. b. Central andperipheral. c. Peripheral andautonomic. d. Hypothalamus andcerebral. ANS: B Thenervoussystemcanbedividedintotwopartscentralandperipheral.Thecentralnervoussystemincludes thebrainandspinalcord.Theperipheralnervoussystemincludesthe12pairsofcranialnerves(CNs),the31 pairs of spinal nerves, and all of theirbranches. DIF:CognitiveLevel:Remembering(Knowledge) MSC: Client Needs:General 2. Thewifeofa65-year-oldmantellsthenursethatsheisconcernedbecauseshehasnoticedachangeinher husbandspersonalityandabilitytounderstandN.UHReSaIlNsoGcTrBie.CsOveMryeasilyandbecomesangry.Thenurserecalls thatthecerebralloberesponsibleforthesebehaviorsisthe lobe. a. Frontal b. Parietal c. Occipital d. Temporal ANS:A The frontal lobe has areas responsible for personality, behavior, emotions, and intellectual function. The parietallobehasareasresponsibleforsensation;theoccipitallobeisresponsibleforvisualreception;andthe temporal lobe is responsible for hearing, taste, andsmell. DIF: Cognitive Level: Understanding (Comprehension) MSC: Client Needs: Physiologic Integrity: Physiologic Adaptation 3. Which statement concerning the areas of the brain istrue? a. The cerebellum is the center for speech andemotions. b. The hypothalamus controls body temperature and regulatessleep. c. The basal ganglia are responsible for controlling voluntarymovements. d. Motorpathwaysofthespinalcordandbrainstemsynapseinthethalamus. ANS: B The hypothalamus is a vital area with many important functions: body temperature controller, sleep center, anterior and posterior pituitary gland regulator, and coordinator of autonomic nervous system activity and emotional status. The cerebellum controls motor coordination, equilibrium, and balance. The basal ganglia controlautonomicmovementsofthebody.Themotorpathwaysofthespinalcordsynapseinvariousareasof the spinal cord, not in thethalamus. DIF:CognitiveLevel:Understanding(Comprehension) MSC: Client Needs:General 4. Theareaofthenervoussystemthatisresponsibleformediatingreflexesisthe: a. Medulla. b. Cerebellum. c. Spinalcord. NURSINGTB.COM d. Cerebralcortex. ANS: C Thespinalcordisthemainhighwayforascendinganddescendingfibertractsthatconnectthebraintothe spinal nerves; it is responsible for mediatingreflexes. DIF:CognitiveLevel:Remembering(Knowledge) MSC: Client Needs:General 5. Whilegatheringequipmentafteraninjection,anurseaccidentallyreceivedaprickfromanimproperly cappedneedle.Tointerpretthissensation,whichoftheseareasmustbeintact? a. Corticospinal tract, medulla, and basalganglia b. Pyramidal tract, hypothalamus, and sensorycortex c. Lateral spinothalamic tract, thalamus, and sensorycortex d. Anteriorspinothalamictract,basalganglia,andsensorycortex ANS:C Thespinothalamictractcontainssensoryfibersthattransmitthesensationsofpain,temperature,andcrudeor lighttouch.Fiberscarryingpainandtemperaturesensationsascendthelateralspinothalamictract,whereasthe sensations of crude touch form the anterior spinothalamic tract. At the thalamus, the fibers synapse with another sensory neuron, which carries the message to the sensory cortex for full interpretation. The other options are notcorrect. DIF:CognitiveLevel:Applying(Application) MSC: Client Needs:General 6. Apatientwithalackofoxygentohisheartwillhavepaininhischestandpossiblyintheshoulder,arms,or jaw.Thenurseknowsthatthebestexplanationwhythisoccursiswhichoneofthesestatements? a. Aproblemexistswiththesensorycortexanditsabilitytodiscriminatethelocation. b. Thelackofoxygeninhishearthasresultedindecreasedamountofoxygentotheareas experiencing thepain. c. Thesensorycortexdoesnothavetheabilitytolocalizepainintheheart;consequently,thepainis feltelsewhere. d. Alesionhasdevelopedinthedorsalroot,whichispreventingthesensationfrombeingtransmitted normally. ANS: C ThesensorycortexisarrangedinaspecificpaNttUerRnS,IfNorGmTiBn.gCaOcMorrespondingmapofthebody.Painintheright hand is perceived at a specific spot on the map. Some organs, such as the heart, liver, and spleen, are absent from the brain map. Pain originating in these organs is referred because no felt image exists in which to have pain.Painisfeltbyproxy,thatis,byanotherbodypartthatdoeshaveafeltimage.Theotherresponsesarenot correctexplanations. DIF: Cognitive Level: Understanding (Comprehension) MSC: Client Needs: Physiologic Integrity: Basic Care and Comfort 7. Theabilitythathumanshavetoperformveryskilledmovementssuchaswritingiscontrolledbythe: a. Basalganglia. b. Corticospinaltract. c. Spinothalamictract. d. Extrapyramidaltract. ANS: B Corticospinal fibers mediate voluntary movement, particularly very skilled, discrete, and purposeful movements, such as writing. The corticospinal tract, also known as the pyramidal tract, is a newer, higher motorsystemthathumanshavethatpermitsveryskilledandpurposefulmovements.Theotherresponsesare not related to skilledmovements. DIF:CognitiveLevel:Understanding(Comprehension) MSC: Client Needs:General 8. A30-year-oldwomantellsthenursethatshehasbeenveryunsteadyandhashaddifficultyinmaintaining herbalance.Whichareaofthebrainthatisrelatedtothesefindingswouldconcernthenurse? a. Thalamus b. Brainstem c. Cerebellum d. Extrapyramidal tract ANS:C Thecerebellarsystemcoordinatesmovement,maintainsequilibrium,andhelpsmaintainposture.Thethalamus is the primary relay station where sensory pathways of the spinal cord, cerebellum, and brainstem form synapses on their way to the cerebral cortex. The brainstem consists of the midbrain, pons, and medulla and has various functions, especially concerning autonomic centers. The extrapyramidal tract maintains muscle tone for gross automatic movements, such aswalking. DIF: Cognitive Level: Understanding (Comprehension) MSC: Client Needs: Physiologic Integrity: Physiologic Adaptation NURSINGTB.COM 9. Whichofthesestatementsabouttheperipheralnervoussystemiscorrect? a. The CNs enter the brain through the spinalcord. b. Efferentfiberscarrysensoryinputtothecentralnervoussystemthroughthespinalcord. c. Theperipheralnervesareinsidethecentralnervoussystemandcarryimpulsesthroughtheirmotor fibers. d. Theperipheralnervescarryinputtothecentralnervoussystembyafferentfibersandawayfrom the central nervous system by efferentfibers. ANS: D Anerveisabundleoffibersoutsideofthecentralnervoussystem.Theperipheralnervescarryinputtothe centralnervoussystembytheirsensoryafferentfibersanddeliveroutputfromthecentralnervoussystemby theirefferentfibers.Theotherresponsesarenotrelatedtotheperipheralnervoussystem. DIF:CognitiveLevel:Remembering(Knowledge) MSC: Client Needs:General 10. Apatienthasaseveredspinalnerveasaresultoftrauma.Whichstatementistrueinthissituation? a. Becausethereare31pairsofspinalnerves,noeffectresultsifonlyonenerveissevered. b. Thedermatomeservedbythisnervewillnolongerexperienceanysensation. c. Theadjacentspinalnerveswillcontinuetocarrysensationsforthedermatomeservedbythe severednerve. d. Aseveredspinalnervewillonlyaffectmotorfunctionofthepatientbecausespinalnerveshaveno sensorycomponent. ANS: C Adermatomeisacircumscribedskinareathatisprimarilysuppliedfromonespinalcordsegmentthrougha particularspinalnerve.Thedermatomesoverlap,whichisaformofbiologicinsurance;thatis,ifonenerveis severed,thenmostofthesensationscanbetransmittedbythespinalnerveaboveandthespinalnervebelow the severednerve. DIF: Cognitive Level: Applying (Application) MSC: Client Needs: Physiologic Integrity: Physiologic Adaptation 11. A21-year-oldpatienthasaheadinjuryresultingfromtraumaandisunconscious.Therearenoother injuries.Duringtheassessmentwhatwouldthenurseexpecttofindwhentestingthepatientsdeeptendon reflexes? a. Reflexes will benormal. b. Reflexescannotbeelicited. NURSINGTB.COM c. All reflexes will be diminished butpresent. d. Some reflexes will be present, depending on the area ofinjury. ANS: A Areflexisadefensemechanismofthenervoussystem.Itoperatesbelowthelevelofconsciouscontroland permits a quick reaction to potentially painful or damagingsituations. DIF: Cognitive Level: Applying (Application) MSC: Client Needs: Safe and Effective Care Environment: Management of Care 12. Amotherofa1-month-oldinfantasksthenursewhyittakessolongforinfantstolearntorollover.The nurse knows that the reason for thisis: a. A demyelinating process must be occurring with herinfant. b. Myelinisneededtoconducttheimpulses,andtheneuronsofanewbornarenotyetmyelinated. c. Thecerebralcortexisnotfullydeveloped;therefore,controlovermotorfunctiongraduallyoccurs. d. Thespinalcordiscontrollingthemovementbecausethecerebellumisnotyetfullydeveloped. ANS: B Theinfantssensoryandmotordevelopmentproceedsalongwiththegradualacquisitionofmyelin,whichis neededtoconductmostimpulses.Verylittlecorticalcontrolexists,andtheneuronsarenotyetmyelinated. The other responses are notcorrect. DIF: Cognitive Level: Applying (Application) MSC: Client Needs: Health Promotion and Maintenance 13. Duringanassessmentofan80-year-oldpatient,thenursenoticesthefollowing:aninabilitytoidentify vibrations at her ankle and to identify the position of her big toe, a slower and more deliberate gait, and a slightlyimpairedtactilesensation.Allotherneurologicfindingsarenormal.Thenurseshouldinterpretthat these findingsindicate: a. CNdysfunction. b. Lesion in the cerebralcortex. c. Normal changes attributable toaging. d. Demyelination of nerves attributable to alesion. ANS: C NURSINGTB.COM Some aging adults show a slower response to requests, especially for those calling for coordination of movements.Thefindingslistedarenormalintheabsenceofothersignificantabnormalfindings.Theother responses areincorrect. DIF: Cognitive Level: Analyzing (Analysis) MSC: Client Needs: Health Promotion and Maintenance 14. A70-year-oldwomantellsthenursethateverytimeshegetsupinthemorningoraftershesbeensitting, shegetsreallydizzyandfeelslikesheisgoingtofallover.Thenursesbestresponsewouldbe: a. Have you been extremely tiredlately? b. You probably just need to drink moreliquids. c. Ill refer you for a complete neurologicexamination. d. Youneedtogetupslowlywhenyouvebeenlyingdownorsitting. ANS: D Agingisaccompaniedbyaprogressivedecreaseincerebralbloodflow.Insomepeople,thisdecreasecauses dizzinessandalossofbalancewithapositionchange.Theseindividualsneedtobetaughttogetupslowly. The other responses areincorrect. DIF: Cognitive Level: Analyzing (Analysis) MSC: Client Needs: Health Promotion and Maintenance 15. Duringthetakingofthehealthhistory,apatienttellsthenursethatitfeelsliketheroomisspinningaround me. The nurse would document this findingas: a. Vertigo. b. Syncope. c. Dizziness. d. Seizureactivity. ANS: A Truevertigoisrotationalspinningcausedbyaneurologicdysfunctionoraprobleminthevestibularapparatus or the vestibular nuclei in the brainstem. Syncope is a sudden loss of strength or a temporary loss of consciousness.Dizzinessisalightheaded,swimmingsensation.Seizureactivityischaracterizedbyalteredor loss of consciousness, involuntary muscle movements, and sensorydisturbances. DIF: Cognitive Level: Applying (Application) MSC: Client Needs: Physiologic Integrity: Physiologic Adaptation 16. WhentakingthehealthhistoryonapatienNtUwRitShIaNsGeTizBu.rCeOdMisorder,thenurseassesseswhetherthepatient hasanaura.Whichofthesewouldbethebestquestionforobtainingthisinformation? a. Does your muscle tone seem tense orlimp? b. After the seizure, do you spend a lot of timesleeping? c. Do you have any warning sign before your seizurestarts? d. Doyouexperienceanycolorchangeorincontinenceduringtheseizure? ANS: C Auraisasubjectivesensationthatprecedesaseizure;itcouldbeauditory,visual,ormotor.Theother questions do not solicit information about anaura. DIF: Cognitive Level: Applying (Application) MSC: Client Needs: Physiologic Integrity: Physiologic Adaptation 17. Whileobtainingahealthhistoryofa3-month-oldinfantfromthemother,thenurseasksabouttheinfants abilitytosuckandgraspthemothersfinger.Whatisthenurseassessing? a. Reflexes b. Intelligence c. CNs d. Cerebralcortexfunction ANS:A Questionsregardingreflexesincludesuchquestionsas,Whathaveyounoticedabouttheinfantsbehavior,Are the infants sucking and swallowing seem coordinated, and Does the infant grasp your finger? The other responses areincorrect. DIF:CognitiveLevel:Understanding(Comprehension) MSC:ClientNeeds:HealthPromotionandMaintenance 18. Inobtainingahealthhistoryona74-year-oldpatient,thenursenotesthathedrinksalcoholdailyandthat hehasnoticedatremorinhishandsthataffectshisabilitytoholdthings.Withthisinformation,whatresponse should the nursemake? a. Does your family know you are drinking everyday? b. Does the tremor change when you drinkalcohol? c. Well do some tests to see what is causing thetremor. NURSINGTB.COM d. Youreallyshouldntdrinksomuchalcohol;itmaybecausingyourtremor. ANS: B Seniletremorisrelievedbyalcohol,althoughnotarecommendedtreatment.Thenurseshouldassesswhether the person is abusing alcohol in an effort to relieve thetremor. DIF: Cognitive Level: Analyzing (Analysis) MSC: Client Needs: Health Promotion and Maintenance 19. A50-year-oldwomanisintheclinicforweaknessinherleftarmandlegthatshehasnoticedforthepast week. The nurse should perform which type of neurologicexamination? a. Glasgow ComaScale b. Neurologic recheckexamination c. Screening neurologicexamination d. Completeneurologicexamination ANS:D Thenurseshouldperformacompleteneurologicexaminationonanindividualwhohasneurologicconcerns (e.g., headache, weakness, loss of coordination) or who is showing signs of neurologic dysfunction. The GlasgowComaScaleisusedtodefineapersonslevelofconsciousness.Theneurologicrecheckexamination isappropriateforthosewhoaredemonstratingneurologicdeficits.Thescreeningneurologicexaminationis performedonseeminglywellindividualswhohavenosignificantsubjectivefindingsfromthehealthhistory. DIF: Cognitive Level: Applying (Application) MSC: Client Needs: Health Promotion and Maintenance 20. During an assessment of the CNs, the nurse finds the following: asymmetry when the patient smiles or frowns,unevenliftingoftheeyebrows,saggingofthelowereyelids,andescapeofairwhenthenursepresses againsttherightpuffedcheek.ThiswouldindicatedysfunctionofwhichoftheseCNs? a. Motor component of CNIV b. Motor component of CNVII c. Motor and sensory components of CNXI d. MotorcomponentofCNXandsensorycomponentofCNVII ANS:B The findings listed reflect a dysfunction of the motor component of the facial nerve (CN VII). DIF:CognitiveLevel:Analyzing(Analysis) NURSINGTB.COM MSC: Client Needs: Health Promotion and Maintenance 21. ThenurseistestingthefunctionofCNXI.Whichstatementbestdescribestheresponsethenurseshould expect if this nerve is intact? Thepatient: a. Demonstrates the ability to hear normalconversation. b. Sticks out the tongue midline without tremors ordeviation. c. Followsanobjectwithhisorhereyeswithoutnystagmusorstrabismus. d. Moves the head and shoulders against resistance with equalstrength. ANS: D The following normal findings are expected when testing the spinal accessory nerve (CN XI): The patients sternomastoid and trapezius muscles are equal in size; the person can forcibly rotate the head both ways against resistance applied to the side of the chin with equal strength; and the patient can shrug the shoulders againstresistancewithequalstrengthonbothsides.Checkingthepatientsabilitytohearnormalconversation checksthefunctionofCNVIII.HavingthepatientstickoutthetonguechecksthefunctionofCNXII.Testing theeyesfornystagmusorstrabismusisperformedtocheckCNsIII,IV,andVI. DIF: Cognitive Level: Applying (Application) MSC: Client Needs: Health Promotion and Maintenance 22. Duringtheneurologicassessmentofahealthy35-year-oldpatient,thenurseaskshimtorelaxhismuscles completely.Thenursethenmoveseachextremitythroughfullrangeofmotion.Whichoftheseresultswould the nurse expect tofind? a. Firm,rigidresistancetomovement b. Mild,evenresistancetomovement c. Hypotonic muscles as a result of totalrelaxation d. Slightpainwithsomedirectionsofmovement ANS:B Toneisthenormaldegreeoftension(contraction)involuntarilyrelaxedmuscles.Itshowsamildresistanceto passivestretching.Normally,thenursewillnoticeamild,evenresistancetomovement.Theotherresponses are notcorrect. DIF: Cognitive Level: Applying (Application) MSC: Client Needs: Health Promotion and Maintenance 23. Whenthenurseasksa68-year-oldpatienttostandwithhisfeettogetherandarmsathissidewithhiseyes closed,hestartstoswayandmoveshisfeetfartherapart.Thenursewoulddocumentthisfindingas: NURSINGTB.COM a. Ataxia. b. Lack ofcoordination. c. NegativeHomanssign. d. PositiveRombergsign. ANS: D Abnormal findings for the Romberg test include swaying, falling, and a widening base of the feet to avoid falling.ApositiveRombergsignisalossofbalancethatisincreasedbytheclosingoftheeyes.Ataxiaisan uncoordinatedorunsteadygait.Homanssignisusedtotestthelegsfordeep-veinthrombosis. DIF: Cognitive Level: Analyzing (Analysis) MSC: Client Needs: Health Promotion and Maintenance 24. The nurse is performing an assessment on a 29-year-old woman who visits the clinic complaining of alwaysdroppingthingsandfallingdown.Whiletestingrapidalternatingmovements,thenursenoticesthatthe woman is unable to pat both of her knees. Her response is extremely slow and she frequently misses. What should the nursesuspect? a. Vestibulardisease b. Lesion of CNIX c. Dysfunction of thecerebellum d. Inabilitytounderstanddirections ANS:C Whenapersontriestoperformrapid,alternatingmovements,responsesthatareslow,clumsy,andsloppyare indicative of cerebellar disease. The other responses areincorrect. DIF: Cognitive Level: Analyzing (Analysis) MSC: Client Needs: Physiologic Integrity: Physiologic Adaptation 25. During the taking of the health history of a 78-year-old man, his wife states that he occasionally has problemswithshort-termmemorylossandconfusion:Hecantevenrememberhowtobuttonhisshirt.When assessinghissensorysystem,whichactionbythenurseismostappropriate? a. Thenursewouldnottestthesensorysystemaspartoftheexaminationbecausetheresultswould not bevalid. b. Thenursewouldperformthetests,knowingthatmentalstatusdoesnotaffectsensoryability. c. ThenursewouldproceedwithaneNxUpRlaSnIaNtiGonTBo.fCeOaMchtest,makingcertainthatthewife understands. d. Beforetesting,thenursewouldassessthepatientsmentalstatusandabilitytofollowdirections. ANS: D The nurse should ensure the validity of the sensory system testing by making certain that the patient is alert, cooperative,comfortable,andhasanadequateattentionspan.Otherwise,thenursemayobtainmisleadingand invalidresults. DIF: Cognitive Level: Analyzing (Analysis) MSC: Client Needs: Health Promotion and Maintenance 26. Theassessmentofa60-year-oldpatienthastakenlongerthananticipated.Intestinghispainperception, thenursedecidestocompletethetestasquicklyaspossible.Whenthenurseappliesthesharppointofthepin on his arm several times, he is only able to identify these as one very sharp prick. What would be the most accurate explanation forthis? a. The patient has hyperesthesia as a result of the agingprocess. b. This response is most likely the result of the summationeffect. c. Thenursewasprobablynotpokinghardenoughwiththepinintheotherareas. d. Thepatientmostlikelyhasanalgesiainsomeareasofarmandhyperalgesiainothers. ANS: B Atleast2secondsshouldbeallowedtoelapsebetweeneachstimulustoavoidsummation.Withsummation, frequentconsecutivestimuliareperceivedasonestrongstimulus.Theotherresponsesareincorrect. DIF: Cognitive Level: Analyzing (Analysis) MSC: Client Needs: Safe and Effective Care Environment: Management of Care 27. Thenurseisperforminganeurologicassessmentona41-year-oldwomanwithahistoryofdiabetes.When testing her ability to feel the vibrations of a tuning fork, the nurse notices that the patient is unable to feel vibrations on the great toe or ankle bilaterally, but she is able to feel vibrations on both patellae. Given this information, what would the nursesuspect? a. Hyperalgesia b. Hyperesthesia c. Peripheralneuropathy d. Lesion of sensorycortex ANS: C NURSINGTB.COM Loss of vibration sense occurs with peripheral neuropathy (e.g., diabetes and alcoholism). Peripheral neuropathyisworseatthefeetandgraduallyimprovesastheexaminermovesuptheleg,asopposedtoa specificnervelesion,whichhasaclearzoneofdeficitforitsdermatome.Theotherresponsesareincorrect. DIF: Cognitive Level: Analyzing (Analysis) MSC: Client Needs: Physiologic Integrity: Physiologic Adaptation 28. Thenurseplacesakeyinthehandofapatientandheidentifiesitasapenny.Whattermwouldthenurse use to describe thisfinding? a. Extinction b. Astereognosis c. Graphesthesia d. Tactile discrimination ANS:B Stereognosis is the persons ability to recognize objects by feeling their forms, sizes, and weights. Astereognosis is an inability to identify objects correctly, and it occurs in sensory cortex lesions. Tactile discriminationtestsfinetouch.Extinctionteststhepersonsabilitytofeelsensationsonbothsidesofthebody at the samepoint. DIF: Cognitive Level: Applying (Application) MSC: Client Needs: Physiologic Integrity: Physiologic Adaptation 29. Thenurseistestingthedeeptendonreflexesofa30-year-oldwomanwhoisintheclinicforanannual physicalexamination.WhenstrikingtheAchillesheelandquadricepsmuscle,thenurseisunabletoelicita reflex. The nurses next response should beto: a. Ask the patient to lock her fingers andpull. b. Complete the examination, and then test these reflexesagain. c. Refer the patient to a specialist for furthertesting. d. Document these reflexes as 0 on a scale of 0 to4+. ANS: A Sometimesthereflexresponsefailstoappear.Documentingthereflexesasabsentisinappropriatethissoonin the examination. The nurse should try to further encourage relaxation, varying the persons position or increasing the strength of the blow. Reinforcement is another technique to relax the muscles and enhance the response.Thepersonshouldbeaskedtoperformanisometricexerciseinamusclegroupsomewhatawayfrom the one being tested. For example, to enhance a patellar reflex, the person should be asked to lock the fingers together andpull. DIF: Cognitive Level: Applying (Application) NURSINGTB.COM MSC: Client Needs: Health Promotion and Maintenance 30. Inassessinga70-year-oldpatientwhohashadarecentcerebrovascularaccident,thenursenoticesright- sidedweakness.Whatmightthenurseexpecttofindwhentestinghisreflexesontherightside? a. Lack ofreflexes b. Normalreflexes c. Diminishedreflexes d. Hyperactive reflexes ANS:D Hyperreflexiaistheexaggeratedreflexobservedwhenthemonosynapticreflexarcisreleasedfromthe influence of higher cortical levels. This response occurs with upper motor neuron lesions (e.g., a cerebrovascular accident). The other responses areincorrect. DIF: Cognitive Level: Applying (Application) MSC: Client Needs: Physiologic Integrity: Physiologic Adaptation 31. Whenthenurseistestingthetricepsreflex,whatistheexpectedresponse? a. Flexion of thehand b. Pronation of thehand c. Extension of theforearm d. Flexionoftheforearm ANS:C Thenormalresponseofthetricepsreflexisextensionoftheforearm.Thenormalresponseofthebicepsreflex causes flexion of the forearm. The other responses areincorrect. DIF: Cognitive Level: Remembering (Knowledge) MSC: Client Needs: Safe and Effective Care Environment: Management of Care 32. The nurse is testing superficial reflexes on an adult patient. When stroking up the lateral side of the sole andacrosstheballofthefoot,thenursenoticestheplantarflexionofthetoes.Howshouldthenursedocument thisfinding? a. Positive Babinskisign b. Plantar reflexabnormal c. Plantar reflexpresent NURSINGTB.COM d. Plantar reflex 2+ on a scale from 0 to4+ ANS: C With the same instrument, the nurse should draw a light stroke up the lateral side of the sole of the foot and acrosstheballofthefoot,similartoanupside-downJ.Thenormalresponseisplantarflexionofthetoesand sometimesoftheentirefoot.ApositiveBabinskisignisabnormalandoccurswiththeresponseofdorsiflexion ofthebigtoeandfanningofalltoes.Theplantarreflexisnotgradedona0to4+scale. DIF: Cognitive Level: Analyzing (Analysis) MSC: Client Needs: Safe and Effective Care Environment: Management of Care 33. Intheassessmentofa1-month-oldinfant,thenursenoticesalackofresponsetonoiseorstimulation.The motherreportsthatinthelastweekhehasbeensleepingallofthetime,andwhenheisawakeallhedoesis cry. The nurse hears that the infants cries are very high pitched and shrill. What should be the nurses appropriate response to thesefindings? a. Refer the infant for furthertesting. b. Talk with the mother about eatinghabits. c. Do nothing; these are expected findings for an infant thisage. d. Tellthemothertobringthebabybackin1weekforarecheck. ANS: A A high-pitched, shrill cry or cat-sounding screech occurs with central nervous system damage. Lethargy, hyporeactivity,andhyperirritability,aswellastheparentsreportofsignificantchangesinbehaviorallwarrant referral. The other options are not correctresponses. DIF: Cognitive Level: Analyzing (Analysis) MSC: Client Needs: Health Promotion and Maintenance 34. Whichofthesetestswouldthenurseusetocheckthemotorcoordinationofan11-month-oldinfant? a. DenverII b. Stereognosis c. Deep tendonreflexes d. Rapidalternatingmovements ANS:A Toscreengrossandfinemotorcoordination,tNhUeRnuSrIsNeGsThBou.CldOuMsetheDenverIIwithitsage-specific developmentalmilestones.Stereognosistestsapersonsabilitytorecognizeobjectsbyfeelingthemandisnot appropriateforan11-month-oldinfant.Testingthedeeptendonreflexesisnotappropriateforcheckingmotor coordination.Testingrapidalternatingmovementsisappropriatefortestingcoordinationinadults. DIF:CognitiveLevel:Understanding(Comprehension) MSC:ClientNeeds:HealthPromotionandMaintenance 35. Toassesstheheadcontrolofa4-month-oldinfant,thenurseliftsuptheinfantinapronepositionwhile supportinghischest.Thenurselooksforwhatnormalresponse?Theinfant: a. Raises the head, and arches theback. b. Extends the arms, and drops down thehead. c. Flexes the knees and elbows with the backstraight. d. Holds the head at 45 degrees, and keeps the backstraight. ANS: A At3monthsofage,theinfantraisestheheadandarchesthebackasifinaswandive.Thisresponseisthe Landaureflex,whichpersistsuntil1yearsofage.Theotherresponsesareincorrect. DIF: Cognitive Level: Applying (Application) MSC: Client Needs: Health Promotion and Maintenance 36. While assessing a 7-month-old infant, the nurse makes a loud noise and notices the following response: abductionandflexionofthearmsandlegs;fanningofthefingers,andcurlingoftheindexfingerandthumbin aCposition,followedbytheinfantbringinginthearmsandlegstothebody.Whatdoesthenurseknowabout thisresponse? a. This response could indicate brachial nervepalsy. b. This reaction is an expected startle response at thisage. c. Thisreflexshouldhavedisappearedbetween1and4monthsofage. d. Thisresponseisnormalaslongasthemovementsarebilaterallysymmetric. ANS: C TheMororeflexispresentatbirthandusuallydisappearsat1to4months.AbsenceoftheMororeflexinthe newborn or its persistence after 5 months of age indicates severe central nervous system injury. The other responses areincorrect. DIF: Cognitive Level: Analyzing (Analysis) MSC: Client Needs: Health Promotion and Maintenance 37. Totestforgrossmotorskillandcoordinationofa6-year-oldchild,whichofthesetechniqueswouldbe appropriate? Ask the child to: a. Hop on onefoot. b. Stand on hishead. c. Touch his finger to hisnose. d. Makefunnyfacesatthenurse. NURSINGTB.COM ANS: A Normally,achildcanhopononefootandcanbalanceononefootforapproximately5secondsby4yearsof ageandcanbalanceononefootfor8to10secondsat5yearsofage.Childrenenjoyperformingthesetests. Failuretohopafter5yearsofageindicatesincoordinationofgrossmotorskills.Askingthechildtotouchhis orherfingertothenosechecksfinemotorcoordination;andaskingthechildtomakefunnyfacestestsCN VII. Asking a child to stand on his or her head is not appropriate. DIF: Cognitive Level: Applying (Application) MSC: Client Needs: Health Promotion and Maintenance 38. Duringtheassessmentofan80-year-oldpatient,thenursenoticesthathishandsshowtremorswhenhe reachesforsomethingandhisheadisalwaysnodding.Noassociatedrigidityisobservedwithmovement. Which of these statements is mostaccurate? a. These findings are normal, resulting fromaging. b. These findings could be related tohyperthyroidism. c. These findings are the result of Parkinsondisease. d. This patient should be evaluated for a cerebellarlesion. ANS: A Senile tremors occasionally occur. These benign tremors include an intention tremor of the hands, head nodding(asifsayingyesorno),andtongueprotrusion.TremorsassociatedwithParkinsondiseaseinclude rigidity,slowness,andaweaknessofvoluntarymovement.Theotherresponsesareincorrect. DIF: Cognitive Level: Analyzing (Analysis) MSC: Client Needs: Health Promotion and Maintenance 39. Whilethenurseistakingthehistoryofa68-year-oldpatientwhosustainedaheadinjury3daysearlier,he tellsthenursethatheisonacruiseshipandis30yearsold.Thenurseknowsthatthisfindingisindicativeof a(n): a. Great sense ofhumor. b. Uncooperativebehavior. c. Inabilitytounderstandquestions. d. Decreasedlevelofconsciousness. NURSINGTB.COM ANS: D A change in consciousness may be subtle. The nurse should notice any decreasing level of consciousness, disorientation,memoryloss,uncooperativebehavior,orevencomplacencyinapreviouslycombativeperson. The other responses areincorrect. DIF: Cognitive Level: Applying (Application) MSC: Client Needs: Physiologic Integrity: Physiologic Adaptation 40. Thenurseiscaringforapatientwhohasjusthadneurosurgery.Toassessforincreasedintracranial pressure, what would the nurse include in theassessment? a. CNs, motor function, and sensoryfunction b. Deep tendon reflexes, vital signs, and coordinatedmovements c. Levelofconsciousness,motorfunction,pupillaryresponse,andvitalsigns d. Mentalstatus,deeptendonreflexes,sensoryfunction,andpupillaryresponse ANS: C Somehospitalizedpersonshaveheadtraumaoraneurologicdeficitfromasystemicdiseaseprocess.These people must be closely monitored for any improvement or deterioration in neurologic status and for any indication of increasing intracranial pressure. The nurse should use an abbreviation of the neurologic examinationinthefollowingsequence:levelofconsciousness,motorfunction,pupillaryresponse,andvital signs. DIF: Cognitive Level: Applying (Application) MSC: Client Needs: Physiologic Integrity: Physiologic Adaptation 41. Duringanassessmentofa22-year-oldwomanwhosustainedaheadinjuryfromanautomobileaccident4 hoursearlier,thenursenoticesthefollowingchanges:pupilswereequal,butnowtherightpupilisfullydilated andnonreactive,andtheleftpupilis4mmandreactstolight.Whatdothesefindingssuggest? a. Injury to the righteye b. Increased intracranialpressure c. Test inaccuratelyperformed d. Normal response after a headinjury ANS: B NURSINGTB.COM In a person with a brain injury, a sudden, unilateral, dilated, and nonreactive pupil is ominous. CN III runs paralleltothebrainstem.Whenincreasingintracranialpressurepushesdownthebrainstem(uncalherniation), itputspressureonCNIII,causingpupildilation.Theotherresponsesareincorrect. DIF: Cognitive Level: Analyzing (Analysis) MSC: Client Needs: Physiologic Integrity: Physiologic Adaptation 42. A 32-year-old woman tells the nurse that she has noticed very sudden, jerky movements mainly in her handsandarms.Shesays,Theyseemtocomeandgo,primarilywhenIamtryingtodosomething.Ihavent noticed them when Im sleeping. This descriptionsuggests: a. Tics. b. Athetosis. c. Myoclonus. d. Chorea. ANS: D Choreaischaracterizedbysudden,rapid,jerky,purposelessmovementsthatinvolvethelimbs,trunk,orface. Choreaoccursatirregularintervals,andthemovementsareallaccentuatedbyvoluntaryactions. DIF: Cognitive Level: Analyzing (Analysis) MSC: Client Needs: Physiologic Integrity: Physiologic Adaptation 43. Duringanassessmentofa62-year-oldman,thenursenoticesthepatienthasastoopedposture,shuffling walk with short steps, flat facial expression, and pill-rolling finger movements. These findings would be consistentwith: a. Parkinsonism. b. Cerebralpalsy. c. Cerebellarataxia. d. Musculardystrophy. ANS: A Thestoopedposture,shufflingwalk,shortsteps,flatfacialexpression,andpill-rollingfingermovementsare all found inparkinsonism. DIF: Cognitive Level: Analyzing (Analysis) MSC: Client Needs: Physiologic Integrity: Physiologic Adaptation 44. Duringanassessmentofa32-year-oldpatientwitharecentheadinjury,thenursenoticesthatthepatient respondstopainbyextending,adducting,andNinUtRerSnIaNllGyTrBot.CatOinMghisarms.Hispalmspronate,andhislower extremities extend with plantar flexion. Which statement concerning these findings is most accurate? This patientsresponse: a. Indicates a lesion of the cerebralcortex. b. Indicates a completely nonfunctionalbrainstem. c. Is normal and will go away in 24 to 48hours. d. Is a very ominous sign and may indicate brainsteminjury. ANS: D Thesefindingsareallindicativeofdecerebraterigidity,whichisaveryominousconditionandmayindicatea brainsteminjury. DIF: Cognitive Level: Analyzing (Analysis) MSC: Client Needs: Physiologic Integrity: Physiologic Adaptation 45. A78-year-oldmanhasahistoryofacerebrovascularaccident.Thenursenotesthatwhenhewalks,hisleft armisimmobileagainstthebodywithflexionoftheshoulder,elbow,wrist,andfingersandadductionofthe shoulder.Hisleftlegisstiffandextendedandcircumductswitheachstep.Whattypeofgaitdisturbanceisthis individualexperiencing? a. Scissorsgait b. Cerebellarataxia c. Parkinsoniangait d. Spastic hemiparesis ANS:D Withspastichemiparesis,thearmisimmobileagainstthebody.Flexionoftheshoulder,elbow,wrist,and fingersoccurs,andadductionoftheshoulder,whichdoesnotswingfreely,isobserved.Thelegisstiffand extendedandcircumductswitheachstep.Causesofthistypeofgaitincludecerebrovascularaccident. DIF: Cognitive Level: Analyzing (Analysis) MSC: Client Needs: Physiologic Integrity: Physiologic Adaptation 46. Inapersonwithanuppermotorneuronlesionsuchasacerebrovascularaccident,whichofthesephysical assessment findings should the nurseexpect? a. Hyperreflexia b. Fasciculations NURSINGTB.COM c. Loss of muscle tone andflaccidity d. Atrophyandwastingofthemuscles ANS:A Hyperreflexia,diminishedorabsentsuperficialreflexes,andincreasedmuscletoneorspasticitycanbe expectedwithuppermotorneuronlesions.Theotheroptionsreflectalesionoflowermotorneurons. DIF: Cognitive Level: Applying (Application) MSC: Client Needs: Physiologic Integrity: Physiologic Adaptation 47. A59-year-oldpatienthasaherniatedintervertebraldisk.Whichofthefollowingfindingsshouldthenurse expect to see on physical assessment of thisindividual? a. Hyporeflexia b. Increasedmuscletone c. PositiveBabinskisign d. Presence of pathologicreflexes ANS: A With a herniated intervertebral disk or lower motor neuron lesion, loss of tone, flaccidity, atrophy, fasciculations,andhyporeflexiaorareflexiaaredemonstrated.NoBabinskisignorpathologicreflexeswould be observed. The other options reflect a lesion of upper motorneurons. DIF: Cognitive Level: Applying (Application) MSC: Client Needs: Physiologic Integrity: Physiologic Adaptation 48. Apatientisunabletoperformrapidalternatingmovementssuchasrapidlypattingherknees.Thenurse should document this inabilityas: a. Ataxia. b. Astereognosis. c. Presence ofdysdiadochokinesia. d. Loss ofkinesthesia. ANS: C Slowclumsymovementsandtheinabilitytoperformrapidalternatingmovementsoccurwithcerebellar disease.Theconditionistermeddysdiadochokinesia.Ataxiaisanuncoordinatedorunsteadygait. Astereognosisistheinabilitytoidentifyanobjectbyfeelingit.Kinesthesiaisthepersonsabilitytoperceive passivemovementoftheextremitiesorthelosNsUoRfSpIoNsGitiToBn.CseOnMse. DIF: Cognitive Level: Applying (Application) MSC: Client Needs: Physiologic Integrity: Physiologic Adaptation 49. Thenurseknowsthatdeterminingwhetherapersonisorientedtohisorhersurroundingswilltestthe functioning of whichstructure(s)? a. Cerebrum b. Cerebellum c. CNs d. Medullaoblongata ANS:A Thecerebralcortexisresponsibleforthought,memory,reasoning,sensation,andvoluntarymovement.The other structures are not responsible for a persons level ofconsciousness. DIF: Cognitive Level: Understanding (Comprehension) MSC: Client Needs: Safe and Effective Care Environment: Management of Care 50. Duringanexamination,thenursenoticesseverenystagmusinbotheyesofapatient.Whichconclusionby the nurse is correct? Severe nystagmus in botheyes: a. Is a normaloccurrence. b. May indicate disease of the cerebellum orbrainstem. c. Is a sign that the patient is nervous about theexamination. d. Indicatesavisualproblem,andareferraltoanophthalmologistisindicated. ANS: B End-pointnystagmusatanextremelateralgazenormallyoccurs;however,thenurseshouldcarefullyassess any other nystagmuses. Severe nystagmus occurs with disease of the vestibular system, cerebellum, or brainstem. DIF: Cognitive Level: Analyzing (Analysis) MSC: Client Needs: Health Promotion and Maintenance 51. The nurse knows that testing kinesthesia is a test of apersons: a. Finetouch. b. Positionsense. c. Motorcoordination. d. Perception ofvibration. NURSINGTB.COM ANS: B Kinesthesia,orpositionsense,isthepersonsabilitytoperceivepassivemovementsoftheextremities.The other options areincorrect. DIF: Cognitive Level: Understanding (Comprehension) MSC: Client Needs: Safe and Effective Care Environment: Management of Care 52. Thenurseisreviewingapatientsmedicalrecordandnotesthatheisinacoma.UsingtheGlasgowComa Scale, which number indicates that the patient is in acoma? a. 6 b. 12 c. 15 d. 24 ANS: A Afullyalert,normalpersonhasascoreof15,whereasascoreof7orlessreflectscomaontheGlasgowComa Scale. DIF: Cognitive Level: Applying (Application) MSC: Client Needs: Physiologic Integrity: Physiologic Adaptation 53. Amanwhowasfoundwanderinginaparkat2AMhasbeenbroughttotheemergencydepartmentforan examination; he said he fell and hit his head. During the examination, the nurse asks him to use his index fingertotouchthenursesfinger,thenhisownnose,thenthenursesfingeragain(whichhasbeenmovedtoa differentlocation).Thepatientisclumsy,unabletofollowtheinstructions,andovershootsthemark,missing the finger. The nurse should suspect which of thefollowing? a. Cerebralinjury b. Cerebrovascularaccident c. Acute alcoholintoxication d. Peripheralneuropathy ANS: C NURSINGTB.COM Duringthefinger-to-fingertest,ifthepersonhasclumsymovementwithovershootingthemark,eithera cerebellardisorderoracutealcoholintoxicationshouldbesuspected.Thepersonsmovementsshouldbe smooth and accurate. The other options are notcorrect. DIF: Cognitive Level: Analyzing (Analysis) MSC: Client Needs: Physiologic Integrity: Physiologic Adaptation 54. Thenurseisassessingtheneurologicstatusofapatientwhohasalate-stagebraintumor.Withthereflex hammer,thenursedrawsalightstrokeupthelateralsideofthesoleofthefootandinward,acrosstheballof the foot. In response, the patients toes fan out, and the big toe shows dorsiflexion. The nurse interprets this resultas: a. Negative Babinski sign, which is normal foradults. b. Positive Babinski sign, which is abnormal foradults. c. Clonus, which is a hyperactiveresponse. d. Achilles reflex, which is an expectedresponse. ANS: B Dorsiflexion of the big toe and fanning of all toes is a positive Babinski sign, also called up-going toes. This responseoccurswithuppermotorneurondiseaseofthecorticospinal(orpyramidal)tractandisanabnormal finding foradults. DIF: Cognitive Level: Analyzing (Analysis) MSC: Client Needs: Physiologic Integrity: Physiologic Adaptation MULTIPLE RESPONSE 1. A 69-year-old patient has been admitted to an adult psychiatric unit because his wife thinks he is getting more and more confused. He laughs when he is found to be forgetful, saying Im just getting old! After the nursecompletesathoroughneurologicassessment,whichfindingswouldbeindicativeofAlzheimerdisease? Select all thatapply. a. Occasionally forgetting names orappointments b. Difficulty performing familiar tasks, such as placing a telephonecall c. Misplacing items, such as putting dish soap in therefrigerator d. Sometimes having trouble finding the rightword e. Rapid mood swings, from calm to tears, for no apparentreason f. Getting lost in ones ownneighborhood NURSINGTB.COM ANS: B, C, E, F Difficulty performing familiar tasks, misplacing items, rapid mood swings, and getting lost in ones own neighborhoodcanbewarningsignsofAlzheimerdisease.Occasionallyforgettingnamesorappointments,and sometimeshavingtroublefindingtherightwordarepartofnormalaging. DIF: Cognitive Level: Applying (Application) MSC: Client Needs: Physiologic Integrity: Physiologic Adaptation SHORT ANSWER 1.Duringtheassessmentofdeeptendonreflexes,thenursefindsthatapatientsresponsesarebilaterally normal.Whatnumberisusedtoindicatenormaldeeptendonreflexeswhenthedocumentingthisfinding? + ANS: 2 Responsestoassessmentofdeeptendonreflexesaregradedona4-pointscale.Aratingof2+indicatesnormal or average response. A rating of 0 indicates no response, and a rating of 4+ indicates very brisk, hyperactive response with clonus, which is indicative ofdisease. DIF: Cognitive Level: Applying (Application) MSC: Client Needs: Physiologic Integrity: Physiologic Adaptation Chapter 25: Male Genitourinary System MULTIPLE CHOICE 1. The external male genital structures includethe: a. Testis. b. Scrotum. c. Epididymis. d. Vasdeferens. ANS: B Theexternalmalegenitalstructuresincludethepenisandscrotum.Thetestis,epididymis,andvasdeferensare internalstructures. DIF:CognitiveLevel:Remembering(Knowledge) MSC: Client Needs:General 2. An accessory glandular structure for the male genital organs isthe: a. Testis. b. Scrotum. c. Prostate. d. Vasdeferens. NURSINGTB.COM ANS: C Glandularstructuresaccessorytothemalegenitalorgansaretheprostate,seminalvesicles,andbulbourethral glands. DIF:CognitiveLevel:Remembering(Knowledge) MSC: Client Needs:General 3. Which of these statements is true regarding thepenis? a. The urethral meatus is located on the ventral side of thepenis. b. Theprepuceisthefoldofforeskincoveringtheshaftofthepenis. c. Thepenisismadeupoftwocylindricalcolumnsoferectiletissue. d. Thecorpusspongiosumexpandsintoaconeoferectiletissuecalledtheglans. ANS: D Atthedistalendoftheshaft,thecorpusspongiosumexpandsintoaconeoferectiletissue,theglans.Thepenis is made up of three cylindrical columns of erectile tissue. The skin that covers the glans of the penis is the prepuce. The urethral meatus forms at the tip of theglans. DIF:CognitiveLevel:Remembering(Knowledge) MSC: Client Needs:General 4. Whenperformingagenitalexaminationona25-year-oldman,thenursenoticesdeeplypigmented,wrinkled scrotalskinwithlargesebaceousfollicles.Onthebasisofthisinformation,thenursewould: a. Squeeze the glans to check for the presence ofdischarge. b. Consider this finding as normal, and proceed with theexamination. c. Assess the testicles for the presence of masses or painlesslumps. d. Obtainamoredetailedhistory,focusingonanyscrotalabnormalitiesthepatienthasnoticed. ANS: B Afteradolescence,thescrotalskinisdeeplypNigUmReSnItNedGaTnBd.ChOasMlargesebaceousfolliclesandappears corrugated. DIF: Cognitive Level: Applying (Application) MSC: Client Needs: Health Promotion and Maintenance 5. Which statement concerning the testes istrue? a. Thelymphaticvesselsofthetestesdrainintotheabdominallymphnodes. b. Thevasdeferensislocatedalongtheinferiorportionofeachtestis. c. Therighttestisislowerthantheleftbecausetherightspermaticcordislonger. d. Thecremastermusclecontractsinresponsetocoldanddrawsthetesticlesclosertothebody. ANS: D Whenitiscold,thecremastermusclecontracts,whichraisesthescrotalsacandbringsthetestesclosertothe body to absorb heat necessary for sperm viability. The lymphatic vessels of the testes drain into the inguinal lymphnodes.Thevasdeferensislocatedalongtheupperportionofeachtestis.Thelefttestisislowerthanthe right because the left spermatic cord islonger. DIF: Cognitive Level: Remembering (Knowledge) MSC: Client Needs: General 6. Amalepatientwithpossiblefertilityproblemsasksthenursewherespermisproduced.Thenurseknows that sperm production occurs inthe: a. Testes. b. Prostate. c. Epididymis. d. Vasdeferens. ANS: A Spermproductionoccursinthetestes,notintheotherstructureslisted. DIF: Cognitive Level: Remembering(Knowledge) MSC: Client Needs: Physiologic Integrity 7. A62-year-oldmanstatesthathisphysiciantoldhimthathehasaninguinalhernia.Heasksthenurseto explain what a hernia is. The nurseshould: a. Tellhimnottoworryandthatmostmenhisagedevelophernias. NURSINGTB.COM b. Explainthataherniaisoftentheresultofprenatalgrowthabnormalities. c. Referhimtohisphysicianforadditionalconsultationbecausethephysicianmadetheinitial diagnosis. d. Explainthataherniaisaloopofbowelprotrudingthroughaweakspotintheabdominalmuscles. ANS: D Aherniaisaloopofbowelprotrudingthroughaweakspotinthemusculature.Theotheroptionsarenot correct responses to the patientsquestion. DIF:CognitiveLevel:Applying(Application) MSC: Client Needs: PhysiologicIntegrity 8. Themotherofa10-year-oldboyasksthenursetodiscusstherecognitionofpuberty.Thenurseshouldreply bysaying: a. Puberty usually begins around 15 years ofage. b. The first sign of puberty is an enlargement of thetestes. c. The penis size does not increase until about 16 years ofage. d. The development of pubic hair precedes testicular or penisenlargement. ANS: B Pubertybeginssometimebetweenage9forAfricanAmericansandage10forCaucasiansandHispanics.The firstsignisanenlargementofthetestes.Pubichairappearsnext,andthenpenissizeincreases. DIF: Cognitive Level: Applying (Application) MSC: Client Needs: Health Promotion and Maintenance 9. Duringanexaminationofanagingman,thenurserecognizesthatnormalchangestoexpectwouldbe: a. Enlargedscrotalsac. b. Increasedpubichair. c. Decreased penissize. d. Increased rugae over thescrotum. ANS: C Intheagingman,theamountofpubichairdecreases,thepenissizedecreases,andtherugaeoverthescrotal sacdecreases.ThescrotalsacdoesnotenlargeN.URSINGTB.COM DIF:CognitiveLevel:Understanding(Comprehension) MSC:ClientNeeds:HealthPromotionandMaintenance 10. Anoldermanisconcernedabouthissexualperformance.Thenurseknowsthatintheabsenceofdisease,a withdrawal from sexual activity later in life may be attributableto: a. Side effects ofmedications. b. Decreased libido withaging. c. Decreased spermproduction. d. Decreased pleasure from sexualintercourse. ANS: A Intheabsenceofdisease,awithdrawalfromsexualactivitymaybeattributabletosideeffectsofmedications suchasantihypertensives,antidepressants,sedatives,psychotropics,antispasmotics,tranquilizersornarcotics, and estrogens. The other options are notcorrect. DIF:CognitiveLevel:Understanding(Comprehension) MSC:ClientNeeds:HealthPromotionandMaintenance 11. A59-year-oldpatienthasbeendiagnosedwithprostatitisandisbeingseenattheclinicforcomplaintsof burning and pain during urination. He isexperiencing: a. Dysuria. b. Nocturia. c. Polyuria. d. Hematuria. ANS: A Dysuria(burningwithurination)iscommonwithacutecystitis,prostatitis,andurethritis.Nocturiaisvoiding duringthenight.Polyuriaisvoidinginexcessivequantities.Hematuriaisvoidingwithbloodintheurine. DIF: Cognitive Level: Applying (Application) MSC: Client Needs: Physiologic Integrity: Physiologic Adaptation 12. A45-year-oldmotheroftwochildrenisseenattheclinicforcomplaintsoflosingmyurinewhenIsneeze. The nurse documents that she isexperiencing: a. Urinaryfrequency. b. Enuresis. c. Stressincontinence. d. Urgeincontinence. NURSINGTB.COM ANS: C Stressincontinenceisinvoluntaryurinelosswithphysicalstrain,sneezing,orcoughingthatoccursasaresult to weakness of the pelvic floor. Urinary frequency is urinating more times than usual (more than five to six times per day). Enuresis is involuntary passage of urine at night after age 5 to 6 years (bed wetting). Urge incontinenceisinvoluntaryurinelossfromoveractivedetrusormuscleinthebladder.Itcontracts,causingan urgent need tovoid. DIF: Cognitive Level: Applying (Application) MSC: Client Needs: Physiologic Integrity: Physiologic Adaptation 13. Whenthenurseisconductingsexualhistoryfromamaleadolescent,whichstatementwouldbemost appropriate to use at the beginning of theinterview? a. Do you usecondoms? b. You dont masturbate, doyou? c. Have you had sex in the last 6months? d. Often adolescents your age have questions about sexualactivity. ANS: D Theinterviewshouldbeginwithapermissionstatement,whichconveysthatitisnormalandacceptableto think or feel a certain way. Sounding judgmental should beavoided. DIF: Cognitive Level: Analyzing (Analysis) MSC: Client Needs: Health Promotion and Maintenance 14. Whichofthesestatementsismostappropriatewhenthenurseisobtainingagenitourinaryhistoryfroman olderman? a. Do you need to get up at night tourinate? b. Do you experience nocturnal emissions, or wetdreams? c. Do you know how to perform a testicularself-examination? d. Hasanyoneevertouchedyourgenitalswhenyoudidnotwantthemto? ANS: A NURSINGTB.COM The older male patient should be asked about the presence of nocturia. Awaking at night to urinate may be attributabletoadiureticmedication,fluidretentionfrommildheartfailureorvaricoseveins,orfluidingestion 3 hours before bedtime, especially coffee and alcohol. The other questions are more appropriate for younger men. DIF: Cognitive Level: Analyzing (Analysis) MSC: Client Needs: Health Promotion and Maintenance 15. Whenthenurseisperformingagenitalexaminationonamalepatient,thepatienthasanerection.The nurses most appropriate action or response isto: a. Ask the patient if he would like someone else to examinehim. b. Continue with the examination as though nothing hashappened. c. Stoptheexamination,leavetheroomwhilestatingthattheexaminationwillresumeatalatertime. d. Reassurethepatientthatthisisanormalresponseandcontinuewiththeexamination. ANS: D When the male patient has an erection, the nurse should reassure the patient that this is a normal physiologic responsetotouchandproceedwiththerestoftheexamination.Theotherresponsesarenotcorrectandmaybe perceived as judgmental. DIF:CognitiveLevel:Applying(Application) MSC:ClientNeeds:PsychosocialIntegrity 16. Thenurseisexaminingtheglansandknowswhichfindingisnormalforthisarea? a. Themeatusmayhaveaslightdischargewhentheglansiscompressed. b. Hair is without pestinhabitants. c. The skin is wrinkled and withoutlesions. d. Smegma may be present under the foreskin of an uncircumcisedmale. ANS: D Theglanslookssmoothandwithoutlesionsanddoesnothavehair.Themeatusshouldnothaveanydischarge when the glans is compressed. Some cheesy smegma may have collected under the foreskin of an uncircumcisedmale. DIF: Cognitive Level: Understanding (Comprehension) MSC: Client Needs: Safe and Effective Care Environment: Management of Care 17. WhenperformingagenitourinaryassessmNenUtR,tShIeNnGuTrsBe.CnOotMicesthattheurethralmeatusisventrally positioned. This findingis: a. Calledhypospadias. b. A result ofphimosis. c. Probably due to astricture. d. Often associated withaging. ANS: A Normally, the urethral meatus is positioned just about centrally. Hypospadias is the ventral location of the urethralmeatus.Thepositionofthemeatusdoesnotchangewithaging.Phimosisistheinabilitytoretractthe foreskin. A stricture is a narrow opening of themeatus. DIF: Cognitive Level: Applying (Application) MSC: Client Needs: Safe and Effective Care Environment: Management of Care 18. Thenurseisperformingagenitalexaminationonamalepatientandnoticesurethraldrainage.When collectingurethraldischargeformicroscopicexaminationandculture,thenurseshould: a. Ask the patient to urinate into a sterilecup. b. Ask the patient to obtain a specimen ofsemen. c. Insert a cotton-tipped applicator into theurethra. d. Compresstheglansbetweentheexaminersthumbandforefinger,andcollectanydischarge. ANS: D If urethral discharge is noticed, then the examiner should collect a smear for microscopic examination and culturebycompressingtheglansanteroposteriorlybetweenthethumbandforefinger.Theotheroptionsareno correctactions. DIF: Cognitive Level: Applying (Application) MSC: Client Needs: Safe and Effective Care Environment: Management of Care 19. Whenassessingthescrotumofamalepatient,thenursenoticesthepresenceofmultiplefirm,nontender, yellow1-cmnodules.Thenurseknowsthatthesenodulesaremostlikely: a. Fromurethritis. b. Sebaceouscysts. c. Subcutaneousplaques. NURSINGTB.COM d. From an inflammation of theepididymis. ANS: B Sebaceouscystsarecommonlyfoundonthescrotum.Theseyellowish1-cmnodulesarefirm,nontender,and often multiple. The other options are notcorrect. DIF: Cognitive Level: Analyzing (Analysis) MSC: Client Needs: Physiologic Integrity: Physiologic Adaptation 20. Whenperformingascrotalassessment,thenursenoticesthatthescrotalcontentsshowaredglowwith transillumination. On the basis of this finding the nursewould: a. Assess the patient for the presence of ahernia. b. Suspect the presence of serous fluid in thescrotum. c. Consider this finding normal, and proceed with theexamination. d. Refer the patient for evaluation of a mass in thescrotum. ANS: B Normal scrotal contents do not allow light to pass through the scrotum. However, serous fluid does transilluminateandshowsasaredglow.Neitheramassnoraherniawouldtransilluminate. DIF: Cognitive Level: Analyzing(Analysis) MSC: Client Needs: Physiologic Integrity: Physiologic Adaptation 21. Whenthenurseisperformingagenitalexaminationonamalepatient,whichactioniscorrect? a. Auscultating for the presence of a bruit over thescrotum b. Palpatingfortheverticalchainoflymphnodesalongthegroin,inferiortotheinguinalligament c. Palpatingtheinguinalcanalonlyifabulgeispresentintheinguinalregionduringinspection d. Havingthepatientshifthisweightontotheleft(unexamined)legwhenpalpatingforaherniaon the rightside ANS: D Whenpalpatingforthepresenceofaherniaontherightside,themalepatientisaskedtoshifthisweightonto the left (unexamined) leg. Auscultating for a bruit over the scrotum is not appropriate. When palpating for lymphnodes,thehorizontalchainispalpated.Theinguinalcanalshouldbepalpatedwhetherabulgeispresent ornot. DIF: Cognitive Level: Applying (Application) MSC:ClientNeeds:SafeandEffectiveCareENnUvRirSoInNmGeTnBt:.CMOaMnagementofCare 22. Thenurseisawareofwhichstatementtobetrueregardingtheincidenceoftesticularcancer? a. Testicularcanceristhemostcommoncancerinmenaged30to50years. b. The early symptoms of testicular cancer are pain andinduration. c. Menwithahistoryofcryptorchidismareatthegreatestriskforthedevelopmentoftesticular cancer. d. The cure rate for testicular cancer islow. ANS: C Men with undescended testicles (cryptorchidism) are at the greatest risk for the development of testicular cancer.Theoverallincidenceoftesticularcancerisrare.Althoughtesticularcancerhasnoearlysymptoms, whendetectedearlyandtreatedbeforemetastasizing,thecurerateisalmost100%. DIF:CognitiveLevel:Understanding(Comprehension) MSC:ClientNeeds:HealthPromotionandMaintenance 23. Thenurseisdescribinghowtoperformatesticularself-examinationtoapatient.Whichstatementismost appropriate? a. Agoodtimetoexamineyourtesticlesisjustbeforeyoutakeashower. b. Ifyounoticeanenlargedtesticleorapainlesslump,callyourhealthcareprovider. c. Thetesticleiseggshapedandmovable.Itfeelsfirmandhasalumpyconsistency. d. Performatesticularexaminationatleastonceaweektodetecttheearlystagesoftesticularcancer. ANS: B Ifthepatientnoticesafirmpainlesslump,ahardarea,oranoverallenlargedtesticle,thenheshouldcallhis healthcareproviderforfurtherevaluation.Thetesticlenormallyfeelsrubberywithasmoothsurface.Agood time to examine the testicles is during the shower or bath, when ones hands are warm and soapy and the scrotumiswarm.Testicularself-examinationshouldbeperformedonceamonth. DIF: Cognitive Level: Applying (Application) MSC: Client Needs: Health Promotion and Maintenance 24. A2-month-olduncircumcisedinfanthasbeenbroughttotheclinicforawell-babycheckup.Howwould the nurse proceed with the genitalexamination? a. Eliciting the cremasteric reflex isrecommended. b. The glans is assessed for redness orlesions. NURSINGTB.COM c. Retractingtheforeskinshouldbeavoideduntiltheinfantis3monthsold. d. Anydirtorsmegmathathascollectedundertheforeskinshouldbenoted. ANS:C Ifuncircumcised,thentheforeskinisnormallytightduringthefirst3monthsandshouldnotberetracted becauseoftheriskoftearingthemembraneattachingtheforeskintotheshaft.Theotheroptionsarenot correct. DIF: Cognitive Level: Applying (Application) MSC: Client Needs: Safe and Effective Care Environment: Management of Care 25. A2-year-oldboyhasbeendiagnosedwithphysiologiccryptorchidism.Consideringthisdiagnosis,during assessment the nurse will most likelyobserve: a. Testes that are hard and painful topalpation. b. Atrophic scrotum and a bilateral absence of thetestis. c. Absenceofthetestisinthescrotum,butthetestiscanbemilkeddown. d. Testesthatmigrateintotheabdomenwhenthechildsquatsorsitscross-legged. ANS: C Migratory testes (physiologic cryptorchidism) are common because of the strength of the cremasteric reflex andthesmallmassoftheprepubertaltestes.Theaffectedsidehasanormallydevelopedscrotumandthetestis can be milked down. The other responses are notcorrect. DIF: Cognitive Level: Applying (Application) MSC: Client Needs: Physiologic Integrity: Physiologic Adaptation 26. Thenurseknowsthatacommonassessmentfindinginaboyyoungerthan2yearsoldis: a. Inflamedandtenderspermaticcord. b. Presenceofaherniainthescrotum. c. Penisthatlookslargeinrelationtothescrotum. d. Presenceofahydrocele,orfluidinthescrotum. ANS: D Acommonscrotalfindinginboysyoungerthan2yearsofageisahydrocele,orfluidinthescrotum.Theothe options are notcorrect. NURSINGTB.COM DIF: Cognitive Level: Applying (Application) MSC: Client Needs: Health Promotion and Maintenance 27. Duringanexaminationofanagingman,thenurserecognizesthatnormalchangestoexpectwouldbe: a. Change in scrotalcolor. b. Decrease in the size of thepenis. c. Enlargement of the testes andscrotum. d. Increase in the number of rugae over the scrotalsac. ANS: B Whenassessingthegenitalsofanolderman,thenursemaynoticethinner,grayingpubichairandadecreasein the size of the penis. The size of the testes may be decreased, they may feel less firm, and the scrotal sac is pendulous with less rugae. No change in scrotal color isobserved. DIF: Cognitive Level: Applying (Application) MSC: Client Needs: Health Promotion and Maintenance 28. When performing a genital assessment on a middle-aged man, the nurse notices multiple soft, moist, painlesspapulesintheshapeofcauliflower-likepatchesscatteredacrosstheshaftofthepenis.Theselesions are characteristic of: a. Carcinoma. b. Syphiliticchancres. c. Genitalherpes. d. Genitalwarts. ANS: D Thelesionsofgenitalwartsaresoft,pointed,moist,fleshy,painlesspapulesthatmaybesingleormultipleina cauliflower-likepatch.Theyoccurontheshaftofthepenis,behindthecorona,oraroundtheanus,wherethey may grow into large grapelikeclusters. DIF: Cognitive Level: Analyzing (Analysis) MSC: Client Needs: Physiologic Integrity: Physiologic Adaptation 29. A 15-year-old boy is seen in the clinic for complaints of dull pain and pulling in the scrotal area. On examination,thenursepalpatesasoft,irregularmassposteriortoandabovethetestisontheleft.Thismass collapseswhenthepatientissupineandrefillswhenheisupright.Thisdescriptionisconsistentwith: a. Epididymitis. b. Spermatocele. c. Testiculartorsion. d. Varicocele. NURSINGTB.COM ANS: D A varicocele consists of dilated, tortuous varicose veins in the spermatic cord caused by incompetent valves withinthevein.Symptomsincludedullpainoraconstantpullingordraggingfeeling,ortheindividualmaybe asymptomatic. When palpating the mass, the examiner will feel a soft, irregular mass posterior to and above thetestisthatcollapseswhentheindividualissupineandrefillswhentheindividualisupright. DIF: Cognitive Level: Analyzing (Analysis) MSC: Client Needs: Physiologic Integrity: Physiologic Adaptation 30. When performing a genitourinary assessment on a 16-year-old male adolescent, the nurse notices a swellinginthescrotumthatincreaseswithincreasedintra-abdominalpressureanddecreaseswhenheislying down.Thepatientcomplainsofpainwhenstraining.Thenurseknowsthatthisdescriptionismostconsistent witha(n) hernia. a. Femoral b. Incisional c. Directinguinal d. Indirectinguinal ANS:D With indirect inguinal hernias, pain occurs with straining and a soft swelling increases with increased intra- abdominalpressure,whichmaydecreasewhenthepatientliesdown.Thesefindingsdonotdescribetheother hernias. DIF: Cognitive Level: Analyzing (Analysis) MSC: Client Needs: Physiologic Integrity: Physiologic Adaptation 31. Whenthenurseisperformingatesticularexaminationona25-year-oldman,whichfindingisconsidered normal? a. Nontender subcutaneousplaques b. Scrotal area that is dry, scaly, andnodular c. Testesthatfeelovalandmovableandareslightlysensitivetocompression d. Single,hard,circumscribed,movaNblUeRmSaIsNsG,lTeBss.CthOaMn1cmunderthesurfaceofthetestes ANS:C Testesnormallyfeeloval,firmandrubbery,smooth,andbilaterallyequalandarefreelymovableandslightly tender to moderate pressure. The scrotal skin should not be dry, scaly, or nodular or contain subcutaneous plaques. Any mass would be an abnormalfinding. DIF: Cognitive Level: Applying (Application) MSC: Client Needs: Safe and Effective Care Environment: Management of Care 32. Thenurseisinspectingthescrotumandtestesofa43-year-oldman.Whichfindingwouldrequire additional follow-up andevaluation? a. Skin on the scrotum istaut. b. Left testicle hangs lower than the righttesticle. c. Scrotal skin has yellowish 1-cm nodules that are firm andnontender. d. Testes move closer to the body in response to coldtemperatures. ANS: A Scrotal swelling may cause the skin to be taut and to display pitting edema. Normal scrotal skin is rugae, and asymmetryisnormalwiththeleftscrotalhalfusuallylowerthantheright.Thetestesmaymoveclosertothe body in response to coldtemperatures. DIF: Cognitive Level: Analyzing (Analysis) MSC: Client Needs: Physiologic Integrity: Physiologic Adaptation 33. A55-year-oldmanisexperiencingseverepainofsuddenonsetinthescrotalarea.Itissomewhatrelieved by elevation. On examination the nurse notices an enlarged, red scrotum that is very tender to palpation. Distinguishing the epididymis from the testis is difficult, and the scrotal skin is thick and edematous. This description is consistent with which ofthese? a. Varicocele b. Epididymitis c. Spermatocele d. Testiculartorsion ANS:B Epididymitispresentsasseverepainofsuddenonsetinthescrotumthatissomewhatrelievedbyelevation.On examination, the scrotum is enlarged, reddened, and exquisitely tender. The epididymis is enlarged and indurated and may be hard to distinguish from the testis. The overlying scrotal skin may be thick and edematous. NURSINGTB.COM DIF: Cognitive Level: Analyzing (Analysis) MSC: Client Needs: Physiologic Integrity: Physiologic Adaptation 34. Thenurseisperformingagenitourinaryassessmentona50-year-oldobesemalelaborer.Onexamination, the nurse notices a painless round swelling close to the pubis in the area of the internal inguinal ring that is easilyreducedwhentheindividualissupine.Thesefindingsaremostconsistentwitha(n) hernia. a. Scrotal b. Femoral c. Directinguinal d. Indirectinguinal ANS:C Directinguinalherniasoccurmostofteninmenovertheageof40years.Itisanacquiredweaknessbroughton by heavy lifting, obesity, chronic cough, or ascites. The direct inguinal hernia is usually a painless, round swellingclosetothepubisintheareaoftheinternalinguinalringthatiseasilyreducedwhentheindividualis supine. DIF: Cognitive Level: Analyzing (Analysis) MSC: Client Needs: Physiologic Integrity: Physiologic Adaptation 35. Thenurseisprovidingpatientteachingaboutanerectiledysfunctiondrug.Oneofthedrugspotentialside effectsisprolonged,painfulerectionofthepeniswithoutsexualstimulation,whichisknownas: a. Orchitis. b. Stricture. c. Phimosis. d. Priapism. ANS: D Priapismisprolonged,painfulerectionofthepeniswithoutsexualdesire.Orchitisisinflammationofthe testes.Strictureisanarrowingoftheopeningoftheurethralmeatus.Phimosisistheinabilitytoretractthe foreskin. DIF: Cognitive Level: Remembering (Knowledge) MSC: Client Needs: Physiologic Integrity: Physiologic Adaptation 36. During an examination, the nurse notices that a male patient has a red, round, superficial ulcer with a yellowishserousdischargeonhispenis.Onpalpation,thenursefindsanontenderbasethatfeelslikeasmall buttonbetweenthethumbandfingers.Atthispointthenursesuspectsthatthispatienthas: NURSINGTB.COM a. Genitalwarts. b. Herpesinfection. c. Syphiliticchancre. d. Carcinomalesion. ANS: C Thislesionindicatessyphiliticchancre,whichbeginswithin2to4weeksofinfection. DIF: Cognitive Level: Analyzing(Analysis) MSC: Client Needs: Physiologic Integrity: Physiologic Adaptation 37. Duringahealthhistory,apatienttellsthenursethathehastroubleinstartinghisurinestream.This problem is knownas: a. Urgency. b. Dribbling. c. Frequency. d. Hesitancy. ANS: D Hesitancy is trouble in starting the urine stream. Urgency is the feeling that one cannot wait to urinate. Dribblingisthelastoftheurinebeforeorafterthemainactofurination.Frequencyisurinatingmoreoften thanusual. DIF: Cognitive Level: Understanding (Comprehension) MSC: Client Needs: Physiologic Integrity: Physiologic Adaptation 38. Duringagenitalexamination,thenursenoticesthatamalepatienthasclustersofsmallvesiclesonthe glans,surroundedbyerythema.Thenurserecognizesthattheselesionsare: a. Peyroniedisease. b. Genitalwarts. c. Genitalherpes. d. Syphiliticcancer. ANS: C NURSINGTB.COM Genitalherpes,orherpessimplexvirus2(HSV-2),infectionsareindicatedwithclustersofsmallvesicleswith surroundingerythema,whichareoftenpainfulanderuptontheglansorforeskin. DIF: Cognitive Level: Applying (Application) MSC: Client Needs: Physiologic Integrity: Physiologic Adaptation 39. Duringaphysicalexamination,thenursefindsthatamalepatientsforeskinisfixedandtightandwillnot retract over the glans. The nurse recognizes that this conditionis: a. Phimosis. b. Epispadias. c. Urethralstricture. d. Peyroniedisease. ANS: A Withphimosis,theforeskinisnonretractable,formingapointytipofthepeniswithatinyorificeattheendof the glans. The foreskin is advanced and so tight that it is impossible to retract over the glans. This condition may be congenital or acquired from adhesions related toinfection. DIF: Cognitive Level: Applying (Application) MSC: Client Needs: Physiologic Integrity: Physiologic Adaptation MULTIPLE RESPONSE 1. A55-year-oldmanisintheclinicforayearlycheckup.Heisworriedbecausehisfatherdiedofprostate cancer.Thenurseknowswhichtestsshouldbeperformedatthistime?Selectallthatapply. a. Blood test for prostate-specific antigen(PSA) b. Urinalysis c. Transrectalultrasound d. Digital rectal examination(DRE) e. Prostatebiopsy ANS: A,D ProstatecanceristypicallydetectedbytestingthebloodforPSAorbyaDRE.Itisrecommendedthatboth PSA and DRE be offered to men annually, beginning at age 50 years. If the PSA is elevated, then further laboratoryworkoratransrectalultrasound(TRUS)andbiopsymayberecommended. DIF: Cognitive Level: Applying (Application)NURSINGTB.COM MSC: Client Needs: Health Promotion and Maintenance 2. A 16-year-old boy is brought to the clinic for a problem that he refused to let his mother see. The nurse examineshim,andfindsthathehasscrotalswellingontheleftside.Hehadthemumpsthepreviousweek,and the nurse suspects that he has orchitis. Which of the following assessment findings support this diagnosis? Select all thatapply. a. Swollentestis b. Mass thattransilluminates c. Mass that does nottransilluminate d. Scrotum that is nontender uponpalpation e. Scrotum that is tender uponpalpation f. Scrotalskinthatisreddened ANS: A, C, E,F With orchitis, the testis is swollen, with a feeling of weight, and is tender or painful. The mass does not transilluminate,andthescrotalskinisreddened.Transilluminationofamassoccurswithahydrocele,not orchitis. DIF: Cognitive Level: Applying (Application) MSC: Client Needs: Health Promotion and Maintenance NURSINGTB.COM Chapter 26: Anus, Rectum, and Prostate MULTIPLE CHOICE 1. Whichstatementconcerningtheanalcanalistrue?Theanalcanal: a. Is approximately 2 cm long in theadult. b. Slantsbackwardtowardthesacrum. c. Containshairandsebaceousglands. d. Is the outlet for the gastrointestinaltract. ANS: D Theanalcanalistheoutletforthegastrointestinaltractandisapproximately3.8cmlongintheadult.Itis linedwithamodifiedskinthatdoesnotcontainhairorsebaceousglands,anditslantsforwardtowardthe umbilicus. DIF:CognitiveLevel:Remembering(Knowledge) MSC: Client Needs:General 2. Which statement concerning the sphincters iscorrect? NURSINGTB.COM a. Theinternalsphincterisundervoluntarycontrol. b. Theexternalsphincterisundervoluntarycontrol. c. Both sphincters remain slightly relaxed at alltimes. d. The internal sphincter surrounds the externalsphincter. ANS: B Theexternalsphinctersurroundstheinternalsphincterbutalsohasasmallsectionoverridingthetipofthe internalsphincterattheopening.Theexternalsphincterisundervoluntarycontrol.Exceptforthepassingof feces and gas, the sphincters keep the anal canal tightlyclosed. DIF:CognitiveLevel:Remembering(Knowledge) MSC: Client Needs:General 3. Thenurseisperforminganexaminationoftheanusandrectum.Whichofthesestatementsiscorrectand important to remember during thisexamination? a. The rectum is approximately 8 cmlong. b. The anorectal junction cannot bepalpated. c. Above the anal canal, the rectum turnsanteriorly. d. No sensory nerves are in the anal canal orrectum. ANS: B Theanalcolumnsarefoldsofmucosathatextendverticallydownfromtherectumandendintheanorectal junction.Thisjunctionisnotpalpablebutisvisibleonproctoscopy.Therectumis12cmlong;justabovethe anal canal, the rectum dilates and turnsposteriorly. DIF:CognitiveLevel:Remembering(Knowledge) MSC: Client Needs: Safe and Effective Care Environment: Management of Care 4. Thestructurethatsecretesathin,milkyalkalinefluidtoenhancetheviabilityofspermisthe: a. Cowpergland. b. Prostategland. c. Mediansulcus. d. Bulbourethralgland. ANS: B NURSINGTB.COM In men, the prostate gland secretes a thin milky alkaline fluid that enhances sperm viability. The Cowper glands(alsoknownasbulbourethralglands)secreteaclear,viscidmucus.Themediansulcusisagroovethat divides the lobes of the prostate gland and does not secretefluid. DIF:CognitiveLevel:Remembering(Knowledge) MSC: Client Needs:General 5. A46-year-oldmanrequiresanassessmentofhissigmoidcolon.Whichinstrumentortechniqueismost appropriate for thisexamination? a. Proctoscope b. Ultrasound c. Colonoscope d. Rectalexaminationwithanexaminingfinger ANS:C Thesigmoidcolonis40cmlong,andthenurseknowsthatitisaccessibletoexaminationonlywiththe colonoscope.Theotherresponsesarenotappropriateforanexaminationoftheentiresigmoidcolon. DIF: Cognitive Level: Understanding (Comprehension) MSC: Client Needs: Safe and Effective Care Environment: Management of Care 6. Thenurseiscaringforanewborninfant.Thirtyhoursafterbirth,theinfantpassesadarkgreenmeconium stool. The nurse recognizes this is important becausethe: a. Stool indicates analpatency. b. Dark green color indicates occult blood in thestool. c. Meconium stool can be reflective of distress in thenewborn. d. Newbornshouldhavepassedthefirststoolwithin12hoursafterbirth. ANS: A Thefirststoolpassedbythenewbornisdarkgreenmeconiumandoccurswithin24to48hoursofbirth, indicating anal patency. The other responses are notcorrect. DIF:CognitiveLevel:Understanding(Comprehension) MSC:ClientNeeds:HealthPromotionandMaintenance 7. Duringtheassessmentofan18-month-oldinfant,themotherexpressesconcerntothenurseaboutthe infantsinabilitytotoilettrain.Whatwouldbethenursesbestresponse? NURSINGTB.COM a. Somechildrenarejustmoredifficulttotrain,soIwouldntworryaboutityet. b. Haveyouconsideredreadinganyofthebooksontoilettraining?Theycanbeveryhelpful. c. Thiscouldmeanthatthereisaprobleminyourbabysdevelopment.Wellwatchhercloselyforthe next fewmonths. d. Thenervesthatwillallowyourbabytohavecontroloverthepassingofstoolsarenotdeveloped until at least 18 to 24 months ofage. ANS: D The infant passes stools by reflex. Voluntary control of the external anal sphincter cannot occur until the nervessupplyingtheareahavebecomefullymyelinated,usuallyaround1to2yearsofage.Toilettraining usually starts after the age of 2years. DIF: Cognitive Level: Applying (Application) MSC: Client Needs: Health Promotion and Maintenance 8. A60-year-oldmanhasjustbeentoldthathehasbenignprostatichypertrophy(BPH).Hehasafriendwho justdiedfromcanceroftheprostate.Heisconcernedthiswillhappentohim.Howshouldthenurserespond? a. The swelling in your prostate is only temporary and will goaway. b. We will treat you with chemotherapy so we can control thecancer. c. Itwouldbeveryunusualforamanyouragetohavecanceroftheprostate. d. Theenlargementofyourprostateiscausedbyhormonalchanges,andnotcancer. ANS: D Theprostateglandcommonlystartstoenlargeduringthemiddleadultyears.BPHispresentin1in10menat theageof40yearsandincreaseswithage.Itisbelievedthatthehypertrophyiscausedbyhormonalimbalance thatleadstotheproliferationofbenignadenomas.Theotherresponsesarenotappropriate. DIF: Cognitive Level: Applying (Application) MSC: Client Needs: Health Promotion and Maintenance 9. A30-year-oldwomanisvisitingtheclinicbecauseofpaininmybottomwhenIhaveabowelmovement. The nurse should assess for whichproblem? a. Pinworms b. Hemorrhoids c. Coloncancer d. Fecalincontinence NURSINGTB.COM ANS: B Havingpainfulbowelmovements,knownasdyschezia,maybeattributabletoalocalcondition(hemorrhoidor fissure) or constipation. The other responses are notcorrect. DIF: Cognitive Level: Applying (Application) MSC: Client Needs: Physiologic Integrity: Physiologic Adaptation 10. Apatientwhoisvisitingthecliniccomplainsofhavingstomachpainsfor2weeksanddescribeshisstools as being soft and black for approximately the last 10 days. He denies taking any medications. The nurse is aware that these symptoms are mostly indicativeof: a. Excessive fat caused bymalabsorption. b. Increased iron intake, resulting from a change indiet. c. Occult blood, resulting from gastrointestinalbleeding. d. Absent bile pigment from liverproblems. ANS: C Blackstoolsmaybetarryasaresultofoccultblood(melena)fromgastrointestinalbleedingornontarryfrom ingestionofironmedications(notdiet).Excessivefatcausesthestooltobecomefrothy.Theabsenceofbile pigment causes clay-coloredstools. DIF: Cognitive Level: Analyzing (Analysis) MSC: Client Needs: Physiologic Integrity: Physiologic Adaptation 11. Aftercompletinganassessmentofa60-year-oldmanwithafamilyhistoryofcoloncancer,thenurse discusseswithhimearlydetectionmeasuresforcoloncancer.Thenurseshouldmentiontheneedfora(n): a. Annualproctoscopy. b. Colonoscopy every 10years. c. Fecal test for blood every 6months. d. DREs every 2years. ANS: B Early detection measures for colon cancer include a DRE performed annually after age 50 years, an annual fecaloccultbloodtestafterage50years,asigmoidoscopicexaminationevery5yearsoracolonoscopyevery 10yearsafterage50years,andaPSAbloodtestannuallyformenover50yearsold,exceptbeginningatage 45 years for black men (American Cancer Society,2006). DIF: Cognitive Level: Applying (Application)NURSINGTB.COM MSC: Client Needs: Health Promotion and Maintenance 12. Themotherofa5-year-oldgirltellsthenursethatshehasnoticedherdaughterscratchingatherbottoma lot the last few days. During the assessment, the nurse finds redness and raised skin in the anal area. This finding most likelyindicates: a. Pinworms. b. Chickenpox. c. Constipation. d. Bacterialinfection. ANS: A Inchildren,pinwormsareacommoncauseofintenseitchingandirritatedanalskin.Theotheroptionsarenot correct. DIF: Cognitive Level: Analyzing (Analysis) MSC: Client Needs: Physiologic Integrity: Physiologic Adaptation 13. Thenurseisexaminingonlytherectalareaofawomanandshouldplacethewomaninwhatposition? a. Lithotomy b. Prone c. Left lateraldecubitus d. Bendingoverthetablewhilestanding ANS:C Thenurseshouldplacethefemalepatientinthelithotomypositionifthegenitaliaarebeingexaminedaswell. The left lateral decubitus position is used for the rectal areaalone. DIF: Cognitive Level: Understanding (Comprehension) MSC: Client Needs: Safe and Effective Care Environment: Management of Care 14. Whileperforminganassessmentoftheperianalareaofapatient,thenursenoticesthatthepigmentationof anus is darker than the surrounding skin, the anal opening is closed, and a skin sac that is shiny and blue is noted. The patient mentioned that he has had pain with bowel movements and has occasionally noted some spotsofblood.Whatwouldthisassessmentandhistorymostlikelyindicate? a. Analfistula b. Pilonidalcyst c. Rectalprolapse d. Thrombosedhemorrhoid NURSINGTB.COM ANS: D Theanusnormallylooksmoistandhairless,withcoarsefoldedskinthatismorepigmentedthantheperianal skin,andtheanalopeningistightlyclosed.Theshinyblueskinsacindicatesathrombosedhemorrhoid. DIF: Cognitive Level: Analyzing (Analysis) MSC: Client Needs: Physiologic Integrity: Physiologic Adaptation 15. Thenurseispreparingtopalpatetherectumandshouldusewhichofthesetechniques?Thenurseshould: a. Flex the finger, and slowly insert it toward theumbilicus. b. Firstinstructthepatientthatthisprocedurewillbepainful. c. Insertanextendedindexfingeratarightangletotheanus. d. Placethefingerdirectlyintotheanustoovercomethetightsphincter. ANS: A The nurse should gently place the pad of the index finger against the anal verge. The nurse will feel the sphinctertightenandthenrelax.Asitrelaxes,thenurseshouldflexthetipofthefingerandslowlyinsertitinto the anal canal in a direction toward the umbilicus. The nurse should never approach the anus at right angles withtheindexfingerextended;doingsowouldcausepain.Thenurseshouldinstructthepatientthatpalpation is not painful but may feel like needing to move thebowels. DIF: Cognitive Level: Understanding (Comprehension) MSC: Client Needs: Safe and Effective Care Environment: Management of Care 16. Whileperformingarectalexamination,thenursenoticesafirm,irregularlyshapedmass.Whatshouldthe nurse donext? a. Continue with the examination, and document the finding in thechart. b. Instruct the patient to return for a repeat assessment in 1month. c. Tellthepatientthatamasswasfelt,butitisnothingtoworryabout. d. Reportthefinding,andreferthepatienttoaspecialistforfurtherexamination. ANS: D Afirmorhardmasswithanirregularshapeorrollededgesmaysignifycarcinoma.Anymassthatis discoveredshouldbepromptlyreportedforfuNrtUheRrSeIxNaGmTiBna.CtiOonM.Theotherresponsesarenotcorrect. DIF: Cognitive Level: Applying (Application) MSC: Client Needs: Health Promotion and Maintenance 17. Duringanassessmentofthenewborn,thenurseexpectstoseewhichfindingwhentheanalareaisslightly stroked? a. Jerking of thelegs b. Flexion of theknees c. Quick contraction of thesphincter d. Relaxationoftheexternalsphincter ANS:C Toassesssphinctertone,thenurseshouldchecktheanalreflexbygentlystrokingtheanalareaandnoticinga quick contraction of the sphincter. The other responses are notcorrect. DIF:CognitiveLevel:Understanding(Comprehension) MSC:ClientNeeds:HealthPromotionandMaintenance 18. A13-year-oldgirlisvisitingtheclinicforasportsphysicalexamination.Thenurseshouldrememberto include which of these tests in theexamination? a. Testing for occultblood b. Valsalvamaneuver c. Internal palpation of theanus d. Inspectionoftheperianalarea ANS:D Theperianalregionoftheschool-agedchildandadolescentshouldbeinspectedduringtheexaminationofthe genitalia.Internalpalpationisnotroutinelyperformedatthisage.Testingforoccultbloodandperformingthe Valsalva maneuver are also notnecessary. DIF: Cognitive Level: Applying (Application) MSC: Client Needs: Health Promotion and Maintenance 19. Duringanassessmentofa20-year-oldman,thenursefindsasmallpalpablelesionwithatuftofhair locateddirectlyoverthecoccyx.Thenurseknowsthatthislesionwouldmostlikelybea: a. Rectalpolyp. b. Pruritusani. c. Carcinoma. d. Pilonidalcyst. NURSINGTB.COM ANS: D Apilonidalcystorsinusisahair-containingcystorsinuslocatedinthemidlineoverthecoccyxorlower sacrum.Itoftenopensasadimplewithavisibletuftofhairand,possibly,anerythematoushalo. DIF: Cognitive Level: Applying (Application) MSC: Client Needs: Physiologic Integrity: Physiologic Adaptation 20. Duringanexamination,thenurseasksthepatienttoperformtheValsalvamaneuverandnoticesthatthe patient has a moist, red, doughnut-shaped protrusion from the anus. The nurse knows that this finding is consistent witha: a. Rectalpolyp. b. Hemorrhoid. c. Rectalfissure. d. Rectalprolapse. ANS: D Inrectalprolapse,therectalmucousmembraneprotrudesthroughtheanus,appearingasamoistreddoughnut withradiatinglines.ItoccursafteraValsalvamaneuver,suchasstrainingatpassingstoolorwithexercising. DIF: Cognitive Level: Understanding (Comprehension) MSC: Client Needs: Physiologic Integrity: Physiologic Adaptation 21. A70-year-oldmanisvisitingtheclinicfordifficultyinpassingurine.Inthehealthhistory,heindicates thathehastourinatefrequently,especiallyatnight.Hehasburningwhenheurinatesandhasnoticedpainin hisback.Consideringthishistory,whatmightthenurseexpecttofindduringthephysicalassessment? a. Asymmetric,hard,andfixedprostategland b. Occultbloodandperianalpaintopalpation c. Symmetricallyenlarged,softprostategland d. Soft nodule protruding from the rectalmucosa ANS: A NURSINGTB.COM Subjective symptoms of carcinoma of the prostate include frequency, nocturia, hematuria, weak stream, hesitancy, pain or burning on urination, and continuous pain in lower back, pelvis, and thighs. Objective symptomsofcarcinomaoftheprostateincludeamalignantneoplasmthatoftenstartsasasinglehardnodule on the posterior surface, producing asymmetry and a change in consistency. As it invades normal tissue, multiplehardnodulesappear,ortheentireglandfeelsstonehardandfixed. DIF: Cognitive Level: Analyzing (Analysis) MSC: Client Needs: Physiologic Integrity: Physiologic Adaptation 22. A40-year-oldblackmanisintheofficeforhisannualphysicalexamination.Whichstatementregarding thePSAbloodtestistrue,accordingtotheAmericanCancerSociety?ThePSA: a. Should be performed with thisvisit. b. Shouldbeperformedatage45years. c. Shouldbeperformedatage50years. d. Is only necessary if a family history of prostate cancerexists. ANS: B AccordingtotheAmericanCancerSociety(2006),thePSAbloodtestshouldbeperformedannuallyforblack menbeginningatage45yearsandannuallyforallothermenoverage50years. DIF: Cognitive Level: Applying (Application) MSC: Client Needs: Health Promotion and Maintenance 23. A62-year-oldmanisexperiencingfever,chills,malaise,urinaryfrequency,andurgency.Healsoreports urethraldischargeandadullachingpainintheperinealandrectalarea.Thesesymptomsaremostconsistent with whichcondition? a. Prostatitis b. Polyps c. Carcinoma of theprostate d. BPH ANS: A Thecommonpresentingsymptomsofprostatitisarefever,chills,malaise,andurinaryfrequencyandurgency. Theindividualmayalsohavedysuria,urethraldischarge,andadullachingpainintheperinealandrectalarea. These symptoms are not consistent withpolyps. DIF: Cognitive Level: Analyzing (Analysis) MSC: Client Needs: Physiologic Integrity: Physiologic Adaptation 24. DuringadiscussionforamenshealthgrouNpU,RthSeINnuGrTseBr.CelOatMesthatthegroupwiththehighestincidenceof prostate canceris: a. AsianAmericans. b. Blacks. c. AmericanIndians. d. Hispanics. ANS: B AccordingtotheAmericanCancerSociety(2010),blackmenhaveahigherrateofprostatecancerthanother racialgroups. DIF: Cognitive Level: Remembering (Knowledge) MSC: Client Needs: Physiologic Integrity: Reduction of Risk Potential 25. Whichcharacteristicoftheprostateglandwouldthenurserecognizeasanabnormalfindingwhile palpating the prostate gland through therectum? a. Palpablecentralgroove b. Tenderness topalpation c. Heartshaped d. Elasticandrubberyconsistency ANS:B Thenormalprostateglandshouldfeelsmooth,elastic,andrubbery;slightlymovable;heart-shapedwitha palpable central groove; and not be tender topalpation. DIF: Cognitive Level: Analyzing (Analysis) MSC: Client Needs: Health Promotion and Maintenance 26. Thenursenoticesthatapatienthashadapale,yellow,greasystool,orsteatorrhea,andrecallsthatthisis causedby: a. Occultbleeding. b. Absent bilepigment. c. Increased fatcontent. d. Ingestion of bismuthpreparations.NURSINGTB.COM ANS: C Steatorrhea(pale,yellow,greasystool)iscausedbyincreasedfatcontentinthestools,asinmalabsorption syndrome.Occultbleedingandingestionofbismuthproductscauseablackstool,andabsentbilepigment causes a gray-tanstool. DIF: Cognitive Level: Understanding (Comprehension) MSC: Client Needs: Physiologic Integrity: Physiologic Adaptation 27. Duringahealthhistoryofapatientwhocomplainsofchronicconstipation,thepatientasksthenurseabout high-fiberfoods.Thenurserelatesthatanexampleofahigh-fiberfoodwouldbe: a. Broccoli. b. Hamburger. c. Iceberglettuce. d. Yogurt. ANS: A High-fiber foods are either soluble type (e.g., beans, prunes, barley, broccoli) or insoluble type (e.g., cereals, wheatgerm).Theotherexamplesarenotconsideredhigh-fiberfoods. DIF: Cognitive Level: Understanding(Comprehension) MSC: Client Needs: Health Promotion and Maintenance 28. Whileassessingapatientwhoishospitalizedandbedridden,thenursenoticesthatthepatienthasbeen incontinentofstool.Thestoolislooseandgray-tanincolor.Thenurserecognizesthatthisfindingindicates which of thefollowing? a. Occultblood b. Inflammation c. Absent bilepigment d. Ingestionofironpreparations ANS:C The presence of gray-tan stool indicates absent bile pigment, which can occur with obstructive jaundice. The ingestionofironpreparationsandthepresenceofoccultbloodturnsthestoolstoablackcolor.Jellylikemucus shreds mixed in the stool would indicateinflammation. DIF: Cognitive Level: Applying (Application) MSC:ClientNeeds:PhysiologicIntegrity:PhNysUioRlSoIgNicGATBda.CpOtaMtion 29. Duringadigitalexaminationoftherectum,thenursenoticesthatthepatienthashardfecesintherectum. The patient complains of feeling full, has a distended abdomen, and states that she has not had a bowel movement for several days. The nurse suspects whichcondition? a. Rectalpolyp b. Fecalimpaction c. Rectalabscess d. Rectalprolapse ANS:B Afecalimpactionisacollectionofhard,desiccatedfecesintherectum.Theobstructionoftenresultsfrom decreasedbowelmotility,inwhichmorewaterisreabsorbedfromthestool. DIF: Cognitive Level: Applying (Application) MSC:ClientNeeds:PhysiologicIntegrity:PhysiologicAdaptation 30. Duringthetakingofahealthhistory,thepatientstates,Itreallyhurtsbackthere,andsometimesititches, too.IhaveevenseenbloodonthetissuewhenIhaveabowelmovement.Istheresomethingthere?Thenurse should expect to see which of these upon examination of theanus? a. Rectalprolapse b. Internalhemorrhoid c. External hemorrhoid that hasresolved d. Externalhemorrhoidthatisthrombosed ANS:D These symptoms are consistent with an external hemorrhoid. An external hemorrhoid, when thrombosed, containsclottedbloodandbecomesapainful,swollen,shinybluemassthatitchesandbleedswithdefecation. Whentheexternalhemorrhoidresolves,itleavesaflabby,painlessskinsacaroundtheanalorifice.Aninternal hemorrhoid is not palpable but may appear as a red mucosal mass when the person performs a Valsalva maneuver.Arectalprolapseappearsasamoist,reddoughnutwithradiatinglines. DIF: Cognitive Level: Applying (Application) MSC: Client Needs: Physiologic Integrity: Physiologic Adaptation MULTIPLE RESPONSE 1.Thenurseisperformingadigitalexaminationofapatientsprostateglandandnoticesthatanormalprostate glandincludeswhichofthefollowingcharacteristics?Selectallthatapply. a. 1cmprotrusionintotherectum NURSINGTB.COM b. Heart-shaped with a palpable centralgroove c. Flat shape with no palpablegroove d. Boggy with a softconsistency e. Smooth surface, elastic, and rubberyconsistency f. Fixedmobility ANS: A, B,E The size of a normal prostate gland should be 2.5 cm long by 4 cm wide and should not protrude more than 1 cmintotherectum.Theprostateshouldbeheart-shaped,withapalpablecentralgroove,asmoothsurface,and elasticwitharubberyconsistency.Abnormalfindingsincludeaflatshapewithnopalpablegroove,boggywith a soft consistency, and fixedmobility. DIF: Cognitive Level: Analyzing (Analysis) MSC: Client Needs: Health Promotion and Maintenance Chapter 27: Female Genitourinary System MULTIPLE CHOICE 1. Duringahealthhistory,a22-yearoldwomanasks,CanIgetthatvaccineforhumanpapillomavirus (HPV)?IhavegenitalwartsandIdlikethemtogoaway!Whatisthenursesbestresponse? a. TheHPVvaccineisforgirlsandwomenages9to26years,sowecanstartthattoday. b. Thisvaccineisonlyforgirlswhohavenotyetstartedtobecomesexuallyactive. c. Letscheckwiththephysiciantoseeifyouareacandidateforthisvaccine. d. ThevaccinecannotprotectyouifyoualreadyhaveanHPVinfection. ANS: D TheHPVvaccineisappropriateforgirlsandwomenage9to26yearsandisadministeredtopreventcervical cancer by preventing HPV infections before girls become sexually active. However, it cannot protect the woman if an HPV infection is alreadypresent. DIF: Cognitive Level: Analyzing (Analysis) MSC: Client Needs: General 2. Duringanexamination,thenurseobservesNaUfeRmSaIlNeGpTaBtie.CnOtsMvestibuleandexpectstoseethe: a. Urethral meatus and vaginalorifice. b. Vaginalorificeandvestibular(Bartholin)glands. c. Urethralmeatusandparaurethral(Skene)glands. d. Paraurethral (Skene) and vestibular (Bartholin)glands. ANS: A The labial structures encircle a boat-shaped space, or cleft, termed the vestibule. Within the vestibule are numerousopenings.Theurethralmeatusandvaginalorificearevisible.Theductsoftheparaurethral(Skene) glands and the vestibular (Bartholin) glands are present but notvisible. DIF: Cognitive Level: Understanding (Comprehension) MSC: Client Needs: Physiologic Integrity: Physiologic Adaptation 3. Duringaspeculuminspectionofthevagina,thenursewouldexpecttoseewhatattheendofthevaginal canal? a. Cervix b. Uterus c. Ovaries d. Fallopiantubes ANS:A Attheendofthecanal,theuterinecervixprojectsintothevagina. DIF: Cognitive Level: Remembering(Knowledge) MSC: Client Needs: Physiologic Integrity: Physiologic Adaptation 4. Theuterusisusuallypositionedtiltingforwardandsuperiortothebladder.Thispositionisknownas: a. Anteverted andanteflexed. b. Retroverted andanteflexed. c. Retroverted andretroflexed. d. Superiorverted andanteflexed. ANS: A NURSINGTB.COM Theuterusisfreelymovable,notfixed,andusuallytiltsforwardandsuperiortothebladder(apositionlabeled as anteverted andanteflexed). DIF:CognitiveLevel:Remembering(Knowledge) MSC: Client Needs:General 5. An11-year-oldgirlisintheclinicforasportsphysicalexamination.Thenursenoticesthatshehasbegunto develop breasts, and during the conversation the girl reveals that she is worried about her development. The nurseshouldusewhichofthesetechniquestobestassisttheyounggirlinunderstandingtheexpectedsequence for development? The nurseshould: a. Use the Tanner scale on the five stages of sexualdevelopment. b. Describeherdevelopmentandcompareitwiththatofothergirlsherage. c. UsetheJacobsentableonexpecteddevelopmentonthebasisofheightandweightdata. d. Reassureherthatherdevelopmentiswithinnormallimitsandtellhernottoworryaboutthenext step. ANS: A The Tanner scale on the five stages of pubic hair development is helpful in teaching girls the expected sequenceofsexualdevelopment.Theotherresponsesarenotappropriate. DIF: Cognitive Level: Applying(Application) MSC: Client Needs: Health Promotion and Maintenance 6. Awomanwhois8weekspregnantisintheclinicforacheckup.Thenursereadsonherchartthathercervix issoftenedandlookscyanotic.Thenurseknowsthatthewomanisexhibiting signand sign. a. Tanner;Hegar b. Hegar;Goodell c. Chadwick;Hegar d. Goodell;Chadwick ANS:D Shortlyafterthefirstmissedmenstrualperiod,thefemalegenitaliashowsignsofthegrowingfetus.Thecervix softens(Goodellsign)at4to6weeks,andthevaginalmucosaandcervixlookcyanotic(Chadwicksign)at8 to 12 weeks. These changes occur because of increased vascularity and edema of the cervix and hypertrophy and hyperplasia of the cervical glands. Hegar sign occurs when the isthmus of the uterus softens at 6 to 8 weeks. Tanner sign is not a correctresponse. DIF:CognitiveLevel:Understanding(ComprNehUeRnSsiIoNnG)TB.COM MSC: Client Needs: Health Promotion and Maintenance 7. Generally,thechangesnormallyassociatedwithmenopauseoccurbecausethecellsinthereproductivetract are: a. Aging. b. Becomingfibrous. c. Estrogendependent. d. Able to respond toestrogen. ANS: C Becausecellsinthereproductivetractareestrogendependent,decreasedestrogenlevelsduringmenopause bring dramatic physical changes. The other options are notcorrect. DIF: Cognitive Level: Remembering (Knowledge) MSC:ClientNeeds:HealthPromotionandMaintenance 8. Thenurseisreviewingthechangesthatoccurwithmenopause.Whichchangesareassociatedwith menopause? a. Uterineandovarianatrophy,alongwithathinningofthevaginalepithelium b. Ovarian atrophy, increased vaginal secretions, and increasing clitoralsize c. Cervicalhypertrophy,ovarianatrophy,andincreasedacidityofvaginalsecretions d. Vaginalmucosafragility,increasedacidityofvaginalsecretions,anduterinehypertrophy ANS:A Theuterusshrinksbecauseofitsdecreasedmyometrium.Theovariesatrophyto1to2cmandarenotpalpable after menopause. The sacral ligaments relax, and the pelvic musculature weakens; consequently, the uterus droops. The cervix shrinks and looks paler with a thick glistening epithelium. The vaginal epithelium atrophies, becoming thinner, drier, and itchy. The vaginal pH becomes more alkaline, and secretions are decreased,whichresultsinafragilemucosalsurfacethatisatriskforvaginitis. DIF:CognitiveLevel:Understanding(Comprehension) MSC:ClientNeeds:HealthPromotionandMaintenance 9. A 54-year-old woman who has just completed menopause is in the clinic today for a yearly physical examination.Whichofthesestatementsshouldthenurseincludeinpatienteducation?Apostmenopausal woman: a. Isnotatanygreaterriskforheartdiseasethanayoungerwoman. NURSINGTB.COM b. Shouldbeawarethatsheisatincreasedriskfordyspareuniabecauseofdecreasedvaginal secretions. c. Hasonlystoppedmenstruating;therereallyarenoothersignificantchangeswithwhichsheshould beconcerned. d. Islikelytohavedifficultywithsexualpleasureasaresultofdrasticchangesinthefemalesexual responsecycle. ANS: B Decreasedvaginalsecretionsleavethevaginadryandatriskforirritationandpainwithintercourse (dyspareunia). The other statements areincorrect. DIF: Cognitive Level: Applying (Application) MSC: Client Needs: Health Promotion and Maintenance 10. Awomanisintheclinicforanannualgynecologicexamination.Thenurseshouldplantobeginthe interview withthe: a. Menstrual history, because it is generallynonthreatening. b. Obstetric history, because it includes the most importantinformation. c. Urinarysystemhistory,becauseproblemsmaydevelopinthisareaaswell. d. Sexual history, because discussing it first will buildrapport. ANS: A Menstrualhistoryisusuallynonthreateningandthereforeagoodtopicwithwhichtobegintheinterview. Obstetric,urinary,andsexualhistoriesarealsopartoftheinterviewbutnotnecessarilythebesttopicswith which tostart. DIF: Cognitive Level: Applying (Application) MSC: Client Needs: Safe and Effective Care Environment: Management of Care 11. Apatienthashadthreepregnanciesandtwolivebirths.Thenursewouldrecordthisinformationasgrav ,para ,AB . a. 2; 2;1 b. 3; 2;0 c. 3; 2;1 d. 3; 3;1 ANS: C NURSINGTB.COM Gravida(grav)isthenumberofpregnancies.Paraisthenumberofbirths.Abortionsareinterrupted pregnancies, including elective abortions and spontaneousmiscarriages. DIF: Cognitive Level: Applying (Application) MSC: Client Needs: Safe and Effective Care Environment: Management of Care 12. Duringtheinterviewwithafemalepatient,thenursegathersdatathatindicatethepatientis perimenopausal.Whichofthesestatementsmadebythispatientleadstothisconclusion? a. IhavenoticedthatmymusclesacheatnightwhenIgotobed. b. IwillbeveryhappywhenIcanstopworryingabouthavingaperiod. c. IhavebeennoticingthatIsweatalotmorethanIusedto,especiallyatnight. d. Ihaveonlybeenpregnanttwice,butbothtimesIhadbreasttendernessasmyfirstsymptom. ANS: C Hormoneshiftsoccurduringtheperimenopausalperiod,andassociatedsymptomsofmenopausemayoccur, such as hot flashes, night sweats, numbness and tingling, headache, palpitations, drenching sweats, mood swings, vaginal dryness, and itching. The other responses are notcorrect. DIF: Cognitive Level: Analyzing (Analysis) MSC: Client Needs: Health Promotion and Maintenance 13. A 50-year-old woman calls the clinic because she has noticed some changes in her body and breasts and wondersifthesechangescouldbeattributabletothehormonereplacementtherapy(HRT)shestarted3months earlier. The nurse should tellher: a. HRT is at such a low dose that side effects are veryunusual. b. HRThasseveralsideeffects,includingfluidretention,breasttenderness,andvaginalbleeding. c. VaginalbleedingwithHRTisveryunusual;Isuggestyoucomeintotheclinicimmediatelytohave thisevaluated. d. Itsoundsasifyourdoseofestrogenistoohigh;Ithinkyoumayneedtodecreasetheamountyou are taking and then call back in aweek. ANS:B SideeffectsofHRTincludefluidretention,breastpain,andvaginalbleeding.Theotherresponsesarenot correct. DIF: Cognitive Level: Applying (Application) MSC: Client Needs: Physiologic Integrity: Pharmacologic and Parenteral Therapies NURSINGTB.COM 14. A52-year-oldpatientstatesthatwhenshesneezesorcoughsshewetsherselfalittle.Sheisveryconcerned thatsomethingmaybewrongwithher.Thenursesuspectsthattheproblemis: a. Dysuria. b. Stressincontinence. c. Hematuria. d. Urgeincontinence. ANS: B Stress incontinence is involuntary urine loss with physical strain, sneezing, or coughing. Dysuria is pain or burningwithurination.Hematuriaisbleedingwithurination.Urgeincontinenceisinvoluntaryurinelossthat occursasaresultofanoveractivedetrusormuscleinthebladderthatcontractsandcausesanurgentneedto void. DIF: Cognitive Level: Understanding (Comprehension) MSC: Client Needs: Physiologic Integrity: Physiologic Adaptation 15. Duringtheinterview,apatientrevealsthatshehassomevaginaldischarge.Sheisworriedthatitmaybea sexuallytransmittedinfection.Thenursesmostappropriateresponsetothiswouldbe: a. Oh, dont worry. Some cyclic vaginal discharge isnormal. b. Have you been engaging in unprotected sexualintercourse? c. Id like some information about the discharge. What color isit? d. Have you had any urinary incontinence associated with thedischarge? ANS: C Questions that help the patient reveal more information about her symptoms should be asked in a nonthreateningmanner.Askingabouttheamount,color,andodorofthevaginaldischargeprovidesthe opportunity for further assessment. Normal vaginal discharge is small, clear or cloudy, and always nonirritating. DIF: Cognitive Level: Analyzing (Analysis) MSC: Client Needs: Physiologic Integrity: Physiologic Adaptation 16. Awomanstatesthat2weeksagoshehadaurinarytractinfectionthatwastreatedwithanantibiotic.Asa part of the interview, the nurse should ask, Have you noticedany: a. Changes in your urinationpatterns? b. Excessive vaginalbleeding? NURSINGTB.COM c. Unusual vaginal discharge oritching? d. Changes in your desire forintercourse? ANS: C Severalmedicationsmayincreasetheriskofvaginitis.Broad-spectrumantibioticsalterthebalanceofnormal flora,whichmayleadtothedevelopmentofvaginitis.Theotherquestionsarenotappropriate. DIF: Cognitive Level: Applying (Application) MSC: Client Needs: Physiologic Integrity: Pharmacologic and Parenteral Therapies 17. Whichstatementwouldbemostappropriatewhenthenurseisintroducingthetopicofsexualrelationships during aninterview? a. Now,itistimetotalkaboutyoursexualhistory.Whendidyoufirsthaveintercourse? b. Womenoftenfeeldissatisfiedwiththeirsexualrelationships.Woulditbeokaytodiscussthis now? c. Womenoftenhavequestionsabouttheirsexualrelationshipandhowitaffectstheirhealth.Doyou have anyquestions? d. Mostwomenyouragehavehadmorethanonesexualpartner.Howmanywouldyousayyouhave had? ANS: C The nurse should begin with an open-ended question to assess individual needs. The nurse should include appropriate questions as a routine part of the health history, because doing so communicates that the nurse accepts the individuals sexual activity and believes it is important. The nurses comfort with the discussion prompts the patients interest and, possibly, relief that the topic has been introduced. The initial discussion establishesadatabaseforcomparisonwithanyfuturesexualactivitiesandprovidesanopportunitytoscreen sexualproblems. DIF:CognitiveLevel:Applying(Application) MSC:ClientNeeds:PsychosocialIntegrity 18. A22-year-oldwomanhasbeenconsideringusingoralcontraceptives.Asapartofherhealthhistory,the nurse shouldask: a. Do you have a history of heartmurmurs? b. Will you be in a monogamousrelationship? c. Have you carefully thought this choicethrough? d. If you smoke, how many cigarettes do you smoke perday? NURSINGTB.COM ANS: D Oralcontraceptives,togetherwithcigarettesmoking,increasetheriskforcardiovascularsideeffects.If cigarettesareused,thenthenurseshouldassessthepatientssmokinghistory.Theotherquestionsarenot appropriate. DIF: Cognitive Level: Applying (Application) MSC: Client Needs: Physiologic Integrity: Pharmacologic and Parenteral Therapies 19. Amarriedcouplehascometotheclinicseekingadviceonpregnancy.Theyhavebeentryingtoconceive for4monthsandhavenotbeensuccessful.Whatshouldthenursedofirst? a. Ascertainwhethereitherofthemhasbeenusingbroad-spectrumantibiotics. b. Explainthatcouplesareconsideredinfertileafter1yearofunprotectedintercourse. c. Immediatelyreferthewomantoanexpertinpelvicinflammatorydiseasethemostcommoncause ofinfertility. d. Explainthatcouplesareconsideredinfertileafter3monthsofengaginginunprotectedintercourse and that they will need a referral to a fertilityexpert. ANS: B Infertilityisconsideredafter1yearofengaginginunprotectedsexualintercoursewithoutconceiving.The other actions are notappropriate. DIF:CognitiveLevel:Applying(Application) MSC:ClientNeeds:PsychosocialIntegrity 20. Anurseisassessingapatientsriskofcontractingasexuallytransmittedinfection(STI).Anappropriate question to ask wouldbe: a. You know that its important to use condoms for protection,right? b. Do you use a condom with each episode of sexualintercourse? c. Do you have a sexually transmittedinfection? d. You are aware of the dangers of unprotected sex, arentyou? ANS: B InreviewingapatientsriskforSTIs,thenurseshouldaskinanonconfrontationalmannerwhethercondoms arebeingusedduringeachepisodeofsexualintercourse.Askingapersonwhetherheorshehasaninfection does not address therisk. DIF: Cognitive Level: Understanding (Comprehension) MSC:ClientNeeds:PhysiologicIntegrity:ReNduUcRtiSoInNoGfTRBi.sCkOPMotential 21. Whenthenurseisinterviewingapreadolescentgirl,whichopeningquestionwouldbeleastthreatening? a. Do you have any questions about growingup? b. What has your mother told you about growingup? c. When did you notice that your body waschanging? d. IrememberbeingveryscaredwhenIgotmyperiod.Howdoyouthinkyoullfeel? ANS: C Open-endedquestionssuchas,Whendidyou?ratherthanDoyou?shouldbeasked.Open-endedquestions arelessthreateningbecausetheyimplythatthetopicisnormalandunexceptional. DIF:CognitiveLevel:Understanding(Comprehension) MSC: Client Needs: PsychosocialIntegrity 22. Whenthenurseisdiscussingsexualityandsexualissueswithanadolescent,apermissionstatementhelps convey that it is normal to think or feel a certain way. Which statement is the best example of a permission statement? a. It is okay that you have become sexuallyactive. b. Girlsyourageoftenhavequestionsaboutsexualactivity.Doyouhaveanyquestions? c. Ifitisokaywithyou,Idliketoaskyousomequestionsaboutyoursexualhistory. d. Girlsyourageoftenengageinsexualactivities.Itisokaytotellmeifyouhavehadintercourse. ANS: B Theexaminershouldstartwithapermissionstatementsuchas,GirlsyourageoftenexperienceApermission statementconveystheideathatitisnormaltothinkorfeelacertainway,andimplyingthatthetopicisnormal and unexceptional isimportant. DIF:CognitiveLevel:Understanding(Comprehension) MSC: Client Needs: PsychosocialIntegrity 23. Thenurseispreparingtointerviewapostmenopausalwoman.Whichofthesestatementsistrueasit applies to obtaining the health history of a postmenopausalwoman? a. Thenurseshouldaskapostmenopausalwomanifshehaseverhadvaginalbleeding. b. Onceawomanreachesmenopause,thenursedoesnotneedtoaskanyhistoryquestions. c. ThenurseshouldscreenformonthNlyUbRrSeIaNstGtTenBd.CerOnMess. d. PostmenopausalwomenarenotatriskforcontractingSTIs;therefore,thesequestionscanbe omitted. ANS: A Postmenopausalbleedingwarrantsfurtherworkupandreferral.Theotherstatementsarenottrue. DIF: Cognitive Level: Understanding(Comprehension) MSC: Client Needs: Physiologic Integrity: Reduction of Risk Potential 24. Duringtheexaminationportionofapatientsvisit,shewillbeinlithotomyposition.Whichstatement reflectssomethingsthatthenursecandotomakethispositionmorecomfortableforher? a. Ask her to place her hands and arms over herhead. b. Elevate her head and shoulders to maintain eyecontact. c. Allowhertochoosetohaveherfeetinthestirrupsorhavethemrestingsidebysideontheedgeof thetable. d. Allowhertokeepherbuttocksapproximately6inchesfromtheedgeofthetabletopreventher from feeling as if she will falloff. ANS: B Thenurseshouldelevateherheadandshoulderstomaintaineyecontact.Thepatientsarmsshouldbeplacedat hersidesoracrossthechest.Placingherhandsandarmsoverherheadonlytightenstheabdominalmuscles. Thefeetshouldbeplacedintothestirrups,kneesapart,andbuttocksattheedgeoftheexaminingtable.The stirrups are placed so that the legs are not abducted toofar. DIF: Cognitive Level: Applying (Application) MSC: Client Needs: Safe and Effective Care Environment: Management of Care 25. An18-year-oldpatientishavingherfirstpelvicexamination.Whichactionbythenurseisappropriate? a. Inviting her mother to be present during theexamination b. Avoidingthelithotomypositionforthisfirsttimebecauseitcanbeuncomfortableand embarrassing c. Raisingtheheadoftheexaminationtableandgivingheramirrorsothatshecanviewthe examination d. Fullydrapingher,leavingthedrapebetweenherlegselevatedtoavoidembarrassingherwitheye contact ANS: C ThetechniquesoftheeducationalormirrorpeNlvUiRcSeIxNamGTinBa.tCioOnMshouldbeused.Thisisaroutineexamination withsomemodificationsinattitude,position,andcommunication.First,thewomanisconsideredanactive participant,onewhoisinterestedinlearningandinsharingdecisionsaboutherownhealthcare.Thewoman props herself up on one elbow, or the head of the table is raised. Her other hand holds a mirror between her legs,abovetheexaminershands.Theyoungwomancanseeallthattheexaminerisdoingandhasafullview ofhergenitalia.Themirrorworkswellforteachingnormalanatomyanditsrelationshiptosexualbehavior. The examiner can ask her if she would like to have a family member, friend, or chaperone present for the examination.Thedrapeshouldbepusheddownbetweenthepatientslegssothatthenursecanseeherface. DIF: Cognitive Level: Applying (Application) MSC: Client Needs: Safe and Effective Care Environment: Management of Care 26. Thenursehasjustcompletedaninspectionofanulliparouswomansexternalgenitalia.Whichofthese would be a description of a finding within normallimits? a. Redness of the labiamajora b. Multiple nontender sebaceouscysts c. Discharge that is foul smelling andirritating d. Gapingandslightlyshriveledlabiamajora ANS:B Nolesionsshouldbenoted,exceptfortheoccasionalsebaceouscysts,whichareyellowish1-cmnodulesthat are firm, nontender, and often multiple. The labia majora are dark pink, moist, and symmetric; redness indicatesinflammationorlesions.Dischargethatisfoulsmellingandirritatingmayindicateinfection.Inthe nulliparouswoman,thelabiamajorameetinthemidline,aresymmetricandplump. DIF: Cognitive Level: Applying (Application) MSC: Client Needs: Safe and Effective Care Environment: Management of Care 27. Thenurseispreparingforaninternalgenitaliaexaminationofawoman.Whichorderoftheexaminationis correct? a. Bimanual,speculum,andrectovaginal b. Speculum,rectovaginal,andbimanual c. Speculum,bimanual,andrectovaginal d. Rectovaginal,bimanual,andspeculum ANS:C Thecorrectsequenceisspeculumexamination,thenbimanualexaminationafterremovingthespeculum,and then rectovaginal examination. The examiner should change gloves before performing the rectovaginal examination to avoid spreading any possibleinfection. DIF:CognitiveLevel:Analyzing(Analysis) NURSINGTB.COM MSC: Client Needs: Safe and Effective Care Environment: Management of Care 28. Duringaninternalexaminationofawomansgenitalia,thenursewillusewhichtechniqueforproper insertion of thespeculum? a. Thewomanisinstructedtobeardown,thespeculumbladesareopenedandappliedinaswift, upwardmovement. b. Thebladesofthespeculumareinsertedonahorizontalplane,turningthemtoa30-degreeangle whilecontinuingtoinsertthem.Thewomanisaskedtobeardownafterthespeculumisinserted. c. Thewomanisinstructedtobeardown,thewidthofthebladesarehorizontallyturned,andthe speculumisinserteddownwardata45-degreeangletowardthesmallofthewomansback. d. Thebladesarelockedopenbyturningthethumbscrew.Oncethebladesareopen,pressureis appliedtotheintroitusandthebladesareinserteddownwardata45-degreeangletobringthe cervix intoview. ANS: C Theexaminershouldinstructthewomantobeardown,turnthewidthofthebladeshorizontally,andinsertthe speculumata45-degreeangledownwardtowardthesmallofthewomansback.(SeethetextunderSpeculum Examination for moredetail.) DIF: Cognitive Level: Applying (Application) MSC: Client Needs: Safe and Effective Care Environment: Management of Care 29. The nurse is examining a 35-year-old female patient. During the health history, the nurse notices that she hashadtwotermpregnancies,andbothbabiesweredeliveredvaginally.Duringtheinternalexamination,the nurseobservesthatthecervicalosisahorizontalslitwithsomehealedlacerationsandthatthecervixhassome nabothian cysts that are small, smooth, and yellow. In addition, the nurse notices that the cervical surface is granularandred,especiallyaroundtheos.Finally,thenursenoticesthepresenceofstringy,opaque,odorless secretions. Which of these findings areabnormal? a. Nabothian cysts arepresent. b. The cervical os is a horizontalslit. c. The cervical surface is granular andred. d. Stringy and opaque secretions arepresent. ANS: C Normalfindings:Nabothiancystsmaybepresentonthecervixafterchildbirth.Thecervicalosisahorizontal, irregular slit in the parous woman. Secretions vary according to the day of the menstrual cycle, and may be clear and thin or thick, opaque, and stringy. The surface is normally smooth, but cervical eversion, or ectropion, may occur where the endocervical canal is rolled out. Abnormal finding: The cervical surface should not be reddened or granular, which may indicate alesion. NURSINGTB.COM DIF: Cognitive Level: Analyzing (Analysis) MSC: Client Needs: Health Promotion and Maintenance 30. ApatientcallstheclinicforinstructionsbeforehavingaPapanicolaou(Pap)smear.Themostappropriate instructions from the nurseare: a. Ifyouaremenstruating,pleaseusepadstoavoidplacinganythingintothevagina. b. Avoidintercourse,insertinganythingintothevagina,ordouchingwithin24hoursofyour appointment. c. Ifyoususpectthatyouhaveavaginalinfection,pleasegatherasampleofthedischargetobring withyou. d. Wewouldlikeyoutouseamildsalinedouchebeforeyourexamination.Youmaypickthisupin ouroffice. ANS: B When instructing a patient before Pap smear is obtained, the nurse should follow these guidelines: Do not obtain during the womans menses or if a heavy infectious discharge is present. Instruct the woman not to douche,haveintercourse,orputanythingintothevaginawithin24hoursbeforecollectingthespecimens.Any specimens will be obtained during the visit, notbeforehand. DIF: Cognitive Level: Applying (Application) MSC: Client Needs: Health Promotion and Maintenance 31. Duringanexamination,whichtestswillthenursecollecttoscreenforcervicalcancer? a. Endocervical specimen, cervical scrape, and vaginalpool b. Endocervical specimen, vaginal pool, and acetic acidwash c. Endocervicalspecimen,potassiumhydroxide(KOH)preparation,andaceticacidwash d. Cervicalscrape,aceticacidwash,salinemount(wetprep) ANS:A Laboratoriesmayvaryinmethod,butusuallythetestconsistsofthreespecimens:endocervicalspecimen, cervicalscrape,andvaginalpool.Theothertests(aceticacidwash,KOHpreparation,andsalinemount)are used to test for sexually transmittedinfections. DIF: Cognitive Level: Understanding (Comprehension) MSC: Client Needs: Safe and Effective Care Environment: Management of Care 32. Whenperformingthebimanualexamination,thenursenoticesthatthecervixfeelssmoothandfirm,is round,andisfixedinplace(doesnotmove).Whencervicalpalpationisperformed,thepatientcomplainsof somepain.ThenursesinterpretationoftheserNeUsuRltSsINshGoTuBld.CbOeMwhichofthese? a. These findings are all within normallimits. b. Cervical consistency should be soft and velvetynotfirm. c. Thecervixshouldmovewhenpalpated;animmobilecervixmayindicatemalignancy. d. Pain may occur during palpation of thecervix. ANS: C Normally,thecervixfeelssmoothandfirm,similartotheconsistencyofthetipofthenose.Itsoftensandfeels velvety at 5 to 6 weeks of pregnancy (Goodell sign). The cervix should be evenly rounded. With a finger on either side, the examiner should be able to move the cervix gently from side to side, and doing so should produce no pain for the patient. Hardness of the cervix may occur with malignancy. Immobility may occur withmalignancy,andpainmayoccurwithinflammationorectopicpregnancy. DIF: Cognitive Level: Analyzing (Analysis) MSC: Client Needs: Health Promotion and Maintenance 33. Thenurseispalpatingafemalepatientsadnexa.Thefindingsincludeafirm,smoothuterinewall;the ovariesarepalpableandfeelsmoothandfirm.Thefallopiantubeisfirmandpulsating.Thenursesmost appropriate course of action would beto: a. Tell the patient that her examination isnormal. b. Give her an immediate referral to agynecologist. c. Suggestthatshereturninamonthforarechecktoverifythefindings. d. Tellthepatientthatshemayhaveanovariancystthatshouldbeevaluatedfurther. ANS: B Normally,theuterinewallfeelsfirmandsmooth,withthecontourofthefundusrounded.Ovariesarenotoften palpable, but when they are, they normally feel smooth, firm, and almond shaped and are highly movable, slidingthroughthefingers.Thefallopiantubeisnotnormallypalpable.Noothermassorpulsationshouldbe felt.Pulsationorpalpablefallopiantubesuggestsectopicpregnancy,whichwarrantsimmediatereferral. DIF: Cognitive Level: Applying (Application) MSC: Client Needs: Health Promotion and Maintenance 34. A 65-year-old woman is in the office for routine gynecologic care. She had a complete hysterectomy 3 monthsagoaftercervicalcancerwasdetected.Whichstatementdoesthenurseknowtobetrueregardingthis visit? a. Her cervical mucosa will be red and drylooking. b. ShewillnotneedtohaveaPapsmNeUarRpSeIrNfoGrTmBe.dC.OM c. Thenursecanexpecttofindthatheruteruswillbesomewhatenlargedandherovariessmalland hard. d. Thenurseshouldplantolubricatetheinstrumentsandtheexamininghandadequatelytoavoida painfulexamination. ANS: D Intheagingadultwoman,naturallubricationisdecreased;therefore,toavoidapainfulexamination,thenurse should take care to lubricate the instruments and the examining hand adequately. Menopause, with the resulting decrease in estrogen production, shows numerous physical changes. The cervix shrinks and looks paleandglistening.Withthebimanualexamination,theuterusfeelssmallerandfirmerandtheovariesarenot normallypalpable.Womenshouldcontinuecervicalcancerscreeninguptoage65yearsiftheyhaveanintact cervix and are in good health. Women who have had a total hysterectomy do not need cervical cancer screening if they have 3 consecutive negative Pap tests or 2 or more consecutive negative HIV and Pap tests within the last 10years. DIF: Cognitive Level: Applying (Application) MSC: Client Needs: Health Promotion and Maintenance 35. Thenurseispreparingtoexaminetheexternalgenitaliaofaschool-agegirl.Whichpositionwouldbemost appropriate in thissituation? a. In the parentslap b. In a frog-leg position on the examiningtable c. In the lithotomy position with the feet instirrups d. Lyingflatontheexaminingtablewithlegsextended ANS:B Forschool-agechildren,placingthemontheexaminingtableinafrog-legpositionisbest.Withtoddlersand preschoolers,havingthechildontheparentslapinafrog-legpositionisbest. DIF: Cognitive Level: Applying (Application) MSC: Client Needs: Health Promotion and Maintenance 36. Whenassessinganewborninfantsgenitalia,thenursenoticesthatthegenitaliaaresomewhatengorged. Thelabiamajoraareswollen,theclitorislookslarge,andthehymenisthick.Thevaginalopeningisdifficult to visualize. The infants mother states that she is worried about the labia being swollen. The nurse should reply: a. Thisisanormalfindinginnewbornsandshouldresolvewithinafewweeks. b. Thisfindingcouldindicateanabnormalityandmayneedtobeevaluatedbyaphysician. NURSINGTB.COM c. Wewillneedtohaveestrogenlevelsevaluatedtoensurethattheyarewithinnormallimits. d. Wewillneedtokeepclosewatchoverthenextfewdaystoseeifthegenitaliadecreaseinsize. ANS: A Itisnormalforanewbornsgenitaliatobesomewhatengorged.Asanguineousvaginaldischargeorleukorrhea is normal during the first few weeks because of the maternal estrogen effect. During the early weeks, the genitalengorgementresolves,andthelabiaminoraatrophyandremainsmalluntilpuberty. DIF: Cognitive Level: Applying (Application) MSC: Client Needs: Health Promotion and Maintenance 37. During a vaginal examination of a 38-year-old woman, the nurse notices that the vulva and vagina are erythematousandedematouswiththick,white,curdlikedischargeadheringtothevaginalwalls.Thewoman reports intense pruritus and thick white discharge from her vagina. The nurse knows that these history and physical examination findings are most consistent with whichcondition? a. Candidiasis b. Trichomoniasis c. Atrophicvaginitis d. Bacterialvaginosis ANS: A Thewomanwithcandidiasisoftenreportsintensepruritusandthickwhitedischarge.Thevulvaandvaginaare erythematousandedematous.Thedischargeisusuallythick,white,andcurdlike.Infectionwithtrichomoniasis causesaprofuse,watery,gray-green,andfrothydischarge.Bacterialvaginosiscausesaprofusedischargethat hasafoul,fishy,rottenodor.Atrophicvaginitismayhaveamucoiddischarge. DIF: Cognitive Level: Analyzing (Analysis) MSC: Client Needs: Physiologic Integrity: Physiologic Adaptation 38. A 22-year-old woman is being seen at the clinic for problems with vulvar pain, dysuria, and fever. On physicalexamination,thenursenoticesclustersofsmall,shallowvesicleswithsurroundingerythemaonthe labia.Inguinallymphadenopathypresentisalsopresent.Themostlikelycauseoftheselesionsis: a. Pediculosispubis. b. Contactdermatitis. c. HPV. d. Herpes simplex virus type2. ANS: D NURSINGTB.COM Herpessimplexvirustype2exhibitsclustersofsmall,shallowvesicleswithsurroundingerythemathaterupt onthegenitalareas.Inguinallymphadenopathyisalsopresent.Thewomanreportslocalpain,dysuria,and fever. DIF: Cognitive Level: Analyzing (Analysis) MSC: Client Needs: Physiologic Integrity: Physiologic Adaptation 39. Whenperforminganexternalgenitaliaexaminationofa10-year-oldgirl,thenursenoticesthatnopubic hairhasgrowninandthemonsandthelabiaarecoveredwithfinevellushair.Thesefindingsareconsistent with stage of sexual maturity, according to the Sexual Maturity Ratingscale. a. 1 b. 2 c. 3 d. 4 ANS: A SexualMaturityRatingstage1isthepreadolescentstage.Thereisnopubichair,andthemonsandlabiaare covered with fine, vellus hair as on theabdomen. DIF: Cognitive Level: Applying (Application) MSC: Client Needs: Health Promotion and Maintenance 40. A46-year-oldwomanisintheclinicforherannualgynecologicexamination.Shevoicesaconcernabout ovarian cancer because her mother and sister died of it. Which statement does the nurse know to be correct regarding ovariancancer? a. Ovarian cancer rarely has anysymptoms. b. The Pap smear detects the presence of ovariancancer. c. Womenathighriskforovariancancershouldhaveannualtransvaginalultrasonographyfor screening. d. Womenoverage40yearsshouldhaveathoroughpelvicexaminationevery3years. ANS: C Withovariancancer,thepatientmayhaveabdominalpain,pelvicpain,increasedabdominalsize,bloating,and nonspecific gastrointestinal symptoms; or she may be asymptomatic. The Pap smear does not detect the presenceofovariancancer.Annualtransvaginalultrasonographymaydetectovariancanceratanearlierstage in women who are at high risk for developingit. DIF: Cognitive Level: Applying (Application) MSC:ClientNeeds:HealthPromotionandMNaiUntReSnIaNncGeTB.COM 41. Duringabimanualexamination,thenursedetectsasolidtumorontheovarythatisheavyandfixed,witha poorly defined mass. This finding is suggestiveof: a. Ovariancyst. b. Endometriosis. c. Ovariancancer. d. Ectopicpregnancy. ANS: C Ovarian tumors that are solid, heavy, and fixed, with poorly defined mass are suggestive of malignancy. Benign masses may feel mobile and solid. An ovarian cyst may feel smooth, round, fluctuant, mobile, and nontender. With an ectopic pregnancy, the examiner may feel a palpable, tender pelvic mass that is solid, mobile,andunilateral.Endometriosismayhavemasses(invariouslocationsinthepelvicarea)thataresmall, firm, nodular, and tender to palpation, with enlargedovaries. DIF: Cognitive Level: Applying (Application) MSC: Client Needs: Physiologic Integrity: Physiologic Adaptation 42. A25-year-oldwomancomestotheemergencydepartmentwithasuddenfeverof38.3Candabdominal pain.Uponexamination,thenursenoticesthatshehasrigid,boardlikelowerabdominalmusculature.When thenursetriestoperformavaginalexamination,thepatienthasseverepainwhentheuterusandcervixare moved.Thenurseknowsthatthesesignsandsymptomsaresuggestiveof: a. Endometriosis. b. Uterinefibroids. c. Ectopicpregnancy. d. Pelvic inflammatorydisease. ANS: D Thesesignsandsymptomsaresuggestiveofacutepelvicinflammatorydisease,alsoknownasacute salpingitis. DIF: Cognitive Level: Analyzing (Analysis) MSC: Client Needs: Physiologic Integrity: Physiologic Adaptation 43. Duringanexternalgenitaliaexaminationofawoman,thenursenoticesseverallesionsaroundthevulva. Thelesionsarepink,moist,soft,andpointedpapules.Thepatientstatesthatsheisnotawareofanyproblems in that area. The nurse recognizes that these lesions maybe: a. Syphiliticchancre. NURSINGTB.COM b. Herpes simplex virus type 2 (herpesgenitalis). c. HPV or genitalwarts. d. Pediculosis pubis (crablice). ANS: C HPVlesionsarepainless,wartygrowthsthatthewomanmaynotnotice.Lesionsarepinkorfleshcolored, soft,pointed,moist,wartypapulesthatoccurinsingleormultiplecauliflower-likepatchesaroundthevulva, introitus, anus, vagina, or cervix. Herpetic lesions are painful clusters of small, shallow vesicles with surroundingerythema.Syphiliticchancresbeginasasolitarysilverypapulethaterodesintoared,roundor oval superficial ulcer with a yellowish discharge. Pediculosis pubis causes severe perineal itching and excoriations and erythematousareas. DIF: Cognitive Level: Analyzing (Analysis) MSC: Client Needs: Physiologic Integrity: Physiologic Adaptation 44. Duringanexamination,thenursewouldexpectthecervicalosofawomanwhohasneverhadchildrento appear: a. Stellate. b. Small andround. c. As a horizontal irregularslit. d. Everted. ANS: B Thecervicalosinanulliparouswomanissmallandround.Intheparouswoman,itisahorizontal,irregular slit that also may show healed lacerations on thesides. DIF:CognitiveLevel:Understanding(Comprehension) MSC:ClientNeeds:HealthPromotionandMaintenance 45. AwomanhasjustbeendiagnosedwithHPVorgenitalwarts.Thenurseshouldcounselhertoreceive regularexaminationsbecausethisvirusmakesheratahigherriskfor cancer. a. Uterine b. Cervical c. Ovarian d. Endometrial NURSINGTB.COM ANS: B HPVisthevirusresponsibleformostcasesofcervicalcancer,nottheotheroptions. DIF: Cognitive Level: Applying(Application) MSC: Client Needs: Health Promotion and Maintenance 46. Duringaninternalexamination,thenursenoticesthatthecervixbulgesoutsidetheintroituswhenthe patient is asked to strain. The nurse will document thisas: a. Uterine prolapse, graded firstdegree. b. Uterine prolapse, graded seconddegree. c. Uterine prolapse, graded thirddegree. d. A normalfinding. ANS: B Thecervixshouldnotbefoundtobulgeintothevagina.Uterineprolapseisgradedasfollows:firstdegreethe cervix appears at the introitus with straining; second degreethe cervix bulges outside the introitus with straining;andthirddegreethewholeuterusprotrudes,evenwithoutstraining(essentially,theuterusisinside out). DIF: Cognitive Level: Applying (Application) MSC: Client Needs: Physiologic Integrity: Physiologic Adaptation 47. A35-year-oldwomanisattheclinicforagynecologicexamination.Duringtheexamination,sheasksthe nurse,HowoftendoIneedtohavethisPaptestdone?Whichreplybythenurseiscorrect? a. It depends. Do yousmoke? b. APaptestneedstobeperformedannuallyuntilyouare65yearsofage. c. IfyouhavetwoconsecutivenormalPaptests,thenyoucanwait5yearsbetweentests. d. Afterage30years,ifyouhavethreeconsecutivenormalPaptests,thenyoumaybescreenedevery 2 to 3years. ANS: D CervicalcancerscreeningwiththePaptestcontinuesannuallyuntilage30years.Afterage21,regardlessof sexualhistoryoractivity,womenshouldbescreenedevery3yearsuntilage30,thenevery5yearsuntilage 65. DIF: Cognitive Level: Applying (Application) MSC:ClientNeeds:HealthPromotionandMNaiUntReSnIaNncGeTB.COM MULTIPLE RESPONSE 1.Thenurseispalpatinganovarianmassduringaninternalexaminationofa63-year-oldwoman.Which findingsofthemassscharacteristicswouldsuggestthepresenceofanovariancyst?Selectallthatapply. a. Heavy andsolid b. Mobile andfluctuant c. Mobile andsolid d. Fixed e. Smooth andround f. Poorlydefined ANS: B,E An ovarian cyst (fluctuant ovarian mass) is usually asymptomatic and would feel like a smooth, round, fluctuant,mobile,nontendermassontheovary.Amassthatisheavy,solid,fixed,andpoorlydefinedsuggests malignancy. A benign mass may feel mobile andsolid. DIF: Cognitive Level: Applying (Application) MSC: Client Needs: Physiologic Integrity: Physiologic Adaptation NURSINGTB.COM Chapter 28: The Complete Health Assessment: Adult MULTIPLE CHOICE 1. An85-year-oldmanhascomeinforaphysicalexamination,andthenursenoticesthatheusesacane.When documentinggeneralappearance,thenurseshoulddocumentthisinformationunderthesectionthatcovers: a. Posture. b. Mobility. c. Mood andaffect. d. Physicaldeformity. ANS: B Useofassistivedeviceswouldbedocumentedunderthemobilitysection.Theotherresponsesareallother categories of the general appearance section of the healthhistory. DIF: Cognitive Level: Remembering (Knowledge) MSC: Client Needs: Safe and Effective Care Environment: Management of Care 2. Thenurseisperformingavisionexamination.Whichofthesechartsismostwidelyusedforvision examinations? a. Snellen b. Shetllen c. Smoollen d. Schwellon NURSINGTB.COM ANS: A TheSnelleneyechartismostwidelyusedforvisionexaminations.Theotheroptionsarenottestsforvision examinations. DIF: Cognitive Level: Remembering (Knowledge) MSC: Client Needs: Safe and Effective Care Environment: Management of Care 3. Afterthehealthhistoryhasbeenobtainedandbeforebeginningthephysicalexamination,thenurseshould first ask the patientto: a. Empty thebladder. b. Completelydisrobe. c. Lie on the examinationtable. d. Walk around theroom. ANS: A Beforebeginningtheexamination,thenurseshouldaskthepersontoemptythebladder(savethespecimenif needed), disrobe except for underpants, put on a gown, and sit with the legs dangling off side of the bed or table. DIF: Cognitive Level: Remembering (Knowledge) MSC: Client Needs: Safe and Effective Care Environment: Management of Care 4. Duringacompletehealthassessment,howwouldthenursetestthepatientshearing? a. Observing how the patient participates in normalconversation b. Using the whispered voicetest c. Using the Weber and Rinnetests d. Testing with anaudiometer ANS: B NURSINGTB.COM Duringthecompletehealthassessment,thenurseshouldtesthearingwiththewhisperedvoicetest.Theother options are notcorrect. DIF: Cognitive Level: Applying (Application) MSC: Client Needs: Safe and Effective Care Environment: Management of Care 5. Apatientstates,WheneverIopenmymouthrealwide,Ifeelthispoppingsensationinfrontofmyears.To further examine this, the nursewould: a. Placethestethoscopeoverthetemporomandibularjoint,andlistenforbruits. b. Placethehandsoverhisears,andaskhimtoopenhismouthreallywide. c. Placeonehandonhisforeheadandtheotheronhisjaw,andaskhimtotrytoopenhismouth. d. Placeafingeronhistemporomandibularjoint,andaskhimtoopenandclosehismouth. ANS: D Thenurseshouldpalpatethetemporomandibularjointbyplacinghisorherfingersoverthejointastheperson opens and closes themouth. DIF: Cognitive Level: Applying (Application) MSC: Client Needs: Safe and Effective Care Environment: Management of Care 6. Thenursehasjustcompletedanexaminationofapatientsextraocularmuscles.Whendocumentingthe findings,thenurseshoulddocumenttheassessmentofwhichcranialnerves? a. II, III, andVI b. II, IV, andV c. III, IV, andV d. III, IV, andVI ANS:D ExtraocularmusclesareinnervatedbycranialnervesIII,IV,andVI. DIF: Cognitive Level: Applying(Application) MSC: Client Needs: Safe and Effective Care Environment: Management of Care 7. Apatientsuvularaisesmidlinewhenshesaysahh,andshehasapositivegagreflex.Thenursehasjust tested which cranialnerves? a. IX andX b. IX andXII c. X andXII d. XI andXII NURSINGTB.COM ANS: A CranialnervesIXandXarebeingtestedbyhavingthepatientsayahh,notingthemobilityoftheuvula,and when assessing the patients gagreflex. DIF: Cognitive Level: Applying (Application) MSC: Client Needs: Safe and Effective Care Environment: Management of Care 8. Duringanexamination,thenursenoticesthatapatientisunabletostickouthistongue.Whichcranialnerve is involved with the successful performance of thisaction? a. I b. V c. XI d. XII ANS: D CranialnerveXIIenablesthepersontostickouthisorhertongue. DIF: Cognitive Level: Understanding(Comprehension) MSC: Client Needs: Safe and Effective Care Environment: Management of Care 9. Apatientisunabletoshrughershouldersagainstthenursesresistanthands.Whatcranialnerveisinvolved with successful shouldershrugging? a. VII b. IX c. XI d. XII ANS: C CranialnerveXIenablesthepatienttoshrughershouldersagainstresistance. DIF:CognitiveLevel:Understanding(ComprNehUeRnSsiIoNnG)TB.COM MSC: Client Needs: Safe and Effective Care Environment: Management of Care 10. During an examination, a patient has just successfully completed the finger-to-nose and the rapid- alternating-movementstestsandisabletoruneachheeldowntheoppositeshin.Thenursewillconcludethat thepatients function isintact. a. Occipital b. Cerebral c. Temporal d. Cerebellar ANS:D Thenurseshouldtestcerebellarfunctionoftheupperextremitiesbyusingthefinger-to-nosetestorrapid- alternating-movementstest.Thenurseshouldtestcerebellarfunctionofthelowerextremitiesbyaskingthe person to run each heel down the oppositeshin. DIF: Cognitive Level: Applying (Application) MSC: Client Needs: Safe and Effective Care Environment: Management of Care 11. When the nurse performs the confrontation test, the nurse hasassessed: a. Extraocular eye muscles(EOMs). b. Pupils(pupilsequal,round,reactivetolight,andaccommodation[PERRLA]). c. Nearvision. d. Visualfields. ANS: D Theconfrontationtestassessesvisualfields.Theotheroptionsarenottestedwiththeconfrontationtest. DIF: Cognitive Level: Applying(Application) MSC: Client Needs: Safe and Effective Care Environment: Management of Care 12. Which statement is true regarding the complete physicalassessment? a. The male genitalia should be examined in the supineposition. b. Thepatientshouldbeinthesittingpositionforexaminationoftheheadandneck. c. Thevitalsigns,height,andweightshouldbeobtainedattheendoftheexamination. NURSINGTB.COM d. Topromoteconsistencybetweenpatients,theexaminershouldnotvarytheorderofthe assessment. ANS: B Theheadandneckshouldbeexaminedinthesittingpositiontobestpalpatethethyroidandlymphnodes.The male patient should stand during an examination of the genitalia. Vital signs are measured early in the assessment.Thesequenceoftheassessmentmayneedtovaryaccordingtodifferentpatientsituations. DIF: Cognitive Level: Remembering (Knowledge) MSC: Client Needs: Safe and Effective Care Environment: Management of Care 13. Which of these is included in an assessment of generalappearance? a. Height b. Weight c. Skincolor d. Vitalsigns ANS: C General appearance includes items such as level of consciousness, skin color, nutritional status, posture, mobility,facialexpression,moodandaffect,speech,hearing,andpersonalhygiene.Height,weight,andvital signs are consideredmeasurements. DIF: Cognitive Level: Remembering (Knowledge) MSC: Client Needs: Safe and Effective Care Environment: Management of Care 14. The nurse should wear gloves for which of theseexaminations? a. Measuring vitalsigns b. Palpation of thesinuses c. Palpation of the mouth andtongue d. Inspectionoftheeyewithanophthalmoscope ANS:C Glovesshouldbewornwhentheexaminerisexposedtothepatientsbodyfluids. DIF: Cognitive Level: Applying(Application) MSC:ClientNeeds:SafeandEffectiveCareENnUvRirSoInNmGeTnBt:.CSOafMetyandInfectionControl 15. The nurse should use which location for eliciting deep tendonreflexes? a. Achilles b. Femoral c. Scapular d. Abdominal ANS:A Deeptendonreflexesareelicitedinthebiceps,triceps,brachioradialis,patella,andAchillesheel. DIF: Cognitive Level: Understanding(Comprehension) MSC: Client Needs: Safe and Effective Care Environment: Management of Care 16. Duringaninspectionofapatientsface,thenursenoticesthatthefacialfeaturesaresymmetric.Thisfinding indicates which cranial nerve isintact? a. VII b. IX c. XI d. XII ANS: A Cranial nerve VII is responsible for facial symmetry. DIF:CognitiveLevel:Understanding(Comprehension) MSC: Client Needs: Safe and Effective Care Environment: Management of Care 17. During inspection of the posterior chest, the nurse should assessfor: a. Symmetricexpansion. b. Symmetry of shoulders andmuscles. c. Tactilefremitus. d. Diaphragmaticexcursion. ANS: B NURSINGTB.COM Duringaninspectionoftheposteriorchest,thenurseshouldinspectforsymmetryofshouldersandmuscles, configuration of the thoracic cage, and skin characteristics. Symmetric expansion and tactile fremitus are assessed with palpation; diaphragmatic excursion is assessed withpercussion. DIF: Cognitive Level: Understanding (Comprehension) MSC: Client Needs: Safe and Effective Care Environment: Management of Care 18. Duringanexamination,thepatienttellsthenursethatshesometimesfeelsasifobjectsarespinningaround her. The nurse would document that she occasionallyexperiences: a. Vertigo. b. Tinnitus. c. Syncope. d. Dizziness. ANS: A Vertigo is the sensation of a person moving around in space (subjective) or of the person sensing objects movingaroundhimorher(objective)andisaresultofadisturbanceofequilibratoryapparatus(seeChapter 24). DIF: Cognitive Level: Applying (Application) MSC: Client Needs: Safe and Effective Care Environment: Management of Care 19. Apatienttellsthenurse,SometimesIwakeupatnightandIhaverealtroublebreathing.Ihavetositupin bedtogetagoodbreath.Whendocumentingthisinformation,thenursewouldnote: a. Orthopnea. b. Acuteemphysema. c. Paroxysmal nocturnaldyspnea. d. Acute shortness of breathepisode. ANS: C Paroxysmalnocturnaldyspneaoccurswhenthepatientawakensfromsleepwithshortnessofbreathandneeds to be upright to achieve comfort (see Chapter19). DIF: Cognitive Level: Analyzing (Analysis) MSC: Client Needs: Safe and Effective Care Environment: Management of Care 20. Duringtheexaminationofapatient,thenursenoticesthatthepatienthasseveralsmall,flatmaculesonthe posteriorportionofherthorax.Thesemaculesarelessthan1cmwide.Anothernameforthesemaculesis: NURSINGTB.COM a. Warts. b. Bullae. c. Freckles. d. Papules. ANS: C Amaculeissolelyalesionwithcolorchange,flatandcircumscribed,lessthan1cm.Maculesarealsoknown as freckles (see Chapter13). DIF: Cognitive Level: Understanding (Comprehension) MSC: Client Needs: Safe and Effective Care Environment: Management of Care 21. Duringanexamination,thenursenoticesthatapatientslegsturnwhitewhentheyareraisedabovethe patients head. The nurse shouldsuspect: a. Lymphedema. b. Raynauddisease. c. Chronicarterialinsufficiency. d. Chronicvenousinsufficiency. ANS: C Elevationalpallor(striking)indicatesarterialinsufficiency(seeChapter21). DIF: Cognitive Level: Analyzing(Analysis) MSC: Client Needs: Safe and Effective Care Environment: Management of Care 22. Thenursedocumentsthatapatienthascoarse,thickenedskinandbrowndiscolorationoverthelowerlegs. Pulses are present. This finding is probably the resultof: a. Lymphedema. b. Raynauddisease. c. Chronicarterialinsufficiency. d. Chronicvenousinsufficiency. ANS: D NURSINGTB.COM Chronicvenousinsufficiencywouldexhibitfirmbrawnyedema,coarsethickenedskin,normalpulses,and brown discoloration (see Chapter21). DIF: Cognitive Level: Analyzing (Analysis) MSC: Client Needs: Safe and Effective Care Environment: Management of Care 23. Thenursenoticesthatapatienthasulcerationsonthetipsofthetoesandonthelateralaspectoftheankles. This findingindicates: a. Lymphedema. b. Raynauddisease. c. Arterialinsufficiency. d. Venousinsufficiency. ANS: C Ulcerationsonthetipsofthetoesandlateralaspectoftheanklesareindicativeofarterialinsufficiency(see Chapter21). DIF: Cognitive Level: Analyzing (Analysis) MSC: Client Needs: Safe and Effective Care Environment: Management of Care 24. Thenursehasjustrecordedapositiveiliopsoastestonapatientwhohasabdominalpain.Thistestisused to confirma(n): a. Inflamedliver. b. Perforatedspleen. c. Perforatedappendix. d. Enlargedgallbladder. ANS: C Aninflamedorperforatedappendixirritatestheiliopsoasmuscle,producingpainintheRLQ. DIF: Cognitive Level: Applying(Application) MSC: Client Needs: Safe and Effective Care Environment: Management of Care 25. The nurse will measure a patients near vision with whichtool? a. Snellen eye chart withletters b. Snellen Echart c. Jaegercard d. Ophthalmoscope NURSINGTB.COM ANS: C TheJaegercardisusedtomeasurenearvision(seeChapter15). DIF: Cognitive Level: Applying(Application) MSC: Client Needs: Safe and Effective Care Environment: Management of Care 26. If the nurse records the results to the Hirschberg test, the nursehas: a. Tested the patellarreflex. b. Assessed forappendicitis. c. Tested the corneal lightreflex. d. Assessed forthrombophlebitis. ANS: C TheHirschbergtestassessesthecorneallightreflex(seeChapter15). DIF: Cognitive Level: Applying(Application) MSC: Client Needs: Safe and Effective Care Environment: Management of Care 27. Duringtheexaminationofapatientsmouth,thenurseobservesanodularbonyridgedownthemiddleof the hard palate. The nurse would chart this findingas: a. Cheilosis. b. Leukoplakia. c. Ankyloglossia. d. Toruspalatinus. ANS: D Anormalvariationofthehardpalateisanodularbonyridgedownthemiddleofthehardpalate;thisvariation is termed torus palatinus (see Chapter17). DIF: Cognitive Level: Applying (Application) MSC: Client Needs: Safe and Effective Care Environment: Management of Care 28. Duringexamination,thenursefindsthataNpUatRieSnItNiGsTuBna.CblOeMtodistinguishobjectsplacedinhishand.The nurse woulddocument: a. Stereognosis. b. Astereognosis. c. Graphesthesia. d. Agraphesthesia. ANS: B Astereognosisistheinabilitytoidentifycorrectlyanobjectplacedinthehand(seeChapter24). DIF: Cognitive Level: Applying(Application) MSC: Client Needs: Safe and Effective Care Environment: Management of Care 29. Aftertheexaminationofaninfant,thenursedocumentsopisthotonos.Thenurserecognizesthatthis finding often occurswith: a. Cerebralpalsy. b. Meningealirritation. c. Lowermotorneuronlesion. d. Uppermotorneuronlesion. ANS: B Opisthotonosisaformofspasminwhichtheheadisarchedback,andastiffnessoftheneckandanextension of the arms and legs are observed. Opisthotonus occurs with meningeal or brainstem irritation (see Chapter 23). DIF: Cognitive Level: Analyzing (Analysis) MSC: Client Needs: Safe and Effective Care Environment: Management of Care 30. Afterassessingafemalepatient,thenursenoticesflesh-colored,soft,pointed,moist,papulesina cauliflower-like patch around her introitus. This finding is mostlikely: a. Urethralcaruncle. b. Syphiliticchancre. c. Herpes simplexvirus. d. Humanpapillomavirus. NURSINGTB.COM ANS: D Humanpapillomavirusappearsinaflesh-colored,soft,moist,cauliflower-likepatchofpapules(seeChapter 27). DIF: Cognitive Level: Analyzing (Analysis) MSC: Client Needs: Safe and Effective Care Environment: Management of Care 31. Whilerecordinginapatientsmedicalrecord,thenursenoticesthatapatientsHematestresultsarepositive. This finding means that thereis(are): a. Crystalsinhisurine. b. Parasitesinhisstool. c. Occult blood in hisstool. d. Bacteria in hissputum. ANS: C If a stool is Hematest positive, then it indicates the presence of occult blood (see Chapter 22). DIF: Cognitive Level: Analyzing (Analysis) MSC: Client Needs: Safe and Effective Care Environment: Management of Care 32. Whileexamininga48-year-oldpatientseyes,thenursenoticesthathehadtomovethehandheldvision screener farther away from his face. The nurse wouldsuspect: a. Myopia. b. Omniopia. c. Hyperopia. d. Presbyopia. ANS: D Presbyopia,thedecreaseinpowerofaccommodationwithaging,issuggestedwhenthehandheldvision screener card is moved farther away (see Chapter15). DIF: Cognitive Level: Analyzing (Analysis) MSC: Client Needs: Safe and Effective Care Environment: Management of Care NURSINGTB.COM Chapter 29: The Complete Physical Assessment: Infant, Child, andAdolescent MULTIPLE CHOICE 1. A5-year-oldchildisintheclinicforacheckup.Thenursewouldexpecthimto: a. Need to be held on his motherslap. b. Be able to sit on the examinationtable. c. Beabletostandonthefloorfortheexamination. d. Beabletoremainaloneintheexaminationroom. ANS: B At4or5yearsold,achildusuallyfeelscomfortableontheexaminationtable.Olderinfantsandyoung childrenaged6monthsto2or3yearsshouldbepositionedintheparentslap. DIF: Cognitive Level: Understanding (Comprehension) MSC: Client Needs: Safe and Effective Care Environment: Management of Care 2. Whichstatementistrueregardingtherecordingofdatafromthehistoryandphysicalexamination? NURSINGTB.COM a. Use long, descriptive sentences to documentfindings. b. Recordthedataassoonaspossibleaftertheinterviewandphysicalexamination. c. Iftheinformationisnotdocumented,thenitcanbeassumedthatitwasdoneasastandardofcare. d. Theexaminershouldavoidtakinganynotesduringthehistoryandexaminationbecauseofthe possibility of decreasing the rapport with thepatient. ANS: B The data from the history and physical examination should be recorded as soon after the event as possible. Fromalegalperspective,ifitisnotdocumented,thenitwasnotdone.Briefnotesshouldbetakenduringthe examination.Whendocumenting,thenurseshoulduseshort,clearphrasesandavoidredundantphrasesand descriptions. DIF: Cognitive Level: Applying (Application) MSC: Client Needs: Safe and Effective Care Environment: Management of Care 3. Whenassessingtheneonate,thenurseshouldtestforhipstabilitywithwhichmethod? a. Eliciting the Mororeflex b. Performing the Rombergtest c. Checking for the Ortolanisign d. Assessingthesteppingreflex ANS:C ThenurseshouldtestforhipstabilityintheneonatebytestingfortheOrtolanisign.Theothertestsarenot appropriate for testing hipstability. DIF: Cognitive Level: Applying (Application) MSC: Client Needs: Health Promotion and Maintenance 4. Afemalepatienttellsthenursethatshehasfourchildrenandhashadthreepregnancies.Howshouldthe nurse documentthis? a. Gravida 3, para4 b. Gravida 4, para3 c. Thisinformationcannotbedocumentedusingthetermsgravidaandpara. d. ThepatientseemstobeconfusedaNbUouRtShINowGTmBa.CnyOtMimesshehasbeenpregnant. ANS: A Gravidareferstothenumberofpregnancies,andparareferstothenumberofchildren.Onepregnancywas withtwins. DIF: Cognitive Level: Applying (Application) MSC: Client Needs: Safe and Effective Care Environment: Management of Care 5. Thenurseisdocumentingtheassessmentofaninfant.Duringtheabdominalassessment,thenursenoticeda veryloudsplashauscultatedovertheupperabdomenwhenthenurserockedherfromsidetoside.Thisfinding wouldindicate: a. Epigastrichernia. b. Pyloricobstruction. c. Hypoactive bowelsounds. d. Hyperactive bowelsounds. ANS: D Asuccussionsplash,whichisunrelatedtoperistalsis,isaveryloudsplashauscultatedovertheupperabdomen whentheinfantisrockedsidetoside.Itindicatesincreasedairandfluidinthestomachasobservedwith pyloric obstruction or large hiatus hernia (see Chapter21). DIF: Cognitive Level: Analyzing (Analysis) MSC: Client Needs: Safe and Effective Care Environment: Management of Care 6. Whichoftheseactionsismostappropriatetoperformona9-month-oldinfantatawell-childcheckup? a. Testing for Ortolanisign b. Assessment forstereognosis c. Blood pressuremeasurement d. Assessmentforthepresenceofthestartlereflex ANS:A Untiltheageof12months,theinfantshouldbeassessedforOrtolanisign.IfOrtolanisignispresent,thenit couldindicatethepresenceofadislocatedhip.Theothertestsarenotappropriatefora9-month-oldchild. DIF: Cognitive Level: Applying (Application) MSC: Client Needs: Safe and Effective Care Environment: Management of Care NURSINGTB.COM Chapter 30: Bedside Assessment and ElectronicDocumentation MULTIPLE CHOICE 1. Atthebeginningofroundswhenenteringtheroom,whatshouldthenursedofirst? a. Check the intravenous (IV) infusion site for swelling orredness. b. Check the infusion pump settings foraccuracy. c. Makeeyecontactwiththepatient,andintroducehimorherselfasthepatientsnurse. d. Offer the patient something todrink. ANS: C Whenenteringapatientsroom,thenurseshouldmakedirecteyecontact,withoutbeingdistractedbyIV pumpsandotherequipment,andintroducehimorherselfasthepatientsnurse. DIF: Cognitive Level: Analyzing (Analysis) MSC: Client Needs: Safe and Effective Care Environment: Management of Care 2. Duringanassessment,thenurseisunabletopalpatepulsesintheleftlowerleg.Whatshouldthenursedo next? NURSINGTB.COM a. Document that the pulses arenonpalpable. b. Reassess the pulses in 1hour. c. Askthepatientturntotheside,andthenpalpateforthepulsesagain. d. Use a Doppler device to assess thepulses. ANS: D ThenurseshouldbepreparedtoassesspulsesinthelowerextremitiesbyDopplermeasurementiftheycannot be detected bypalpation. DIF: Cognitive Level: Applying (Application) MSC: Client Needs: Safe and Effective Care Environment: Management of Care 3. Duringamorningassessment,thenursenoticesthatapatientsurineoutputisbelowtheexpectedamount. What should the nurse donext? a. Obtain an order for a Foleycatheter. b. Obtain an order for a straightcatheter. c. Perform a bladder scantest. d. Refer the patient to anurologist. ANS: C Ifurineoutputisbelowtheexpectedvalue,thenthenurseshouldperformabladderscanaccordingto institutional policy to check forretention. DIF: Cognitive Level: Applying (Application) MSC: Client Needs: Safe and Effective Care Environment: Management of Care 4. Whatshouldthenurseassessbeforeenteringthepatientsroomonmorningrounds? a. Posted conditions, such as isolationprecautions b. Patients input and output chart from the previousshift c. Patients generalappearance d. Presence of any visitors in theroom ANS: A NURSINGTB.COM Onthewaytothepatientsroom,thenurseshouldassessthepresenceofconditionssuchasisolation precautions, latex allergies, or fallprecautions. DIF:CognitiveLevel:Applying(Application) MSC: Client Needs: Safe and Effective Care Environment: Safety and Infection Control 5. ThenursehasadministeredapainmedicationtoapatientbyanIVinfusion.Thenurseshouldreassessthe patientsresponsetothepainmedicationwithin minutes. a. 5 b. 15 c. 30 d. 60 ANS: B Ifpainmedicationisgiven,thenthenurseshouldreassessthepatientsresponsein15minutesforIV administration or 1 hour for oraladministration. DIF: Cognitive Level: Applying (Application) MSC: Client Needs: Physiologic Integrity: Pharmacologic and Parenteral Therapies 6. Duringanassessmentofahospitalizedpatient,thenursepinchesafoldofskinundertheclavicleoronthe forearm to testthe: a. Mobility andturgor. b. Patients response topain. c. Percentage of the patients fat-to-muscleratio. d. Presence ofedema. ANS: A Pinchingafoldofskinundertheclavicleorontheforearmisdonebythenursetodeterminemobilityand turgor. DIF: Cognitive Level: Applying (Application) MSC: Client Needs: Safe and Effective Care Environment: Management of Care 7. Whenassessingtheneurologicsystemofahospitalizedpatientduringmorningrounds,thenurseshould include which of these during theassessment? a. Bloodpressure NURSINGTB.COM b. Patients rating of pain on a scale of 1 to10 c. Patients ability tocommunicate d. Patientspersonalhygienelevel ANS:C Assessmentofapatientsabilitytocommunicateispartoftheneurologicassessment.Bloodpressureandpain ratingaremeasurements,andpersonalhygieneisassessedundergeneralappearance. DIF: Cognitive Level: Applying (Application) MSC: Client Needs: Safe and Effective Care Environment: Management of Care 8. Whenassessingapatientsgeneralappearance,thenurseshouldincludewhichquestion? a. Is the patients muscle strength equal in botharms? b. Is ptosis or facial drooppresent? c. Does the patient appropriately respond toquestions? d. Are the pupils equal in reaction andsize? ANS: C Assessingwhetherthepatientappropriatelyrespondstoquestionsisacomponentofanassessmentofthe patientsgeneralappearance.Theotheranswersreflectcomponentsoftheneurologicexamination. DIF: Cognitive Level: Applying (Application) MSC: Client Needs: Safe and Effective Care Environment: Management of Care 9. Whenassessingapatientinthehospitalsetting,thenurseknowswhichstatementtobetrue? a. The patient will need a brief assessment at least every 4hours. b. Thepatientwillneedaconsistent,specializedexaminationevery8hoursthatfocusesoncertain parameters. c. Thepatientwillneedacompletehead-to-toephysicalexaminationevery24hours. d. Mostpatientsrequireaminimalexaminationeachshiftunlesstheyareincriticalcondition. ANS: B Inahospitalsetting,thepatientdoesnotrequireacompletehead-to-toephysicalexaminationduringevery24- hourstay.Thepatientdoes,however,requireaconsistentspecializedexaminationevery8hoursthatfocuses on certain parameters. NURSINGTB.COM DIF: Cognitive Level: Applying (Application) MSC: Client Needs: Safe and Effective Care Environment: Management of Care 10. The nurse is giving report to the next shift and is using the situation, background, assessment, recommendation(SBAR)frameworkforcommunication.WhichofthesestatementsreflectstheBackground portion of thereport? a. Im worried that his gastrointestinal bleeding is gettingworse. (assessment) b. We need an order foroxygen. (recommendation) c. MynameisMs.Smith,andImgivingthereportonMrs.Xinroom1104. (situation) d. He is 4 days postoperative, and his incision is open toair. (background) ANS: D During the Background portion, the nurse should state data pertinent to the moments problem such as the condition of the patients incision. During the Situation portion, the nurse provides his or her name and the patients name. During the Assessment portion, the nurse states what he or she thinks is happening (e.g., gastrointestinalbleeding).DuringtheRecommendationportion,thenurseshouldofferprobablesolutionsor orders that may beimplemented. DIF: Cognitive Level: Analyzing (Analysis) MSC: Client Needs: Safe and Effective Care Environment: Safety and Infection Control MULTIPLE RESPONSE 1. ThenurseisassessingtheIVinfusionatthebeginningoftheshift.Whichfactorsshouldbeincludedinthe assessment of the infusion? Select all thatapply. a. Proper IV solution is infusing, according to the physiciansorders. b. TheIVsolutionisinfusingattheproperrate,accordingtophysiciansorders. c. Theinfusionisproper,accordingtothenursesassessmentofthepatientsneeds. d. Capillary refill in the fingers is checked andnoted. e. The IV site date isnoted. f. Whether the patient is sufficiently voiding isnoted. ANS: A, B, C, E ThenurseshouldverifythattheproperIVsolutionishangingandisflowingattheproperrateaccordingtothe physicians orders and the nurses own assessment of the patients needs. In addition, the nurse should note the date of the IV site and surrounding skin condition. Checking capillary refill is part of the cardiovascular assessment;checkingthepatientsvoidingispNarUtRofSIthNeGgTeBn.iCtoOuMrinaryassessment. DIF: Cognitive Level: Applying (Application) MSC: Client Needs: Physiologic Integrity: Pharmacologic and Parenteral Therapies 2. Thenurseiscompletinganassessmentonapatientwhowasjustadmittedfromtheemergencydepartment. Whichassessmentfindingswouldrequireimmediateattention?Selectallthatapply. a. Temperature: 38.6C b. Systolic blood pressure: 150 mmHg c. Respiratory rate: 22 breaths perminute d. Heart rate: 130 beats perminute e. Oxygen saturation:95% f. Sudden restlessness ANS: A, D,F The following examination findings require immediate attention: High or low temperature: (36.1 C or 37.8 C) Highorlowbloodpressure:(systolicpressure90mmHgor160mmHg) Highorlownumberofrespirations: (12 or28breathsperminute) Highorlowheartrate:(60or90beatsperminute) Oxygen saturation:92% Suddenrestlessnessoranxiety,alteredlevelofconsciousness,confusion,ordifficultyinarousing DIF: Cognitive Level: Applying(Application) MSC: Client Needs: Physiologic Integrity: Physiologic Adaptation NURSINGTB.COM Chapter 31: The Pregnant Woman MULTIPLE CHOICE 1. Whichofthesestatementsbestdescribestheactionofthehormoneprogesteroneduringpregnancy? a. Progesterone produces the hormone human chorionicgonadotropin. b. Duct formation in the breast is stimulated byprogesterone. c. Progesteronepromotessloughingoftheendometrialwall. d. Progesteronemaintainstheendometriumaroundthefetus. ANS: D Progesteronepreventsthesloughingoftheendometrialwallandmaintainstheendometriumaroundthefetus. Progesteroneincreasesthealveoliinthebreastandkeepstheuterusinaquiescentstate.Theotheroptionsare notcorrect. DIF:CognitiveLevel:Remembering(Knowledge) MSC: Client Needs:General 2. Afemalepatienthasnausea,breasttenderness,fatigue,andamenorrhea.Herlastmenstrualperiodwas6 weeksago.ThenurseinterpretsthatthispatieNntUiRsSexINpeGrTieBn.cCiOngM signsofpregnancy. a. Positive b. Possible c. Probable d. Presumptive ANS:D Presumptive signs of pregnancy are those that the woman experiences and include amenorrhea, breast tenderness,fatigue,nausea,andincreasedurinaryfrequency.Probablesignsarethosethataredetectedbythe examiner, such as an enlarged uterus or changes in the cervix. Positive signs of pregnancy are those that documentdirectevidenceofthefetussuchasfetalhearttonesorpositivecardiacactivityonultrasound. DIF: Cognitive Level: Applying (Application) MSC: Client Needs: Health Promotion and Maintenance 3. Awomanwhois8weekspregnantisvisitingtheclinicforacheckup.Hersystolicbloodpressureis30mm Hg higher than her prepregnancy systolic blood pressure. The nurseshould: a. Consider this a normalfinding. b. Expectthebloodpressuretodecreaseastheestrogenlevelsincreasethroughoutthepregnancy. c. Considerthisanabnormalfindingbecausebloodpressureistypicallyloweratthispointinthe pregnancy. d. Recommendthatshedecreasehersaltintakeinanattempttodecreaseherperipheralvascular resistance. ANS: C Duringtheseventhgestationalweek,bloodpressurebeginstodropasaresultoffallingperipheralvascular resistance. Early in the first trimester, blood pressure values are similar to those of prepregnancy measurements.Inthiscase,thewomansbloodpressureishigherthanitshouldbe. DIF: Cognitive Level: Analyzing (Analysis) MSC: Client Needs: Health Promotion and Maintenance 4. Apatientisbeingseenattheclinicforher10-weekprenatalvisit.Sheaskswhenshewillbeabletohearthe babys heartbeat. The nurse shouldreply: a. The babys heartbeat is not usually heard until the secondtrimester. b. Thebabysheartbeatmaybeheardanywherefromtheninthtothetwelfthweek. c. ItisoftendifficulttohearthehearNtbUeaRtSaItNtGhiTsBp.oCiOntM,butwecantry. d. Itisnormaltoheartheheartbeatat6weeks.Wemaybeabletohearittoday. ANS: B FetalhearttonescanbeheardwiththeuseoftheDopplerdevicebetween9and12weeks.Theotherresponses areincorrect. DIF: Cognitive Level: Applying (Application) MSC: Client Needs: Health Promotion and Maintenance 5. Apatientwhoisinherfirsttrimesterofpregnancytellsthenursethatsheisexperiencingsignificantnausea and vomiting and asks when it will improve. The nurse shouldreply: a. Did your mother have significant nausea andvomiting? b. Many women experience nausea and vomiting until the thirdtrimester. c. Usually,bythebeginningofthesecondtrimester,thenauseaandvomitingimprove. d. Atapproximatelythetimeyoubegintofeelthebabymove,thenauseaandvomitingwillsubside. ANS: C Thenausea,vomiting,andfatigueofpregnancyimprovebythe12thweek.Quickening,whenthemother recognizes fetal movement, occurs at approximately 18 to 20weeks. DIF: Cognitive Level: Analyzing (Analysis) MSC: Client Needs: Physiologic Integrity: Physiologic Adaptation 6. Duringtheexaminationofawomaninhersecondtrimesterofpregnancy,thenursenoticesthepresenceofa smallamountofyellowdrainagefromthenipples.Thenurseknowsthatthisis: a. An indication that the womans milk is comingin. b. A sign of possible breast cancer in a pregnantwoman. c. Mostlikelycolostrumandconsideredanormalfindingatthisstageofthepregnancy. d. Tooearlyinthepregnancyforlactationtobeginandrefersthewomantoaspecialist. ANS: C Duringthesecondtrimester,colostrum,theprecursorofmilk,maybeexpressedfromthenipples.Colostrumis yellow and contains more minerals and protein but less sugar and fat than mature milk. The other options are incorrect. DIF: Cognitive Level: Analyzing (Analysis) NURSINGTB.COM MSC: Client Needs: Health Promotion and Maintenance 7. Awomaninhersecondtrimesterofpregnancycomplainsofheartburnandindigestion.Whendiscussing thiswiththewoman,thenurseconsiderswhichexplanationfortheseproblems? a. Toneandmotilityofthegastrointestinaltractincreaseduringthesecondtrimester. b. Sluggishemptyingofthegallbladder,resultingfromtheeffectsofprogesterone,oftencauses heartburn. c. Lowerbloodpressureatthistimedecreasesbloodflowtothestomachandgastrointestinaltract. d. Enlarginguterusandalteredesophagealsphinctertonepredisposethewomantohaveheartburn. ANS: D Stomachdisplacementfromtheenlarginguterusplusalteredesophagealsphincterandgastrictoneasaresult of progesterone predispose the woman to heartburn. The tone and motility of the gastrointestinal tract are decreased,notincreased,duringpregnancy.Emptyingofthegallbladdermaybecomemoresluggishduring pregnancy but is not related to indigestion. Rather, some women are predisposed to gallstone formation. A lowerbloodpressuremayoccurduringthesecondsemester,butitdoesnotaffectdigestion. DIF: Cognitive Level: Understanding (Comprehension) MSC: Client Needs: Physiologic Integrity: Physiologic Adaptation 8. Apatientwhois20weekspregnanttellsthenursethatshefeelsmoreshortnessofbreathasherpregnancy progresses. The nurse recognizes which statement to betrue? a. High levels of estrogen cause shortness ofbreath. b. Feelings of shortness of breath are abnormal duringpregnancy. c. Hormones of pregnancy cause an increased respiratoryeffort. d. Thepatientshouldgetmoreexerciseinanattempttoincreaseherrespiratoryreserve. ANS: C Progesteroneandestrogencauseanincreaseinrespiratoryeffortduringpregnancybyincreasingtidalvolume. Increasedtidalvolumecausesaslightdropinpartialpressureofarterialcarbondioxide(PaCO2),causingthe woman to have dyspnea occasionally. The other options are notcorrect. DIF: Cognitive Level: Understanding (Comprehension) MSC: Client Needs: Physiologic Integrity: Physiologic Adaptation 9. Thenurseauscultatesafunctionalsystolicmurmur,gradeII/IV,onawomaninweek30ofherpregnancy. Theremainderofherphysicalassessmentiswithinnormallimits.Thenursewould: a. Consider this finding abnormal, and refer her for additionalconsultation. NURSINGTB.COM b. Askthewomantorunbrieflyinplaceandthenassessforanincreaseinintensityofthemurmur. c. Knowthatthisfindingisnormalandisaresultoftheincreaseinbloodvolumeduringpregnancy. d. Askthewomantorestrictheractivitiesandreturntotheclinicin1weekforre-evaluation. ANS: C Becauseoftheincreaseinbloodvolume,afunctionalsystolicmurmur,gradeII/IVorless,canbeheardin 95% of pregnant women. The other actions are notappropriate. DIF: Cognitive Level: Analyzing (Analysis) MSC: Client Needs: Health Promotion and Maintenance 10. Awomanwhois28weekspregnanthasbilateraledemainherlowerlegsafterworking8hoursadayasa cashieratalocalgrocerystore.Sheisworriedaboutherlegs.Whatisthenursesbestresponse? a. Youwillbeatriskfordevelopmentofvaricoseveinswhenyourlegsareedematous. b. Iwouldliketolistentoyourheartsounds.Edemacanindicateaproblemwithyourheart. c. Edemaisusuallytheresultoftoomuchsaltandfluidsinyourdiet.Youmayneedtocutdownon saltyfoods. d. Asyourbabygrows,itslowsbloodreturnfromyourlegs,causingtheswelling.Thisoftenoccurs with prolongedstanding. ANS: D Edemaofthelowerextremitiesoccursbecauseoftheenlargingfetus,whichimpairsvenousreturn.Prolonged standingworsenstheedema.Typically,thebilateral,dependentedemaexperiencedwithpregnancyisnotthe result of a cardiac pathologiccondition. DIF: Cognitive Level: Applying (Application) MSC: Client Needs: Health Promotion and Maintenance 11. Whenassessingawomanwhoisinherthirdtrimesterofpregnancy,thenurselooksfortheclassic symptoms associated with preeclampsia, whichinclude: a. Edema, headaches, andseizures. b. Elevated blood pressure andproteinuria. c. Elevated liver enzymes and high plateletcounts. d. Decreased blood pressure andedema. ANS: B NURSINGTB.COM Theclassicsymptomsofpreeclampsiaarehypertensionandproteinuria.Headachesmayoccurwithworsening symptoms,andseizuresmayoccurifpreeclampsiaisleftuntreatedandleadstoeclampsia.Aseriousvariantof preeclampsia, the hemolysis, elevated liver enzymes, low platelet count (HELLP) syndrome, is an ominous picture. Edema is a common occurrence inpregnancy. DIF: Cognitive Level: Remembering (Knowledge) MSC: Client Needs: Physiologic Integrity: Physiologic Adaptation 12. Thenurseknowsthatthebesttimetoassessawomansbloodpressureduringaninitialprenatalvisitis: a. Attheendoftheexaminationwhenshewillbethemostrelaxed. b. Atthebeginningoftheinterviewasanonthreateningmethodofgainingrapport. c. Duringthemiddleofthephysicalexaminationwhensheisthemostcomfortable. d. Beforebeginningthepelvicexaminationbecauseherbloodpressurewillbehigherafterthepelvic examination. ANS: A Assessingthewomansbloodpressureattheendoftheexamination,whenitishopedthatshewillbemost relaxed,isthebesttimetoassessbloodpressure.Theotheroptionsarenotcorrect. DIF: Cognitive Level: Applying (Application) MSC: Client Needs: Safe and Effective Care Environment: Management of Care 13. When examining the face of a woman who is 28 weeks pregnant, the nurse notices the presence of a butterfly-shapedincreaseinpigmentationontheface.Thepropertermforthisfindinginthedocumentationis: a. Striae. b. Chloasma. c. Lineanigra. d. Mask ofpregnancy. ANS: B Chloasmaisabutterfly-shapedincreaseinpigmentationontheface.Itisknownasthemaskofpregnancy,but when documenting, the nurse should use the correct medical term, chloasma. Striae is the term for stretch marks.Thelineanigraisahyperpigmentedlinethatbeginsatthesternalnotchandextendsdowntheabdomen through the umbilicus to thepubis. DIF: Cognitive Level: Applying (Application) MSC: Client Needs: Safe and Effective Care Environment: Management of Care 14. WhichfindingisconsiderednormalandexNpUeRctSeIdNwGhTeBn.CthOeMnurseisperformingaphysicalexaminationon a pregnantwoman? a. Palpable, fullthyroid b. Edema in one lowerleg c. Significant diffuse enlargement of thethyroid d. Palemucousmembranesofthemouth ANS:A Thethyroidmaybepalpableduringpregnancy.Itshouldfeelfull,butsmooth.Significantdiffuseenlargement occurswithhyperthyroidism,thyroiditis,andhypothyroidism.Palemucousmembranesmayindicateanemia. Bilateral lower extremity edema is common in pregnancy, but edema with pain in only one leg occurs with deep veinthrombosis. DIF: Cognitive Level: Applying (Application) MSC: Client Needs: Health Promotion and Maintenance 15. Whenauscultatingtheanteriorthoraxofapregnantwoman,thenursenoticesthepresenceofamurmur over the second, third, and fourth intercostal spaces. The murmur is continuous but can be obliterated by pressure with the stethoscope or finger on the thorax just lateral to the murmur. The nurse interprets this finding tobe: a. Murmur of aorticstenosis. b. Most likely a mammarysouffle. c. Associated with aorticinsufficiency. d. Indication of a patent ductusarteriosus. ANS: B Bloodflowthroughthebloodvessels,specificallytheinternalmammaryartery,canoftenbeheardoverthe second,third,andfourthintercostalspaces.Thisfindingiscalledamammarysouffle,butitmaybemistaken for a cardiac murmur. The other options areincorrect. DIF: Cognitive Level: Analyzing (Analysis) MSC: Client Needs: Health Promotion and Maintenance 16. Whenthenurseisassessingthedeeptendonreflexes(DTRs)onawomanwhois32weekspregnant, whichofthesewouldbeconsideredanormalfindingona0to4+scale? a. AbsentDTRs b. 2+ c. 4+ NURSINGTB.COM d. Brisk reflexes and the presence ofclonus ANS: B Normally during pregnancy, the DTRs are 1+ to 2+ and bilaterally equal. Brisk or greater than 2+ DTRs and thepresenceofclonusareabnormalandmaybeassociatedwithanelevatedbloodpressureandcerebraledema in the preeclampticwoman. DIF: Cognitive Level: Applying (Application) MSC: Client Needs: Health Promotion and Maintenance 17. Whenperforminganexaminationofawomanwhois34weekspregnant,thenursenoticesamidlinelinear protrusion in the abdomen over the area of the rectus abdominis muscles as the woman raises her head and shoulders off of the bed. Which response by the nurse iscorrect? a. The presence of diastasis recti should bedocumented. b. Thisconditionshouldbediscussedwiththephysicianbecauseitwillmostlikelyneedtobe surgicallyrepaired. c. Thepossibilitythatthewomanhasaherniaattributabletotheincreasedpressurewithinthe abdomen from the pregnancy should besuspected. d. Thewomanshouldbetoldthatshemayhaveadifficulttimewithdeliverybecauseofthe weakness in her abdominalmuscles. ANS: A Theseparationoftheabdominalmusclesiscalleddiastasisrectiandfrequentlyoccursduringpregnancy.The rectusabdominismuscleswillreturntogetherafterpregnancywithabdominalexercise.Thisconditionisnota truehernia. DIF: Cognitive Level: Applying (Application) MSC: Client Needs: Health Promotion and Maintenance 18. Thenurseispalpatingthefundusofapregnantwoman.Whichstatementaboutpalpationofthefundusis true? a. Thefundusshouldbehardandslightlytendertopalpationduringthefirsttrimester. b. Fetalmovementmaynotbefeltbytheexamineruntiltheendofthesecondtrimester. c. After20weeksgestation,thenumberofcentimetersshouldapproximatethenumberofweeks gestation. d. Fundalheightisusuallylessthanthenumberofweeksgestation,unlessanabnormalcondition suchasexcessiveamnioticfluidisNpUreRsSeInNt.GTB.COM ANS: C After 20 weeks gestation, the number of centimeters should approximate the number of weeks gestation. In addition,at20weeksgestation,theexaminermaybeabletofeelfetalmovementandtheheadcanbeballoted. DIF:CognitiveLevel:Understanding(Comprehension) MSC:ClientNeeds:HealthPromotionandMaintenance 19. Thenurseispalpatingtheabdomenofawomanwhois35weekspregnantandnoticesthatthefetalheadis facingdownwardtowardthepelvis.Thenursewoulddocumentthisasfetal: a. Lie. b. Variety. c. Attitude. d. Presentation. ANS: D Fetalpresentationdescribesthepartofthefetusthatisenteringthepelvisfirst.Fetallieisorientationofthe fetalspinetothematernalspine.Fetalattitudeisthepositionoffetalpartsinrelationtoeachother,andfetal varietyisthelocationofthefetalbacktothematernalpelvis. DIF: Cognitive Level: Applying(Application) MSC: Client Needs: Health Promotion and Maintenance 20. Thenurseispalpatingtheuterusofawomanwhois8weekspregnant.Whichfindingwouldbeconsidered to be most consistent with this stage ofpregnancy? a. The uterus seems slightly enlarged andsoftened. b. Itreachesthepelvicbrimandisapproximatelythesizeofagrapefruit. c. Theuterusrisesabovethepelvicbrimandisapproximatelythesizeofacantaloupe. d. Itisaboutthesizeofanavocado,approximately8cmacrossthefundus. ANS: D The8-weekpregnantuterusisapproximatelythesizeofanavocado,7to8cmacrossthefundus.The6-week pregnantuterusisslightlyenlargedandsoftened.The10-weekpregnantuterusisapproximatelythesizeofa grapefruitandmayreachthepelvicbrim.The12-weekpregnantuteruswillfillthepelvis.At12weeks,the uterus is sized from theabdomen. DIF: Cognitive Level: Understanding (Comprehension) MSC:ClientNeeds:HealthPromotionandMNaiUntReSnIaNncGeTB.COM 21. Which of these correctly describes the average length ofpregnancy? a. 38weeks b. 9 lunarmonths c. 280 days from the last day of the last menstrualperiod d. 280daysfromthefirstdayofthelastmenstrualperiod ANS:D Theaveragelengthofpregnancyis280daysfromthefirstdayofthelastmenstrualperiod,whichisequalto 40 weeks, 10 lunar months, or roughly 9 calendarmonths. DIF: Cognitive Level: Remembering (Knowledge) MSC: Client Needs: Physiologic Integrity: Physiologic Adaptation 22. Apatientspregnancytestispositive,andshewantstoknowwhenthebabyisdue.Thefirstdayofherlast menstrualperiodwasJune14,andthatperiodendedJune20.UsingtheNgelerule,whatisherexpecteddate ofdelivery? a. March7 b. March14 c. March21 d. March27 ANS:C TodeterminetheexpecteddateofdeliveryusingtheNgelerule,7daysareaddedtothefirstdayofthelast menstrualperiod;then3monthsaresubtracted.Therefore,adding7daystoJune14wouldbeJune21and subtracting 3 months would make the expected delivery date March21. DIF: Cognitive Level: Analyzing (Analysis) MSC: Client Needs: Safe and Effective Care Environment: Management of Care 23. Duringtheassessmentofawomaninher22ndweekofpregnancy,thenurseisunabletohearfetalheart tones with the fetoscope. The nurseshould: a. Immediately notify the physician, then wait 10 minutes and tryagain. b. Ask the woman if she has felt the baby movetoday. c. Wait10minutes,andtryagain. NURSINGTB.COM d. Use ultrasound to verify cardiacactivity. ANS: D If no fetal heart tones are heard during auscultation with a fetoscope, then the nurse should verify cardiac activityusingultrasonography.Anultrasoundshouldbeimmediatelydoneandbeforenotifyingthephysician or causing the woman distress by asking about fetalmovement. DIF: Cognitive Level: Analyzing (Analysis) MSC: Client Needs: Physiologic Integrity: Physiologic Adaptation 24. Apatientwhois24weekspregnantasksaboutwearingaseatbeltwhiledriving.Whichresponsebythe nurse iscorrect? a. Seat belts should not be worn duringpregnancy. b. Placethelapbeltbelowtheuterusandusetheshoulderstrapatthesametime. c. Placethelapbeltbelowtheuterusbutomittheshoulderstrapduringpregnancy. d. Placethelapbeltatyourwaistabovetheuterusandusetheshoulderstrapatthesametime. ANS: B Formaternalandfetalsafety,thenurseshouldinstructthewomantoplacethelapbeltbelowtheuterusandto use the shoulder strap. The other instructions areincorrect. DIF: Cognitive Level: Applying (Application) MSC: Client Needs: Physiologic Integrity: Reduction of Risk Potential 25. Duringahealthhistoryinterview,a38-year-oldwomansharesthatsheisthinkingabouthavinganother baby.Thenurseknowswhichstatementtobetrueregardingpregnancyafter35yearsofage? a. Fertility does not start to decline until age 40years. b. OccurrenceofDownsyndromeissignificantlymorefrequentaftertheageof35years. c. Geneticcounselingandprenatalscreeningarenotroutineuntilafterage40years. d. Womenolderthan35yearswhoarepregnanthavethesamerateofpregnancy-related complications as those who are younger than 35years. ANS: B TheriskofDownsyndromeincreasesasthewomanages,fromapproximately1in1250atage25yearsto1in 400 at age 35 years. Fertility declines with advancing maternal age. Women 35 years and older or with a historyofageneticabnormalityareofferedgeneticcounselingandtheoptionsofprenataldiagnosticscreening tests.BecausetheincidenceofchronicdiseaseNsUinRcSrIeNasGeTsBw.CitOhMage,womenolderthan35yearswhoare pregnantmoreoftenhavemedicalcomplicationssuchasdiabetes,obesity,andhypertension. DIF: Cognitive Level: Understanding(Comprehension) MSC: Client Needs: Physiologic Integrity: Reduction of Risk Potential 26. A25-year-oldwomanisintheclinicforherfirstprenatalvisit.Thenursewillpreparetoobtainwhich laboratory screening test at thistime? a. Urinetoxicology b. Complete blood cellcount c. Alpha-fetoprotein d. Carrierscreeningforcysticfibrosis ANS:B At the onset of pregnancy, a routine prenatal panel usually includes a complete blood cell count, serologic testing,rubellaantibodies,hepatitisBscreening,bloodtypeandRhesusfactor,andantibodyscreen.Aclean- catch urine sample is collected for urinalysis to rule out cystitis. Urine toxicology, although beneficial for womenifactivesubstanceabuseissuspectedorknown,isnotroutinelyperformed.Inthesecondtrimester, maternal serum is analyzed for alpha-fetoprotein. Carrier screening for cystic fibrosis is offered to check whetherapersoncarriestheabnormalgenethatcausescysticfibrosisbutisnotpartofroutinetesting. DIF: Cognitive Level: Applying (Application) MSC: Client Needs: Safe and Effective Care Environment: Management of Care 27. A woman at 25 weeks gestation comes to the clinic for her prenatal visit. The nurse notices that her face andlowerextremitiesareswollen,andherbloodpressureis154/94mmHg.Thewomanstatesthatshehashad headachesandblurryvisionbutthoughtshewasjusttired.Whatshouldthenursesuspect? a. Eclampsia b. Preeclampsia c. Diabetes type1 d. Pretermlabor ANS:B Classicsymptomsofpreeclampsiaincludeelevatedbloodpressure(greaterthan140mmHgsystolicor90mm Hg diastolic in a woman with previously normal blood pressure) and proteinuria. Onset and worsening symptomsmaybesudden,andsubjectivesignsincludeheadachesandvisualchanges.Eclampsiaismanifested by generalized tonic-clonic seizures. These symptoms are not indicative of diabetes mellitus (type 1 or 2) or pretermlabor. DIF: Cognitive Level: Applying (Application) MSC:ClientNeeds:SafeandEffectiveCareENnUvRirSoInNmGeTnBt:.CMOaMnagementofCare 28. Duringauscultationoffetalhearttones(FHTs),thenursedeterminesthattheheartrateis136beatsper minute. The nurses next action should beto: a. Document the results, which are within normalrange. b. Takethematernalpulsetoverifythesefindingsastheuterinesouffle. c. Have the patient change positions and count the FHTsagain. d. Immediately notify the physician for possible fetaldistress. ANS: A Thenormalfetalheartrateisbetween110and160beatsperminute.Thenurseshoulddocumenttheresultsas within the normal range. The other options are notcorrect. DIF: Cognitive Level: Analyzing (Analysis) MSC: Client Needs: Safe and Effective Care Environment: Management of Care 29. Duringawomans34thweekofpregnancy,sheistoldthatshehaspreeclampsia.Thenurseknowswhich statement concerning preeclampsia istrue? a. Preeclampsia has little effect on thefetus. b. Edema is one of the main indications ofpreeclampsia. c. Eclampsia only occurs before delivery of thebaby. d. Untreated preeclampsia may contribute to restriction of fetalgrowth. ANS: D Untreatedpreeclampsiamayprogresstoeclampsia,whichismanifestedbygeneralizedtonic-clonicseizures. Eclampsiamaydevelopaslateas10dayspostpartum.Beforethesyndromebecomesclinicallymanifested,it isaffectingtheplacentathroughvasospasmandaseriesofsmallinfarctions.Theplacentascapacitytodeliver oxygenandnutrientsmaybeseriouslydiminished,andfetalgrowthmayberestricted.Edemaiscommonin pregnancy and is not an indicator ofpreeclampsia. DIF: Cognitive Level: Applying (Application) MSC: Client Needs: Physiologic Integrity: Physiologic Adaptation MULTIPLE RESPONSE 1.Duringagroupprenatalteachingsession,thenurseteachesKegelexercises.Whichstatementswouldbe appropriate for this teaching session? Select all thatapply. a. Kegel exercises help keep your uterus strong during thepregnancy. NURSINGTB.COM b. Kegel exercises should be performed twice aday. c. Kegel exercises should be performed 50 to 100 times aday. d. ToperformKegelexercises,slowlysqueezetoapeakatthecountofeight,andthenslowlyrelease to a count ofeight. e. ToperformKegelexercises,rapidlyperformalternatingsqueeze-releaseexercisesuptothecount ofeight. ANS: C, D Kegelexercisescanbeperformedtoprepareforandtorecoverfrombirth.Thenurseshoulddirectthewoman tosqueezeslowlytoapeakatthecountofeightandthentoreleaseslowlytothecountofeight.Thenursecan prescribe this exercise to be performed 50 to 100 times aday. DIF: Cognitive Level: Applying (Application) MSC: Client Needs: Health Promotion and Maintenance Chapter 32: Functional Assessment of the Older Adult MULTIPLE CHOICE 1. Thenurseisassessinganolderadultsfunctionalability.Whichdefinitioncorrectlydescribesonesfunctiona ability? Functionalability: a. Isthemeasureoftheexpectedchangesofagingthatoneisexperiencing. b. Referstotheindividualsmotivationtoliveindependently. c. Referstothelevelofcognitionpresentinanolderperson. d. Referstoonesabilitytoperformactivitiesnecessarytoliveinmodernsociety. ANS: D Functionalabilityreferstoonesabilitytoperformactivitiesnecessarytoliveinmodernsocietyandcan includedriving,usingthetelephone,orperformingpersonaltaskssuchasbathingandtoileting. DIF:CognitiveLevel:Understanding(Comprehension) MSC:ClientNeeds:HealthPromotionandMaintenance 2. Thenurseispreparingtoperformafunctionalassessmentofanolderpatientandknowsthatagood approach would be to: NURSINGTB.COM a. Observe the patients ability to perform thetasks. b. Ask the patients wife how he does when performingtasks. c. Review the medical record for information on the patientsabilities. d. Ask the patients physician for information on the patientsabilities. ANS: A Two approaches are used to perform a functional assessment: (1) asking individuals about their ability to performthetasks(self-reports),or(2)actuallyobservingtheirabilitytoperformthetasks.Forpersonswith memoryproblems,theuseofsurrogatereporters(proxyreports),suchasfamilymembersorcaregivers,may benecessary,keepinginmindthattheymayeitheroverestimateorunderestimatethepersonsactualabilities. DIF: Cognitive Level: Applying (Application) MSC: Client Needs: Safe and Effective Care Environment: Management of Care 3. Thenurseneedstoassessapatientsabilitytoperformactivitiesofdailyliving(ADLs)andshouldchoose which tool for thisassessment? a. Direct Assessment of Functional Abilities(DAFA) b. Lawton Instrumental Activities of Daily Living (IADL)scale c. BarthelIndex d. OlderAmericansResourcesandServicesMultidimensionalFunctionalAssessment QuestionnaireIADL(OMFAQ-IADL) ANS: C TheBarthelIndexisusedtoassessADLs.TheotheroptionsareusedtomeasureIADLs. DIF: Cognitive Level: Understanding(Comprehension) MSC: Client Needs: Health Promotion and Maintenance 4. ThenurseispreparingtousetheLawtonIADLinstrumentaspartofanassessment.Whichstatementabout the Lawton IADL instrument istrue? a. The nurse uses direct observation to implement thistool. b. TheLawtonIADLinstrumentisdesignedasaself-reportmeasureofperformanceratherthan ability. c. This instrument is not useful in the acute hospitalsetting. NURSINGTB.COM d. This tool is best used for those residing in an institutionalsetting. ANS: B TheLawtonIADLinstrumentisdesignedasaself-reportmeasureofperformanceratherthanability.Direct testingisoftennotfeasible,suchasdemonstratingtheabilitytopreparefoodwhileahospitalinpatient. Attentiontothefinalscoreislessimportantthanidentifyingapersonsstrengthsandareaswhereassistanceis needed. The instrument is useful in acute hospital settings for discharge planning and continuously in outpatient settings. It would not be useful for those residing in institutional settings because many of these tasks are already being managed for theresident. DIF: Cognitive Level: Applying (Application) MSC: Client Needs: Safe and Effective Care Environment: Management of Care 5. Thenurseisassessinganolderadultsadvancedactivitiesofdailyliving(AADLs),whichwouldinclude: a. Recreationalactivities. b. Mealpreparation. c. Balancing thecheckbook. d. Self-groomingactivities. ANS: A AADLs are activities that an older adult performs such as occupational and recreational activities. Self- groomingactivitiesarebasicADLs;mealpreparationandbalancingthecheckbookareconsideredIADLs. DIF: Cognitive Level: Applying (Application) MSC: Client Needs: Health Promotion and Maintenance 6. WhenusingthevariousinstrumentstoassessanolderpersonsADLs,thenurseneedstorememberthata disadvantage of these instrumentsincludes: a. Reliability of thetools. b. Self or proxy reporting of functionalactivities. c. Lack of confidentiality during theassessment. d. Insufficient details concerning the deficienciesidentified. ANS: B AdisadvantageofmanyoftheADLandIADLinstrumentsistheselforproxyreportingoffunctional activities. The other responses are notcorrect. DIF: Cognitive Level: Understanding (Comprehension) NURSINGTB.COM MSC: Client Needs: Health Promotion and Maintenance 7. A patient will be ready to be discharged from the hospital soon, and the patients family members are concernedaboutwhetherthepatientisabletowalksafelyoutsidealone.Thenursewillperformwhichtestto assessthis? a. Get Up and GoTest b. PerformanceADLs c. Physical PerformanceTest d. TinettiGaitandBalanceEvaluation ANS:A TheGetUpandGoTestisareliableandvalidtesttoquantifyfunctionalmobility.Thetestisquick,requires little training and no special equipment, and is appropriate to use in many settings including hospitals and clinics. This instrument has been shown to predict a persons ability to go safely outside alone. The PerformanceofADLstesthasatrainedobserveractuallyobservingasapatientperformsvariousADLs.The Physical Performance Test assesses upper body fine motor and coarse motor activities, as well as balance, mobility,coordination,andendurance.TheTinettiGaitandBalanceEvaluationassessesgaitandbalanceand provides information about fallrisk. DIF: Cognitive Level: Understanding (Comprehension) MSC: Client Needs: Health Promotion and Maintenance 8. Thenurseisassessingtheformsofsupportanolderpatienthasbeforesheisdischarged.Whichofthese examples is an informal source ofsupport? a. Local seniorcenter b. Patients Medicarecheck c. Meals on Wheels meal deliveryservice d. Patientsneighbor,whovisitswithherdaily ANS:D Informalsupportincludesfamilyandclose,long-timefriendsandisusuallyprovidedfreeofcharge.Formal supportsincludeprogramssuchassocialwelfareandothersocialserviceandhealthcaredeliveryagencies such as home healthcare. DIF:CognitiveLevel:Applying(Application) MSC:ClientNeeds:PsychosocialIntegrity 9. An 85-year-old man has been hospitalized after a fall at home, and his 86-year-old wife is at his bedside. Shetellsthenursethatsheishisprimarycaregiver.Thenurseshouldassessthecaregiverforsignsofpossible caregiver burnout, suchas: NURSINGTB.COM a. Depression. b. Weightgain. c. Hypertension. d. Socialphobias. ANS: A Caregiver burden is the perceived strain by the person who cares for an older adult or for a person who is chronicallyillordisabled.Caregiverburnoutislinkedtothecaregiversabilitytocopeandhandlestress.Signs of possible caregiver burnout include multiple somatic complaints, increased stress and anxiety, social isolation,depression,andweightloss.Screeningcaregiversfordepressionmayalsobeappropriate. DIF:CognitiveLevel:Applying(Application) MSC:ClientNeeds:PsychosocialIntegrity 10. Duringamorningassessment,thenursenoticesthatanolderpatientislessattentiveandisunabletorecall yesterdaysevents.Whichtestisappropriateforassessingthepatientsmentalstatus? a. Geriatric Depression Scale, shortform b. Rapid Disability RatingScale-2 c. Mini-Cog d. Get Up and GoTest ANS:C For nurses in various settings, cognitive assessments provide continuing comparisons to the individuals baselinetodetectanyacutechangesinmentalstatus.TheMini-Cogisamentalstatustestthattestsimmediate and delayed recall and visuospatial abilities. The Geriatric Depression Scale, short form, assesses for depression and changes in the level of depression, not mental status. The Rapid Disability Rating Scale-2 measures what the person can actually do versus what he or she could do, but not mental status. The Get Up and Go Test assesses functional mobility, not mentalstatus. DIF:CognitiveLevel:Applying(Application) MSC:ClientNeeds:PsychosocialIntegrity 11. An older patient has been admitted to the intensive care unit (ICU) after falling at home. Within 8 hours, hisconditionhasstabilizedandheistransferredtoamedicalunit.Thefamilyiswonderingwhetherhewillbe abletogobackhome.Whichassessmentinstrumentismostappropriateforthenursetochooseatthistime? a. Lawton IADLinstrument b. Hospital Admission Risk Profile(HARP) NURSINGTB.COM c. Mini-Cog d. NEECHAMConfusionScale ANS:B Hospital-acquired functional decline may occur within 2 days of a hospital admission. The HARP helps identifyolderadultswhoareatgreatestriskoflosingtheirabilitytoperformADLsormobilityatthiscritical time.TheLawtonIADLmeasuresinstrumentalactivitiesofdailyliving,whichmaybedifficulttoobservein thehospitalsetting.TheMini-Cogisanassessmentofmentalstatus.TheNEECHAMConfusionScaleisused to assess fordelirium. DIF:CognitiveLevel:Applying(Application) MSC:ClientNeeds:PsychosocialIntegrity 12. Duringafunctionalassessmentofanolderpersonshomeenvironment,whichstatementorquestionbythe nurse is most appropriate regarding common environmentalhazards? a. Theselowtoiletseatsaresafebecausetheyarenearertothegroundincaseoffalls. b. Doyouhavearelativeorfriendwhocanhelptoinstallgrabbarsinyourshower? c. Thesesmallrugsareidealforpreventingyoufromslippingonthehardfloor. d. Itwouldbesafertokeepthelightinglowinthisroomtoavoidglareinyoureyes. ANS: B Environmental hazards within the home can be a potential constraint on the older persons day-to-day functioning.Commonenvironmentalhazards,includinginadequatelighting,loosethrowrugs,curledcarpet edges,obstructedhallways,cordsinwalkways,lackofgrabbarsintubandshower,andlowandloosetoilet seats,arehazardsthatcouldleadtoanincreasedriskoffallsandfractures.Environmentalmodificationscan promote mobility and reduce the likelihood of the older adultfalling. DIF: Cognitive Level: Analyzing (Analysis) MSC: Client Needs: Safe and Effective Care Environment: Safety and Infection Control 13. Whenbeginningtoassessapersonsspirituality,whichquestionbythenursewouldbemostappropriate? a. Do you believe inGod? b. How does your spirituality relate to your health caredecisions? c. What religious faith do youfollow? d. Do you believe in the power ofprayer? ANS: B NURSINGTB.COM Open-endedquestionsprovideafoundationforfuturediscussions.Theotherresponsesareeasilyansweredby one-word replies and are closedquestions. DIF: Cognitive Level: Analyzing (Analysis) MSC: Client Needs: Psychosocial Integrity 14. Thenurseispreparingtoassessanolderadultanddiscoversthattheolderadultisinseverepain.Which statement about pain and the older adult istrue? a. Pain is inevitable withaging. b. Older adults with cognitive impairments feel lesspain. c. Alleviatingpainshouldbeapriorityoverotheraspectsoftheassessment. d. Theassessmentshouldtakeprioritysothatcaredecisionscanbemade. ANS: C Iftheolderadultisexperiencingpainordiscomfort,thenthedepthofknowledgegatheredthroughthe assessmentswillsuffer.Alleviatingpainshouldbeapriorityoverotheraspectsoftheassessment. Remembering that older adults with cognitive impairment do not feel less pain is paramount. DIF: Cognitive Level: Analyzing (Analysis) MSC: Client Needs: Physiologic Integrity: Basic Care and Comfort MULTIPLE RESPONSE 1.Thenurseisassessingtheabilitiesofanolderadult.WhichactivitiesareconsideredIADLs?Selectallthat apply. a. Feedingoneself b. Preparing ameal c. Balancing acheckbook d. Walking e. Toileting f. Groceryshopping ANS: B, C,F Typically,IADLtasksincludeshopping,mealpreparation,housekeeping,laundry,managingfinances,taking medications,andusingtransportation.TheothNeUroRpStIiNonGsTlBis.tCeOdMareADLsrelatedtoself-care. DIF: Cognitive Level: Applying (Application) MSC: Client Needs: Health Promotion and Maintenance [Show More]

Last updated: 11 months ago

Preview 1 out of 928 pages

Reviews( 0 )

$21.00

Add to cart

Instant download

Can't find what you want? Try our AI powered Search

OR

GET ASSIGNMENT HELP
73
0

Document information


Connected school, study & course


About the document


Uploaded On

Feb 17, 2021

Number of pages

928

Written in

Seller


seller-icon
securegrades

Member since 3 years

117 Documents Sold


Additional information

This document has been written for:

Uploaded

Feb 17, 2021

Downloads

 0

Views

 73

Document Keyword Tags

Recommended For You


$21.00
What is Browsegrades

In Browsegrades, a student can earn by offering help to other student. Students can help other students with materials by upploading their notes and earn money.

We are here to help

We're available through e-mail, Twitter, Facebook, and live chat.
 FAQ
 Questions? Leave a message!

Follow us on
 Twitter

Copyright © Browsegrades · High quality services·